You are on page 1of 244

MCQ OF

Davidson’s
Principles and Practice of

th
20 Edition
This book is downloaded from the
Website www.studentconsult.com
CONTENTS

Part 1 Part 2 No of
Serial Subject
Question Answer Question
1 Good prescribing 3 118 11
2 Molecular mechanisms of disease 5 120 14
3 Immunological factors in disease 7 123 16
4 Environmental & nutritional factors in disease 9 125 21
5 Principles of infectious disease 12 128 17
6 Ageing and disease 15 131 8
7 Critical care and emergency medicine 16 133 16
8 Poisoning 19 136 19
9 Medical psychiatry 23 139 15
10 Oncology 25 142 11
11 Palliative care and pain management 27 144 6
12 Infectious diseases 28 146 44
13 HIV infection and the human AIDS 35 153 15
14 Sexually transmitted infections 37 156 9
15 Clinical biochemistry and metabolism 39 158 25
16 Kidney and urinary tract disease 42 163 44
17 Cardiovascular disease 48 170 80
18 Respiratory disease 59 181 48
19 Endocrine disease 68 188 36
20 Diabetes mellitus 73 195 24
21 Alimentary tract and pancreatic disease 77 200 54
22 Liver and biliary tract disease 85 208 40
23 Blood disorders 91 213 43
24 Musculoskeletal disorders 98 221 42
25 Neurological disease 104 227 68
26 Skin disease 113 238 25

1
PART 1
QUESTIONS

2
Chapter 1 GOOD PRESCRIBING
Answers page 118

Question 1. Warfarin has a low life-threatening haemorrhage, compared


therapeutic index. What exactly does this with 40 of the treatment group. Based only
term mean? on these data, what would be the number
A) It causes adverse effects at doses close of patients treated to cause one life-
to the usual therapeutic range threatening haemorrhage (i.e. the number
B) It is effective at low plasma needed to treat to cause one episode of
concentrations harm, NNTH, sometimes called 'the
C) It causes toxicity at low plasma number needed to harm')?
concentrations A) 2
D) It has a low incidence of adverse B) 16
effects and toxicity C) 25
E) It is only effective at high plasma D) 31
concentrations E) 50

Question 2. In a randomised, double-blind Question 4. Before you prescribe a new


placebo-controlled trial of a new inhaler drug for heart disease, your patient asks
for asthma there were half as many acute you how likely it is to help him. After a
severe exacerbations in patients who took quick search you find the following
the inhaler every day for 1 year compared information from clinical trials relating to
with placebo. The study authors quote a the risk of myocardial infarction over 5
number needed to treat (NNTB) of 25 for years while taking the drug:
the benefit of preventing one severe • Risk of a myocardial infarction without
exacerbation in 1 year. Based on these the new drug: 20%
data, which one of the following • Risk of a myocardial infarction with the
statements is valid? new drug: 17%
A) Only 1 in 25 patients will experience • Absolute risk reduction: 3%
any benefit from taking the new inhaler • Relative risk reduction: 15%
B) On average, two severe exacerbations • Number needed to treat (to prevent one
will be prevented for every 50 patients myocardial infarction): 33 (95%
treated for 1 confidence interval: 30.4 to 35.6)
year with the new inhaler How would you express this information
C) On average, two severe exacerbations to the patient?
will be prevented for every 25 patients A) Taking this drug for 5 years will reduce
treated with the new inhaler for 2 years your chance of a heart attack by 15%
D) Patients treated with the new inhaler B) If you take the drug for 5 years, there is
for 1 year are 25% less likely to have a a 15% chance that it will prevent you from
severe exacerbation during that time than having a heart attack
those not treated C) Taking this drug for 5 years will reduce
E) The number needed to harm is greater the chances of your having a heart attack
than 25 by 3%
D) If 100 people like you were to take this
Question 3. A new drug is shown to drug for 5 years it would prevent 15 heart
reduce mortality from sepsis from 310 out attacks
of 1000 in the placebo group to 250 out of E) There is a 1 in 33 chance that taking
1000 in the treatment group. A total of 20 this drug for 5 years will reduce your
of the placebo group develop chance of a heart attack by 95%

3
Question 5. A drug company A) The prescribing doctor's General
representative tells you that a new drug, Medical Council (GMC) registration
'Potentol', is 10 times more potent than the number
currently popular drug of the same class, B) The route of administration of the drug
'Weakol'. Which of the following C) The condition that is being treated
statements concerning the two drugs can D) The proprietary name of the prescribed
be made with greatest confidence? drug
A) The maximum effect of Potentol will E) The duration of treatment
be greater than the maximum effect of
Weakol Question 9. Which one of the following is
B) Potentol will be a more effective drug not required on a prescription for a
C) Potentol and Weakol must act on controlled drug?
different receptors A) Patient's date of birth
D) Potentol will have a more favourable B) Doctor's address
adverse effects profile C) Patient's address
E) The dose-response curve for Potentol D) Exact size of each dose specified in
lies to the left of the curve for Weakol words and figures
E) Total quantity to be dispensed
Question 6. The dose of which one of the
following drugs should be reduced in Question 10. Treatment with which one of
patients with mild renal insufficiency? the following drugs is often initiated with
A) Co-amoxiclav (amoxicillin plus a loading dose?
clavulanic acid) A) Codeine
B) Paracetamol B) Amiodarone
C) Digoxin C) Atenolol
D) Simvastatin D) Allopurinol
E) Furosemide E) Furosemide

Question 7. Which one of the following Question 11. In a trial of a new treatment
drug combinations is most likely to be for fulminant sepsis the mortality in the
harmful? treatment group is 20%, whereas the
A) Digoxin and warfarin mortality in the placebo group is 40%.
B) Paracetamol and dihydrocodeine What is the relative risk of death with the
C) Trimethoprim and sulfamethoxazole new treatment?
D) Allopurinol and azathioprine A) 0.1
E) Atenolol and ciclosporin B) 0.2
C) 0.4
Question 8. Which one of the following D) 0.5
pieces of information must be included in E) 2.0
a drug prescription outside hospital in the
UK?

4
MOLECULAR
Chapter 2 MECHANISMS OF
DISEASE
Answers page 120

Question 1. Which of the following Question 5. Which of the following


human cells contain 46 chromosomes? statements about cell death by apoptosis is
A) Erythrocytes true?
B) Spermatozoa A) The process only occurs in elderly
C) Platelets individuals
D) Neutrophils B) Cell destruction occurs by an antibody-
E) Ova mediated process
C) The cell membrane remains intact
Question 2. The term 'transcription' refers D) There is an associated acute
to which of the following cellular inflammatory response
processes? E) It is the major mechanism underlying
A) Activation of second messengers such myocyte death in acute myocardial
as adenyl cyclase infarction
B) Formation of messenger RNA from
DNA by RNA polymerase Question 6. A 45-year-old man is
C) Splicing out of introns from RNA diagnosed with Huntington's disease. He
D) Protein synthesis from RNA on and his wife have non-identical twins, now
ribosomes aged 19. After genetic counselling, one of
E) Protein modification in the Golgi the twins undergoes testing for the
apparatus Huntingtin gene which is found to be
positive. What is the risk of the other twin
Question 3. Mutation of the CFTR gene in inheriting the Huntingtin gene?
patients with cystic fibrosis mediates its A) Zero
effect by interfering with which of the B) 25%
following cellular processes? C) 50%
A) Protein modification in the Golgi D) 75%
apparatus E) 100%
B) Active transport of sodium and
potassium via the Na/K ATPase pump on Question 7. Which of the following
the cell membrane conditions is not correctly paired with its
C) Chloride transport through channels on usual mode of inheritance?
the cell membrane A) Wilson's disease - autosomal recessive
D) Endocytosis of extracellular debris B) Friedreich's ataxia - autosomal
E) Binding of transcription factors in lung recessive
tissue to upstream gene promoters C) Familial adenomatous polyposis coli -
autosomal dominant
Question 4. All of the following D) Haemochromatosis - autosomal
molecules except one bind to intracellular dominant
receptors. Which is the exception? E) Myotonic dystrophy - autosomal
A) Vitamin A dominant
B) Glucocorticoids
C) Noradrenaline (norepinephrine)
D) Triiodothyronine
E) Oestrogen

5
Question 8. What is the underlying Question 12. A man with haemophilia A
chromosomal abnormality in Down's is married to a woman who is not a carrier
syndrome? for haemophilia. They are planning to start
A) 45,XO a family and so attend the genetic
B) 47,XXY counselling clinic. What advice would you
C) Trisomy 13 give them?
D) Trisomy 18 A) A son would have haemophilia and a
E) Trisomy 21 daughter would have a 50% chance of
being a carrier
Question 9. Which one of the following B) A son would have haemophilia and a
familial cancer syndromes is associated daughter would have a 100% chance of
with mutations in BRCA genes? being a carrier
A) Von Hippel-Lindau disease C) A son would be unaffected and a
B) Peutz-Jeghers syndrome daughter would have a 50% chance of
C) Li-Fraumeni syndrome being a carrier
D) Hereditary non-polyposis colorectal D) A son would be unaffected and a
cancer (HNPCC) daughter would have a 100% chance of
E) Familial breast/ovarian cancer being a carrier
E) A son would have a 50% chance of
Question 10. Which one of the following being a carrier and a daughter would have
conditions does not show autosomal haemophilia
dominant inheritance?
A) Gilbert's disease Question 13. Which one of the following
B) Congenital adrenal hyperplasia terms refers to the proportion of
C) Familial hypercholesterolaemia individuals inheriting an abnormal gene
D) Marfan's syndrome who go on to exhibit clinical disease?
E) Adult polycystic kidney disease A) Anticipation
B) Imprinting
Question 11. Which one of the following C) Penetrance
features is not characteristic of Turner's D) Lyonisation
syndrome? E) Translocation
A) Only females are affected
B) Short stature Question 14. The cluster of cerebellar
C) Webbed neck haemangioblastoma, renal cell carcinoma
D) Secondary amenorrhoea and phaeochromocytoma occurs in which
E) Coarctation of the aorta one of the following conditions?
A) Peutz-Jeghers syndrome
B) Von Hippel-Lindau disease
C) Li-Fraumeni syndrome
D) Wermer's syndrome
E) Gardner's syndrome

6
Chapter 3 IMMUNOLOGICAL
FACTORS IN DISEASE
Answers page 123

Question 1. Which of the following is a A) Phagocyte deficiency


component of the acquired immune B) Complement deficiency
response in humans? C) T-lymphocyte deficiency
A) Low pH of skin D) Antibody deficiency
B) Secretion of lysozyme E) Neutrophil deficiency
C) Colonisation with commensal bacteria
D) Phagocytes Question 5. A 35-year-old man presents
E) T lymphocytes with his third episode of meningococca
meningitis. What is the most likely
Question 2. A gardener cuts his finger primary immune deficiency that could
while using a rusty knife. Over the account for
following 24 hours, his finger becomes This ?
red, warm and painful. Which cell type is A) Complement deficiency
most important in immune defence during B) T-cell deficiency
these initial stages of infection? C) Neutrophil deficiency
A) Eosinophils D) Antibody deficiency
B) Neutrophils E) Primary cytokine deficiency
C) T cells
D) B cells Question 6. A 30-year-old woman, who is
E) Basophils known to be hepatitis C-positive, develops
joint pain, microscopic haematuria and a
Question 3. The infection described in the purpuric rash. Which diagnosis would best
previous question (Q2) spreads and the explain her symptoms?
gardener is given a course of antibiotics by A) Anti-phospholipid syndrome
his general practitioner. Which of the B) Autoimmune thrombocytopenia
following blood tests would be most C) Liver cirrhosis
useful in monitoring his response to D) Cryoglobulinaemia
treatment? E) Autoimmune hepatitis
A) C-reactive protein (CRP)
B) T-cell count Question 7. A 30-year-old woman is
C) Complement (C3 and C4) being investigated for recurrent
D) Immunoglobulin IgM levels miscarriages. She has recently developed a
E) Immunoglobulin IgG levels deep venous thrombosis and is on
warfarin. Which of the following tests
Question 4. A 48-year-old man presents would be most useful in investigating her
with progressive visual loss and is found underlying disease?
to have fulminant cytomegalovirus (CMV) A) Erythrocyte sedimentation rate
infection of the retina. He also has B) Rheumatoid factor
recurrent oral candidiasis. A deficiency of C) T-cell count
which of the following immune D) Anticardiolipin antibodies
components is most likely to cause these E) Lupus anticoagulant
problems?

7
Question 8. An 8-year-old boy presents B) Acute cell-mediated transplant
with recurrent bacterial chest infections rejection
and bronchiectasis. What is the most likely C) ABO blood transfusion reaction
primary immune deficiency that could D) Farmer's lung
account for this? E) Nickel hypersensitivity
A) Complement deficiency
B) T-cell deficiency Question 13. Which of the following
C) Neutrophil function disorder conditions is associated with anti-
D) Antibody deficiency centromere antibodies?
E) Cytokine defect A) CREST syndrome
B) Rheumatoid arthritis
Question 9. Which of the following C) Systemic lupus erythematosus
measures is least likely to form part of the D) Sjögren's syndrome
management of a patient with an antibody E) Mixed connective tissue disease
deficiency syndrome?
A) Long-term prophylactic antibiotics Question 14. Which of the following
B) Vaccination with live attenuated conditions is particularly associated with
vaccines anti-Ro and anti-La antibodies?
C) Intravenous immunoglobulin infusion A) Dermatomyositis
D) Aggressive treatment of even mild B) Drug-induced lupus
infections C) Mixed connective tissue disease
E) Monitoring of immunoglobulin levels D) Diffuse scleroderma
E) Sjögren's syndrome
Question 10. In DiGeorge syndrome,
abnormal development of which of the Question 15. c-ANCA (anti-neutrophil
following structures leads to immune cytoplasmic antibody with cytoplasmic
deficiency? fluorescence) is commonly associated with
A) Thymus which one of the following conditions?
B) Pituitary A) Churg-Strauss syndrome
C) Thyroid B) Wegener's granulomatosis
D) Parathyroid C) Henoch-Schönlein purpura
E) Spleen D) Temporal arteritis
E) Rheumatoid vasculitis
Question 11. Which one of the following
human leucocyte antigen (HLA) Question 16. Which of the following best
genotypes is strongly associated with describes the characteristic features of the
ankylosing spondylitis? amyloid diseases?
A) HLA-DR3 A) Extracellular deposition of insoluble
B) HLA-B27 proteins
C) HLA-DR4 B) Intracellular deposition of insoluble
D) HLA-B12 proteins
E) HLA-DR2 C) Extracellular deposition of soluble
proteins
Question 12. According to the Gell and D) Intracellular deposition of soluble
Coombs classification of hypersensitivity, proteins
which of the following is an example of a E) Intracellular deposition of insoluble
type II (antibody-mediated) reaction? carbohydrates
A) Acute anaphylaxis

8
ENVIRONMENTAL AND
Chapter 4 NUTRITIONAL FACTORS IN
DISEASE
Answers page 125

Question 1. Which one of the following Question 4. After which one of the
terms describes the number of new cases following interventions is a patient most
of a disease occurring in a population at likely to be abstinent from cigarettes 6
risk during a defined period of time? months later?
A) Rate of the disease A) Bupropion
B) Prevalence of the disease B) Nicotine replacement therapy (NRT)
C) Relative risk of the disease C) NRT plus behavioural support
D) Attributable risk of the disease D) Behavioural support
E) Incidence of the disease E) Opportunistic advice from the doctor

Question 2. A nuclear power station is Question 5. Which one of the following


built in close proximity to a village with a statements about the physiological
population of 3000 (village X). The adaptations that occur at high altitude is
residents are concerned about the rising true?
incidence of disease Z. The incidence of A) The oxyhaemoglobin dissociation
disease Z in village X in 1 year was 150 curve moves to the right
cases. In a matched population of 3000 B) In those acclimatised to high altitude,
people not living in proximity to a nuclear 2,3-DPG production is decreased
power station, the incidence of disease Z is C) In those acclimatised to high altitude,
30 cases per year. What is the relative risk erythropoietin (Epo) production is
of disease Z in village X? decreased
A) 5 D) Full physiological adaptation to altitude
B) 10 is complete in 3-4 hours in most
C) 3 individuals
D) 30 E) Hypoventilation is one of the adaptive
E) 50 mechanisms used by the body when
ascending to high altitude
Question 3. Regular cigarette smoking has
been associated with an increased risk of Question 6. A traveller on an expedition
developing all of the following diseases to the Andes (altitude 3850 m) becomes
except one. Which is the exception? unwell on the third day of the trip. He
A) Peripheral vascular disease initially complains of breathlessness, dry
B) Peptic ulcer disease cough and fatigue. The cough
C) Bronchogenic carcinoma subsequently becomes productive of
D) Ulcerative colitis bloody sputum and on examination he is
E) Stroke found to be hypoxic with crepitations in
both lung fields. What is the likeliest
diagnosis?
A) Acute mountain sickness
B) Pulmonary embolism
C) High-altitude pulmonary oedema
D) Acute respiratory distress syndrome
E) Pneumothorax

9
Question 7. In a patient diagnosed with Question 12. Which one of the following
acute mountain sickness, the appearance statements regarding artificial nutritional
of which of the following clinical features support in hospital is true?
should lead to a review of the diagnosis? A) Aspiration pneumonia is a recognised
A) Headache complication of parenteral feeding
B) Visual loss B) High-energy supplement drinks are an
C) Difficulty sleeping adequate replacement for normal diet in
D) Fatigue the hospital inpatient
E) Nausea and vomiting C) Parenteral nutrition is preferable to
enteral feeding in most cases
Question 8. Which one of the following is D) Hyperosmolar feeding solutions can be
not a complication of near-drowning? administered through a wide-bore venous
A) Haemoptysis cannula inserted into a large peripheral
B) Hypotension vein
C) Metabolic alkalosis E) Feeding via a central (subclavian or
D) Dehydration internal jugular) line carries a higher risk
E) Cardiac arrhythmias of sepsis than feeding via a percutaneous
endoscopic gastrostomy (PEG) tube
Question 9. Non-starch polysaccharides
(NSP) are the most important source of Question 13. Which one of the following
fibre in the human diet. Which one of the drug treatments is most likely to lead to
following is not classed as an NSP? weight gain?
A) Hemicellulose A) Thyroxine
B) Cellulose B) Combined oral contraceptive pill
C) Pectin C) Metformin
D) Linoleic acid D) Sibutramine
E) Gums E) Omeprazole

Question 10. The calorific value of one Question 14. Which one of the following
gram of protein is 5 kcal. How many statements regarding obesity is true?
calories are in one gram of fat? A) Obesity is defined by World Health
A) 5 kcal Organization criteria as a body mass index
B) 7 kcal (BMI) of 25 kg/m2 and above
C) 9 kcal B) Obesity is more common in smokers
D) 12 kcal C) Hypothyroidism is an irreversible cause
E) 17 kcal of obesity
D) Known genetic mutations account for
Question 11. Which one of the following 60% of the susceptibility to obesity
statements regarding dietary proteins is E) Obese people have a higher basal
false? metabolic rate than lean people
A) The term 'biological value' of a protein
refers to the relative proportions of Question 15. Obesity is associated with an
essential amino acids which it contains increased risk of all of the following
B) Proteins typically contribute < 20% to conditions except one. Which is the
overall energy intake exception?
C) Some essential amino acids cannot be A) Coronary heart disease
obtained from vegetarian diets, B) Non-alcoholic steatohepatitis
necessitating the use of supplements C) Osteoporosis
D) Essential amino acids are so called D) Endometrial cancer
because they cannot be synthesised in E) Obstructive sleep apnoea
humans, but are essential for the synthesis
of important proteins
E) The 'energy density' of protein (in
kcal/g) is lower than that of fat

10
Question 16. Which of the following large bruises and has been bleeding from
interventions is likely to produce the his gums. Routine blood tests have also
greatest magnitude of weight loss in revealed a mild iron-deficiency anaemia.
patients with morbid obesity? You suspect that there is a nutritional
A) Bariatric gastric surgery element to his complaints. Which of the
B) Modified diet following vitamins is the patient most
C) Orlistat likely to be deficient in?
D) Graded exercise programme A) Vitamin C
E) Sibutramine B) Vitamin A
C) Vitamin B1
Question 17. Which of the following is D) Vitamin B12
not a fat-soluble vitamin? E) Vitamin K
A) Vitamin K
B) Vitamin A Question 20. Which one of the following
C) Vitamin D inorganic nutrients is incorrectly paired
D) Vitamin C with its deficiency state?
E) Vitamin E A) Iodine Congenital hypothyroidism
B) Calcium Impaired bone mineralisation
Question 18. Which one of the following C) Fluoride Discoloration of teeth
vitamins is incorrectly matched with its D) Zinc Dermatitis and chronic diarrhoea
deficiency state? E) Selenium Cardiomyopathy in children
A) Vitamin C Scurvy
B) Vitamin B2 (riboflavin) Beri-beri Question 21. All of the following except
C) Niacin (nicotinic acid) Pellagra one are clinical features suggestive of
D) Folate (folic acid) Neural tube defects severe adult malnutrition. Which is the
in the fetus exception?
E) Vitamin A Night blindness A) Thinning of hair
B) Increased skinfold thickness over the
Question 19. The district nurse asks you triceps
to review an 84-year-old man at his home. C) Dependent oedema
She has been administering regular D) Amenorrhoea or impotence
dressings for a chronic venous stasis ulcer E) Dryness of skin with occasional
that has been slow to heal, but is now pigmented patches
concerned that he has developed several

11
Chapter 5 PRINCIPLES OF
INFECTIOUS DISEASE
Answers page 128

Question 1. Which one of the following Question 5. Which of the following


statements regarding infectious agents is measures is of least importance in the
incorrect? management of asplenic patients?
A) Bacteria possess a cell wall A) Yearly receipt of the polyvalent flu
B) Viruses contain either DNA or RNA vaccine
C) Prions are highly complex structures B) Yearly receipt of the pneumococcal
consisting of > 100 different proteins vaccine
D) Fungal cells contain a defined nucleus C) Receipt of the meningococcal C
E) Viruses cannot reproduce autonomousl vaccine
D) Prophylactic antibiotics prior to dental
Question 2. Which one of the following procedures
organisms is incorrectly matched with its E) Long-term daily dose of penicillin
mode of transmission?
A) Legionella Water aerosol Question 6. Which one of the following
B) Salmonella Faecal-oral conditions is a notifiable disease in the UK
C) Listeria Ingestion under the Public Health (Infectious
D) Hepatitis A Needlestick injury Diseases) Regulations 1988?
E) Tetanus Direct skin penetration A) Chicken pox
B) HIV
Question 3. Which one of the following C) Scabies
vaccines is inactivated (as opposed to D) Syphilis
live)? E) Meningococcal meningitis
A) Pertussis
B) Measles Question 7. Which one of the following
C) Mumps human pathogens is correctly matched
D) BCG (tuberculosis) with its most common method of
E) Rubella identification?
A) Streptococcus pyogenes Polymerase
Question 4. Which one of the following chain reaction (PCR)
vaccines is not given to children until the B) Hepatitis B virus Blood culture
second year of life? C) Neisseria gonorrhoeae Direct
A) HiB (Haemophilus influenzae type B) microscopy
vaccine D) Mycobacterium tuberculosis Enzyme-
B) Oral polio vaccine linked immunosorbent assay (ELISA)
C) Meningitis group C vaccine E) Candida albicans Complement fixation
D) MMR (measles, mumps, rubella) test
vaccine
E) DPT (diphtheria, pertussis, tetanus)
vaccine

12
Question 8. Which one of the antibiotics Question 10. In which one of the
below is correctly paired with its following clinical scenarios is the
predominant mode of action? suggested prophylactic agent inappropriate
A) Penicillin Disrupts bacterial protein for the purpose described?
synthesis by binding to the 50S subunit of A) Prevention of spontaneous bacterial
bacterial ribosomes peritonitis in a patient with
B) Gentamicin Inhibits cell wall synthesis cirrhosis Norfloxacin
C) Ciprofloxacin Inhibits DNA gyrase, B) Prevention of bacteraemia in a post-
thereby blocking DNA replication splenectomy
D) Minocycline Inhibits dihydrofolate and patient Phenoxymethylpenicillin
tetradihydrofolate reductase to block C) Prevention of endocarditis in a patient
bacterial folate synthesis with a prosthetic mitral valve undergoing
E) Erythromycin Forms superoxides that tooth extraction Amoxicillin
damage proteins, nuclear acid and lipids D) Prevention of gas gangrene in a patient
with a contaminated wound Flucloxacillin
Question 9. One of the following E) Prevention of varicella infection in a
definitions of terms relating to the healthy newborn exposed to varicella
pharmacokinetics of antibiotics is zoster during delivery Varicella immune
incorrect.Which is it? globulin
A) Minimum inhibitory concentration
(MIC) is the lowest concentration of an Question 11. Which one of the following
antibiotic required to inhibit 50% of the statements regarding ß-lactam antibiotics
colonies of a particular organism is incorrect?
B) t{1/2} (plasma half-life) of an antibiotic A) Benzylpenicillin, flucloxacillin and
is the time taken for the concentration of meropenem are all subtypes of this class
the drug in the body to be reduced by half B) They are bactericidal antibiotics
C) Post-antibiotic effect (PAE) is the C) Most patients with established allergy
clinically useful property of some classes to benzylpenicillin are also allergic to
of antibiotic whereby inhibition of cephalosporins
microbial replication continues beyond the D) Synergism occurs in combination with
time when the MIC is reached in plasma the aminoglycosides (e.g. gentamicin)
D) Inoculum effect is the reduction in E) Potential side-effects include
activity of an antibiotic by the presence of gastrointestinal upset and interstitial
large numbers of organisms despite nephritis
apparent MIC-based sensitivity
E) Synergism is the enhanced efficacy Question 12. A previously healthy 14-
achieved by co-administration of two year-old girl is admitted to the paediatric
different antibiotics that exceeds the effect intensive care unit with a severe lower
predicted by their combined MICs respiratory tract infection (LRTI).
Serological investigations subsequently
reveal very high titres of Mycoplasma
pneumoniae. Which of the following
antibiotics is most effective against this
organism?
A) Flucloxacillin
B) Metronidazole
C) Clarithromycin
D) Co-amoxiclav
E) Ceftriaxone

13
Question 13. An 83-year-old lady is B) It has potent anti-Gram-negative
admitted to the stroke unit with a right properties, but little or no anti-Gram-
partial anterior circulation stroke after positive activity
being found at home on the floor, C) Side-effects involving the
surrounded by vomit. On day 2 of her gastrointestinal tract and skin are frequent
admission, she develops a productive and troublesome
cough, and oxygen saturations fall from D) It has a small volume of distribution
96% to 89% on air. Her temperature is and poor tissue penetration
38.5°C. She has coarse crepitations at the E) Oral absorption is enhanced by
right base and opacification of the right administration with food
lower zone on chest X-ray. While awaiting
the results of microbiological Question 16. Which one of the following
investigations, which one of the following statements about glycopeptide antibiotics
antibiotic regimens would you prescribe? (such as vancomycin) is incorrect?
A) IV ceftriaxone and metronidazole A) Rapid infusion may produce an
B) IV vancomycin alone anaphylactoid reaction from histamine
C) IV benzylpenicillin and flucloxacillin release
D) Oral co-amoxiclav and clarithromycin B) They exhibit good activity against
E) IV amoxicillin alone meticillin-resistant Staphylococcus aureus
(MRSA)
Question 14. Which of the following C) Therapeutic monitoring of drug levels
adverse effects is least likely to occur as a is required
result of aminoglycoside (e.g. gentamicin) D) Cover against Gram-negative
administration? organisms is poor
A) Renal toxicity E) They have good oral bioavailability
B) Cochlear toxicity
C) Neuromuscular blockade following Question 17. Which one of the following
rapid IV infusion antimicrobials is incorrectly matched with
D) Hepatotoxicity its adverse side-effect?
E) Nausea and vomiting A) Chloramphenicol 'Grey baby syndrome'
B) Minocycline Tooth discoloration in
Question 15. Which one of the following children
statements regarding ciprofloxacin is true? C) Doxycycline Photosensitivity
A) It has poor oral bioavailability D) Ketoconazole Nephrotoxicity
E) Erythromicin Cholestatic jaundice

14
Chapter 6 AGEING AND DISEASE
Answers page 131

Question 1. Features of ageing include all of D) Alzheimer's disease


the following except one. Which is the E) Hyponatraemia
exception?
A) Reduced glomerular filtration rate Question 6. Which one of the following
B) Lens opacification statements regarding urinary incontinence is
C) Neuronal loss true?
D) Increased insulin secretion A) It is a feature of normal ageing
E) Reduced maximum heart rate B) It can be caused by hypocalcaemia
C) It affects 50% of women over the age of 65
Question 2. Which of the following drugs is D) Bladder catheterisation is usually the first-
least likely to contribute to recurrent falls in an line treatment
elderly patient? E) Laxatives may be helpful
A) Codeine phosphate
B) Temazepam Question 7. An 82-year-old man is referred to
C) Amitriptyline the medical outpatient clinic with recurrent
D) Omeprazole dizzy spells. He describes periodic
E) Furosemide lightheadedness occurring several times a
week, typically lasting for less than 1 minute.
Question 3. Which one of the following is not Symptoms are unrelated to posture. He has
an established risk factor for recurrent falls in never lost consciousness and denies chest pain
the elderly? or palpitations. He has a past history of
A) Cognitive impairment hypertension and osteoarthritis and normally
B) Depression takes atenolol and paracetamol. Examination is
C) Osteoarthritis unremarkable and the resting ECG normal.
D) Visual impairment What would the most useful initial
E) Bisphosphonate therapy investigation be?
A) Cervical spine X-ray
Question 4. A 79-year-old man is referred to B) 24-hour ambulatory ECG
the day hospital with recurrent falls. He has a C) Echocardiogram
history of hypertension and type II diabetes. D) CT head scan
He normally takes aspirin, amlodipine, E) MRI head scan
metformin and simvastatin. Blood pressure is
121/86 mmHg when supine, falling to 96/61 Question 8. An 86-year-old lady on the care of
mmHg when standing. What would the most the elderly ward is found to have a coin-shaped
appropriate initial treatment be? lesion suggestive of a malignancy on a routine
A) Continue current treatment chest X-ray. She has a profound expressive
B) Continue current treatment and start dysphasia resulting from a previous stroke and
fludrocortisone suffers from severe chronic obstructive
C) Stop amlodipine pulmonary disease. Which of the following
D) Stop amlodipine and start bendrofluazide factors should least influence further
E) Stop all medication except aspirin investigation and management?
A) The patient's comorbidity
Question 5. Causes of acute confusion in the B) The patient's ability to tolerate the available
elderly include all of the following except one. treatments
Which is the exception? C) Chronological age
A) Pulmonary embolism D) Biological age
B) Urinary tract infection E) An advance directive prepared by the
C) Recent prescription for tramadol patient

15
Chapter 7 CRITICAL CARE AND
EMERGENCY MEDICINE
Answers page 133

Question 1. You are asked to prescribe Question 4. A 65-year-old patient in the


fluids for a 66-year-old, previously well ICU appears to be making an excellent
man on the high-dependency unit, who recovery from pneumonia but then
returned from a radical prostatectomy 8 deteriorates acutely. He has the following
hours ago. Preoperatively, his blood observations:
pressure was 145/78 mmHg. He lost 2 Which of the following diagnoses is the
litres of blood in theatre and has since most likely cause of his deterioration?
received intravenous fluids and 4 units of A) Major haemorrhage
red cells. In the past 4 hours his B) Pulmonary embolism
intravenous intake has consisted of a 500 C) Septic shock (pre-volume load)
ml bag of 0.9% saline. He is alert and D) Neurogenic shock
complains of feeling thirsty. He does not E) Acute left ventricular failure secondary
have an epidural. The observation chart to myocardial infarction
shows:
Which of the following is the most Question 5. Which one of the following
appropriate fluid regimen to prescribe? would be the best intramuscular dose of
A) 6-hourly Hartman's solution adrenaline (epinephrine) in a patient with
B) 8-hourly alternating 0.9% saline and acute anaphylactic shock and airway
5% dextrose, with potassium compromise?
supplementation A) 1 ml of 1 in 1000 adrenaline
C) 4-hourly 0.9% saline B) 1 ml of 1 in 10 000 adrenaline
D) Immediate transfusion with O-negative C) 1 ml of 1 in 100 000 adrenaline
blood D) 10 ml of 0.1% adrenaline
E) 250 ml of gelofusine over 15 minutes E) 10 ml of 1% adrenaline

Question 2. Which of the following Question 6. Which of the following drugs


haemodynamic parameters is represented causes an increase in systemic vascular
by pulmonary artery wedge pressure resistance?
(PAWP)? A) Sodium nitroprusside
A) Central venous pressure B) Noradrenaline (norepinephrine)
B) Pulmonary arterial pressure C) Dobutamine
C) Left atrial pressure D) Dopexamine
D) Systemic capillary pressure E) Glyceryl trinitrate (GTN)
E) Intrathoracic pressure
Question 7. Which one of the following
Question 3. Which of the following forms part of the diagnostic criteria for
factors has the least direct bearing on acute respiratory distress syndrome
central venous pressure? (ARDS)?
A) Pulmonary artery pressure A) Normal chest X-ray
B) Systemic arterial pressure B) PaCO2 > 6.5
C) Venous tone C) Blood pressure < 90/50 mmHg
D) Intravascular volume D) Pulmonary artery wedge pressure
E) Intrathoracic pressure (PAWP) < 15 mmHg
E) Increased lung compliance

16
Question 8. A patient presents critically ill E) There is a metabolic alkalosis in the
in type 1 respiratory failure with the second ABG
following arterial blood gas
measurements: Question 11. Which of the following
metabolic derangements is most likely to
cause coma?
A) Hyperkalaemia
B) Hypercalcaemia
C) Hypokalaemia
D) Hyponatraemia
E) Hyperuricaemia

Which of the following conditions is least Question 12. A 35-year-old man is


likely to explain these findings? admitted to the ICU with abdominal sepsis
A) Diamorphine overdose following a laparotomy for a ruptured
B) Pulmonary embolus appendix. He is systemically unwell and
C) Acute asthma requires ventilation. One day post-
D) Pneumothorax operatively, bleeding is noticed from his
E) Pulmonary oedema abdominal wound and sites of line
insertion. The following results are
Question 9. In which of the following obtained from tests of coagulation:
conditions is continuous positive airways
pressure (CPAP) usually most effective?
A) Acute severe asthma
B) Pulmonary oedema secondary to left
ventricular failure (LVF)
C) Exacerbation of chronic obstructive
pulmonary disease (COPD)
D) Narcotic drug overdose
E) Neuromuscular disorder leading to Which of the following is the most likely
respiratory failure underlying problem?
A) Warfarin effect
Question 10. This series of arterial blood B) Heparin effect
gas (ABG) measurements was taken from C) Disseminated intravascular coagulation
the same patient 24 hours apart: (DIC)
D) Haemolytic uraemic syndrome (HUS)
E) Idiopathic thrombocytopaenic purpura

Question 13. Which one of the following


is an appropriate strategy for controlling
intracranial pressure in a patient with a
head injury?
A) 30° head-down tilt
B) Use intravenous dextrose to reduce
Which one of the following statements is plasma sodium
true? C) Use of positive end-expiratory pressure
A) There is a respiratory alkalosis in the (PEEP) ventilation
first ABG D) Mild hyperventilation to reduce PaCO2
B) There is a respiratory acidosis in the to 4-4.5 kPa
second ABG E) Aggressive fluid resuscitation to keep
C) Arterial bicarbonate (HCO3-) must be CVP above 20 cmH2O
lower in the second ABG
D) Pulmonary gas exchange has improved
in the second ABG

17
Question 14. Which of the following He is transferred to the ICU. Which of the
complications is associated with treatment following measures would be most
of ventilated patients with ranitidine? appropriate as part of his further
A) Stress ulceration management?
B) Ischaemic bowel A) Intubation and ventilation
C) Hyperbilirubinaemia B) Administration of a diuretic
D) Pneumonia C) Central venous catheterisation
E) Acute tubular necrosis D) Commencement of inotrope infusion
E) Haemofiltration
Question 15. A 72-year-old patient with a
history of ischaemic heart disease is being Question 16. The patient from the
monitored in the surgical high-dependency previous question is resuscitated over the
unit following drainage of an intra- next 6 hours in the ICU. Despite him
abdominal abscess. At 2000 hours he receiving 4 litres of intravenous fluid, his
complains of feeling nauseated and blood pressure and urine output fail to
feverish. He is referred to the critical care improve significantly. An oesophageal
team. Below is his observation chart for cardiac output monitor is placed and
the previous 6 hours (breathing air): demonstrates a cardiac output of 8
litres/min. (A normal cardiac output would
be 5 litres/min for a man of this size.)
Which of the following is the likeliest
diagnosis?
A) Hypovolaemic shock
B) Septic shock
C) Anaphylactic shock
D) Cardiogenic shock
E) Obstructive shock

18
Chapter 8 POISONING
Answers page 136

Question 1. A rural hospital Accident and Question 3. Which of the following drugs
Emergency department is overwhelmed is most effectively removed by
when 21 people present with poisoning haemodialysis?
due to contamination of the local water A) Paracetamol
supply. They all have small pupils, muscle B) Venlafaxine
fasciculation, excessive salivation and C) Amitriptyline
lacrimation, vomiting, diarrhoea and D) Metformin
hyper-reflexia. What is the likeliest E) Aspirin
contaminant?
A) Amphetamine-related compounds Question 4. Activated charcoal may be
B) Methadone useful in the treatment of overdose with all
C) Organophosphate insecticides of the following drugs except one. Which
D) Arsenic is the exception?
E) Fluoride A) Theophylline
B) Ethanol
Question 2. A 19-year-old girl is brought C) Paracetamol
to hospital by her partner, having been D) Quinine
found unconscious at home after a night E) Carbamazepine
out. Her partner has brought an empty
bottle of the tricyclic antidepressant, Question 5. Which of the following drugs
amitriptyline, which he believes she has is the most appropriate treatment for
taken in overdose. You meet her on her paracetamol overdose?
arrival at the hospital Accident and A) Glucagon
Emergency department. What is the B) Gamma-hydroxybutyrate
priority on her arrival at the hospital? C) Desferrioxamines
A) A history from the partner of the D) N-acetylcysteine
estimated dose and any possible co- E) Pralidoxime
ingestants, such as alcohol
B) Cardiac monitoring and/or a portable
defibrillator in case of cardiac arrhythmias
C) Assess the patient's pulse rate to
identify any tachyarrhythmia
D) Ensure a patent airway
E) Administer 100% oxygen

19
Question 6. A 16-year-old female with Question 8. A known drug addict is
anorexia is brought to hospital by her brought to hospital, having been found in a
mother 2 hours after attempting suicide by collapse. He has a respiratory rate of 3
taking 22 g of paracetamol. She was breaths per minute and pinpoint pupils,
drowsy when her mother found her, and and is completely unresponsive to painful
on arrival at hospital she is unresponsive stimuli. Which of the following drug
to pain. Observations on admission are as treatments should he receive?
follows: A) Flumazenil
B) Naloxone
C) Desferrioxamine
D) Adrenaline (epinephrine)
E) Glucagon
After assessment and appropriate
management of airway, breathing and Question 9. A 62-year-old lady is brought
circulation, which of the following is the to hospital by her son-in-law after being
most appropriate next step in her found at home behaving strangely. She is
management? on several medications for cardiac disease,
A) Give activated charcoal via a including warfarin, and has recently
nasogastric tube started a drug for depression. Observations
B) Give N-acetylcysteine intravenously on admission are as follows
C) Administer vitamin K intravenously to
prevent hepatic coagulopathy
D) Urgently measure blood paracetamol
levels
E) Communicate with the mother to be
certain of the timing of the overdose and
to identify possible co-ingestants

Question 7. A 21-year-old patient is


brought to the hospital Accident and
Emergency department by the police on a When you see her she is confused and
Sunday morning. He had been arrested at disorientated but appears to be able to hear
an all-night party but on the way to the normally. She complains of seeing a
police station, became very agitated and yellow hue over everything and vomits
appeared unwell. On arrival he complains three times. The cardiac monitor shows
of thirst and nausea, and on examination sinus bradycardia with brief self-
exhibits large pupils, hypertension and terminating runs of ventricular
global hyper-reflexia. He admits to illicit tachycardia. Her ECG shows downward-
drug use the night before. Which of the sloping ST segment depression. Toxicity
following substances is the most likely from which one of the following agents
cause of his symptoms? best explains her presentation?
A) Methanol A) Warfarin
B) Gammahydroxybutyrate (GHB) B) Verapamil
C) Ecstasy (MDMA) C) Quinine
D) Heroin D) Fluoxetine
E) D-lysergic acid diethylamide (LSD) E) Digoxin

20
Question 10. An unkempt 43-year-old Question 12. A patient with an overdose
man staggers into the hospital Accident has been successfully treated with
and Emergency department smelling desferrioxamine. What agent was
strongly of alcohol and complaining that ingested?
he cannot see. He vomits twice on arrival A) Iron
and is incontinent of urine. His blood B) Paracetamol
results are as follows: C) Aspirin
D) Lead
E) Cyanide

Question 13. A 23-year-old man is


brought to the hospital Accident and
Emergency department by the ambulance
service. He is drowsy, speaking
incoherently and has small pupils.
Observations on admission are as follows:

Toxicity from which of the following


drugs best explains his presentation?
A) Amitriptyline
B) Diamorphine
C) Temazepam
D) Cocaine
E) Venlafaxine

What is the diagnosis?


A) Diabetic ketoacidosis (DKA)
B) Ethanol poisoning
C) Salicylate poisoning
D) Methanol poisoning
E) Carbon monoxide (CO) poisoning

Question 11. Which one of the following


Question 14. Treatment of paracetamol
agents is used in the treatment of overdose
with ß-blockers? overdoses is usually decided by referring
to this nomogram:
A) N-acetylcysteine
A patient with which of the following
B) Calcium gluconate
conditions should be treated according to
C) Naloxone
the normal risk line?
D) Methionine
E) Glucagon A) Anorexia nervosa
B) Epilepsy treated with phenytoin
C) Chronic alcoholism
D) Chronic malnourishment
E) Co-ingestion of ecstasy (MDMA)

21
Question 15. A 45-year-old woman is Question 17. A pet snake is brought to the
brought to hospital at 2330 hours by her hospital Accident and Emergency
partner, who found her lying in bed with department by the friends of a patient who
three empty packets of paracetamol. She was bitten by it some 12 hours previously.
tells you that she attempted suicide by The patient was dead on arrival at the
taking some of the tablets that morning, hospital. A medical student examining the
but then took more tablets in the snake to identify it is accidentally bitten on
afternoon, and some more after watching the forearm. Which of the following is the
the ten o'clock news. Her partner has most important action to take before
brought the packets to hospital. She is intravenous administration of the specific
normally healthy and is on no regular antivenin?
medications. How would you decide A) Inject 0.02 ml of saline-diluted
whether to administer N-acetylcysteine? antivenin subcutaneously and observe the
A) Treat if the most recent dose was more injection site for 10 minutes
than 75 mg/kg B) Apply a tourniquet to the affected limb
B) Treat if a paracetamol level taken 4 C) Incise the bite and aspirate the poison
hours after the last dose is above the D) Check prothrombin time and fibrinogen
normal-risk treatment line on a standard levels
nomogram E) Check renal function and creatine
C) Treat if a paracetamol level taken 4 kinase (CK) levels
hours after the last dose is above the high-
risk treatment line on a standard
nomogram Question 18. Overdose with which one of
D) Treat if the total 24-hour dose was the following agents causes large pupils?
more than 150 mg/kg A) Morphine
E) Treat if the total 24-hour dose was B) Aspirin
more than 75 mg/kg C) Paracetamol
D) Amitriptyline
Question 16. Which one of the following E) Atenolol
drugs does not usually cause bradycardia
in overdose? Question 19. A blood level would be most
A) Atenolol useful in the management of poisoning
B) Digoxin with which of the following agents?
C) Lithium A) Aspirin
D) Verapamil B) Fluoxetine
E) Diltiazem C) Morphine
D) Quinine
E) Organophosphates

22
Chapter 9 MEDICAL PSYCHIATRY
Answers page 139

Question 1. Pressure of speech and Question 5. Which one of the following


grandiose ideas are most suggestive of statements regarding electroconvulsive
which of the following psychiatric therapy (ECT) is false?
conditions? A) Use has declined since the advent of
A) Mania antidepressant drugs
B) Schizophrenia B) It is of proven efficacy for severe
C) Generalised anxiety disorder depressive illness
D) Obsessive-compulsive disorder C) It is rarely used due to the high
E) Depression incidence of serious side-effects
D) It may cause both anterograde and
Question 2. Common somatic symptoms retrograde amnesia
of anxiety disorders include all of the E) It requires a general anaesthetic and
following except one. Which is the paralysis with muscle relaxants
exception?
A) Sweating Question 7. Which of the following
B) Palpitations psychiatric drugs may interact with foods
C) Constipation rich in tyramine, such as cheese and red
D) Dizziness wine, to produce a potentially fatal
E) Breathlessness hypertensive crisis?
A) Tricyclic antidepressants
Question 3. Which one of the following is B) Dopamine receptor antagonists
not a risk factor for suicide? C) Monoamine oxidase inhibitors
A) Male sex D) Selective serotonin re-uptake inhibitors
B) Unemployed E) Benzodiazepines
C) Alcohol misuse
D) Living alone Question 8. Which one of the following
E) Age under 45 statements regarding antidepressant drug
treatments is true?
Question 4. As the house officer on call A) They are more effective than
you are asked to review a patient who psychological treatments for mild to
appears to be hallucinating. The patient moderate depression
describes seeing large spiders crawling B) Selective serotonin re-uptake inhibitors
over the floor and the bed. Which of the (SSRIs) have greater anticholinergic side-
following features, elicited through a effects and are more dangerous in
careful mental state examination, would overdose than tricyclic antidepressants
point most strongly to an organic cause for C) The majority of patients experience an
these hallucinations? improvement in symptoms within one
A) Impaired cognition and drowsiness week of starting treatment
B) Pressure of speech D) Newer antidepressants have greater
C) Unshakeable belief that spiders are efficacy than older agents such as tricyclic
from another planet antidepressants
D) Associated auditory hallucinations E) Treatment should be continued for at
E) Flat, blunted affect least 6 months to reduce the high risk of
relapse

23
Question 9. Adverse effects associated D) Driving under the influence of alcohol
with lithium therapy include all of the E) Reinstatement of drinking behaviour
following except one. Which is the after abstinence
exception?
A) Tremor Question 13. A 55-year-old male with a
B) Teratogenicity long history of alcohol excess is admitted
C) Hypothyroidism to a medical ward, having been found by
D) Diabetes mellitus neighbours in a state of gross self-neglect.
E) Convulsions On admission he is noted to be confused
and very unsteady on his feet. On
Question 10. Which one of the following examination he is clearly malnourished,
is not a 'first-rank' symptom of disorientated in time, person and place and
schizophrenia? exhibits horizontal jerk nystagmus on
A) Thought insertion lateral gaze. He does not smell of alcohol
B) Poverty of speech and there are no features of chronic liver
C) Delusional perception disease. Which of the following is the
D) Passivity experiences likeliest diagnosis?
E) Auditory hallucinations in the third A) Korsakoff's syndrome
person B) Hepatic encephalopathy
C) Delirium tremens
Question 11. A 23-year-old male is D) Wernicke's encephalopathy
brought to Accident & Emergency by the E) Acute alcohol intoxication
police after exhibiting bizarre behaviour.
He is extremely agitated and claims that he Question 14. With reference to the patient
is being followed by various agencies, all from the previous question (Q13) who has
with apparently sinister intent. Which one Wernicke's encephalopathy, which of the
of the following would make the diagnosis following steps should be undertaken first
of schizophrenia least likely? in his management?
A) Incomprehensible speech A) Rehydration with IV dextrose
B) Claims by the patient that the above B) CT scan of the brain
agencies were stealing thoughts from his C) High-dose intravenous thiamin
head (Pabrinex)
C) No previous history of mental illness D) Oral benzodiazepines
D) Ingestion of amphetamines within the E) Full septic screen
last 24 hours
E) Lack of insight Question 15. All of the following except
one are diagnostic criteria for anorexia
Question 12. Which one of the following nervosa. Which is the exception?
is not a recognised criterion for alcohol A) Body mass index 17.5 or less
dependence? B) Avoidance of high-calorie foods
A) Feeling of compulsion to drink C) Self-induced vomiting or purgation
B) Priority of drinking over other activities D) Distortion of body image
C) Relief of withdrawal symptoms by E) Amenorrhea for at least 3 month
further drinking

24
Chapter 10 ONCOLOGY Answers page 142

Question 1. All of the following Question 5. A 63-year-old woman


statements except one are correct receiving chemotherapy for breast cancer
epidemiological associations of attends the hospital accident and
malignancy. Which is the exception? emergency department, complaining of
A) BRCA 1 gene and ovarian cancer headache, malaise and a dry cough.
B) Asbestos exposure and mesothelioma Routine observations show a temperature
C) Schistosomiasis and squamous bladder of 38.5°C, pulse of 104 bpm, BP of 128/83
cancer mmHg, respiratory rate of 20 and oxygen
D) Human papillomavirus and saturation of 97% on air. Examination is
oesophageal cancer otherwise unremarkable. Initial
E) Adenopolyposis coli (APC) gene and investigations reveal the
colorectal cancer following

Question 2. Which one of the following is


a risk factor for breast carcinoma?
A) Early menarche
B) Fibrocystic breast disease
C) Premature menopause
D) First full-term pregnancy at age 16
E) Multiple endocrine neoplasia (MEN)
type I

Question 3. Which one of the following is


not a recognised extrapulmonary
manifestation of bronchial carcinoma?
A) Syndrome of inappropriate antidiuretic
hormone secretion (SIADH)
B) Acanthosis nigricans
C) Finger clubbing What should the next step in her treatment
D) Eaton-Lambert syndrome be?
E) Ectopic adrenocorticotrophic hormone A) Perform a full septic screen and start
(ACTH) production antibiotics if cultures are positive
B) Take blood cultures and start broad-
Question 4. Which one of the following spectrum antibiotics immediately
tumour markers is not paired with the C) Insert a central line to allow
correct malignancy? measurement of central venous pressure
A) Carcinoembryonic antigen (CEA) and D) Administer granulocyte-colony
colonic carcinoma stimulating factor (G-CSF) to stimulate
B) Alpha-fetoprotein (AFP) and bladder white blood cell production
carcinoma E) Discharge her home to prevent her
C) Prostate-specific antigen (PSA) and contracting a hospital-acquired infection
prostatic carcinoma
D) CA-125 and ovarian carcinoma
E) Human chorionic gonadotrophin
(HCG) and choriocarcinoma

25
Question 6. A 73-year-old man with Question 9. A 55-year-old woman with
known prostate cancer is admitted to breast cancer is admitted to hospital
hospital with a 24-hour history of back complaining of headaches which are worst
pain and difficulty walking. Examination in the morning, together with nausea and
of the lower limbs reveals bilaterally vomiting. The result of a CT head scan is
increased tone with reduced power, brisk show:
reflexes and extensor plantars. What
would the most useful investigation be?
A) Plain X-ray of the spine
B) Spinal CT scan
C) Spinal MRI scan
D) Nerve conduction studies
E) Electromyography

Question 7. A 76-year-old man


undergoing treatment for lung cancer
attends hospital with polyuria, thirst and
mild confusion. Initial blood tests reveal a
corrected calcium level of 3.02 mmol/l
(reference range 2.12-2.62). What is the Which one of the following would be the
most important initial treatment? best initial treatment?
A) Intravenous saline A) Furosemide
B) Intravenous bisphosphonate B) Dexamethasone
C) Intravenous furosemide C) Radiotherapy
D) Calcium gluconate D) Chemotherapy
E) Calcitonin E) Surgical resection of the abnormal
lesions
Question 8. A 68-year-old man with
small-cell lung cancer complains of Question 10. Which of the following side-
worsening shortness of breath and a effects is least likely to result from
persistent headache. On examination, the radiotherapy?
jugular venous pressure is raised but non- A) Pulmonary fibrosis
pulsatile, prominent vessels are seen on B) Erythema and desquamation of the skin
the chest wall and there is facial oedema. C) Ischaemic heart disease
What is the most likely diagnosis? D) Peripheral neuropathy
A) Congestive heart failure E) Diarrhoea
B) Lymphangitis carcinomatosa
C) Nephrotic syndrome Question 11. Which one of the following
D) Superior vena caval obstruction statements about the drug tamoxifen is
E) Side-effects of radiotherapy true?
A) It commonly causes neutropenia
B) It should be offered to all patients with
breast cancer
C) It is an alkylating agent
D) Evidence supports its use in metastatic
breast cancer only
E) It exerts its effect by acting as an
oestrogen receptor antagonist

26
Chapter 11 PALLIATIVE CARE AND
PAIN MANAGEMENT
Answers page 144

Question 1. All of the following features B) Excessive salivation is common


except one suggest a neuropathic origin of C) Constipation should be managed with a
pain. Which is the exception? reduction in dose rather than laxative agents
A) Occurs in an area of sensory loss D) Respiratory depression is more likely to be
B) Is burning in nature a problem in acute dosing than with patients on
C) Occurs in response to non-painful stimuli regular opioids
D) Has a better response to non-steroidal anti- E) Sedation and drowsiness are uncommon in
inflammatory drugs (NSAIDs) than tricyclic patients under 65 years
antidepressants
E) Is associated with abnormal unpleasant Question 5. A patient with metastatic
sensations pancreatic carcinoma was admitted to hospital
from home with a 4-day history of worsening
Question 2. Which one of the following pain drowsiness and confusion. Her dose of 12-
treatments is least appropriately paired with hourly controlled-release (CR) morphine had
the type of pain? been doubled 1 week previously, as her pain
A) Bone pain Non-steroidal anti-inflammatory had been poorly controlled. On admission she
drugs (NSAIDs) has a Glasgow Coma score of 13 (E3 M6 V4),
B) Liver capsule pain Opioids with small pupils and periodic myoclonic
C) Raised intracranial pressure Corticosteroids jerks. After 24 hours free of opiates she is now
D) Neuropathic pain Anticonvulsants much more alert but starting to experience pain
E) Abdominal colic Antispasmodics again. Which of the following would be the
best strategy for managing her pain?
Question 3. A patient with widespread A) Restart her CR morphine at the same dose
metastatic prostatic carcinoma complains of and also prescribe 'as required' doses of
troublesome, constant pain at multiple sites. immediate-release (IR) morphine
He takes regular paracetamol, codeine and B) Restart her CR morphine at the same dose
ibuprofen at full dose but continues to but withhold 'as required' doses
experience severe pain. According to the WHO C) Reduce the CR morphine to the previous
analgesic ladder, what is the most appropriate dose (prior to adjustment) and also prescribe
next step in his pain management? 'as required' doses of IR morphine
A) Replace codeine with a strong opioid such D) Change from CR morphine to regular IR
as morphine morphine at a lower equivalent dose and also
B) Replace ibuprofen with a strong opioid such prescribe 'as required' doses of IR morphine
as morphine E) Change from CR morphine to a fentanyl
C) Replace ibuprofen with an alternative non- patch at a lower equivalent dose and also
steroidal anti-inflammatory drug (NSAID) prescribe 'as required' doses of oral IR
such as diclofenac morphine
D) Add dihydrocodeine to existing
medications Question 6. A patient with advanced bowel
E) Add a neuropathic agent to existing cancer presents with persistent large-volume
medications vomiting and severe colicky abdominal pain.
Abdominal imaging reveals multiple levels of
Question 4. Which one of the following gastrointestinal obstruction. Which one of the
statements regarding side-effects of strong following antiemetics is contraindicated?
opioids used in cancer pain is true? A) Cyclizine
A) Nausea and vomiting tend to worsen with B) Metoclopramide
increased duration of treatment C) Levomepromazine
D) Ondansetron
E) Prochlorperazin

27
Chapter 12 INFECTIOUS DISEASES
Answers page 146

Question 1. A gap-year student returns Question 4. A traveller, returning from


from the Far East with a temperature of East Africa to the UK with a fever of
39°C. You take a detailed history of 38.7°C, has a full blood count performed
exposures. Which one of the following as part of his initial work-up. This reveals
exposures is incorrectly paired with the an eosinophilia (eosinophil count 0.6 ×
resulting infection? 109). Which one of the following
A) Fresh-water swimming Schistosomiasis diagnoses is least consistent with this
B) Unprotected sexual contact Hepatitis B finding?
C) Unpasteurised milk Leptospirosis A) Schistosomiasis
D) Tick bite Typhus B) Strongyloidiasis
E) Mosquito bite Dengue fever C) Ascariasis
D) Tuberculosis
Question 2. All of the following except E) Filariasis
one are infective causes of splenomegaly.
Which is the exception? Question 5. Regarding measles, which
A) Brucellosis one of the following statements is correct?
B) Falciparum malaria A) The incubation period is 5 days to
C) Giardiasis onset of rash
D) Leishmaniasis B) Koplik's spots on the retina are
E) Subacute bacterial endocarditis pathognomonic
C) Antibiotics should be given to prevent
Question 3. A 24-year-old medical the development of bacterial pneumonia
student returns from her elective in D) Infants are protected for the first 6
Mauritius and attends the tropical diseases months of life by passive immunity from
unit with an intensely itchy rash on her maternal antibodies
foot, present since just before she came E) Children with the disease should be
home. She admits to having walked offered vaccination as soon as they
barefoot a lot while abroad. On recover
examination, there is a serpiginous linear
lesion on the sole of her foot. It is now 7-8 Question 6. Which of the following
cm in length, and she says that it has been pairings of childhood exanthems and their
elongating. What is the most likely complications is incorrect?
diagnosis? A) Rubella Myocarditis
A) Staphyloccus aureus cellulitis B) Measles Subacute sclerosing
B) Cutaneous larva migrans panencephalitis (SSPE)
C) Scabies C) Human erythrovirus 19 (parvovirus
D) Onchocerciasis B19) Anaemia
E) Strongyloidiasis D) Mumps Orchitis
E) Chickenpox Pneumonia

28
Question 7. A 76-year-old gentleman Question 10. All of the following
presents to his general practitioner with a statements about cytomegalovirus (CMV)
2-day history of burning discomfort are true except one. Which is the
around the left side of his chest. On exception?
examination, you find a vesicular rash in a A) In immunosuppressed patients,
band-like distribution around his left side: infection may cause sight-threatening
retinitis
B) In pregnant women, the infection may
have neurological sequelae for the fetus
C) In healthy young adults, infection is
usually asymptomatic
D) The peak incidence of acquisition is in
the sixth decade
E) The virus is shed in saliva, urine and
semen
Which of the following is the most likely Question 11. A 19-year-old male attends
causative organism? his general practitioner 3 days after
A) Treponema pallidum returning from back-packing in South-east
B) Herpes simplex virus Asia. He complains of headache,
C) Varicella zoster virus generalised aches and pains, painful eye
D) Staphylococcus aureus movements and a rash. He recalls a
E) Human erythrovirus 19 (parvovirus number of mosquito bites in the week
B19) prior to returning home but appears to
have taken appropriate antimalarial
Question 8. Which one of the following prophylaxis throughout the trip and since
statements about smallpox is true? his return. On examination, he has
A) It has been used as a bioterrorist agent lymphadenopathy, a temperature of
B) Infection carries a mortality rate of 38.2°C, and a maculopapular morbilliform
90% rash on his trunk. Which one of the
C) The vesiculo-pustular rash is most following diagnoses is most likely?
dense on the trunk, with sparing of the A) Salmonella
extremities B) Schistosomiasis
D) No successful vaccine has ever been C) Leishmaniasis
developed D) Dengue
E) It is caused by a DNA poxvirus E) Strongyloidiasis
Question 9. A 14-year-old boy, living in Question 12. Fifteen patients in a large
London, is brought to the hospital accident care of the elderly unit in the UK develop
and emergency department by his mother nausea and vomiting in the space of 3
with a 6-day history of malaise, fever and days. A number of nursing and medical
sore throat. He gives no history of recent staff also report vomiting. Which one of
foreign travel. His temperature is 38°C. the following agents is the most likely
On examination, you note that he has an cause of the outbreak?
inflamed pharynx, cervical A) Meticillin-resistant Staphylococcus
lymphadenopathy, a tender enlarged liver aureus (MRSA)
and a palpable spleen. What is the most B) Clostridium difficile
likely diagnosis? C) E. coli
A) Infectious mononucleosis D) Calicivirus
B) Malaria E) Norovirus
C) Influenza
D) Mumps
E) Listeriosis

29
Question 13. Infection with Question 16. All of the following except
Staphylococcus aureus is commonly one are recognised clinical features of
responsible for all of the following brucellosis. Which is the exception?
conditions except one. Which is the A) Pyoderma gangrenosum
exception? B) Uveitis
A) Endocarditis C) Meningitis
B) Pneumonia D) Endocarditis
C) Hepatitis E) Septic arthritis
D) Osteomyelitis
E) Cellulitis Question 17. A 24-year-old student
presents to the dermatology clinic with an
Question 14. A 16-year-old girl is brought expanding red macule on her left calf; this
to the hospital accident and emergency appeared 2 weeks previously, shortly after
department by her parents, having returning from holiday in Connecticut,
developed a high fever, headache and sore USA. She reports that the lesion has been
throat over the last 12 hours. She tells you progressively increasing in size since it
that she has just finished menstruating. On appeared. On examination, the lesion now
examination, she has a temperature of measures approximately 15 cm across and
38.6°C, blood pressure of 90/40 mmHg appears to be clearing from the centre
and a generalised erythematous blanching outwards. Which of the following is the
rash. There are no signs of meningism or likely diagnosis of her rash?
petechiae. Which of the following is the A) Necrobiosis lipoidica
most likely diagnosis? B) Erythema nodosum
A) Infectious mononucleosis C) Pyoderma gangrenosum
B) Staphylococcal toxic shock syndrome D) Cellulitis
C) Scarlet fever E) Erythema chronicum migrans
D) Rubella
E) Rheumatic fever Question 18. With reference to the patient
from the previous question (Q17), the
Question 15. Which of the following diagnosis of Lyme borreliosis is confirmed
conditions is least likely to occur as a by polymerase chain reaction (PCR)
consequence of streptococcal infection? testing of her urine. She is not pregnant
A) Scarlet fever and has no known allergies. Which of the
B) Endocarditis following is the most appropriate
C) Rheumatic fever treatment?
D) Glomerulonephritis A) 14-day course of doxycycline
E) Haemolytic uraemic syndrome B) 1-month course of clarithromycin
C) 1-month course of metronidazole
D) Observation only
E) 14-day course of penicillin

30
Question 19. A 45-year-old male who is a Question 21. Which of the following
keen water-skier is brought to the statements about staphylococcal food
infectious diseases unit with a 24-hour poisoning is false?
history of jaundice, fever and nosebleeds. A) Dairy products and cooked meat are an
There is no history of recent foreign travel. important source of infection
On examination, his temperature is 39°C, B) Refrigeration of food products
he is deeply jaundiced and has an enlarged encourages the growth of heat-stable
liver. His conjunctivae are hyperaemic and enterotoxins
there is a marked purpuric rash on the C) Symptoms arise within 1-6 hours of
trunk and limbs. Blood results are as ingestion of contaminated foodstuffs
follows: D) Vomiting is a more common symptom
than diarrhoea
E) A marked neutrophilia is common

Question 22. Which one of the following


statements is incorrect with regard to
Escherichia coli?
A) It is the most common cause of
travellers' diarrhoea
B) The incubation period of
enterotoxigenic E. coli (ETEC) is 1-2 days
C) Enteropathic E. coli (EPEC) is an
important cause of infant diarrhoea
D) Enterohaemorrhagic E. coli (EHEC)
may lead to haemolytic uraemic syndrome
E) Entero-invasive E. coli (EIEC) causes
symptoms through production of an
enterotoxin

Question 23. Which of the following


antibiotics is used as first-line therapy for
Which of the following is the most likely symptomatic infection with Clostridium
diagnosis? difficile?
A) Listeriosis A) Co-amoxiclav
B) Hepatitis A B) Metronidazole
C) Meningococcal sepsis C) Cefuroxime
D) Weil's disease (leptospirosis) D) Clarithromycin
E) Infectious mononucleosis (Epstein-Barr E) Clindamycin
virus)
Question 24. Which one of the following
Question 20. Which type of transmission gastrointestinal infections does not cause
does typhoid fever demonstrate? bloody diarrhoea?
A) Droplet spread A) Cholera
B) A bite from an infected animal vector B) Salmonella
C) Faecal-oral C) Shigella
D) Sexual D) Entamoeba histolytica
E) Blood-borne, i.e. needlestick injuries E) Campylobacter

31
Question 25. Which one of the following Question 27. A 45-year-old Indian man
statements is correct regarding the visiting his family in the UK is brought to
correction of fluid and electrolyte balance the infectious diseases unit by his son who
in the patient with acute diarrhoea? is concerned about the appearance of what
A) 48 hours of diarrhoea in the average he terms a 'rash' on his father's back. On
adult will result in a fluid deficit of half a examination, there are three well-defined
litre hypopigmented macules that are
B) Intravenous fluid replacement is completely anaesthetic. You note that both
preferable to oral rehydration for most the patient's median and ulnar nerves are
patients with acute diarrhoea palpably thickened bilaterally, with
C) Oral rehydration solution (ORS) variable sensory loss distal to the wrists in
contains carbohydrate, sodium, potassium both upper limbs. Which of the following
and chloride is the most likely underlying diagnosis?
D) Prolonged bouts of diarrhoea result in A) Toxoplasmosis
the development of a metabolic alkalosis B) Botulism
E) Infants with gastroenteritis should not C) Cutaneous leishmaniasis
be breastfed while they have diarrhoea D) Leprosy
E) Tuberculosis
Question 26. A sales executive, recently
returned to the UK from Ukraine, is Question 28. With regard to the patient in
referred to the local infectious diseases the previous question (Q27), which one of
unit by his general practitioner with a sore the following investigations will confirm
throat of unusual appearance. On arrival, the diagnosis of leprosy?
he is tachycardic with a heart rate of 110 A) Leprosy serology
bpm, and has a mild pyrexia of 37.7°C. B) Polymerase chain reaction (PCR) of
There is a slight blood-stained discharge at serum for Mycobacterium leprae DNA
the nasal orifices. His neck is visibly C) Microscopy of split skin smears
swollen. Examination of the mouth reveals D) Nasal swab
a greyish-green membrane overlying the E) Blood culture
tonsils, which does not come off with
gentle scraping with the tongue depressor. Question 29. Which of the following
What is the likely diagnosis? antimicrobial agents is commonly used in
A) Streptococcal throat infection the treatment of leprosy?
B) Diphtheria A) Clindamycin
C) Oral candidiasis B) Chloroquine
D) Mumps C) Chloramphenicol
E) Tonsillar carcinoma D) Rifampicin
E) Penicillin V

Question 30. Which of the following


organisms is responsible for cat scratch
disease?
A) Toxoplasma gondii
B) Chlamydia psittaci
C) Rickettsia rickettsii
D) Bartonella henselae
E) Coxiella burnetti

32
Question 31. A 19-year-old gap-year Question 34. Which of the following
student returns to the UK after working for statements regarding the treatment and
6 weeks in West Africa on a charity chemoprophylaxis of malaria is correct?
project. Shortly after returning, he presents A) Chloroquine resistance is rare
to his general practitioner with a 4-day B) Falciparum malaria is still largely
history of intermittent fevers, tiredness, sensitive to quinine
headache and general malaise. On C) Chemoprophylaxis with doxycycline
examination, he is mildly jaundiced. His should be continued for 7 days after
heart rate is 110 bpm and temperature is leaving a malarious area
38.2°C. He has a palpable spleen and a D) Quinine is associated with troublesome
tender liver edge but no rash or neuropsychiatric side-effects
lymphadenopathy. E) Fansidar is suitable for
chemoprophylaxis

Question 35. Which one of the following


statements regarding visceral
leishmaniasis is incorrect?
A) It is transmitted from human to human
by the sandfly
B) Both splenomegaly and hepatomegaly
are common clinical features
C) Dermatological manifestations include
black discoloration of the skin
D) Blood tests typically reveal
thrombocytosis and neutrophilia
E) The treatment of choice is pentavalent
antimony
Which one of the following is the most Question 36. A 19-year-old woman is
likely diagnosis? referred by her general practitioner to the
A) Leishmaniasis local dermatology department, 3 months
B) Falciparum malaria after returning from a trip to Central
C) HIV seroconversion illness America. She has an ulcer on the lateral
D) Q fever aspect of her calf, which has failed to heal
E) Infectious mononucleosis with a 2-week course of flucloxacillin. It
began 1 month previously as a raised red
Question 32. All of the following except 'lump', then increased in size to about 6
one are potential complications of cm, before ulcerating and developing a
falciparum malaria. Which is the crust. Swabs of the ulcer have grown skin
exception? commensals only, but a biopsy shows
A) Convulsions numerous parasites. What is the most
B) Acute renal failure likely diagnosis?
C) Nephrotic syndrome A) Toxoplasmosis
D) Hypoglycaemia B) Cutaneous leishmaniasis
E) Metabolic acidosis C) Babesiosis
D) Plasmodium vivax malaria
Question 33. Which one of the following E) Trypanosomiasis
investigations is diagnostic for malaria?
A) Thin blood film
B) Full blood count
C) Blood culture
D) Serology for P. falciparum
E) Coagulation screen

33
Question 37. Infection with which one of E) The infection is responsive to penicillin
the following organisms is most antibiotics
commonly associated with cardiac
complications, including dilated Question 41. Which one of the following
cardiomyopathy and conduction defects? features is least characteristic of
A) Toxoplasma gondii strongyloidiasis?
B) Trypanosoma cruzi A) Abdominal pain
C) Leishmania donovani B) Urticaria
D) Ancylostoma braziliense C) Diarrhoea
E) Cytomegalovirus D) Motile larvae on stool microscopy
E) Neutrophilia
Question 38. Which of the following is a
vector for African trypanosomiasis Question 42. A 53-year-old man, recently
(African sleeping sickness)? returned to the UK from a holiday in
A) African sandfly Indonesia, presents with a tender,
B) Tsetse fly erythematous right leg and thigh. On
C) Mosquito examination, his right leg is swollen,
D) Ixodes tick warm, painful and red along the length of
E) Freshwater snail its medial aspect. There is marked
oedema. He also has an acutely tender
Question 39. With regard to right testicle and swelling of the right
toxoplasmosis, which one of the following inguinal lymph nodes. A full blood count
statements is incorrect? reveals eosinophilia. Which one of the
A) Domestic cats are important hosts in following is the most likely diagnosis?
the life cycle of Toxoplasma gondii A) Acute streptococcal cellulitis
B) Infection is asymptomatic in the B) Filiariasis
majority of individuals C) Cutaneous larva migrans infection
C) Congenital toxoplasmosis is D) Tinea pedis
characterised by retinochoroiditis, E) Histoplasmosis
microcephaly and hydrocephalus
D) Serological tests are available for the Question 43. Which of the five senses is
detection of past and present infection most commonly affected by infection with
with Toxoplasma gondii Onchocerca volvulus (onchocerciasis)?
E) The most common presenting features A) Vision
of acquired toxoplasmosis are fever, B) Taste
photophobia and a non-specific rash C) Hearing
D) Touch
Question 40. Which one of the following E) Smell
statements regarding amoebiasis is
correct? Question 44. The following are all
A) The parasite typically invades the small recognised sequelae of chronic infection
bowel with schistosomiasis except one. Which is
B) The incubation period of the the exception?
Entamoeba histolytica cyst is 48 hours A) Hydronephrosis
C) Clinically, the disease manifests as B) Oesophageal varices
diarrhoea alternating with constipation, C) Splenomegaly
with the presence of blood and mucus D) Urticaria
D) Diagnosis is made on stool culture E) Recurrent urinary tract infection

34
Chapter 13 HIV INFECTION AND
THE AIDS
Answers page 153

Question 1. Which of the following best Question 5. Which one of the following is
describes the human immunodeficiency not an AIDS-defining disease?
virus (HIV)? A) Pulmonary tuberculosis
A) Single-stranded RNA virus B) Kaposi's sarcoma
B) Double-stranded RNA virus C) Cytomegalovirus retinitis
C) Single-stranded DNA virus D) Oropharyngeal candidiasis
D) Double-stranded DNA virus E) Cerebral toxoplasmosis
E) Prion
Question 6. Which one of the following
Question 2. What is the main mode of statements regarding Cryptosporidium
transmission of HIV in northern Europe infection is incorrect?
and the United States? A) Patients typically present with profuse,
A) Blood transfusion bloody diarrhea
B) Heterosexual transmission B) It tends to occur when the CD4 count is
C) Homosexual transmission < 200 cells/mm3
D) Intravenous drug use C) Definitive diagnosis is made on stool
E) Vertical (mother to child) transmission microscopy
D) Complications include cholecystitis,
Question 3. Which of the following cholangitis and pneumonitis
exposures has the highest transmission risk E) Cryptosporidium is a protozoal enteric
for HIV? pathogen
A) Genital mucous membrane contact
B) Intravenous drug use using a Question 7. Which one of the following
contaminated needle pathogens is least likely to be responsible
C) Being born to a mother who is HIV- for chronic diarrhoea in the HIV-positive
positive but not on highly active patient?
antiretroviral therapy (HAART) A) Isospora
D) Homosexual intercourse between two B) Mycobacterium avium intracellulare
men C) Microsporidium
E) Receipt of infected blood products D) Cytomegalovirus
E) Toxoplasma
Question 4. Which one of the following
options best characterises the virological Question 8. Which one of the following
and immunological progression of HIV options would not be supportive of a
infection in the first 4-8 weeks following diagnosis of Pneumocystis carinii
exposure? (jirovecii) pneumonia?
A) CD4 count rises, viral load falls A) 2-3-week history of shortness of breath
B) CD4 count falls, viral load rises B) A cough productive of green sputum
C) Both CD4 count and viral load rise C) Marked oxygen desaturation on
D) Both CD4 count and viral load fall exercise
E) CD4 count remains stable, viral load D) A lack of response to penicillin
rises antibiotics
E) Pyrexia

35
Question 9. Which one of the following Question 12. Which one of the following
statement regarding cryptococcal is not a characteristic side-effect of
meningitis is true? treatment with nucleoside reverse
A) The organism is identified on transcriptase inhibitors (NRTIs)?
cerebrospinal fluid (CSF) microscopy with A) Cranial nerve palsies
Ziehl-Neelsen staining B) Peripheral neuropathy
B) The majority of affected patients have a C) Pancreatitis
CD4 count > 200 cells/mm3 D) Anaemia
C) Deafness and blindness are the most E) Lipoatrophy
common complications
D) Neck stiffness is invariably present Question 13. Which one of the following
E) Penicillin is the treatment of choice statements about protease inhibitors (PIs)
is incorrect?
Question 10. A 27-year-old HIV-positive A) They act by preventing post-
patient with a CD4 count of 20 cells/mm3 translational cleavage of polypeptides into
presents to the hospital accident and functional viral proteins
emergency department with visual B) They cause glucose intolerance and
difficulties. He reports a 6-week history of frank diabetes in a proportion of patients
flashing lights and dark 'clouds' floating in C) Metabolism is via the P450 system of
front of his eyes. In the past fortnight he enzymes
has experienced problems with driving, D) They improve the lipid profile of most
narrowly escaping a collision with a patients by lowering total cholesterol and
cyclist whom he did not see on the road low-density lipoprotein (LDL), and by
beside him. On examination, the raising high-density lipoprotein (HDL)
conjunctivae are injected and fundoscopy E) Monitoring of plasma levels helps to
reveals haemorrhagic exudates near the optimise antiviral effect and reduce
retinal vessels. What is the likeliest toxicity
diagnosis?
A) Bacterial conjunctivitis Question 14. All of the following except
B) Herpes simplex keratitis one are advantages of non-nucleoside
C) Anterior uveitis reverse transcriptase inhibitors (NNRTIs)
D) Cytomegalovirus (CMV) retinitis over other anti-HIV agents. Which is the
E) Acute retinal necrosis exception?
A) Once-daily dosing
Question 11. A 37-year-old homosexual B) Good oral bioavailability
male who is HIV-positive (CD4 count 300 C) Lower tablet numbers than with NRTIs
cells/mm3) is found on a routine visit to D) No association with fat redistribution
the infectious diseases clinic to have E) Low incidence of drug rashes
purplish-blue papules across the bridge of
his nose; these have appeared in the last 4 Question 15. Which one of the following
months. He is otherwise asymptomatic. On opportunistic infections is incorrectly
examination, he has cervical matched with its appropriate prophylactic
lymphadenopathy and agent?
hepatosplenomegaly. Which one of the A) Tuberculosis Rifampicin and isoniazid
following diagnoses is most likely? B) Toxoplasmosis Erythromycin
A) Seborrhoeic dermatitis C) Pneumocystis carinii (jirovecii)
B) Disseminated cutaneous herpes simplex pneumonia Co-trimoxazole
C) Molluscum contagiosum D) Cryptococcus Itraconazole
D) HIV-associated lymphoma E) Mycobacterium avium intracellulare
E) Kaposi's sarcoma (MAI) Azithromycin

36
Chapter 14 SEXUALLY TRANSMITTED
INFECTIONS
Answers page 156

Question 1. Worldwide, which of the Question 4. Which of the following


following is the most common curable genotypes of human papillomavirus
sexually transmitted infection (STI)? (HPV) is associated with dysplastic
A) Syphilis conditions and cancers of the genital tract?
B) Gonorrhoea A) HPV 6 and 11
C) Chlamydia B) HPV 16 and 18
D) Trichomonas vaginalis C) HPV 10 and 15
E) HIV D) HPV 6 only
E) HPV 10 and 13
Question 2. A 22-year-old heterosexual
male presents to the STI clinic with a 2- Question 5. A 34-year-old Englishwoman
week history of purulent urethral discharge is referred to the local gynaecology
and dysuria. He has been sexually active outpatient department with a 2-year
in the past month with two new partners. history of intermittent pelvic pain. In her
Gram-stained smear of urethral exudate history, she also describes dyspareunia and
shows Gram-negative diplococci, along occasional post-coital bleeding. She and
with prominent polymorphonuclear cells. her husband have been trying to conceive
What is the most likely cause for the without success for the past 18 months.
gentleman's urethral discharge? Examination of the external genitalia is
A) Non-gonococcal urethritis unremarkable. On per vaginam
B) Syphilis examination, you see an inflamed cervix,
C) Genital warts which is partly covered by a mucopurulent
D) Gonorrhoea discharge. Which of the following
E) Staphylococcal infection diagnoses is most likely?
A) Syphilis
Question 3. A 23-year-old girl presents to B) Herpes simplex type 1 infection
her GP, with a 3-day history of a painful C) Herpes simplex type 2 infection
rash over her perineum. This was preceded D) Trichomoniasis
by 2 days of 'flu-like symptoms'. She has E) Chlamydial infection
been sexually active with a new partner for
the last 6 weeks and takes the oral
contraceptive pill, but does not use barrier
contraception. On examination, she has a
cluster of small vesicles on both labia
majora. Which is the most appropriate
next investigation?
A) High vaginal swab
B) Urethral swab
C) HIV test
D) Swab of the contents of a vesicle
E) Urine microscopy and cultu

37
Question 6. A 35-year-old lady is referred E) Syphilitic chancre
to Gynaecology triage with a 7-day history
of a profuse, foul-smelling vaginal Question 8. Which one of the following is
discharge. She is very distressed by this, as not a feature of congenital syphilis?
the quantities produced are necessitating A) Hutchinson's incisors
the use of sanitary towels. On speculum B) Saddle nose
examination, the external genitalia are C) Sabre tibia
entirely normal but the vagina itself is D) Webbed neck
slightly inflamed. High and low vaginal E) Mulberry molars
swabs are taken, as well as a wet film of
some of the discharge, which is sent Question 9. A 17-year-old female
immediately for microscopy. Motile reluctantly attends her GP to discuss the
flagellate organisms are seen in large problem of a 2-month history of a whitish
numbers. What is the diagnosis? vaginal discharge and vulval pruritus. She
A) Trichomoniasis is not sexually active and her only past
B) Bacterial vaginosis medical history is of acne, for which she
C) Herpes simplex type 2 infection takes regular minocycline. On
D) Chlamydial infection examination, her external genitalia are
E) Gonorrhoea inflamed and traces of a curd-like
discharge can be seen at the vaginal
Question 7. Which one of the following is orifice. What is the most likely diagnosis?
not a feature of secondary syphilis? A) Human papillomavirus infection
A) Condylomata lata B) Vaginal candidiasis
B) Meningitis C) Chlamydial infection
C) Generalised lymphadenopathy D) Toxic shock syndrome (TSS)
D) A maculo-papular rash on the trunk and E) Gonorrhoea
limbs

38
Chapter 15 CLINICAL BIOCHEMISTRY
AND METABOLISM
Answers page 158

Question 1. A typical adult male has a Question 5. What proportion of filtered


total body water of 40 litres. How much of sodium is normally reabsorbed within the
this is located within the extracellular fluid proximal tubule?
compartment? A) 90%
A) 25 litres B) 65%
B) 15 litres C) 40%
C) 9 litres D) 25%
D) 6 litres E) 5%
E) 3 litres
Question 6. Which one of the following
Question 2. The most abundant cation clinical signs would be an unexpected
within the intracellular fluid compartment finding in the presence of hypovolaemia?
is: A) Low jugular venous pulse
A) Sodium B) Reduced skin turgor
B) Chloride C) Bradycardia
C) Potassium D) Postural hypotension
D) Hydrogen E) Reduced urine output
E) Calcium
Question 7. The normal physiological
Question 3. A blood sample is analysed response to loss of sodium and water
after significant in vitro haemolysis has comprises all of the following except one.
occurred. Which of the following spurious Which is the exception?
results is most likely to occur in the A) Enhanced conversion of
measured electrolytes? angiotensinogen to angiotensin I
A) High sodium B) Reduced glomerular filtration rate
B) High potassium C) Activation of the sympathetic nervous
C) Low sodium system
D) Low potassium D) Increased release of atrial natriuretic
E) High chloride peptide
E) Stimulation of thirst
Question 4. The total body content of
which ion is a principal determinant of Question 8. Which one of the following
ECF volume? clinical conditions would be least likely to
A) Sodium cause sodium and water excess?
B) Potassium A) Chronic renal failure
C) Calcium B) Nephrotic syndrome
D) Chloride C) Conn's syndrome
E) Bicarbonate D) Congestive cardiac failure
E) Addison's disease

39
Question 9. The diuretic furosemide acts C) Increased oral salt intake
principally at which of the following sites D) Fluid restriction of 1-1.5 litres per day
within the kidney? E) Demeclocycline therapy
A) Glomerulus
B) Proximal tubule Question 14. Which of the following
C) Loop of Henle drugs is most likely to cause SIADH?
D) Early distal tubule A) Metformin
E) Collecting ducts B) Carbamazepine
C) Digoxin
Question 10. Which one of the following D) Amiodarone
metabolic abnormalities is most likely to E) Alendronate
be caused by treatment with a thiazide
diuretic? Question 15. Which one of the following
A) Hypocalcaemia ECG changes is most typical of
B) Hyponatraemia hyperkalaemia?
C) Hyperkalaemia A) Peaked P wave
D) Hypermagnesaemia B) Presence of U wave
E) Hypoglycaemia C) ST segment depression
D) Peaked T wave
Question 11. Which one of the following E) Narrowed QRS complex
conditions is a cause of hypovolaemic
hyponatraemia? Question 16. Which one of the following
A) Primary polydipsia would be most likely to produce
B) Hyperlipidaemia hypokalaemia (plasma K < 3.5 mmol/l)?
C) Nephrotic syndrome A) Addison's disease
D) Syndrome of inappropriate ADH B) Amiloride therapy
secretion (SIADH) C) Conn's syndrome
E) Diuretic therapy D) Rhabdomyolysis
E) Acute renal failure
Question 12. During an employment
medical, a 59-year-old male is found to Question 17. Treatment with which one of
have a plasma sodium of 126 mmol/l. He the following drugs is most likely to lead
is asymptomatic and clinical examination to hyperkalaemia?
reveals no abnormal findings other than A) Bendroflumethiazide
mild obesity. Further blood tests reveal a B) Warfarin
plasma osmolality of 250 mmol/kg C) Ramipril
(normal range 280-290 mmol/kg) and a D) Salbutamol
urine osmolality of 360 mmol/kg. Which E) Calcium resonium
of the following is the likeliest diagnosis?
A) Diuretic abuse Question 18. Which one of the following
B) Primary polydipsia treatments for hyperkalaemia is not used
C) Nephrotic syndrome to lower the concentration of plasma
D) Syndrome of inappropriate ADH potassium?
secretion (SIADH) A) Insulin
E) Cranial diabetes insipidus B) Salbutamol
C) Calcium resonium
Question 13. Aside from identifying and D) Dialysis
treating possible underlying causes, what E) Calcium gluconate
is the most appropriate initial management
for the patient in the previous question
who has SIADH?
A) Infusion of 0.9% saline (normal saline)
B) Infusion of hypertonic saline

40
Question 19. Which one of the following C) Low-density lipoprotein
conditions would be most likely to cause a D) Intermediate-density lipoprotein
normal anion gap metabolic acidosis? E) High-density lipoprotein
A) Sepsis
B) Salicylate poisoning Question 23. Which one of the following
C) Starvation clinical features is more characteristic of
D) Diabetic ketoacidosis hypertriglyceridaemia than
E) Renal tubular acidosis hypercholesterolaemia?
A) Xanthelasma
Question 20. The following laboratory B) Corneal arcus
results are obtained from a patient on a C) Eruptive xanthomas
general surgical ward: D) Achilles tendon xanthomas
E) Premature cardiovascular disease

Question 24. Regarding therapy with


HMGCoA reductase inhibitors (statins),
which one of the following statements is
true?
A) The incidence of rhabdomyolysis is
Urine pH: 5.5 around 10%
What is the acid-base abnormality? B) In patients with hypercholesterolaemia
A) Raised anion gap metabolic acidosis and risk factors for atherosclerosis, they
B) Normal anion gap metabolic acidosis reduce the risk of coronary events but not
C) Metabolic alkalosis stroke
D) Respiratory acidosis C) They inhibit endogenous cholesterol
E) Respiratory alkalosis synthesis
D) They lead to a modest reduction in
Question 21. Which one of the following high-density lipoprotein (HDL) cholesterol
disorders is most likely to be responsible E) They have a greater effect on
for the results seen in the previous triglyceride levels than low-density
question? (shown again below) lipoprotein (LDL) cholesterol levels
Urine pH: 5.5
A) Gastric outlet obstruction Question 25. Which one of the following
B) Acute renal failure clinical features would be an unexpected
C) Traveller's diarrhea finding in an attack of acute intermittent
D) Hypothyroidism porphyria?
E) Salicylate ingestion A) Acute abdominal pain
B) Bullae on sun-exposed areas of skin
Question 22. Which of the following C) Behavioural disturbance
lipoproteins is primarily responsible for D) Hypertension and tachycardia
reverse cholesterol transport? E) Constipation
A) Chylomicrons
B) Very low-density lipoprotein

41
Chapter 16 KIDNEY AND URINARY
TRACT DISEASE
Answers page 163

Question 1. A renal bruit is associated Question 5. Serum creatinine is often used


with which of the following disorders? as a surrogate measure of glomerular
A) Conn's syndrome filtration rate (GFR). Which of the
B) Hypertensive nephropathy following best describes the relationship
C) Wegener's granulomatosis between the change in serum creatinine
D) Polyarteritis nodosa concentration and GFR?
E) Renal artery stenosis A) Negative exponential
B) Direct proportion (positive linear)
Question 2. The following are all causes C) Inverse proportion (negative linear)
of a palpably enlarged kidney except one. D) Positive exponential
Which is the exception? E) 'Build-up' exponential
A) Polycystic kidney disease
B) Renal cell carcinoma Question 6. Which of the following is
C) Diabetic nephropathy freely filtered in the normal glomerulus?
D) Hydronephrosis A) Albumin
E) Renal amyloidosis B) Glucose
C) Red blood cells
Question 3. In a 70 kg man who has a D) Platelets
normal diet, what is the minimum volume E) Fibrinogen
of urine production required each day to
excrete the daily solute load? Question 7. The presence of red cell casts
A) 15 ml on urine microscopy is most likely to be
B) 50 ml due to which one of the following?
C) 100 ml A) Urethral trauma
D) 500 ml B) Urinary tract infection
E) 1200 ml C) Bladder carcinoma
D) Ureteric stone
Question 4. All of the following except E) Post-streptococcal glomerulonephritis
one would be expected to cause an
increase in serum urea concentration. Question 8. When serum creatinine is
Which is the exception? used as a guide to renal excretory function,
A) Severe burns which one of the following might lead to
B) Reduction in glomerular filtration rate under-estimation of the degree of renal
(GFR) impairment?
C) Malnutrition A) Upper gastrointestinal haemorrhage
D) Dehydration B) Reduced muscle bulk
E) Upper gastrointestinal haemorrhage C) Excessive dietary protein
D) Liver failure
E) Catabolic state

42
Question 9. Intravenous urography (IVU) microscopy reveals no red cells. What is
is superior to renal ultrasound for which the most likely diagnosis?
one of the following purposes? A) Renal calculus
A) Detection of urothelial malignancy B) Urinary sepsis
B) Assessing size and symmetry of C) Rapidly progressive glomerulonephritis
kidneys D) Renal infarction
C) Distinguishing between solid and cystic E) Rhabdomyolysis
masses
D) Identifying hydronephrosis Question 14. A 68-year-old man reports
E) Assessing renal blood flow visible blood in his urine for the past 8
weeks. He is otherwise asymptomatic and
Question 10. Which of the following has no history of renal disease. Urine
investigations provides most information culture has been negative on two
on the relative contribution of each kidney occasions and serum creatinine is within
to overall renal function? the normal range. Repeat urinalysis shows:
A) Renal biopsy Blood +++, Protein nil. Which of the
B) Renal ultrasound following pairs of investigations would
C) Magnetic resonance imaging (MRI) of now be most appropriate for this patient?
kidneys A) Renal biopsy and intravenous
D) Dimercaptosuccinic acid (DMSA) urography (IVU)
isotope renogram B) Cytological analysis of urine and
E) Renal arteriogram microscopy for acid-fast bacilli
C) Cystoscopy and renal ultrasound
Question 11. Which one of the following D) Prostatic biopsy and prostate-specific
is a contraindication to renal biopsy? antigen (PSA) level
A) Haematuria E) Anti-glomerular basement membrane
B) Kidneys < 60% normal size (GBM) and c-ANCA (antineutrophil
C) Anaemia cytoplasmic antibodies)
D) Rapid deterioration in renal function
E) Angiotensin-converting enzyme (ACE) Question 15. Which of the following
inhibitor therapy conditions would not account for the
finding of proteinuria on routine testing
Question 12. Which of the following with standard dipsticks?
organisms is the most common cause of A) Urinary tract trauma
community-acquired lower urinary tract B) Urinary tract infection
infection (UTI)? C) Transitional cell bladder carcinoma
A) Escherichia coli D) Minimal change nephropathy
B) Staphylococcus aureus E) Microalbuminuria
C) Pseudomonas aeruginosa
D) Klebsiella spp. Question 16. A 55-year-old man with
E) Proteus mirabilis persistent proteinuria on dipstick analysis
has 3.6 g of protein in a 24-hour urine
Question 13. A 73-year-old man collapses collection. Which one of the following
at home following a stroke. He is found 24 disorders would account for this?
hours later, still lying on the floor. On A) Persistent urinary tract infection
admission to hospital he appears only B) Acute tubular necrosis
mildly dehydrated but is in acute renal C) Transitional cell carcinoma
failure. He is treated with IV fluids but D) Chronic interstitial nephritis
remains oliguric and his renal function E) Focal segmental glomerulosclerosis
continues to deteriorate. Dipstick
urinalysis is strongly positive for blood but

43
Question 17. Which of the following Question 21. A patient presents for the
abnormalities is most commonly first time with renal failure and no
associated with the nephrotic syndrome? previous blood results are available.
A) Impaired clotting Which of the following features would
B) Lymphatic obstruction most strongly suggest chronic impairment
C) Excessive urinary sodium loss of renal function?
D) Centripetal obesity A) Presence of haematuria and proteinuria
E) Hypercholesterolaemia B) Serum potassium > 6.5 mmol/l
C) Urine output < 500 ml/day
Question 18. Which of the following is D) Bilateral small kidneys on renal
least likely to cause polyuria? ultrasound
A) Polydipsia E) Clinical evidence of dehydration
B) Cranial diabetes insipidus
C) Lithium therapy Question 22. Which of the following
D) Hypocalcaemia drugs is most likely to increase the risk of
E) Diabetes mellitus renal failure in a patient receiving
intravenous X-ray contrast for a
Question 19. Anticholinergics are most radiological procedure?
useful in the treatment of which of the A) Heparin
following patterns of incontinence? B) Ibuprofen
A) Urge incontinence C) Insulin
B) Stress incontinence D) Diazepam
C) Overflow incontinence E) N-acetylcysteine
D) Continual incontinence
E) Neurogenic incontinence Question 23. A patient is diagnosed with
pre-renal acute renal failure. She is taking
Question 20. A patient complains of the drugs listed below. Which one is least
erectile dysfunction. He is taking the likely to worsen her condition?
following regular medications. Which one A) Furosemide
is most likely to be responsible? B) Lisinopril
A) Valsartan C) Diclofenac
B) Ibuprofen D) Amoxicillin
C) Sildenafil E) Bendroflumethiazide
D) Fluoxetine
E) Atenolol Question 24. What is the main
physiological function of the medullary
concentration gradient?
A) Promotes water reabsorption from
urine
B) Secretes water into urine
C) Increases glomerular filtration rate
(GFR)
D) Promotes sodium reabsorption from
urine
E) Controls renal vasomotor tone

44
Question 25. A previously well patient is Question 27. Which of the following
admitted to the renal ward with severe biochemical abnormalities in venous blood
acute renal failure and the following blood is least likely to be present in a patient
results: with untreated chronic renal failure?
A) Reduced bicarbonate
B) Elevated parathyroid hormone (PTH)
C) Reduced phosphate
D) Elevated cholesterol
E) Reduced calcium

Question 28. Which of the following


statements about the management of
anaemia in patients with chronic renal
She is treated with 20 units of intravenous failure is most accurate?
Actrapid insulin and 50 ml of 50% A) Treatment with human recombinant
dextrose, as well as salbutamol nebulisers. erythropoietin is only beneficial in patients
Her blood tests are repeated 2 hours later with a subnormal plasma erythropoietin
and show: level
????NOT FOUND IN THE WEB SITE???????
B) Target haemoglobin should be 130-140
Which of the following is the most
g/l to achieve maximal benefit
appropriate treatment at this stage to
C) Transferrin saturation should be
reduce her serum potassium
maintained below 20% to avoid iron
concentration?
overload
A) A further 20 units of soluble short-
D) Patients not receiving dialysis should
acting insulin (Actrapid) with 50 ml of
receive oral rather than parenteral
50% dextrose
erythropoietin
B) IV calcium gluconate
E) Complications of erythropoietin include
C) IV sodium bicarbonate 1.26%
hypertension and hypercoagulability
D) Calcium resonium
E) Haemodialysis
Question 29. In patients with hypertension
and proteinuria, which of the following
Question 26. A 62-year-old lady presents
classes of drug has been shown to be most
in acute renal failure with severe bilateral
effective in retarding the progression of
loin pain. A renal tract ultrasound shows
chronic renal failure?
dilatation of both ureters and dilatation of
A) Beta-blockers
the collecting system within both kidneys,
B) Angiotensin-converting enzyme (ACE)
probably due to retroperitoneal fibrosis.
inhibitors
Blood results are shown below:
C) Calcium channel blockers
D) Nitrates
E) Thiazide diuretics

Question 30. Which of the following


factors does not contribute to the increased
risk of cardiovascular mortality in patients
with chronic renal failure?
A) Hypercholesterolaemia
B) Vascular calcification
Which of the following is the most
C) Hypertension
appropriate treatment?
D) Enhanced platelet function
A) Haemodialysis
E) Anaemia
B) Continuous veno-venous
haemofiltration (CVVH)
C) Bilateral nephrostomies
D) 1.5 litre/day fluid restriction
E) Suprapubic bladder catheterisation

45
Question 31. Deficiency of which one of Question 36. A 26-year-old man presents
the following contributes to renal with the nephrotic syndrome. Which of the
osteodystrophy? following conditions is most likely to be
A) 1V-hydroxylase the underlying cause?
B) Phosphate A) Acute tubular necrosis
C) Aluminium B) Membranous nephropathy
D) 25-hydroxycholecalciferol C) Post-streptococcal glomerulonephritis
E) Parathyroid hormone D) Anti-glomerular basement membrane
(GBM) disease
Question 32. Which of the following E) Small-vessel vasculitis
adverse features of chronic renal failure is
least effectively treated by haemodialysis? Question 37. Which of the following
A) Accumulation of uraemic toxins causes of glomerular disease is most likely
B) Abnormal electrolyte balance to present as rapidly progressive
C) Fluid overload glomerulonephritis?
D) Anaemia A) Membranous nephropathy
E) Acidosis B) Focal segmental glomerulosclerosis
C) Goodpasture's disease (anti-glomerular
Question 33. Which of the following basement membrane (anti-GBM) disease)
statements about the differences between D) Diabetic nephropathy
peritoneal dialysis (PD) and haemodialysis E) Mesangiocapillary glomerulonephritis
(HD) is true?
A) PD requires a more restrictive diet than Question 38. Which of the following
HD glomerular diseases is most likely to
B) PD enables a more efficient transfer of respond to treatment with corticosteroids?
solutes than HD A) Minimal change nephropathy
C) PD may be performed less frequently B) IgA nephropathy
than HD C) Primary focal segmental
D) Symptomatic fluid shift is less common glomerulosclerosis
with PD than HD D) Diabetic nephropathy
E) Previous abdominal surgery is more E) Post-streptococcal glomerulonephritis
likely to impede HD than PD
Question 39. In addition to renal failure in
Question 34. Which one of the following early adulthood, which of the following is
is an absolute contraindication to renal associated with Alport's syndrome?
transplantation? A) Pulmonary haemorrhage
A) Age > 55 years B) Gastrointestinal telangiectasia
B) Active anti-glomerular basement C) Osteopenia
membrane (GBM) disease D) Intellectual impairment
C) Diabetes mellitus E) Sensorineural deafness
D) Past history of malignant melanoma
excised 4 years ago Question 40. Which of the following is
E) Previous failed renal transplant not associated with adult polycystic kidney
disease (APKD)?
Question 35. Deterioration of renal A) Hepatomegaly
function on commencement of B) Hypertension
angiotensin-converting enzyme (ACE) C) Subarachnoid haemorrhage
inhibitor therapy is most characteristic of: D) Urinary tract infection
A) Diabetic nephropathy E) Renal cell carcinoma
B) Hypertensive nephropathy
C) Adult polycystic kidney disease
D) Renal artery stenosis
E) Goodpasture's syndrome

46
Question 41. Which of the following is Question 43. Which of the following
least likely to precipitate acute urinary hormones is least likely to be secreted by a
retention in a patient with benign prostatic renal adenocarcinoma?
hyperplasia (BPH)? A) Erythropoietin
A) Alcohol excess B) Adrenaline (epinephrine)
B) Alpha-adrenoceptor blockers C) Parathyroid hormone-related peptide
C) Constipation D) Human chorionic gonadotrophin
D) Prostatic infection E) Renin
E) Anticholinergic agents

Question 42. All of the following except


one are characteristic findings in a patient
presenting with metastatic prostate cancer. Question 44. Which one of the following
Which is the exception? drugs is associated with the development
A) Lytic bony metastases of glomerulonephritis?
B) Elevated alkaline phosphatase A) Ciclosporin
C) Positive radioisotope bone scan B) Gentamicin
D) Prostate specific antigen (PSA) > 100 C) Angiotensin-converting enzyme (ACE)
ng/ml inhibitors
E) Enlarged, irregular prostate on rectal D) Penicillamine
examination E) Lithium

47
Chapter 17 CARDIOVASCULAR
DISEASE
Answers page 170

Question 1. Which one of the following Question 4. Which one of the following
statements about the jugular venous pulse statements regarding the nerve supply of
(JVP) is true? the heart is correct?
A) The height reflects left atrial pressure A) Beta1-adrenoceptors mediate negatively
B) The JVP falls with abdominal chronotropic effects
compression B) Beta2-adrenoceptors mediate positively
C) It is best examined with the patient inotropic effects
supine C) Adrenergic fibres reach the heart via
D) Large systolic waves are associated the vagus nerves
with tricuspid regurgitation D) Cholinergic nerve fibres act on the
E) Prominent a waves suggest atrial sinoatrial node to slow heart rate
fibrillation E) Parasympathetic nerve fibres act on
muscle fibres in the atria and ventricles to
Question 2. In the majority of individuals, reduce the strength of myocardial
the atrioventricular (AV) node is supplied contraction
by the:
A) Left main stem artery Question 5. Which one of the following is
B) Left anterior descending artery not an appropriate indication for an
C) Circumflex artery exercise (stress) ECG?
D) Right coronary artery A) Evaluation of stable angina
E) Obtuse marginal artery B) Evaluation of unstable angina
C) Assessment of prognosis following a
Question 3. A 57-year-old man presents myocardial infarction
with an acute myocardial infarction D) Assessment of outcome following
affecting the anterior part of the septum coronary angioplasty
and anterior wall of the left ventricle. E) Diagnosis of exercise-induced
Which of the following vessels is most arrhythmias
likely to have been occluded?
A) Left anterior descending artery Question 6. Which of the following
B) Circumflex artery symptoms is most suggestive of
C) Right coronary artery myocardial ischaemia?
D) Right posterior descending artery A) Epigastric discomfort after eating
E) Obtuse marginal artery B) Exertional breathlessness and wheeze
C) Exertional breathlessness and throat
tightness
D) Breathlessness with perioral and digital
paraesthesia
E) Rapid palpitation on exertion

48
Question 7. A 58-year-old woman Question 10. Which of the following
presents with dyspnoea and central chest disorders is most likely to present with
and throat tightness, which occur when isolated right heart failure?
walking her dog and force her to stop and A) Aortic stenosis
rest. She has smoked 20 cigarettes a day B) Mitral regurgitation
for the last 30 years. Examination is C) Systemic hypertension
unremarkable apart from some lesions D) Dilated cardiomyopathy
around her eye: E) Recurrent pulmonary emboli

Question 11. Which of the following


findings on clinical examination is most
suggestive of left heart failure over right
heart failure?
A) Raised jugular venous pressure (JVP)
B) Hepatomegaly
C) Fine bi-basal crepitations
D) Peripheral oedema
E) Ascites
What abnormality is likely to be found on
blood tests? Question 12. A 68-year-old man presents
A) Renal impairment with acute pulmonary oedema. Which one
B) Hypercholesterolaemia of the following examination findings is
C) Hyperthyroidism most suggestive of pre-existing structural
D) Hyperglycaemia heart disease?
E) Hypocalcaemia A) Elevated jugular venous pressure
B) Basal lung crepitations
Question 8. The patient from the previous C) Displaced apex beat
question (Q7) is found to have a normal D) Gallop rhythm
12-lead ECG and chest X-ray. Which of E) Cool, clammy peripheries
the following would be the most
appropriate next investigation? Question 13. Which of the following
A) Coronary angiogram cardiac medications most commonly
B) Exercise ECG causes peripheral oedema?
C) Echocardiogram A) Digoxin
D) 24-hour tape B) Amlodipine
E) Pulmonary function tests C) Atenolol
D) Nicorandil
Question 9. The patient from the previous E) Ramipril
two questions (Q7 and Q8) undergoes an
exercise ECG, which is positive but shows Question 14. A patient presents with
no high-risk features. Blood tests confirm severe bilateral lower limb pitting oedema.
a raised plasma cholesterol of 7.2 and Which of the following features would
blood pressure is 158/92 mmHg. All of the suggest a cause other than chronic cardiac
following are appropriate secondary failure?
prevention measures except one. Which is A) Raised jugular venous pressure
the exception? B) Hepatomegaly
A) Aspirin C) Peripheral muscle wasting
B) Ramipril D) Hyponatraemia
C) Spironolactone E) Heavy proteinuria
D) Simvastatin
E) Smoking cessation

49
Question 15. A 63-year-old man with Question 19. Which one of the following
ischaemic heart disease attends the diagnoses is least likely to account for an
cardiology clinic with worsening episode of loss of consciousness?
symptoms of chronic heart failure. His A) Carotid sinus syndrome
treatment is modified. Three weeks later, B) Transient ischaemic attack
routine blood tests by the GP reveal a C) Stokes-Adams attack
serum potassium of 6.5 mmol/l (reference D) Ventricular tachycardia
range 3.6-5.1). Which of the following E) Hypertrophic obstructive
drugs is the most likely culprit? cardiomyopathy
A) Furosemide
B) Ramipril Question 20. In differentiating cardiac
C) Digoxin syncope from seizures, which one of the
D) Bisoprolol following most strongly suggests cardiac
E) Aspirin syncope?
A) No warning prior to blackout
Question 16. Which one of the following B) Cyanosis during unconscious period
treatments should be avoided in patients C) Urinary incontinence
with acute left ventricular failure? D) Brief twitching during unconscious
A) Intravenous glyceryl trinitrate period
B) Non-invasive positive pressure E) Rapid recovery (< 1 min) following the
ventilation collapse
C) Beta-blockers
D) Loop diuretics Question 21. A patient admitted to the
E) Intravenous morphine coronary care unit with an acute anterior
myocardial infarction loses consciousness
Question 17. Angiotensin receptor during the morning ward round. Below is
blockers (ARBs) are frequently used in a rhythm strip taken from his continuous
patients intolerant of ACE inhibitors. cardiac telemetry:
Which of the following adverse effects of What is the most urgent priority in this
ACE inhibitor therapy is most likely to be patient's management?
improved by switching to an ARB?
A) Renal impairment
B) Postural hypotension
C) Dry cough
D) Hyperkalaemia A) IV amiodarone
E) Hyponatraemia B) External cardiac pacing
C) IV adrenaline (epinephrine)
Question 18. All of the following agents D) Endotracheal intubation and ventilation
except one have been shown to reduce E) Defibrillation
mortality in patients with chronic heart
failure. Which is the exception? Question 22. A 74-year-old man with
A) Angiotensin receptor blockers severe pneumonia on the medical high-
B) ACE inhibitors dependency unit suffers a cardiac arrest.
C) Beta-blockers The cardiac monitor shows asystole.
D) Loop diuretics Cardiopulmonary resuscitation (CPR) is
E) Spironolactone commenced and IV adrenaline
(epinephrine) is administered. Which one
other drug should be given at this stage?
A) Amiodarone
B) Atropine
C) Lidocaine
D) Magnesium sulphate
E) Sodium bicarbonate

50
Question 23. Which of the following Question 29. A 58-year-old woman
disorders characteristically produces a presents to the hospital Accident and
loud first heart sound (S1)? Emergency department with sudden onset
A) Atrial septal defect of palpitations. Below is a rhythm strip of
B) Mitral stenosis her ECG before and after carotid sinus
C) Aortic regurgitation massage:
D) Left ventricular hypertrophy What is the cause of her palpitations?
E) Constrictive pericarditis

Question 24. Wide fixed splitting of the


second heart sound occurs in which one of
the following disorders? A) Atrial fibrillation
A) Patent ductus arteriosus B) Atrial tachycardia
B) Aortic stenosis C) Atrial flutter
C) Ventricular septal defect D) Atrioventricular (AV) nodal re-entry
D) Mitral regurgitation tachycardia
E) Atrial septal defect E) Sinus tachycardia

Question 25. A fourth heart sound is least Question 30. A 47-year-old man presents
likely to be heard in which of the to the hospital Accident and Emergency
following conditions? department with palpitations and is found
A) Systemic hypertension to be in atrial fibrillation (AF) with a rapid
B) Aortic stenosis ventricular rate. He cardioverts
C) Hypertrophic cardiomyopathy spontaneously but this is his third such
D) Atrial fibrillation episode in the last 2 years. He is otherwise
E) Ischaemic heart disease well and drinks no alcohol. Blood pressure
is 152/88 mmHg. Routine blood tests are
Question 26. The combination of a third normal and an echocardiogram reveals no
heart sound and soft first heart sound is structural heart disease. Which of the
most characteristic of which of the following medications would be the most
following disorders? appropriate first-line therapy to prevent
A) Aortic regurgitation further episodes?
B) Pulmonary stenosis A) Bisoprolol
C) Mitral regurgitation B) Verapamil
D) Atrial septal defect C) Ramipril
E) Ventricular septal defect D) Digoxin
E) Amiodarone
Question 27. Which of the following
conditions is most likely to produce a Question 31. Which one of the following
murmur in early diastole? is a contraindication to DC cardioversion
A) Ventricular septal defect (VSD) for persistent atrial fibrillation?
B) Mitral stenosis A) Age over 75
C) Tricuspid regurgitation B) Atrial fibrillation present for > 1 year
D) Aortic regurgitation C) Moderate to severe left ventricular
E) Pulmonary stenosis impairment
D) Subtherapeutic INR
Question 28. Which one of the following E) Relapse after two previous
not a cause of sinus tachycardia? cardioversions
A) Anaemia
B) Raised intracranial pressure
C) Thyrotoxicosis
D) Phaeochromocytoma
E) Nebulised salbutamol

51
Question 32. Which one of the following Question 36. A 55-year-old man is
medications would not be suitable for rate brought to the emergency department by
control in persistent atrial fibrillation? ambulance after collapsing at work. His
A) Digoxin ECG shows a broad complex tachycardia:
B) Atenolol
C) Verapamil
D) Diltiazem
E) Amlodipine
Which of the following features most
strongly favours a diagnosis of ventricular
Question 33. Intravenous adenosine is
tachycardia (VT) over supraventricular
most likely to restore sinus rhythm in
tachycardia with bundle branch block?
which of the following tachyarrhythmias?
A) History of ischaemic heart disease
A) Atrial flutter
B) Termination with IV adenosine
B) Atrial fibrillation
C) Presence of capture and fusion beats on
C) Atrioventricular (AV) nodal re-entrant
ECG
tachycardia
D) QRS duration of 150 ms
D) Atrial tachycardia
E) Right axis deviation on ECG
E) Ventricular tachycardia
Question 37. The patient in the previous
Question 34. A 24-year-old woman
question (Q36) is diagnosed with
presents to her GP with recurrent episodes
ventricular tachycardia (VT). He is aware
of palpitation. She describes several
of his heart racing but is otherwise
episodes where she has suddenly felt her
asymptomatic. Heart rate is 170 bpm.
heart beating very rapidly. They last
Blood pressure is 110/70 mmHg. There is
anywhere from a few minutes to an hour
no evidence of pulmonary oedema. What
before self-terminating. She feels very
is the most appropriate next treatment?
frightened during these episodes and is
A) IV metoprolol
worried she may have a serious heart
B) IV amiodarone
condition. Examination is normal and she
C) IV flecainide
appears to be in sinus rhythm. A segment
D) DC cardioversion
of her 12-lead ECG is shown below.
E) Overdrive pacing

Question 38. Below is an ECG recording


What is the likely diagnosis? from a patient on the coronary care unit,
A) Paroxysmal atrial fibrillation who is on continuous telemetry. She had
B) Panic attacks been admitted the previous day with an
C) Wolff-Parkinson-White syndrome episode of syncope:
D) Ventricular ectopic beats What is the diagnosis?
E) Recurrent ventricular tachycardia

Question 35. The patient in the previous


question (Q34) finds these episodes of
palpitation distressing and asks about A) Idioventricular rhythm ('slow
treatment to prevent further episodes. ventricular tachycardia')
Which of the following is the treatment of B) Ventricular fibrillation
choice? C) Torsades de pointes
A) Digoxin D) Supraventricular tachycardia with
B) Dual chamber pacemaker aberrant conduction
C) Verapamil E) Sick sinus syndrome
D) Catheter ablation of accessory pathway
E) Amiodarone

52
Question 39. What abnormality is most Question 43. What is the most appropriate
likely to be seen on a 12-lead ECG from management of the patient in the previous
the patient in the previous question (Q38) question (Q42)?
when she was in sinus rhythm? A) Review in 6 months and consider
A) ST elevation treatment if any syncopal episodes
B) Prolonged PR interval B) Start amiodarone and repeat 24-hour
C) Broad QRS complexes tape
D) Prolonged QT interval C) Atrial pacemaker
E) Delta wave D) Dual-chamber pacemaker
E) Implantable cardiac defibrillator
Question 40. Which of the following
drugs is most likely to cause a prolonged Question 44. Which one of the following
QT interval? is not a manifestation of digoxin toxicity?
A) Omeprazole A) Nephrotoxicity
B) Digoxin B) Altered colour vision
C) Clarithromycin C) Bradycardia
D) Diazepam D) Vomiting
E) Morphine E) Ventricular tachycardia

Question 41. Which of the following drug Question 45. Which of the following
treatments is most appropriate for a patient features renders an atherosclerotic plaque
with torsades de pointes? most vulnerable to rupture?
A) Magnesium A) Thick fibrous cap
B) Amiodarone B) Calcification
C) Lidocaine C) Lipid-rich core
D) Digoxin D) Extensive collagenous cross-struts
E) Sotalol E) High concentration of smooth muscle
cells
Question 42. A 77-year-old patient is
referred to the cardiology outpatient clinic Question 46. Which one of the following
with recurrent dizzy spells, and a 24-hour is not an independent risk factor for
tape is performed. The ECG strip below is atherosclerosis?
recorded during a typical episode of A) Systolic hypertension
dizziness: B) Raised high-density lipoprotein (HDL)
cholesterol
C) Hyperhomocysteinaemia
D) Central obesity
E) Diabetes mellitus
What rhythm abnormality is shown?
A) First-degree atrioventricular (AV)
block
B) Mobitz type I second-degree AV block
C) Mobitz type II second-degree AV block
D) Third-degree (complete) heart block
E) Junctional rhythm

53
Question 47. A 60-year-old man is Question 50. A 68-year-old man with a
referred to the cardiology outpatient long history of angina presents with a
department with a 6-month history of worsening of anginal symptoms. He was
exertional chest pain relieved by rest and previously able to walk up to 1 mile before
glyceryl trinitrate (GTN) spray. As part of the onset of symptoms but for the past few
his assessment he undergoes an exercise days has been experiencing pain after just
ECG. Which of the following features on 50 yards and, on one occasion, at rest.
exercise ECG would be the strongest Which of the following factors would have
indication for proceeding to coronary the least impact on his short-term risk of
angiography? death or acute myocardial infarction?
A) Rise in systolic blood pressure of 60 A) Recurrent pain at rest
mmHg during test B) ST segment depression on ECG
B) Widespread 1-2 mm upward-sloping C) Elevated serum cholesterol
ST depression during stage 2 of the Bruce D) Persistent deep T-wave inversion on
protocol without symptoms ECG
C) Failure to complete stage 1 of the E) Elevated serum troponin
Bruce protocol due to hip pain
D) Chest pain and 2-3 mm horizontal ST Question 51. The patient from the
depression in lateral leads during stage 4 previous question (Q50) is admitted to
of the Bruce protocol hospital and an admission ECG reveals 1
E) Chest pain and 1-2 mm downward- mm ST depression in leads V5 and V6. In
sloping ST depression in inferior leads the absence of any contraindications,
during stage 1 of the Bruce protocol which of the following would be the
optimal combination of antiplatelet agents
Question 48. Which one of the following and anticoagulants to prescribe?
drugs is not an anti-anginal agent? A) Aspirin alone
A) Clopidogrel B) Aspirin and low molecular weight
B) Atenolol heparin
C) Diltiazem C) Aspirin and clopidogrel
D) Nicorandil D) Aspirin, clopidogrel and low molecular
E) Isosorbide mononitrate weight heparin
E) Aspirin, clopidogrel and warfarin
Question 49. Which one of the following
features is least consistent with a diagnosis Question 52. After 48 hours of optimal
of unstable angina? medical treatment, the patient from the
A) Chest pain on minimal exertion previous question (Q51) is still
B) Persistent ST segment elevation on experiencing occasional short-lived
ECG episodes of chest pain at rest. His ECGs
C) No preceding history of stable angina while he is pain-free are normal, but those
D) More than two episodes of chest pain at taken during pain show recurrence of ST
rest in the last 24 hours segment depression in the lateral leads.
E) T wave inversion on ECG Troponin T is modestly elevated. What
would be the best investigation to perform
next in this patient?
A) 24-hour ECG
B) Exercise ECG
C) Echocardiogram
D) Myocardial perfusion scan
E) Coronary angiogram

54
Question 53. A 48-year-old man presents Question 55. A 62-year-old woman
with a 6-hour history of severe central presents with a 2-hour history of severe
chest pain. He is unable to give much chest pain and has the following ECG:
history but declares that the pain is the
worst he has ever felt. He was previously
well with no history of cardiovascular
disease. On examination he appears pale,
sweaty and distressed. His ECG is shown
below:
What is the diagnosis?

While awaiting thrombolysis, she


complains of lightheadedness. Her pulse
rate is 38 beats per minute and telemetry
shows sinus bradycardia. What is the most
appropriate next treatment?
A) Insertion of temporary pacing wire
B) IV atropine
A) Aortic dissection C) External cardiac pacing
B) Acute anterior myocardial infarction D) IV isoprenaline
C) Acute inferior myocardial infarction E) IV atenolol
D) Unstable angina
E) Pericarditis Question 56. Which one of the following
long-term treatments does not confer
Question 54. The patient from the prognostic benefit in survivors of acute
previous question (Q53) with acute myocardial infarction (MI)?
anterior ST elevation myocardial A) Aspirin
infarction (MI) is treated immediately with B) Angiotensin-converting enzyme (ACE)
IV morphine, sublingual glyceryl trinitrate inhibitor
and aspirin. He appears less distressed but C) Beta-blocker
his heart rate is 118 bpm and blood D) Statin
pressure is 195/115 mmHg. Heart sounds E) Isosorbide mononitrate
1 and 2 are present with no added sounds
or murmurs and the chest is clear to Question 57. In a patient with peripheral
auscultation. Which of the following arterial disease, which one of the
options would be the +best next step in his following features suggests the worst
management? prognosis?
A) Thrombolysis A) Claudication distance of 50-100 metres
B) IV magnesium B) Night pain
C) Sublingual nifedipine C) Diminished pedal pulses
D) IV metoprolol D) Ankle:brachial pressure index (ABPI)
E) Oral ramipril ratio of 0.8
E) Bruit over femoral artery

55
Question 58. In a patient presenting with Question 63. A 53-year-old male with
acute lower limb ischaemia, which one of hypertension has been taking 10 mg of
the following clinical features would ramipril for several months. Despite this,
favour a diagnosis of embolism over his blood pressure control remains
thrombosis in situ? suboptimal. There are no compliance
A) Onset of symptoms over hours issues. Which one of the following agents
B) History of intermittent claudication would be the best add-on therapy?
C) Previously undiagnosed atrial A) Angiotensin receptor blocker
fibrillation B) Beta-blocker
D) Absent contralateral lower limb pulses C) Calcium channel antagonist
E) Presence of bruits D) Hydralazine
E) Alpha-blocker
Question 59. Which one of the following
clinical features is least consistent with a Question 64. A 19-year-old man presents
type B aortic dissection? with recent onset of breathlessness and
A) Severe tearing chest pain sharp, central chest pain exacerbated by
B) Elevated blood pressure movement and coughing. His heart rate is
C) Asymmetry of femoral pulses 110 beats per minute. On auscultation
D) New early diastolic murmur there is a soft pansystolic murmur and a
E) Acute renal failure pericardial friction rub. Echocardiography
demonstrates mitral regurgitation.
Question 60. Which of the following is Antistreptolysin O antibody titres (ASOT)
not a recognised risk factor for essential are 500 U/ml (normal range < 200). What
hypertension? is the likeliest diagnosis?
A) Hyperlipidaemia A) Infective endocarditis
B) Obesity B) Viral myocarditis
C) High salt intake C) Acute rheumatic fever
D) Intrauterine growth retardation D) Viral pericarditis
E) Alcohol excess E) Dressler's syndrome

Question 61. The following endocrine Question 65. In the patient from the
diseases are all recognised causes of previous question (Q64), which one of the
secondary hypertension except one. Which following features would clinch the
is the exception? diagnosis of rheumatic fever?
A) Acromegaly A) Temperature > 38oC
B) Congenital adrenal hyperplasia B) Positive throat swab culture
C) Conn's syndrome C) Cardiac dilatation on echocardiography
D) Phaeochromocytoma D) First-degree block on ECG
E) Addison's disease E) Flitting polyarthritis

Question 62. A 77-year-old woman in


good health is found by her general
practitioner to have persistently elevated
blood pressure, with a typical reading of
162/84 mmHg. Which of the following
agents would be the most appropriate
initial treatment?
A) Angiotensin-converting enzyme (ACE)
inhibitor
B) Angiotensin receptor blocker
C) Beta-blocker
D) Thiazide diuretic
E) Alpha-blocker

56
Question 66. A 53-year-old lady Question 71. A 64-year-old man is found
complains of progressively worsening to have an ejection systolic murmur during
exertional dyspnoea associated with a routine medical examination. Which of
deterioration in exercise tolerance over the the following associated clinical signs
past 2 years. On examination her pulse is would lend most support to a diagnosis of
irregularly irregular and of small volume. aortic stenosis?
There is a low-pitched mid-diastolic A) Displaced apex beat
murmur audible at her apex. What is the B) Loud aortic component of second heart
likeliest cause of her breathlessness? sound (A2)
A) Dilated cardiomyopathy C) Louder in inspiration
B) Ischaemic heart disease D) Wide pulse pressure
C) Ventricular septal defect E) Slow-rising carotid pulse
D) Mitral stenosis
E) Aortic regurgitation Question 72. A patient with known aortic
Question 67. With reference to the patient stenosis is followed up annually in the
from the previous question (Q66) who has cardiology outpatient clinic. Which of the
mitral stenosis, which one of the following following would be the strongest
features would suggest that her condition indication for aortic valve replacement?
has been complicated by pulmonary A) Syncopal episodes
hypertension? B) Prolonged murmur and thrusting apex
A) Loud first heart sound (S1) beat on examination
B) Opening snap C) Left bundle branch block on ECG
C) Pansystolic murmur radiating to the D) Reduced valve area on
axilla echocardiography
D) Tapping apex beat E) High pressure gradient between left
E) Loud pulmonary component of second ventricle and aorta at cardiac
heart sound (P2) catheterisation

Question 68. Which one of the following Question 73. You are asked to examine a
would be a contraindication to mitral dyspnoeic patient on the cardiology ward.
balloon valvuloplasty? She has bounding peripheral pulses, a
A) Mitral valve orifice < 2 cm2 blood pressure of 190/70 mmHg, a
B) Concomitant mitral regurgitation displaced, heaving apex beat and a loud
C) Mobile valvular apparatus early diastolic murmur. Which of the
D) Atrial fibrillation following underlying pathologies would
E) Enlarged left atrium be least likely to account for these
physical findings?
Question 69. The following are all A) Rheumatic heart disease
recognised causes of mitral regurgitation B) Acute aortic dissection
except one. Which is the exception? C) Syphilitic aortitis
A) Papillary muscle rupture D) Marfan's syndrome
B) Dilated cardiomyopathy E) Ankylosing spondylitis
C) Acute rheumatic fever
D) Subacute bacterial endocarditis
E) Congenitally bicuspid mitral valve

Question 70. Which one of the following


features is likely to be found in mitral
regurgitation but not in mitral stenosis?
A) Irregularly irregular pulse
B) Third heart sound (S3)
C) Right ventricular heave
D) Accentuated pulmonary component of
second heart sound
E) Bi-basal crepitations
57
Question 74. A patient with a history of Question 78. An 18-year-old boy, who
long-standing rheumatic mitral stenosis previously attended the paediatric
presents with a 6-week history of malaise, cardiology clinic but defaulted from
drenching sweats and lethargy. On follow-up after his 14th birthday, now
examination she has poor dentition, a presents to his GP with increasingly severe
temperature of 38.5°C and a loud breathlessness, fatigue and reduced
pansystolic murmur at apex radiating to exercise tolerance. On examination he is
axilla. You suspect a diagnosis of subacute centrally cyanosed with clubbing of the
bacterial endocarditis. Which of the fingers and toes. Which of the following is
following microorganisms is the likeliest the most likely underlying congenital
culprit? cardiac abnormality?
A) Viridans streptococci A) Ventricular septal defect
B) Staphylococcus aureus B) Coarctation of the aorta
C) Coxiella burnetti C) Pulmonary stenosis
D) Coagulase-negative staphylococci D) Bicuspid aortic valve
E) Haemophilus E) Patent foramen ovale

Question 75. Echocardiography confirms Question 79. Which one of the following
the presence of a vegetation on the mitral does not constitute a component of Fallot's
valve and mitral regurgitation. Six sets of tetralogy?
blood cultures are sent. While awaiting A) Pulmonary stenosis
results, which of the following antibiotic B) Ventricular septal defect
regimens would be the most appropriate C) Overriding aorta
empirical treatment? D) Patent ductus arteriosus
A) Flucloxacillin E) Right ventricular hypertrophy
B) Benzyl penicillin
C) Gentamicin Question 80. A patient who immigrated to
D) Benzyl penicillin and gentamicin the UK from West Africa 18 months ago
E) Vancomycin and flucloxacillin presents with progressive ankle swelling
and abdominal distension. On examination
Question 76. According to the modified he has marked bilateral lower limb
Duke criteria, which one of the following oedema, a palpably enlarged liver and
is considered a major criterion for the ascites. His jugular venous pulse is
diagnosis of infective endocarditis? elevated and rises during inspiration. Lung
A) Pyrexia > 38°C fields are clear to auscultation. His chest
B) Pre-existing aortic stenosis X-ray is shown below:
C) Splinter haemorrhages
D) Vegetations on echocardiography
E) Osler's nodes

Question 77. A continuous machinery


murmur, loudest over the left second
intercostal space, is associated with which
of the following congenital cardiac
abnormalities?
A) Atrial septal defect
B) Ventricular septal defect
C) Patent ductus arteriosus Which of the following is the likeliest
D) Tricuspid atresia underlying aetiology?
E) Pulmonary stenosis A) Tuberculosis
B) TrypanosomiasisC) Syphilis
D) Malaria
E) Haemochromatosis

58
Chapter 18 RESPIRATORY DISEASE
Answers page 181

Question 1. A 21-year-old woman Question 3. The patient with COPD from


presents to the hospital accident and the previous question (Q2) has the
emergency department with an acute following arterial blood gas (ABG) results:
exacerbation of asthma. Which of the
following features in her presentation
would give greatest cause for concern?
A) Peak expiratory flow (PEF) 40% of
predicted
B) Inability to complete sentences in one
breath
C) PaCO2 of 5.8 kPa
D) Respiratory rate of 30 breaths per
minute
E) Sinus tachycardia of 120 beats per How should these results be interpreted?
minute A) Compensated metabolic acidosis
B) Compensated respiratory acidosis
Question 2. A 67-year-old lifelong C) Mixed respiratory and metabolic
smoker with a history of one previous alkalosis
myocardial infarction attends the D) Compensated metabolic alkalosis
respiratory clinic with shortness of breath. E) Mixed respiratory and metabolic
On examination she is clinically acidosis
hyperinflated, and has a respiratory rate of
19 breaths per minute at rest and quiet Question 4. In the treatment of stable
vesicular breath sounds bilaterally. COPD, which one of the following
Bilateral ankle oedema is present to the interventions will provide the greatest
mid-calves. Pulmonary function tests show survival benefit?
a forced vital capacity (FVC) of 70% A) Home nebuliser with salbutamol
predicted, a forced expiratory volume in 1 B) Long-acting ß2-agonist
second (FEV1) of 25% predicted and an C) Oral steroid therapy
FEV1:FVC ratio of 55%. These D) Smoking cessation
abnormalities are only partially reversed E) Daily inhaled tiotropium
by inhaled salbutamol. Which of the
following conditions is the most likely
diagnosis?
A) Asthma
B) Chronic obstructive pulmonary disease
(COPD)
C) Congestive cardiac failure
D) Sarcoidosis
E) Bronchial carcinoma

59
Question 5. A previously well, non- Question 7. With reference to the patient
smoking, 24-year-old Turkish woman in the previous question (Q6), which of the
presents to the hospital accident and following investigations would be of
emergency department with a 1-day greatest use in establishing a diagnosis?
history of non-productive cough, chills A) CT scan of thorax
and sore throat. Her voice is not hoarse but B) Lateral chest X-ray
there is a harsh rasping sound from her C) Serum rheumatoid factor, anti-nuclear
throat with each breath and she is unable factor and anti-DNA antibodies
to swallow her own saliva. Her routine D) Pleural aspiration
observations are: E) Arterial blood gas

Question 8. The patient from the previous


two questions (Q6 and Q7) has 30 ml of
serous fluid aspirated from his left pleural
Which of the following is the most space and sent for analysis. Initial
appropriate initial action? biochemistry results are shown below,
A) Carefully examine the throat under along with the results of some routine
direct vision blood tests:
B) Check arterial blood gases
C) Perform a CT scan of the chest and
neck
D) Transfer to a critical care environment
and consider endotracheal intubation by an
experienced anaesthetist
E) Administer nebulised bronchodilator
Which of the following conditions is the
most likely cause of his pleural effusion?
A) Nephrotic syndrome
B) Parapneumonic effusion
C) Thoracic duct occlusion
D) Acute pancreatitis
E) Congestive cardiac failure

Question 9. A 34-year-old woman


presents to the hospital accident and
emergency
department with sudden onset of acute
Question 6. This chest X-ray was taken severe breathlessness. She is able to talk in
from a 69-year-old man who presented broken sentences. She reports that she
with confusion and a vague history of cannot get enough air and also feels
increasing shortness of breath: tingling in both fingertips. She has had
Which of the following clinical signs is several similar episodes in the past. On
most likely to be present on examination examination, she appears very distressed.
of the chest? Respiratory rate is 30 breaths per minute
A) Bronchial breathing at left base but examination of her chest reveals no
B) Reduced chest expansion on right side abnormal findings. Oxygen saturations on
C) Stony dull percussion note at left base air are 99%. What is the most likely
D) Inspiratory crepitations at left base diagnosis?
E) Reduced cricosternal distance A) Psychogenic hyperventilation
B) Acute asthma
C) Pneumothorax
D) Pulmonary oedema
E) Pneumonia

60
Question 10. A previously healthy 35-
year-old man presents with fever,
tachypnoea and hypoxia on air. Which one
of the following features would not lend
support to a diagnosis of Pneumocystis
carinii (jirovecii) pneumonia?
A) Oropharyngeal candidiasis
B) Rusty, golden yellow-coloured sputum
C) Poor response to standard antibiotics
D) Marked worsening of hypoxia on Which of the following would be the most
exertion appropriate next step?
E) Normal chest X-ray but nodular A) Discharge with advice to return if the
ground-glass attenuation on high- wheeze worsens
resolution CT B) Admit to the medical ward for
overnight observation
Question 11. An 18-year-old asthmatic C) Refer to the intensive treatment unit for
patient presents to the hospital accident consideration of mechanical ventilation
and emergency department 1 hour after D) Refer to the high-dependency unit for a
developing breathlessness and wheeze. On trial of non-invasive ventilation
admission she is struggling to complete E) Reduce oxygen concentration to 28%
sentences in one breath, and has a peak and repeat the ABG in 1 hour
flow of 150 l/min (her best peak expiratory
flow rate (PEFR) is 310 l/min) and Question 13. Which of the following best
widespread wheeze throughout her chest. describes the mode of action of
Observations are shown below: ipratropium?
A) ß2-adrenoceptor agonist
B) Leukotriene receptor antagonist
C) Corticosteroid
D) Anticholinergic
E) Cholinergic agonist
All of the following are appropriate steps
in her immediate management except one. Question 14. A patient presents to the
Which is the exception? respiratory clinic complaining of 6 months'
A) High-concentration oxygen history of cough. Respiratory examination
B) Nebulised salbutamol and chest X-ray are normal. Which of the
C) Systemic corticosteroids following is least likely to explain her
D) Nebulised ipratropium symptom?
E) Intravenous aminophylline A) Treatment with ramipril
B) Post-nasal drip
Question 12. After 1 hour of treatment C) Gastro-oesophageal reflux disease
with salbutamol, oxygen, systemic D) Viral pneumonia
corticosteroids and intravenous E) Asthma
magnesium, the patient in the previous
question (Q11) has made no improvement
and appears to be tiring. Her PEFR is now
100 l/min and auscultation of the chest
reveals quiet wheeze bilaterally. Below are
the results of arterial blood gas
measurements taken on admission (first
ABG) then repeated after treatment
(second ABG), both on 60% O2:

61
Question 15. A previously fit and healthy What is the likeliest cause of her
54-year-old woman returns to the UK presentation?
from a family holiday in Spain. She A) Infective exacerbation of COPD
reports fever, rigors, right-sided pleuritic B) Pulmonary embolism
chest pain and general fatigue over the last C) Acute left ventricular failure
24 hours. On examination her temperature D) Psychogenic hyperventilation
is 37.8°C, she appears confused, E) Pneumonia
respiratory rate is 32/min, heart rate
110/min, BP 88/50 mmHg and SaO2 92% Question 17. With reference to the patient
on high-concentration oxygen. in the previous question (Q16), which of
Examination findings relating to the right the following investigations would be
side of the chest are as follows: reduced most likely to confirm the diagnosis?
expansion, dull percussion note, bronchial A) Computed tomography pulmonary
breathing, increased vocal resonance and angiogram (CTPA)
whispering pectoriloquy. Initial blood tests B) Echocardiogram
show a D-dimer of 1000 U, white cell C) Plasma D-dimer level
count of 15 × 109/l and serum urea of 10 D) Ventilation/perfusion (V/Q) scan
mmol/l. Which one of the following E) High-resolution CT (HRCT) scan of
statements is true concerning the thorax
management of this patient?
A) Therapeutic dose low-molecular weight Question 18. A 23-year-old man is rushed
heparin should be commenced to treat to hospital after complaining of severe
pulmonary embolism left-sided chest pain whilst drinking in a
B) Aggressive fluid resuscitation is bar. On arrival at the hospital accident and
necessary immediately emergency department he is in
C) Antibiotic therapy should begin considerable distress and too short of
following the results of sputum culture breath to give a meaningful history. His
D) The most appropriate antibiotics are an blood pressure in the ambulance was
oral aminopenicillin in combination with 83/40 mmHg. He is maintaining his
an oral macrolide airway but his trachea is shifted to the
E) She should be admitted to a general right and he has distended neck veins.
medical or general respiratory unit Which of the following is the most
appropriate way to get enough diagnostic
Question 16. A 62-year-old lady with information to begin treatment?
mild chronic obstructive pulmonary A) Chest X-ray
disease is admitted following sudden onset B) Percussion and auscultation of the lung
of shortness of breath at rest and left-sided fields
pleuritic chest pain. Her chronic cough is C) ECG
unchanged and she denies any D) Arterial blood gas analysis
haemoptysis or change in sputum E) Check for flapping tremor (asterixis)
production. Her only medications are an
as-required ß2-agonist inhaler and
oestrogen replacement therapy. Clinical
examination reveals tachypnoea at rest and
signs of hyperinflation. There is no audible
wheeze on auscultation. Observations are
as follows:
• respiratory rate 22/min, heart rate 102
beats per minute, BP 140/72 mmHg, SpO2
88% on air
• ECG shows sinus tachycardia but no
other abnormality
• chest X-ray shows hyperexpanded lung
fields with no evidence of pneumothorax,
collapse or consolidation.
62
Question 19. A 50-year-old patient is
referred to the respiratory clinic with
shortness of breath. On examination he has
finger clubbing, central cyanosis and fine
end-inspiratory crepitations at both lung
bases. Which of the following types of
pathology would be most consistent with
all these examination findings?
A) Chronic obstructive pulmonary disease
(COPD)
B) Extrathoracic restrictive lung disease
C) Fibrotic lung disease Question 23. This flow-volume loop
D) Left ventricular failure (labelled 'patient A') was taken during
E) Cystic fibrosis pulmonary function testing at a respiratory
clinic:
Question 20. What is the primary Which of the following statements is true?
anatomical source of normal healthy A) The pattern of flow limitation suggests
breath sounds? large airway obstruction
A) Larynx and central airways B) Obstruction is equally severe during
B) Small bronchi inspiration and expiration
C) Terminal bronchioles C) The limitation of vital capacity
D) Respiratory bronchioles indicates a restrictive defect
E) Opening of alveoli D) Reduced mid-expiratory flow indicates
collapse of small intrathoracic airways
Question 21. Which of the following is E) Patient A could produce a normal
the correct description of the normal lobes reading on a peak flow meter
of the lungs?
A) Two lobes on the right, three on the left
B) Two lobes on the right, one on the left
C) Three lobes on the right and left
D) Three lobes on the right, two on the left
E) One lobe on the right, two on the left

Question 22. A 29-year-old lady presents


to the dermatology clinic with tender, red,
raised lesions on her shins. She also
complains of joint pains and pyrexial Question 24. This is a graph of lung
symptoms. A chest X-ray shows bilateral volume plotted against time:
hilar lymphadenopathy (BHL) but no other What is the parameter labelled 'A' (red
abnormality. Which of the following is the arrow)?
most likely explanation for these findings? A) Residual volume
A) Tuberculosis B) Inspiratory capacity
B) Lymphoma C) Tidal volume
C) Sarcoidosis D) Vital capacity
D) Wegener's granulomatosis E) Functional residual capacity
E) Rheumatoid arthritis

63
Question 25. A 23-year-old known Question 29. At 0200 hrs, the on-call
asthmatic is admitted with acute severe respiratory physician is called to see a
breathlessness and tightness in his chest. patient who was admitted at 1600 hrs with
On examination, polyphonic wheeze is an acute exacerbation of chronic
heard throughout both lung fields, but is obstructive pulmonary disease (COPD).
quieter on the right. There is some This is her first admission with chest
response to initial medical therapy, but problems. She is centrally cyanosed and
after 15 minutes of standard treatment he drowsy and has bounding peripheral
remains tachypnoeic and distressed. pulses. What is the most urgent
Which of the following investigations is investigation?
most urgent? A) Chest X-ray
A) Chest X-ray B) Arterial blood gas (ABG) analysis
B) Urea and electrolytes C) Blood samples for salicylates and
C) ECG paracetamol levels
D) Doppler ultrasound of leg veins D) ECG
E) Direct laryngoscopy E) CT of the head

Question 26. Which of the following Question 30. Below are the arterial blood
treatments has been shown to prolong life gases of the patient in the previous
in chronic obstructive pulmonary disease question (Q29):
(COPD)?
A) Long-term oxygen therapy
B) Inhaled tiotropium
C) PRN salbutamol
D) Furosemide
E) Sildenafil (Viagra)

Question 27. In the investigation of a


solitary lung nodule, which of the
following tends to suggest benign, rather
than malignant, pathology? Which one of the following treatments is
A) Spiculated appearance most appropriate?
B) Size > 3 cm A) Increase to 40% Venturi oxygen
C) Long smoking history B) Decrease to 28% Venturi oxygen
D) Calcification on chest X-ray C) Administer 15 l/min oxygen via trauma
E) Hilar lymphadenopathy (reservoir bag) mask
D) Remove oxygen
Question 28. Which of the following is E) Mechanical ventilatory support
not a risk factor for obstructive sleep
apnoea/hypopnoea syndrome (OSAHS)? Question 31. Which one of the following
A) Male sex conditions is more likely to cause type II
B) Obesity respiratory failure rather than type I?
C) Protuberant mandible A) Pulmonary fibrosis
D) Acromegaly B) Pneumothorax
E) Hypothyroidism C) Acute respiratory distress syndrome
D) Pulmonary embolus
E) Severe kyphoscoliosis

64
Question 32. A 56-year-old man is Question 35. A 34-year-old man with
referred for investigation of recurrent asthma sees his general practitioner and
pneumonia. He suffers from asthma, explains that his asthma is still
which seems poorly controlled despite troublesome despite the recent addition of
regular use of high doses of fluticasone a low-dose inhaled corticosteroid to his
and salmeterol. He reports on-going prescription. He is currently on regular
cough, wheeze and sticky, difficult-to- low-dose inhaled steroid therapy and an
clear sputum and requires frequent courses as-needed short-acting ß2-adrenoceptor
of oral corticosteroids. He has never agonist. Which of the following is the
smoked, keeps no pets, complies with most appropriate next step in drug
therapy and has worked in the same office treatment?
for 12 years. Chest X-rays show evidence A) Short-term oral corticosteroid therapy
of a left lower lobe collapse 1 year ago and B) Addition of long-acting ß2-
more recently a right middle lobe collapse. adrenoceptor agonist
The peripheral blood count shows an C) Addition of short-acting anticholinergic
eosinophil count of 0.6 × 109/l. Which one bronchodilator
of the following statements is true? D) One-week course of empirical
A) He should be investigated for antibiotic therapy
carcinoma of the bronchus E) Addition of leukotriene receptor
B) Cystic fibrosis is the most likely antagonist
diagnosis
C) A trial of itraconazole should be Question 36. All of the options below
considered describe clinically stable patients with
D) Induced sputum is likely to be rich in chronic obstructive pulmonary disease and
neutrophils forced expiratory volume in 1 second
E) Prolonged therapy with high-dose oral (FEV1) < 1.5 litres on optimal medical
corticosteroids should be considered therapy. Patients breathed air for at least
20 minutes before arterial blood gases
Question 33. Which of the following (ABGs) were taken. Which one patient
classes of drug may precipitate an acute meets the criteria for long-term oxygen
attack of asthma? therapy (LTOT)?
A) Beta-adrenoceptor agonists A) 65-year-old smoker with maximum
B) Non-steroidal anti-inflammatory drugs PaO2 of 5.9 kPa on two ABGs 3 weeks
C) Calcium channel blocking drugs apart
D) Histamine H1 receptor antagonists B) 71-year-old ex-smoker with maximum
E) Leukotriene receptor antagonists PaO2 of 7.5 kPa on two ABGs 3 weeks
apart
Question 34. In a patient with a clinical C) 76-year-old ex-smoker with bilateral
history compatible with asthma, which one pitting oedema to knees and maximum
of the following clinches the diagnosis? PaO2 of 7.7 kPa on two ABGs 3 weeks
A) Clinical improvement during a course apart
of oral steroids D) 79-year-old ex-smoker with congestive
B) A 20% improvement in forced cardiac failure and maximum PaO2 of 7.6
expiratory volume in 1 second (FEV1) kPa on two ABGs 3 weeks apart
following administration of a E) 85-year-old ex-smoker with pulmonary
bronchodilator hypertension and PaO2 of 8.7 kPa on two
C) An 8% spontaneous change in peak ABGs 3 weeks apart
expiratory flow (PEF) during 1 week of
home monitoring
D) A normal chest X-ray
E) Elevated peripheral blood eosinophil
count

65
Question 37. What is the usual pattern of Question 40. A previously healthy 58-
inheritance in cystic fibrosis? year-old smoker presents with a 2-month
A) Autosomal recessive history of cough productive of blood-
B) Autosomal dominant streaked sputum. His chest X-ray is shown
C) X-linked recessive below:
D) X-linked dominant
E) New mutations are most common

Question 38. Which of the following


drugs is most likely to cause alveolitis in
chronic use?
A) Digoxin
B) Amiodarone
C) Nifedipine
D) Furosemide
E) Atenolol

Question 39. A 64-year-old lady who


recently travelled to the UK from Baghdad
presents with a 3-month history of loss of Which one of the following abnormalities
appetite, weight loss, pyrexial symptoms is apparent on the chest X-ray?
and productive cough. On examination she A) Left upper lobe collapse
is pyrexial at 38.4°C and has B) Left lower lobe collapse
hepatosplenomegaly. After 2 weeks in C) Right upper lobe collapse
hospital all cultures have been negative so D) Right middle lobe collapse
a liver biopsy is taken. This shows E) Right lower lobe collapse
caseating liver granulomas. What is the
underlying diagnosis? Question 41. The same patient from the
A) Sarcoidosis previous question (Q40) then has some
B) Tuberculosis routine blood
C) Lymphoma tests:
D) Bronchogenic carcinoma
E) Amyloidosis

Question 39. A 64-year-old lady who


recently travelled to the UK from Baghdad
presents with a 3-month history of loss of
appetite, weight loss, pyrexial symptoms
and productive cough. On examination she
is pyrexial at 38.4°C and has
hepatosplenomegaly. After 2 weeks in
hospital all cultures have been negative so
a liver biopsy is taken. This shows
caseating liver granulomas. What is the Which of the following is the most likely
underlying diagnosis? unifying diagnosis?
A) Sarcoidosis A) Tuberculosis
B) Tuberculosis B) Small-cell bronchogenic carcinoma
C) Lymphoma C) Bronchial adenocarcinoma
D) Bronchogenic carcinoma D) Sarcoidosis
E) Amyloidosis E) Aspergilloma

66
Question 42. Which of the following Question 47. A 28-year-old gentleman
histological subtypes of lung cancer is with no prior history of chest trouble is
most commonly associated with referred following the receipt of a chest X-
hypercalcaemia in the absence of bony ray report performed as part of routine
metastases? occupational health screening. For the last
A) Small-cell carcinoma 10 years he has been employed by a
B) Bronchoalveolar carcinoma fireplace firm as a stone cutter. The
C) Squamous carcinoma factory is extremely dusty and although he
D) Adenocarcinoma has been supplied with a face mask, he
E) Adenosquamous carcinoma finds it uncomfortable to wear. His chest
X-ray shows multiple well-circumscribed
Question 43. All of the following except 3-5 mm nodular opacities predominantly
one are contraindications to surgical in the mid- and upper zones Which one of
resection in bronchial carcinoma. Which is the following statements is correct?
the exception? A) The most likely diagnosis is miliary
A) Continued cigarette smoking tuberculosis
B) Malignant pleural effusion B) The most likely diagnosis is silicosis
C) Involvement of contralateral C) The disease is unlikely to progress
mediastinal lymph nodes following cessation of exposure
D) Forced expiratory volume in 1 second D) Lung function is likely to show an
(FEV1) of 0.8 litres obstructive defect
E) Presence of bony metastases E) Screening of the other employees is
unnecessary
Question 44. Which of the following is
not normally associated with persistent Question 48. A 60-year-old man has been
pulmonary arterial hypertension? undergoing treatment for stage IIIA
A) Chronic venous thromboembolism Hodgkins disease. He received
B) Progressive systemic sclerosis chemotherapy with adriamycin,
C) Chronic obstructive pulmonary disease bleomycin, vinblastine and dacarbazine.
D) Chronic exposure to high altitude Several months later he developed a dry
E) Asthma cough, exertional breathlessness and
Question 45. Which of the following intermittent pyrexia. Examination revealed
disorders is not associated with the fine late inspiratory crackles. The full
development of pulmonary fibrosis? blood count was normal (including white
A) Systemic lupus erythematosus cell count), and sputum samples were
B) Rheumatoid arthritis negative for acid-fast bacilli and bacterial
C) Systemic sclerosis pathogens. Serology showed no evidence
D) Rheumatic fever of infection with atypical pathogens. The
E) Sarcoidosis chest X-ray showed diffuse
reticulonodular shadowing. Which one of
Question 46. All of the following the following statements is correct?
treatments except one are used in the A) Bronchoalveolar lavage would be the
management of primary pulmonary investigation of choice to diagnose
hypertension. Which is the exception? Pneumocystis carinii (jirovecii)
A) Sildenafil (Viagra) pneumonia (PCP)
B) Warfarin B) The clinical scenario could be
C) Intravenous prostacyclin infusion explained by bleomycin toxicity
D) Heart-lung transplantation C) Lung function tests would be expected
E) Intravenous endothelin to show an obstructive defect
D) Lung toxicity could be decreased by
the administration of G-CSF therapy
E) The appearances on high-resolution CT
(HRCT) scan are likely to be diagnostic

67
Chapter 19 ENDOCRINE DISEASE
Answers page 188

Question 1. Which of the following Question 4. Which of the following ocular


molecules does not contain iodine? signs of thyrotoxicosis is not specific to
A) Thyroxine (T4) Graves' disease?
B) Thyroid-stimulating hormone (TSH) A) Exophthalmos
C) Triiodothyronine (T3) B) Lid lag
D) Reverse triiodothyronine (rT3) C) Periorbital oedema
E) Amiodarone D) Chemosis
E) Ophthalmoplegia
Question 2. A 38-year-old woman is
referred to the medical outpatient clinic Question 5. Which of the following
with a 6-month history of heat intolerance, autoantibodies are most strongly
palpitations and weight loss despite a associated with Graves' disease?
normal appetite. Examination reveals a A) Anti-thyroid peroxidase antibodies
fine tremor and sinus tachycardia but is B) Anti-thyroglobulin antibodies
otherwise unremarkable. Blood tests show C) Anti-nuclear antibodies (ANA)
the following results: D) Thyroid-stimulating hormone (TSH)
receptor antibodies (TRAb)
E) Anti-neutrophil cytoplasmic antibodies
(ANCA)

Question 6. A 75-year-old woman attends


TSH receptor antibodies are detected in the hospital accident and emergency
her serum. What is the most likely department with palpitations. Her past
diagnosis? medical history is unremarkable, although
A) Toxic multinodular goiter she has recently developed a troublesome
B) Toxic thyroid adenoma tremor. ECG reveals atrial fibrillation with
C) Graves' thyrotoxicosis a ventricular rate of 145 bpm. Blood
D) Secondary thyrotoxicosis due to a pressure is 129/68 mmHg. Blood tests
pituitary tumour show biochemical evidence of
E) Factitious thyrotoxicosis thyrotoxicosis. What would be the most
appropriate initial treatment?
Question 3. A 59-year-old man with A) Digoxin
clinical and biochemical evidence of B) Propranolol
thyrotoxicosis undergoes 99mtechnetium C) Verapamil
scanning (scintigraphy) of the thyroid. D) Amiodarone
This reveals a well-defined area of E) DC cardioversion
increased isotope uptake with no uptake
through the rest of the gland. What is the
most likely diagnosis?
A) Thyroiditis
B) Papillary thyroid carcinoma
C) Toxic multinodular goiter
D) Medullary carcinoma of the thyroid
E) Toxic thyroid adenoma

68
Question 7. All of following except one Question 10. A 74-year-old woman is
are recognised causes of hypothyroidism. brought to the hospital accident and
Which is the exception? emergency department with severe
A) Hashimoto's thyroiditis drowsiness. According to her daughter,
B) Post-partum thyroiditis she has become increasingly fatigued and
C) Graves' disease lethargic over the past year and has gained
D) Hyperprolactinaemia considerable weight. Her memory has also
E) Radioactive iodine treatment for deteriorated but over the last few days she
hyperthyroidism has become significantly more confused
and drowsy. On examination, her Glasgow
Question 8. A 52-year-old woman attends Coma Score is 12/15, temperature is
her general practitioner with a 1-year 32.4°C, blood pressure 96/53 mmHg and
history of fatigue, cold intolerance, weight pulse 48 bpm. She is pale and her face and
gain and pain in her left hand at night. hands appear puffy. A provisional
Blood tests reveal the following: diagnosis of myxoedema coma is made
and urgent thyroid function tests are
requested. The most appropriate initial
management would be to:
A) Wait for the thyroid function test
results
B) Give oral thyroxine via a nasogastric
tube
C) Give IV triiodothyronine
D) Give IV triiodothyronine and IM
hydrocortisone
The most appropriate initial management E) Insert a temporary transvenous pacing
would be: wire
A) Arrange an ultrasound scan of the
thyroid Question 11. All of the following except
B) Arrange a 99mtechnetium scan of the one are recognised complications of
thyroid subtotal thyroidectomy for Graves'
C) Start treatment with thyroxine thyrotoxicosis. Which is the exception?
D) Start carbimazole A) Facial nerve injury
E) Do not start treatment now but recheck B) Transient hypocalcaemia
thyroid function tests in 3 months C) Keloid scar formation
D) Hoarse voice
Question 9. Which of the following might E) Primary hypothyroidism
necessitate a decrease in the maintenance
dose of thyroxine in patients with primary Question 12. Which of the following
hypothyroidism? statements is true of the drug carbimazole
A) Pregnancy in the treatment of Graves' disease?
B) Normal ageing A) It reduces peripheral conversion of
C) Treatment for epilepsy with phenytoin thyroxine (T4) to triiodothyronine (T3)
D) Treatment for tuberculosis with B) There is a 30% risk of agranulocytosis
rifampicin C) The risk of relapse is greater than 30%
E) Development of coeliac disease within 2 years of stopping the drug
D) It is preferable to propylthiouracil in
treating hyperthyroidism in breastfeeding
mothers
E) It is associated with an increased risk of
thyroid malignancy

69
Question 13. Serum calcitonin levels are Question 18. All of the following
useful in monitoring response to treatment measures except one have a role in the
of which thyroid condition? treatment of polycystic ovarian syndrome
A) Medullary carcinoma (PCOS). Which is the exception?
B) Thyroid lymphoma A) Metformin
C) Riedel's thyroiditis B) Combined oral contraceptive pill
D) Follicular carcinoma C) Gliclazide
E) Hashimoto's thyroiditis D) Weight loss
E) Cyproterone acetate
Question 14. Which one of the following
is not employed in the investigation and Question 19. All of the following are
treatment of papillary carcinoma of the recognised features of Turner's syndrome
thyroid? except one. Which is the exception?
A) Fine needle aspiration of the thyroid A) Short stature
B) Carbimazole B) Coarctation of the aorta
C) Total thyroidectomy C) Widely spaced nipples
D) Radioactive iodine D) Webbing of the neck
E) Measurement of serum thyroglobulin E) Low follicle-stimulating hormone
(FSH) and luteinising hormone (LH)
Question 15. All of the following except levels
one are causes of hypogonadotrophic
hypogonadism. Which is the exception? Question 20. Which one of the following
A) Pituitary adenoma statements is true of Klinefelter's
B) Anorexia nervosa syndrome?
C) Cystic fibrosis A) It is associated with a 47XYY
D) Autoimmune gonadal failure karyotype
E) Excessive physical exercise B) Affected individuals suffer from
hypogonadotrophic hypogonadism
Question 16. Which of the following C) There is an increased incidence of
statements is true regarding post- gynaecomastia
menopausal oestrogen-containing D) Treatment with growth hormone is
hormone replacement therapy (HRT)? usually required
A) Treatment is usually continued until the E) Testes are usually absent
age of 65
B) Women with an intact uterus should Question 21. Which of the following
receive both oestrogen and progesterone tumour markers is most useful in the
C) It has been demonstrated to reduce the diagnosis and prognosis of testicular
risk of ischaemic heart disease tumours?
D) It is a useful adjuvant treatment for A) Carcinoembryonic antigen (CEA)
thrombophilia B) Prostate-specific antigen (PSA)
E) It reduces the risk of breast cancer C) Lactate dehydrogenase (LDH)
D) Alpha-fetoprotein (AFP)
Question 17. Which of the following is E) CA125
least likely to cause hyperprolactinaemia?
A) Bromocriptine
B) Metoclopramide
C) Pituitary adenoma
D) Pregnancy
E) Hypothyroidism

70
Question 22. Which one of the following D) Low calcium, high phosphate, high
statements regarding parathyroid hormone parathyroid hormone
PTH) is false? E) Low calcium, high phosphate, normal
A) It is a polypeptide hormone parathyroid hormone
B) It inhibits 1V-hydroxylation of vitamin
D in the kidney Question 26. Which one of the following
C) It stimulates calcium resorption from skeletal changes is not a feature of primary
bone hyperparathyroidism?
D) It promotes calcium resorption from A) Osteophyte formation
renal tubules B) Reduced bone mineral density
E) Its release is inhibited by C) Osteitis fibrosa cystica
hypercalcaemia D) Chondrocalcinosis
E) Terminal resorption in the phalanges
Question 23. All of the following except
one are recognised causes of Question 27. A 43-year-old woman
hypercalcaemia. Which is the exception? attends her GP with a 1-year history of
A) Primary hyperparathyroidism progressive weight gain and malaise. She
B) Secondary hyperparathyroidism bruises easily and has trouble climbing
C) Tertiary hyperparathyroidism stairs. She also has increased facial hair.
D) Multiple myeloma On examination she has a body mass index
E) Squamous cell bronchial carcinoma of 32 kg/m2. Her abdomen is obese with
visible striae, although her arms and thighs
Question 24. Which cluster of features is appear wasted. What is the most likely
classically seen in multiple endocrine diagnosis?
neoplasia (MEN) type 2 (Sipple's A) Conn's syndrome
syndrome)? B) Cushing's syndrome
A) Hyperparathyroidism, pituitary C) Addison's disease
adenoma, pancreatic neuro-endocrine D) Phaeochromocytoma
tumour E) Congenital adrenal hyperplasia
B) Hyperparathyroidism, follicular thyroid
carcinoma, phaeochromocytoma Question 28. What is the most common
C) Hyperparathyroidism, medullary cause of Cushing's syndrome?
thyroid carcinoma, phaeochromocytoma A) Pituitary adenoma
D) Hyperparathyroidism, small cell lung B) Adrenal adenoma
carcinoma, colon carcinoma C) Adrenal carcinoma
E) Phaeochromocytoma, renal cell D) Chronic glucocorticoid therapy
carcinoma, cerebellar haemangioblastoma E) Ectopic adrenocorticotrophic (ACTH)
syndromes
Question 25. A 41-year-old woman with
Graves' disease undergoes subtotal Question 29. What is the most useful
thyroidectomy. Twenty-four hours post- initial diagnostic investigation in a patient
operatively she complains that she is with suspected Cushing's syndrome?
unable to straighten her fingers. She also A) Random plasma cortisol measurement
describes tingling around her mouth. What B) Short Synacthen test
pattern of abnormalities is most likely to C) Overnight dexamethasone suppression
be seen on blood testing? test
A) High calcium, high phosphate, low D) Oral glucose tolerance test
parathyroid hormone E) Twenty-four-hour urine metanephrines
B) High calcium, low phosphate, high
parathyroid hormone
C) Low calcium, high phosphate, low
parathyroid hormone

71
Question 30. Which of the following with IV dextrose. She has had three
clinical and serum biochemical parameters similar episodes over the preceding 6
is most consistent with Addison's disease? months. What is the most likely diagnosis?
A) High blood pressure, low sodium, high A) Insulinoma
potassium B) Alcohol excess
B) Low blood pressure, low sodium, high C) Diabetes mellitus
potassium D) Administration of exogenous insulin
C) Low blood pressure, high sodium, high E) Sulphonylurea ingestion
potassium
D) Low blood pressure, high sodium, low Question 34. Excessive secretion of which
potassium hormone is responsible for carcinoid
E) High blood pressure, high sodium, high syndrome?
potassium A) Gastrin
B) Vasoactive intestinal peptide (VIP)
Question 31. Which of the following C) Glucagon
drugs is the most appropriate medical D) 5-hydroxytryptamine (5-HT)
treatment for hypertension associated with E) Somatostatin
Conn's syndrome?
A) Bendroflumethiazide Question 35. A 29-year-old man is
B) Atenolol referred to the medical outpatient clinic
C) Perindopril with a 3-month history of polyuria. He
D) Amlodipine describes increasing thirst and drinks large
E) Spironolactone volumes of fluid. On examination he
appears euvolaemic. A water deprivation
Question 32. A 32-year-old man describes test reveals the following results:
a 4-month history of recurrent headaches
associated with sweating and palpitations.
During one of these episodes he is found
to have a blood pressure of 210/105
mmHg and pulse of 112 bpm. A 24-hour
urine collection shows elevated The most likely diagnosis is:
metanephrine and normetanephrine A) Diabetes mellitus
excretion. An abdominal CT scan shows a B) Nephrogenic diabetes insipidus
large left adrenal mass. What is the most C) Cranial diabetes insipidus
appropriate first-line antihypertensive D) Addison's disease
agent? E) Syndrome of inappropriate antidiuretic
A) Atenolol hormone secretion (SIADH)
B) Phenoxybenzamine
C) Ramipril Question 36. A 43-year-old man is
D) Nifedipine referred to the endocrinology clinic with a
E) Bendroflumethiazide 1-year history of headaches and sweating.
His shoe size has increased and his
Question 33. A 17-year-old woman with a wedding ring no longer fits. Which of the
strong family history of diabetes mellitus following is the investigation of choice in
is brought to the hospital accident and establishing the cause of his symptoms?
emergency department after collapsing at A) Oral glucose tolerance test
home. On arrival she has a Glasgow Coma B) CT head scan
Score of 9/15. Laboratory blood glucose is C) MRI head scan
1.2 mmol/l. Plasma insulin levels are D) Insulin tolerance test
elevated and C-peptide levels are low. Her E) 24-hour urinary cortisol measurement
condition improves following treatment

72
Chapter 20 DIABETES MELLITUS
Answers page 195

Question 1. Which one of the following is Question 5. A 58-year-old lady presents to


a physiological action of insulin? her GP with a 6-month history of general
A) Increased lipolysis malaise and fatigue. The GP does routine
B) Increased gluconeogenesis blood tests and finds her to have a fasting
C) Increased ketogenesis glucose of 7.5 mmol/l. A repeat fasting
D) Decreased glycogenolysis glucose 1 month later is 7.4 mmol/l. The
E) Decreased protein synthesis most appropriate next step would be to:
A) Repeat a fasting glucose measurement
Question 2. All of the following are in 1 month's time
causes of secondary diabetes mellitus B) Perform an oral glucose tolerance test
except one. Which is the exception? C) Measure C-peptide levels
A) Acromegaly D) Explain to the patient that she has
B) Corticosteroid treatment diabetes mellitus
C) Haemochromatosis E) Reassure the patient that she has normal
D) Phaeochromocytoma glucose tolerance
E) Addison's disease
Question 6. What is the most common
Question 3. All of the following cause of glycosuria?
statements about type 1 diabetes are true A) Type 1 diabetes
except one. Which is the exception? B) Type 2 diabetes
A) Viral infection has been implicated in C) A low renal threshold for glucose in an
the aetiology otherwise healthy individual
B) There is an association with pernicious D) Urine testing immediately after a meal
anaemia in an otherwise healthy individual
C) There is destruction of the pancreatic E) Pyelonephritis
islet ß-cells
D) Treatment with insulin is essential Question 7. What is the most informative
E) Concordance rates between method of assessing a patient's glycaemic
monozygotic twins are greater than 90% control in the diabetic outpatient clinic?
A) Performing urine dipstick analysis for
Question 4. All of the following glucose
statements except one are true of type 2 B) Performing urine dipstick analysis for
diabetes. Which is the exception? protein
A) It is frequently detected incidentally on C) Measuring plasma glucose
blood testing D) Measuring glycated haemoglobin
B) It is associated with the metabolic (HbA1C)
syndrome E) Careful analysis of the patient's dietary
C) Ketoacidosis is a common complication intake according to carbohydrate
D) There is an increased risk of ischaemic exchanges
heart disease
E) The prevalence is higher than that of
type 1 diabetes

73
Question 8. A 23-year-old man with type Question 10. Features of hypoglycaemia
1 diabetes presents to the hospital include all the following except one.
Accident and Emergency department with Which is the exception?
a 24-hour history of nausea, poor appetite A) Sweating
and feeling unwell. Initial investigations B) Kussmaul breathing
show the following results: C) Confusion
D) Coma
E) Tachycardia

Question 11. You are asked to review a


67-year-old lady on the ward with insulin-
treated type 2 diabetes, who is
complaining of nausea, sweating and
palpitations. Blood glucose strip testing
gives a reading of 2.6 mmol/l. What would
be the most appropriate initial
management?
What is the most immediate management A) Intravenous dextrose
priority? B) Intramuscular glucagon
A) Administration of intravenous insulin C) Intravenous glucagon
and intravenous saline D) Oral carbohydrate
B) Administration of intravenous insulin E) Taking a blood sample for a formal
and intravenous potassium laboratory measurement of blood glucose
C) Administration of intravenous insulin
and broad-spectrum antibiotics Question 12. A 73-year-old lady with type
D) Administration of intravenous insulin 2 diabetes presents to hospital with an
and intravenous sodium bicarbonate acute myocardial infarction (MI). She
E) Central line insertion to enable accurate normally takes metformin. What is the
monitoring of fluid balance most appropriate way to maintain
glycaemic control in the peri-infarct
Question 9. A 63-year-old man with period?
diabetes presents to the hospital Accident A) Continue metformin at the current dose
and Emergency department with a 4-day B) Continue metformin at a higher dose
history of feeling tired and unwell. He C) Stop metformin and monitor blood
complains of severe thirst. On examination glucose readings over the first 24 hours
he appears dehydrated with a pulse of 116 D) Stop metformin and commence a
bpm and blood pressure of 114/68 mmHg. sulphonylurea
After routine investigations, he is found to E) Stop metformin and commence
be in a hyperosmolar non-ketotic intravenous insulin
hyperglycaemic state. Which one of the
following statements regarding this Question 13. A 29-year-old pregnant
condition is true? woman is referred to clinic with a
A) Higher doses of insulin are usually diagnosis of gestational diabetes. What
required compared to diabetic ketoacidosis would the best treatment be?
B) Subcutaneous heparin should be A) Gliclazide alone
administered within the first 24 hours B) Metformin alone
C) There is usually a profound metabolic C) Insulin alone
acidosis D) Insulin plus gliclazide
D) People with types 1 and 2 diabetes are E) Insulin plus metformin
affected equally
E) Large volumes of intravenous sodium
bicarbonate are often needed

74
Question 14. What is the correct peri- Question 18. What is the biggest cause of
operative management of an insulin- mortality in diabetic patients?
treated type 2 diabetic patient undergoing A) Ischaemic heart disease
a laparotomy? B) Hypoglycaemia
A) Position the patient at the head of the C) Renal failure
operation list and omit insulin until after D) Diabetic ketoacidosis
the operation E) Infection
B) Administer his or her usual
subcutaneous insulin pre-operatively and Question 19. Which of the following
monitor blood glucose fundal abnormalities is not typical of
C) Administer half his or her usual diabetic retinopathy?
subcutaneous dose pre-operatively and A) Hard exudates
monitor blood glucose B) Microaneurysms
D) Start an intravenous infusion of C) Flame haemorrhages
dextrose and insulin in place of the D) Cotton wool spots
patient's usual therapy E) Venous loops and beading
E) Give him or her one dose of an oral
anti-diabetic agent to prevent high blood Question 20. Which one of the following
glucose levels peri-operatively retinal appearances is a key feature of pre-
proliferative diabetic retinopathy?
Question 15. A patient with type 2 A) Microaneurysms with hard exudates
diabetes and a body mass index of 34 B) Papilloedema
kg/m2 has suboptimal glycaemic control C) Neovascularisation
with dietary therapy alone. Which of the D) Venous beading with cotton wool spots
following would be the most appropriate E) Pre-retinal haemorrhage
first-line pharmacological treatment?
A) Gliclazide Question 21. A 61-year-old man with type
B) Metformin 2 diabetes has evidence of background
C) Rosiglitazone diabetic retinopathy when examined at the
D) Insulin diabetic clinic. He normally takes
E) Metformin plus insulin gliclazide, metformin and lisinopril. What
is the most appropriate management?
Question 16. Which of the following A) Urgent referral to an ophthalmologist
statements about the thiazolidinediones for laser photocoagulation
(glitazones) is false? B) Routine referral to an ophthalmologist
A) They act by binding to peroxisome C) Regular follow-up in the diabetic clinic
proliferator-activated receptor-W (PPAR-W) but no referral to ophthalmology
B) They do not increase insulin secretion D) Stop metformin and follow up in the
C) They are particularly useful in diabetic diabetic clinic
patients with left ventricular dysfunction E) Stop lisinopril and follow up in the
D) They are useful agents in obese type 2 diabetic clinic
diabetic patients
E) They rarely cause hypoglycaemia Question 22. In the absence of
contraindications, which of the drugs in
Question 17. Which one of the following the following list is the optimal anti-
drugs is correctly paired with its hypertensive agent in a diabetic patient
predominant side-effect? with microalbuminuria?
A) Metformin X hypoglycaemia A) Bendroflumethiazide (bendrofluazide)
B) Metformin X weight gain B) Atenolol
C) Gliclazide X lipohypertrophy C) Ramipril
D) Glipizide X hypoglycaemia D) Doxazosin
E) Rosiglitazone X lactic acidosis E) Amlodipine

75
Question 23. All of the following E) It is typically associated with painful
statements except one are true of diabetic foot ulcers
neuropathy. Which is the exception?
A) The incidence can be reduced by strict Question 24. Which one of the following
glycaemic control is a feature of diabetic autonomic
B) Motor, sensory and autonomic nerves neuropathy?
can all be involved A) VI nerve palsy
C) The majority of patients are B) Carpal tunnel syndrome
asymptomatic C) Diabetic amyotrophy
D) Charcot joints are a recognised D) Gastroparesis
complication E) Stocking distribution of sensory loss in
the legs

76
Chapter 21 ALIMENTARY TRACT AND
PANCREATIC DISEASE
Answers page 200

Question 1. Which one of the following Question 5. A 55-year-old lady with a 3-


features on clinical examination would month history of progressive dysphagia
suggest a kidney rather than a spleen as and weight loss is investigated with a
the cause of a palpable left upper quadrant barium swallow:
mass?
A) Cannot palpate above it
B) Presence of a notch
C) Dullness to percussion
D) Extends towards the right iliac fossa
E) Rounded mass

Question 2. The major enzyme


responsible for protein digestion in the
small bowel is:
A) Trypsin What is the most likely diagnosis?
B) Amylase A) Achalasia
C) Lipase B) Oesophageal candidiasis
D) Cholecystokinin C) External oesophageal compression by
E) Pepsin bronchial carcinoma
D) Oesophageal carcinoma
Question 3. The enzyme amylase is E) Myasthenia gravis
principally responsible for the breakdown
of which one of the following? Question 6. Which of the following tests
A) Protein is most useful in the assessment of
B) Cholesterol pancreatic exocrine function?
C) Glycogen A) Faecal elastase
D) Bile salts B) CT scan of abdomen
E) Triglycerides C) 3-day faecal fat
D) Fasting blood glucose
Question 4. The gut hormone E) Lactose hydrogen breath test
cholecystokin (CCK) produces all of the
following effects except one. Which is the Question 7. The urea breath test is useful
exception? in the diagnosis of which one of the
A) Gallbladder contraction following conditions?
B) Increased secretion of pancreatic A) Bacterial overgrowth
enzymes B) Helicobacter pylori infection
C) Satiety C) Lactose intolerance
D) Increased gastric acid secretion D) Fat malabsorption
E) Relaxation of the sphincter of Oddi E) Coeliac disease

77
Question 8. In which of the following Question 12. A patient presents with acute
causes of dysphagia would oesophageal lower gastrointestinal bleeding and
manometry be most likely to provide the features of shock. Which of the following
diagnosis? conditions would be least likely to account
A) Myasthenia gravis for this presentation?
B) Bulbar palsy A) Diverticular disease
C) Pseudobulbar palsy B) Angiodysplasia
D) Carcinoma of oesophagus C) Meckel's diverticulum
E) Achalasia D) Bowel ischaemia
E) Anal fissure
Question 9. An otherwise fit and well 40-
year-old man presents with a history of Question 13. In the assessment of
troublesome dyspepsia. Which of the diarrhoea, which one of the following
following features in his presentation clinical features most strongly suggests a
would be the strongest indication for colonic aetiology?
proceeding to upper gastrointestinal A) Right iliac fossa pain
endoscopy? B) Undigested food in stool
A) Poor response to antacids C) Blood and mucus in stool
B) Symptoms worse at night D) Large-volume stool
C) Family history of peptic ulcer disease E) Abdominal bloating
D) Symptoms persistent for > 3 months
prior to treatment Question 14. Which one of the following
E) Associated difficulty in swallowing physical consequences of malabsorption is
incorrectly matched with the
Question 10. Which of the following corresponding vitamin or mineral
drugs is least likely to be implicated in a deficiency?
patient presenting with vomiting? A) Acrodermatitis enteropathica Zinc
A) Erythromycin B) Purpura and bruising Vitamin K
B) Codeine phosphate C) Koilonychia Iron
C) Atenolol D) Peripheral neuropathy Vitamin C
D) Cisplatin E) Night blindness Vitamin A
E) Digoxin
Question 15. A patient recently
Question 11. In patients presenting with commenced on a new medication
acute upper gastrointestinal haemorrhage, complains of constipation. Which of the
all of the following factors except one are following drugs is most likely to be
associated with an increased risk of death. implicated?
Which is the exception? A) Erythromycin
A) Pulse > 100 bpm B) Digoxin
B) Mallory-Weiss tear as the cause of C) Orlistat
bleeding D) Metformin
C) Visible but non-bleeding vessel present E) Verapamil
at endoscopy
D) Age over 60 years Question 16. Which of the following
E) Associated renal failure conditions is least likely to cause oral
ulceration?
A) Coeliac disease
B) Kaposi's sarcoma
C) Behçet's disease
D) Herpes simplex virus
E) Psoriasis

78
Question 17. Which one of the following A barium swallow is performed:
factors does not predispose to gastro-
oesophageal reflux disease (GORD)?
A) Pregnancy
B) High intake of coffee
C) Weight loss
D) Delayed gastric emptying
E) Hiatus hernia

Question 18. Which one of the following


statements is true of Barrett's oesophagus?
A) It refers to neoplastic change in the
normal squamous cells of the lower What is the most likely diagnosis?
oesophagus A) Diffuse oesophageal spasm
B) The majority of patients with the B) Achalasia
condition ultimately develop carcinoma of C) Systemic sclerosis
the oesophagus D) Carcinoma of oesophagus
C) It is associated with increased alcohol E) Myasthenia gravis
intake
D) Patients with the condition should Question 21. Which one of the following
undergo regular endoscopic surveillance statements is true concerning carcinoma of
with biopsies the oesophagus?
E) Antireflux surgery has been shown to A) The overall 5-year survival is
induce regression of the condition approximately 40%
B) CT is superior to endoscopic ultrasound
Question 19. A 34-year-old man presents in local staging
with severe retrosternal burning C) A history of painful, intermittent
discomfort, worse at night and after large dysphagia for solids and liquids is the
meals and eased by various 'over-the- usual presenting complaint
counter' antacid remedies. There are no D) The presence of hoarseness suggests
other symptoms and his general health is mediastinal invasion
good. Which of the following statements E) Symptomatic relief of swallowing
concerning his condition is true? difficulties is best achieved by palliative
A) Upper gastrointestinal endoscopy is radiotherapy
mandatory to exclude Barrett's oesophagus
B) A history of weight gain prior to the Question 22. A 45-year-old male presents
onset of symptoms would raise doubt as to with a 10-month history of recurrent,
the diagnosis 'gnawing' epigastric pain. The pain does
C) Proton pump inhibitors (PPIs) and H2- not radiate and can be absent for up to a
receptor antagonists are equally effective month at a time but, when present, seems
in relieving symptoms and inducing to be relieved by eating. He has also
healing of oesophagitis experienced occasional vomiting. With
D) Recurrence of symptoms following which of the following diagnoses would
cessation of medical treatment is common his symptoms fit best?
E) Antireflux surgery is rarely successful A) Chronic peptic ulcer
in relieving symptoms B) Biliary colic
C) Chronic pancreatitis
Question 20. A 57-year-old woman D) Mesenteric ischaemia
presents with a 12-month history of slowly E) Crohn's disease
progressive dysphagia for solids, eased by
drinking liquids. She also experiences
intermittent episodes of severe chest pain.

79
Question 23. The patient from the Question 26. Which of the following is
previous question (Q22) who has least likely to occur as a complication of
suspected peptic ulcer disease has an partial gastrectomy?
endoscopy performed. This reveals a A) Weight gain
chronic duodenal ulcer. The CLO test is B) Flushing, sweating and lightheadedness
positive. Which one of the following after meals with large carbohydrate
statements regarding further management content
is false? C) Postprandial diarrhoea and abdominal
A) Treatment with acid-suppressant discomfort
therapy and H. pylori eradication is likely D) Anaemia
to cure the ulcer E) Early satiety
B) Advice should be given that cigarette
smoking increases the risk of ulcer Question 27. A 32-year-old woman
complications and reduces the chance of presents with severe recurrent epigastric
healing pain. Endoscopy reveals multiple ulcers in
C) It is unlikely the patient will require the duodenum, stomach and oesophagus.
surgery for the ulcer There is no history of ingestion of non-
D) The patient should be given specific steroidal anti-inflammatory drugs
advice on dietary restriction (NSAIDs) and tests for H. pylori are
E) Non-steroidal anti-inflammatory drugs negative. Serum gastrin levels are grossly
(NSAIDs) should be avoided elevated. What is the likely diagnosis?
A) Covert NSAID abuse
Question 24. Which one of the following B) Carcinoid syndrome
statements concerning the bacterium C) Zollinger-Ellison syndrome
Helicobacter pylori is true? D) Crohn's disease
A) It is a Gram-positive rod E) Biliary reflux
B) It produces an enzyme called 'urease'
that lowers the surrounding pH Question 28. All of the following except
C) It exclusively colonises gastric-type one are recognised risk factors for gastric
epithelium carcinoma. Which is the exception?
D) Strains expressing the enzyme cagA A) Autoimmune chronic gastritis
are less often associated with disease B) Zollinger-Ellison syndrome
E) On colonising the stomach it induces C) Cigarette smoking
increased secretion of somatostatin from D D) H. pylori infection
cells E) Familial adenomatous polyposis (FAP)

Question 25. All of the following Question 29. Which one of the following
statements concerning the consequences of conditions is not associated with coeliac
H. pylori infection are true except one. disease?
Which is the exception? A) Type 1 diabetes mellitus
A) The majority of colonised individuals B) Pernicious anaemia
remain healthy and asymptomatic C) Conn's syndrome
B) It is the most common cause of chronic D) Primary biliary cirrhosis
gastritis E) Dermatitis herpetiformis
C) Around 40% of patients with duodenal
ulcer are infected with H. pylori
D) It is a recognised risk factor for the
development of gastric cancer
E) Some gastric lymphomas may be cured
solely by H. pylori eradication

80
Question 30. The differential diagnosis of Question 34. Which one of the following
subtotal villous atrophy includes all of the is not an adverse prognostic factor in acute
following conditions except one. Which is pancreatitis?
the exception? A) White blood cell count 16 ×
A) Tropical sprue 109/l (normal range 4-11 × 109/l)
B) AIDS enteropathy B) Urea 20 mmol/l (normal range 2.5-6.6
C) Giardiasis mmol/l)
D) Dermatitis herpetiformis C) Albumin 28 g/l (normal range 35-50
E) Mesenteric ischaemia g/l)
D) Amylase 1000 U/l (normal range < 100
Question 31. Which of the following U/l)
statements is true concerning coeliac E) Glucose 14 mmol/l (normal range 3.6-
disease in adults? 5.8 mmol/l)
A) Patients are intolerant of maize and rice
B) Peak onset is in the third decade Question 35. Concerning the management
C) Almost all patients who do not follow a of acute pancreatitis, which one of the
gluten-free diet are symptomatic following statements is true?
D) Adhering to a strict gluten-free diet A) Hypocalcaemia should be corrected
increases the risk of metabolic bone urgently if the corrected calcium falls
disease below 2.0 mmol/l
E) There is an increased risk of T-cell B) Enteral feeding should be started at an
lymphoma early stage in patients with severe
pancreatitis
Question 32. A 55-year-old male presents C) Prophylaxis of thromboembolism is
with a 6-hour history of severe, constant inadvisable in the first 48 hours
upper abdominal pain radiating to the D) In gallstone pancreatitis, emergency
back. He drinks approximately 80 units of endoscopic retrograde
alcohol per week. On examination there is cholangiopancreatography (ERCP) and
marked epigastric tenderness but no stone extraction should be avoided if there
guarding or rebound tenderness. Which of is evidence of ascending cholangitis
the following tests is likely to be most E) Opiate analgesia is usually not require
useful in reaching a positive diagnosis?
A) Plain abdominal X-ray Question 36. Which one of the following
B) C-reactive protein would be considered an unexpected
C) Erect chest X-ray clinical feature in chronic pancreatitis?
D) ECG A) Steatorrhoea
E) Serum amylase B) Weight loss
C) Pain relieved by drinking alcohol
Question 33. All of the following except D) Recurrent hypoglycaemia
one are uncommon but recognised causes E) Skin pigmentation over the back
of acute pancreatitis. Which is the
exception? Question 37. A 56-year-old lady presents
A) Azathioprine therapy with a 4-week history of weight loss and
B) Mumps progressively worsening jaundice with
C) Hypocalcaemia pruritus. On examination she is deeply
D) Hypothermia icteric and has a palpable gallbladder.
E) Seat belt injury What is the likeliest diagnosis?
A) Chronic cholecystitis
B) Hepatocellular carcinoma
C) Ascending cholangitis
D) Carcinoma of pancreas
E) Amoebic abscess

81
Question 38. Which one of the following Question 42. With reference to the patient
statements regarding carcinoma of the in the previous question (Q41) who has
pancreas is true? bloody diarrhoea and lower abdominal
A) The majority of tumours are amenable pain, stool is negative for culture,
to potentially curative resection Clostridium difficile toxin, ova and cysts.
B) The body of the pancreas is the most Sigmoidoscopy and rectal biopsy reveal
common site of origin typical features of ulcerative colitis. In the
C) Coeliac plexus neurolysis is useful in assessment of the severity of this attack,
relieving pruritus all of the following except one suggest
D) The liver is a common site for severe disease. Which is the exception?
metastatic deposits A) 10 bowel motions per day
E) The prognosis is better than for B) Stool volume 200 g/24 hrs
carcinoma of the ampulla of Vater C) Temperature of 38°C on 2 consecutive
days
Question 39. Which one of the following D) Pulse 110 bpm
pathological features would favour a E) Erythrocyte sedimentation rate 80
diagnosis of ulcerative colitis over Crohn's mm/hr
disease?
A) Presence of fistulae Question 43. The patient from the
B) Proximal colitis with rectal sparing previous question (Q42) has several
C) Lymphoid aggregates and clinical and laboratory parameters
microgranulomas indicating severe ulcerative colitis. Which
D) Ulceration within the small intestine one of the following is true regarding the
E) Inflammation limited to the mucosa management of this attack?
A) Nutritional support, if required, should
Question 40. Which of the following be provided intravenously rather than
would be least expected to provoke a enterally
relapse ('flare') of ulcerative colitis? B) The patient should be treated with
A) Loss of job intravenous corticosteroids
B) Cigarette smoking C) Surgery should not be considered until
C) Ankle sprain treated with oral he has had at least 10 days of maximal
diclofenac medical therapy
D) Viral laryngitis D) Prophylaxis of venous
E) Course of antibiotics for falsely thromboembolism with heparin should be
suspected urinary tract infection avoided
E) Sulfasalazine overcomes the need for
Question 41. A 19-year-old male presents surgery in around 80% of patients who fail
with a 10-day history of bloody diarrhoea to respond to corticosteroids
and lower abdominal pain associated with
fever, malaise and anorexia. Which of the
following investigations would be least
helpful in establishing a diagnosis?
A) Stool culture and microscopy
B) Sigmoidoscopy and rectal biopsy
C) Blood cultures and serological tests of
infection
D) Serum albumin
E) Examination of stool for Clostridium
difficile toxin

82
Question 44. On the fourth day of medical Question 46. Which one of the following
treatment the patient from the previous systemic complications of inflammatory
question (Q43) does not show any signs of bowel disease is not related to the activity
improvement. A plain abdominal X-ray is of bowel disease?
performed: A) Conjunctivitis
B) Primary sclerosing cholangitis
C) Erythema nodosum
D) Arthralgia
E) Pyoderma gangrenosum

Question 47. Concerning the management


of Crohn's disease, which one of the
following statements is false?
A) Due to high rates of relapse, infliximab
therapy should be combined with disease-
modifying agents
B) Maintenance of remission is improved
by cessation of cigarette smoking
Which one of the following statements is C) Corticosteroids are preferred to
true? methotrexate for maintaining remission
A) It is unusual for this complication to D) Surgical intervention should be
occur in the first attack of colitis conservative to minimise loss of viable
B) The original diagnosis of ulcerative intestine
colitis was probably incorrect E) Azathioprine is an effective
C) The patient should be referred for maintenance treatment
urgent colectomy
D) No further action should be taken but Question 48. Which one of the following
the X-ray should be repeated in 24 hours clinical features is least suggestive of
E) Azathioprine treatment may avoid the irritable bowel syndrome (IBS)?
need for surgery in patients with this A) Colicky abdominal pain relieved by
complication defaecation
B) Sensation of incomplete defaecation
Question 45. Which one of the following C) Symptoms disturbing sleep
statements concerning intestinal D) Alternating diarrhoea and constipation
complications of inflammatory bowel E) Passage of mucus
disease (IBD) is incorrect?
A) In ulcerative colitis, colonic perforation Question 49. Regarding the management
may occur without the development of of irritable bowel syndrome (IBS), which
toxic megacolon one of the following statements is true?
B) In Crohn's disease, enterovesical A) Patients with constipation as the
fistulation may present with pneumaturia predominant symptom benefit most from
C) The risk of developing colon cancer in tricyclic antidepressant therapy
ulcerative colitis is related to both the B) Elimination diets are the mainstay of
extent and the duration of colitis treatment
D) Both conditions may present with acute C) Probiotics are of proven benefit
life-threatening haemorrhage D) Reassurance alone may lead to
E) Diarrhoea in ulcerative colitis may be resolution of symptoms
due to enteroenteric fistulae E) All patients should be advised to
increase dietary fibre

83
Question 50. A 75-year-old lady presents Question 52. With regard to colonic
with a 2-hour history of sudden-onset polyps, which one of the following
severe abdominal pain and per rectum statements is true?
bleeding. Other than a stroke 2 years ago A) They usually present with lower
she keeps well. She has been on aspirin gastrointestinal bleeding or anaemia
and ramipril since her stroke and takes B) The size of the polyp has no bearing on
digoxin for an irregular heart rhythm. On the risk of malignant change
examination she is very distressed; her C) Asymptomatic polyps found at
abdomen is soft with no evidence of colonoscopy should be biopsied but not
peritonism. Bowel sounds are audible. An removed
arterial blood gas shows a metabolic D) Polyps with a villous architecture carry
acidosis. What is the likeliest diagnosis? a higher risk of malignant change than
A) Ruptured aortic aneurysm those with tubular architecture
B) Acute small bowel ischaemia E) The polyps in Peutz-Jeghers syndrome
C) Perforated peptic ulcer are predominantly colonic adenomas
D) Diverticulitis
E) Irritable bowel syndrome Question 53. All of the following except
one are recognised risk factors for
Question 51. A 73-year-old lady is colorectal cancer. Which is the exception?
admitted to hospital with a severe A) Presence of colorectal adenomas
community-acquired pneumonia. She is B) Smoking
treated with intravenous antibiotics and on C) High dietary calcium
the sixth day of her admission develops D) Acromegaly
profuse diarrhoea associated with E) Long-standing extensive ulcerative
cramping lower abdominal pain. What test colitis
is most likely to reveal the cause of her
diarrhoea? Question 54. A 68-year-old lady presents
A) Stool microscopy for ova and parasites to her GP with tiredness and weight loss
B) Barium enema and is found to have iron deficiency
C) Stool culture for Escherichia coli anaemia. On examination there is a
D) Legionella serology palpable mass in the right iliac fossa.
E) Stool analysis for Clostridium difficile Which of the following is the most
toxin appropriate next investigation to establish
the diagnosis?
A) Flexible sigmoidoscopy
B) Abdominal CT scan
C) Abdominal ultrasound
D) Colonoscopy
E) Serum carcinoembryonic antigen
(CEA)

84
Chapter 22 LIVER AND BILIARY TRACT
DISEASE
Answers page 208

Question 1. Which of the following is not Question 4. Of the following conditions,


a function of the liver? which is the most likely cause of jaundice
A) Synthesis of clotting factors accompanied by a seven-fold elevation of
B) Production of vitamin K alanine aminotransferase and two-fold
C) Production of glucose during fasting elevation of alkaline phosphatase?
D) Storage of copper A) Viral hepatitis
E) Excretion of potentially toxic B) Acute cholecystitis
compounds C) Gilbert's syndrome
D) Haemolysis
Question 2. In a hepatic acinus, zone 3 is E) Pancreatic carcinoma
closest to the central hepatic vein. Which
of the following statements concerning Question 5. Which one of the following is
zone 3 is true? most suggestive of haemolysis as the
A) It is exposed to the highest cause of jaundice?
concentrations of toxins A) Pale stools
B) It is closer to the bile duct than zones 1 B) Raised alanine aminotransferase (ALT)
and 2 C) Presence of spider naevi
C) It is the primary site of bile salt D) Bilirubinuria
formation E) Raised reticulocyte count
D) It has a good oxygen supply relative to
zones 1 and 2 Question 6. Which one of the following
E) It is the primary site of lipolysis in the coagulation factors depends on vitamin K
liver for complete synthesis in the liver?
A) Factor IV
Question 3. A hospital inpatient is noted B) Factor V
to have an elevated gamma-glutamyl C) Factor VIII
transferase (GGT) with otherwise normal D) Factor IX
liver function tests. She is taking the E) Factor XI
following regular medications. Which one
is most likely to have caused this Question 7. Which of the following tests
abnormality? provides the most information about
A) Omeprazole hepatic synthetic function within the last
B) Paracetamol 12 hours?
C) Phenytoin A) Activated partial thromboplastin time
D) Ibuprofen (APTT)
E) Sodium valproate B) Albumin
C) Fibrinogen
D) Urea
E) Prothrombin time (PT)

85
Question 8. Which one of the following is D) Rhabdomyolysis
a contraindication to percutaneous liver E) Dehydration
biopsy?
A) Chronic obstructive pulmonary disease Question 12. A 47-year-old man with a
(COPD) with forced expiratory volume in history of alcoholic cirrhosis is well
1 second (FEV1)/forced vital capacity known on the general medical ward after
(FVC) of 37% recurrent admissions for drainage of
B) Failure to comply with fasting ascites. He presents with confusion, fever
requirements (38.5°C), ascites and abdominal pain.
C) Platelet count of 190 Which of the following investigations is
D) Prothrombin time prolonged by 2 most important to establish the diagnosis?
seconds A) Stool culture and microscopy for ova,
E) Family history of haemochromatosis cysts and parasites
B) Diagnostic laparoscopy
Question 9. A jaundiced patient has the C) Abdominal ultrasound scan
following liver function test (LFT) results: D) CT scan of abdomen
E) Ascitic fluid white cell count

Question 13. Which of the following is


the first-line drug of choice to treat chronic
Which one of the following is the most ascites secondary to hepatic cirrhosis?
appropriate initial investigation to identify A) Furosemide
the underlying problem? B) Spironolactone
A) Endoscopic retrograde C) Digoxin
cholangiopancreatography (ERCP) D) Bendroflumethiazide
B) Hepatic portal venography E) Vasopressin
C) Abdominal X-ray
D) Abdominal ultrasound Question 14. Which of the following
E) Liver biopsy blood results confers the worst prognosis
on a patient with acute liver failure
Question 10. Only one of the following secondary to paracetamol overdose?
options associates the disease with the A) Prothrombin time (PT) > 100 secs
correct initial diagnostic test. Which is it? B) Aspartate aminotransferase (AST) > 6
A) Wilson's disease Transferrin saturation times normal
B) Primary biliary cirrhosis C) Alanine aminotransferase (ALT) > 2.5
(PBC) Antimitochondrial antibody (AMA) times normal
C) Alpha1-antitrypsin D) Platelets < 100 × 109/l
deficiency Caeruloplasmin E) Albumin < 35 g/dl
D) Primary sclerosing cholangitis Anti-
endomysial antibody Question 15. Which of the following
E) Coeliac disease 24-hour urinary copper examination findings is not characteristic
of a palpable spleen?
Question 11. A patient presents to her GP A) A palpable mass that the examiner
after noticing that the whites of her eyes cannot get above
have become yellow. Direct questioning B) Dull to percussion
also reveals that her urine has been very C) May be balloted
dark for several days. Which of the D) Moves with respiration
following diagnoses is the most likely E) Felt superficially
explanation for these symptoms?
A) Carotenaemia
B) Cholestatic jaundice
C) Hyperbilirubinaemia secondary to
Gilbert's syndrome

86
Question 16. Which of the following D) No vaccine is available
statements about primary biliary cirrhosis E) It may cause cirrhosis
is true?
A) It is more common in men Question 21. In a patient with suspected
B) Transplant is contraindicated because hepatitis B infection, which of the
of the high risk of disease recurrence following serological tests is most
C) Pruritus is a more common presenting suggestive of previous exposure to
feature than jaundice hepatitis B virus?
D) A positive antinuclear antibody is A) HBsAg alone
diagnostic B) HBsAg and HBeAg
E) Immunosuppression prevents disease C) HBsAb alone
progression D) Anti-HBc IgM alone
E) Anti-HBc IgG and HBsAb
Question 17. Which of the following
drugs is most useful in a patient with a Question 22. A 23-year-old nurse is found
suspected variceal haemorrhage, where to have abnormal liver function tests, and
urgent upper gastrointestinal endoscopy is serological tests for hepatitis B are
not available? performed. He has a positive titre for anti-
A) Noradrenaline (norepinephrine) HBs but no other positive serological tests.
B) Activated protein C What is the significance of this finding?
C) Dopamine A) It indicates a hepatitis B chronic carrier
D) Terlipressin state
E) Ciprofloxacin B) It suggests acute hepatitis B
C) It effectively rules out hepatitis B as the
Question 18. Which of the following cause of his illness
viruses is not capable of causing infection D) It is consistent with convalescence
without concurrent infection with another following acute hepatitis B
hepatitis virus? E) It indicates concurrent infection with
A) Hepatitis A hepatitis D
B) Hepatitis B
C) Hepatitis C Question 23. Which one of the following
D) Hepatitis D statements about infection with hepatitis B
E) Hepatitis E virus (HBV) is false?
A) The majority of patients make a full
Question 19. Which one of the following recovery from acute infection
statements about hepatitis A virus (HAV) B) Chronic infection may lead to
infection is true? hepatocellular carcinoma
A) It is commonly spread by sexual C) HBeAg-negative patients respond less
transmission well to treatment
B) It may be spread by the faecal-oral D) Antiviral therapy usually results in
route resistant mutants of HBV
C) A chronic carrier state occurs in a E) Treatment with interferon is ineffective
minority of infected patients in chronic infection
D) No vaccine is available
E) Anti-HAV of IgG type is useful Question 24. Which one of the following
diagnostically statements is true of hepatitis C infection?
A) It is usually sexually transmitted
Question 20. Which one of the following B) A minority of patients exposed to the
statements about hepatitis B infection is virus will become chronically infected
true? C) Once infected, cure is impossible
A) It may be spread in faeces D) Liver transplantation is contraindicated
B) The route of infection does not affect due to the risk of reinfection of the
clinical course transplanted liver
C) It is caused by an RNA virus E) Acute infection is usually
asymptomatic
87
Question 25. Which of the following regular ibuprofen. Which of the following
statements about alcoholic liver disease provides the best unifying diagnosis?
(ALD) is true? A) Wilson's disease
A) Once cirrhosis develops, abstinence B) Diabetes insipidus
does not affect prognosis C) Acute viral hepatitis
B) There is a genetic predisposition D) Infectious mononucleosis
C) Binge drinkers are more likely to E) Hereditary haemochromatosis
develop ALD than those with a consistent
intake Question 29. With reference to the patient
D) The presence of fatty liver (steatosis) in the previous question (Q28) who has
on biopsy suggests an alternative diagnosis hereditary haemochromatosis, which of
E) Transplantation is contraindicated the following is the most appropriate
initial treatment?
Question 26. A 34-year-old woman is A) Liver transplant
referred to the gastrointestinal clinic after B) Weekly venesection of 500 ml blood
her GP discovers abnormal liver function C) Warfarin
tests (LFTs). She denies alcohol abuse or D) Hepatic lobe resection
sexual risk factors. She received a blood E) Metformin
transfusion in the UK in 2002 following a
road traffic accident. Her past medical Question 30. A 25-year-old patient
history includes obesity (body mass index presents with an episode of acute hepatitis
= 42), type 2 diabetes mellitus and from which he recovers spontaneously and
anxiety/depression. What is the most is discharged from hospital with a
likely cause of her deranged LFTs? presumptive diagnosis of occult drug use.
A) Occult drug overdose A year later he presents with fatigue,
B) Alcoholic liver disease tremor at rest affecting both hands, and
C) Non-alcoholic fatty liver disease occasional wild, purposeless movements
D) Hepatitis B infection of his upper limbs. The admitting hospital
E) Hepatitis A infection doctor
notes an abnormality on examination of
Question 27. A 42-year-old hospital the eyes:
inpatient, with previously normal liver
function and weighing 73 kg, develops
abnormal liver function tests. She has
taken the following medications at
therapeutic doses in the last 28 days.
Which one is most likely to have caused
the problem?
A) Co-amoxiclav
B) Paracetamol
C) Temazepam
D) Morphine What is the most likely diagnosis?
E) Furosemide A) Ecstasy (MDMA) abuse
B) Budd-Chiari syndrome
Question 28. A 37-year-old man presents C) Wilson's disease
to his GP complaining of tiredness, D) Hereditary haemochromatosis
polyuria and thirst. On examination, he is E) Amanita phalloides poisoning
found to have 4 cm hepatomegaly and a
greyish pigmentation to his skin.
Urinalysis reveals a large amount of
glycosuria. His past medical history
includes osteoarthritis, for which he takes

88
Question 31. Which of the following Question 35. Which of the following
treatments is most appropriate for Wilson's patterns of symptoms is most
disease (hepatolenticular degeneration)? characteristic of acute blockage of the
A) Penicillamine cystic duct by a gallstone?
B) Desferrioxamine A) Recurrent epigastric pain, radiating to
C) Exchange transfusion the back, lasting 2 hours then subsiding
D) Venesection spontaneously
E) Smoking cessation B) Constant severe interscapular pain
associated with sweating and hypotension
Question 32. World-wide, which of the C) Constant severe epigastric pain and
following is the most important risk factor recurrent vomiting
for hepatocellular carcinoma? D) Intermittent burning epigastric pain
A) Alcoholic liver disease relieved by drinking milk
B) Haemochromatosis E) Crushing, heavy epigastric pain
C) Non-alcoholic steatohepatitis (NASH) radiating to the jaw associated with
D) Alpha1-antitrypsin deficiency sweating, nausea and a terror of imminent
E) Chronic viral hepatitis death

Question 33. An overweight 41-year-old Question 36. Which of the following


woman with a body mass index of 32 investigations is most appropriate for the
presents with a 5-day history of severe initial diagnosis of acute symptomatic
right upper quadrant pain. Initially the pain gallstone disease?
was intermittent, lasting for 2 hours then A) CT of the abdomen
subsiding, but for the past 12 hours it has B) Abdominal ultrasound scan
been constant. On examination she is C) Chest X-ray
pyrexial (38.5°C) and there is tenderness D) Plain abdominal X-ray
and rigidity in the right upper quadrant. E) Kidney-ureter-bladder (KUB) X-ray
Her white cell count is 18 × 109/l and C-
reactive protein 130 mg/l. Liver function Question 37. Which of the following is
tests (LFTs) and amylase are within the most important before performing a
normal range. Which of the following transjugular intrahepatic portosystemic
diagnoses is most likely? shunt insertion (TIPSS)?
A) Biliary colic A) Sclerotherapy to oesophageal varices
B) Acute pancreatitis B) Correction of any coagulopathy with
C) Acute cholecystitis fresh-frozen plasma
D) Mirizzi's syndrome C) Normalisation of urea and electrolytes
E) Choledocholithiasis D) Cessation of alcohol abuse
E) Psychiatric evaluation
Question 34. Which one of the following
hormones causes contraction of the Question 38. Which one of the following
gallbladder and release of bile acids in the statements about alcoholic liver disease is
fed state? true?
A) Insulin A) The characteristic pathological lesion is
B) Somotostatin microvesicular steatosis
C) Cholecystokinin B) Once fatty change has developed, the
D) Glucagon condition is irreversible
E) Noradrenaline (norepinephrine) C) Cholestasis and hepatitis are both
common presentations
D) Cirrhosis is an inevitable consequence
of chronic alcoholism
E) Beverages containing less than 10%
alcohol are unlikely to cause liver damage

89
Question 39. In which one of the Question 40. Which of the following
following conditions is hepatomegaly least factors is least likely to be a precipitant of
likely to be found on clinical examination? hepatic encephalopathy?
A) Alcoholic hepatitis A) Constipation
B) Hepatic venous outflow obstruction B) Upper gastrointestinal bleed
(Budd-Chiari syndrome) C) Hypokalaemia
C) Hepatic metastases D) Hypoglycaemia
D) Haemochromatosis E) Urinary tract infection
E) Chronic hepatitis C infection

90
Chapter 23 BLOOD DISORDERS
Answers page 213

Question 1. Which one of the following Question 5. The presence of target cells
statements regarding red blood cells is on the peripheral blood film is most likely
false? to be explained by which of the following
A) They are derived from megakaryocytes disorders?
B) Production is stimulated by A) Myelofibrosis
erythropoietin from the kidneyss B) Cirrhosis
C) Increased production is associated with C) Haemolytic uraemic syndrome
increased numbers of circulating D) Lead poisoning
reticulocytes E) Vitamin B12 deficiency
D) Mature red blood cells have a
biconcave disc shape Question 6. All of the following except
E) Mature red blood cells do not possess a one are causes of splenomegaly. Which is
nucleus or mitochondria the exception?
A) Chronic myeloid leukaemia
Question 2. Which one of the following B) Multiple myeloma
conditions is not typically associated with C) Alcoholic cirrhosis
an eosinophilia? D) Malaria
A) Asthma E) Myelofibrosis
B) Hay fever
C) Eczema Question 7. Which one of the following
D) Pneumococcal infection statements regarding the coagulation
E) Hookworm infection cascade is true?
A) Most clotting factors are synthesised in
Question 3. Which of the following drugs the bone marrow
is most likely to induce neutropenia? B) Coagulation by the extrinsic pathway is
A) Omeprazole initiated by the interaction of factor VII
B) Prednisolone with tissue factor
C) Carbimazole C) The intrinsic pathway is the major
D) Ciprofloxacin mechanism underlying coagulation in vivo
E) Alendronate D) Heparin exerts its anticoagulant effect
by binding to protein C
Question 4. Which one of the following E) The final step in the cascade is the
conditions is most likely to produce a conversion of prothrombin to thrombin
microcytosis on blood film examination?
A) Autoimmune haemolytic anaemia Question 8. Which one of the following
B) Alcohol excess clotting factors does not require vitamin K
C) Thalassaemia for its activation?
D) Vitamin B12 deficiency A) Factor II
E) Hypothyroidism B) Factor V
C) Factor VII
D) Factor IX
E) Factor X

91
Question 9. Which one of the following Question 13. Which one of the following
statements regarding the prothrombin time statements about ABO red cell groups is
is false? true?
A) It assesses the extrinsic pathway of the A) The most common blood group in the
coagulation cascade UK is group B
B) It is prolonged in patients taking B) Individuals with blood group AB will
warfarin have circulating antibodies against both A
C) It provides an accurate assessment of and B antigens
platelet function C) Infusion of group A red cells to a group
D) It is normal in haemophilia A O recipient should cause no more than a
E) It is prolonged in patients with factor X mild transfusion reaction
deficiency D) Patients with group B blood group can
only receive group B red cells
Question 10. In a patient presenting with a E) ABO incompatibility reactions are
bleeding disorder, which of the following mediated by complement
clinical features would be most suggestive
of a coagulation defect as opposed to a Question 14. Which one of the following
platelet disorder? statements regarding the Rhesus D (RhD)
A) Epistaxis blood group is false?
B) Haemarthrosis A) Around 85% of Caucasians express the
C) Menorrhagia RhD red cell antigen
D) Prolonged bleeding from superficial B) Anti-RhD antibodies of IgG type can
cuts cross the placenta
E) Purpura C) Transfusion of RhD-negative blood to
an RhD-positive woman may result in
Question 11. All of the following production of anti-RhD antibodies
statements regarding haemophilia A are D) Rhesus antibodies may cause severe
correct except one. Which is the anaemia and hyperbilirubinaemia in RhD-
exception? positive fetuses
A) The condition shows X-linked E) RhD-negative women of child-bearing
recessive inheritance potential exposed to RhD-positive blood
B) There is deficiency of factor VIII should receive anti-RhD immunoglobulin
C) Muscle haematomas are a characteristic (anti-D)
manifestation
D) Desmopressin aggravates the tendency Question 15. A 73-year-old woman on a
to bleeding regular transfusion programme is admitted
E) It may be complicated by secondary for a blood transfusion. Her baseline
osteoarthritis observations are satisfactory. Twenty
minutes into the transfusion she is noted to
Question 12. A 29-year-old woman have a temperature of 37.9°C. She feels
attends hospital with a painful swollen well in herself and examination is
right leg. She flew back to the UK from otherwise unremarkable. What is the most
Australia 2 days previously. According to appropriate course of action?
the Wells scoring system, her clinical risk A) Stop transfusion and give oral iron
of deep vein thrombosis (DVT) is 'high'. instead
Which of the following tests is the most B) Stop transfusion and administer IV
appropriate first step in her investigation? chlorphenamine and hydrocortisone
A) Coagulation screen C) Administer broad-spectrum antibiotics
B) Plasma D-dimer D) Give paracetamol and continue
C) Doppler ultrasound scan of the leg transfusion at a slower rate
D) Venous plethysmography E) Take an urgent blood sample for a
E) Thrombophilia screen repeat 'group and screen' test

92
Question 16. In the UK blood products Which of the following would be the most
are screened for all of the following appropriate initial investigation?
infections except one. Which is the A) Abdominal ultrasound scan
exception? B) Abdominal CT scan
A) Human immunodeficiency virus (HIV) C) Upper gastrointestinal endoscopy
B) Syphilis D) Cystoscopy
C) Hepatitis B E) Laparoscopy
D) Hepatitis C
E) Creutzfeldt-Jakob disease (CJD) Question 19. All of the following except
one are recognised causes of iron
Question 17. Long-term use of which of deficiency anaemia. Which is the
the following drugs can lead to iron exception?
malabsorption? A) Pregnancy
A) Omeprazole B) Coeliac disease
B) Ferrous sulphate C) Cold agglutinin disease
C) Aspirin D) Partial gastrectomy
D) Mesalazine E) Menorrhagia
E) Sodium valproate
Question 20. Which one of the following
Question 18. A 76-year-old man is tests is the most specific indicator of iron
admitted to hospital with a 4-month deficiency in the investigation of patients
history of increasing shortness of breath, with anaemia?
lethargy and weight loss. He has a past A) Reduced mean cell volume
history of angina, and takes aspirin, B) Reduced transferrin saturation
atenolol and simvastatin. Examination is C) Low serum iron
unremarkable. Blood tests reveal the D) Reduced ferritin
following: E) Raised total iron binding capacity
(TIBC)

Question 21. Which one of the following


statements regarding vitamin B12
deficiency is false?
A) Mean cell volume is usually raised
B) It may be caused by terminal ileal
Crohn's disease
C) Neutrophil hypersegmentation on the
blood film is characteristic
D) It is often due to dietary deficiency
E) There may be associated pancytopenia

Question 22. Which of the following


neurological findings is not a feature of
vitamin B12 deficiency?
A) Cognitive impairment
B) Nystagmus
C) Ataxic gait
D) Extensor plantar responses
E) Symmetrical peripheral neuropathy

93
Question 23. Which one of the following Question 25. In a patient with haemolytic
statements about folate is true? anaemia, which of the following findings
A) Deficiency may result from pernicious would be most suggestive of intravascular
anaemia haemolysis?
B) Dairy products offer a rich source of A) Raised reticulocyte count
folate B) Haemosiderinuria
C) Total body stores are small C) Raised lactate dehydrogenase (LDH)
D) The Schilling test is useful in D) Splenomegaly
establishing the reason for folate E) Positive direct Coombs test
deficiency
E) Serum folate is the most accurate Question 26. Which one of the following
indicator of overall body stores statements about hereditary spherocytosis
is false?
Question 24. A 29-year-old woman A) It usually shows autosomal dominant
attends her GP with increasing lethargy inheritance
and malaise. Blood tests reveal the B) Pregnancy can precipitate a
following: megaloblastic crisis
C) It may lead to the development of
cholesterol gallstones
D) Patients should receive treatment with
folic acid
E) The anaemia may be improved by
splenectomy

Question 27. Deficiency of which of the


following enzymes is associated with
haemolytic anaemia?
A) Alpha-glucosidase
B) Red cell transketolase
C) Glucose-6-phosphate dehydrogenase
D) Lactate dehydrogenase
E) Tyrosine kinase

Question 28. Which one of the following


statements regarding sickle-cell disease is
true?
A) Sickle-cell trait results in an increased
susceptibility to falciparum malaria
B) The condition shows X-linked
Which of the following diagnoses is most inheritance
consistent with these results? C) Haemoglobin electrophoresis
A) Coeliac disease demonstrates a predominance of HbF
B) Pernicious anaemia D) Sickle-cell anaemia is usually
C) Autoimmune haemolytic anaemia accompanied by a reticulocytosis
D) Hereditary spherocytosis E) With optimal medical care for sickle-
E) Anaemia of chronic disease cell anaemia, life expectancy is unaffected

94
Question 29. Which one of the following Investigations reveal the following:
is not a clinical manifestation of sickle-cell
disease?
A) Peripheral neuropathy
B) Dactylitis
C) Avascular necrosis of the hip
D) Splenic infarction
What is the most likely diagnosis?
E) Pulmonary fat embolism
A) Aplastic anaemia
B) Chronic myeloid leukaemia (CML)
Question 30. A 25-year-old Spanish man
C) Acute myeloid leukaemia (AML)
is referred to the haematology clinic after a
D) Non-Hodgkin lymphoma
routine full blood count is found to be
E) Myelodysplastic syndrome
abnormal. Blood tests reveal the
following:
Question 32. A 58-year-old woman is
referred to the haematology clinic with an
abnormal full blood count. She describes a
6-month history of tiredness, weight loss
and shortness of breath. On examination
there is marked hepatosplenomegaly but
no lymphadenopathy. Investigations reveal
the following:

What is the most likely diagnosis?


A) Alpha-thalassaemia
B) Beta-thalassaemia major
C) Beta-thalassaemia minor
D) Sickle-cell disease
E) Congenital sideroblastic anaemia

Question 31. A 59-year-old man is What is the most likely diagnosis?


referred to the haematology clinic with a A) Hodgkin's disease
3-month history of progressive lethargy. B) Acute myeloid leukaemia (AML)
He complains of recurrent sore throat and C) Acute lymphoblastic leukaemia
has recently noticed bleeding from his D) Chronic myeloid leukaemia (CML)
gums when he brushes his teeth. He has E) Chronic lymphocytic leukaemia
also noticed that he bruises easily. He
appears pale but examination is otherwise Question 33. Which haematological
unremarkable. condition is associated with the
Philadelphia chromosome [t(9;22)]?
A) Myelofibrosis
B) Myelodysplastic syndrome
C) Multiple myeloma
D) Chronic lymphocytic leukaemia
E) Chronic myeloid leukaemia (CML)

95
Question 34. Which one of the following Question 37. A 69-year-old man describes
statements about chronic lymphocytic a 6-month history of lethargy, weight loss
leukaemia (CLL) is false? and lower back pain. Blood tests reveal the
A) In the majority of cases, the diagnosis following:
is made incidentally following a routine
full blood count
B) There is a monoclonal proliferation of
B lymphocytes
C) Hyposplenism is a frequent finding
D) The condition is associated with
autoimmune haemolytic anaemia
E) Chlorambucil may be used as first-line
therapy for patients receiving active
treatment

Question 35. Which one of the following


statements about myelodysplasia is false?
A) It predominantly affects elderly
individuals
B) It often progress to acute myeloid
leukaemia (AML)
C) Peripheral blood cytopenias are a
characteristic feature
D) Bone marrow aspiration reveals a
hypocellular marrow
E) Curative treatment is not possible in the
majority of patients

Question 36. A 26-year-old man presents


with a 6-week history of a painless
swelling in his neck. On examination he
has painless rubbery cervical
lymphadenopathy but no other palpable
lymph nodes or splenomegaly. Following
lymph node biopsy (see below) What is the most likely diagnosis?
A) Waldenström's macroglobulinaemia
B) Multiple myeloma
C) Renal cell carcinoma with bony
metastases
D) Primary hyperparathyroidism
E) Non-Hodgkin lymphoma

the diagnosis of Hodgkin lymphoma is


made:
What abnormality is shown in the biopsy?
A) Auer rod
B) Reed-Sternberg cell
C) Ringed sideroblast
D) Howell-Jolly body
E) Burr cell

96
Question 38. The diagnosis of multiple cervical, axillary and inguinal
myeloma requires 2 out of 3 diagnostic lymphadenopathy. There is also palpable
criteria. Which of the following pairs of hepatosplenomegaly. Blood tests show a
results would clinch the diagnosis? mild normocytic, normochromic anaemia.
A) Presence of serum paraprotein and CT confirms the clinical findings and also
raised plasma calcium demonstrates bulky mediastinal and
B) Erythrocyte sedimentation rate (ESR) > retroperitoneal lymph nodes. What is the
100 mm/hr and increased malignant most likely diagnosis?
plasma cells in bone marrow A) Chronic myeloid leukaemia
C) Raised alkaline phosphatase (ALP) and B) Multiple myeloma
positive radioisotope bone scan C) Hodgkin's disease
D) Bence Jones protein in urine and lytic D) Non-Hodgkin lymphoma
lesions on plain X-ray E) Myelofibrosis
E) Presence of serum paraprotein and
widespread lymphadenopathy Question 42. Which one of the following
statements regarding disseminated
Question 39. Which one of the following intravascular coagulation (DIC) is false?
statements regarding polycythaemia rubra A) The condition may be precipitated by
vera is false? Gram-negative septicaemia
A) Splenomegaly is a common finding B) Thrombocytopenia is a characteristic
B) Progression to myelofibrosis may occur finding
C) There is an increased risk of ischaemic C) Fibrinogen levels are elevated
stroke D) D-dimer levels are elevated
D) Symptoms are improved by E) There is a significant risk of
venesection haemorrhage
E) Aspirin is contraindicated
Question 43. All of the following
Question 40. Which condition is statements about heparin are correct
characterised by the combination of except one. Which is the exception?
microangiopathic haemolytic anaemia, A) Unfractionated intravenous heparin
neurological signs and renal impairment? potentiates the activity of antithrombin
A) Idiopathic thrombocytopenic purpura B) Severe bleeding secondary to
B) Thrombotic thrombocytopenic purpura intravenous heparin can be reversed with
C) Hereditary haemorrhagic telangiectasia protamine
D) Primary thrombocythaemia C) The anticoagulant effect of low
E) Henoch-Schönlein purpura molecular weight heparin can be assessed
using the activated partial thromboplastin
Question 41. A 74-year-old man is time (APTT)
referred to the medical outpatient clinic D) Thrombocytopenia is a recognised
with a 6-month history of weight loss, complication of treatment with heparin
fevers and night sweats. He has no E) Heparin exerts its anticoagulant effect
significant past medical history. On more quickly than warfarin
examination there is painless rubbery

97
Chapter 24 MUSCULOSKELETAL
DISORDERS
Answers page 221

Question 1. A 70-year-old lady who has Question 4. Which one of the following
recently noticed some hearing difficulty is statements is true with regard to
found to have a greatly increased plasma fibromyalgia?
alkaline phosphatase concentration with A) The erythrocyte sedimentation rate
normal calcium, phosphate and (ESR) is usually elevated
parathyroid hormone (PTH) levels. What B) Symptoms of pain predominantly arise
is the most likely diagnosis? from peripheral joints
A) Osteomalacia C) Exacerbations of symptoms are often
B) Rickets associated with stressful life events
C) Renal osteodystrophy D) Systemic steroids form the cornerstone
D) Paget's disease of treatment
E) Osteoporosis E) Muscle biopsy is the definitive
investigation
Question 2. A 76-year-old man with type
2 diabetes and longstanding osteoarthritis Question 5. Which of the following
of the knees and hips presents to the clinical features is least typical of
hospital accident and emergency osteoarthritis?
department with a 72-hour history of A) Joint pain aggravated by activity and
progressive pain and swelling of his right relieved by rest
knee. He was recently treated by his B) Involvement of large joints
general practitioner for an episode of C) Prolonged morning stiffness
cellulitis. On examination, his temperature D) Presence of Bouchard's nodes
is 38.5°C and there is a warm, tense E) Palpable crepitus over affected joint
effusion with marked tenderness and
overlying erythema. Which of the Question 6. All of the following except
following is the most likely diagnosis? one may form part of the management of
A) Haemarthrosis osteoarthritis. Which is the exception?
B) Trauma A) Aerobic and strengthening exercises
C) Septic arthritis B) Non-steroidal anti-inflammatory drugs
D) Acute gout C) Total joint replacement
E) First presentation of rheumatoid D) Methotrexate
arthritis E) Provision of a walking stick

Question 3. With reference to the patient


from the previous question (Q2) who has
suspected septic arthritis, which of the
following investigations would be of
greatest help in confirming the diagnosis?
A) Full blood count
B) C-reactive protein (CRP)
C) X-ray of knee joint
D) Joint aspiration
E) Ultrasound scan of knee joint

98
Question 7. A lady is referred to a Question 11. Which one of the following
rheumatologist by her general practitioner agents is not classified as a disease-
with suspected rheumatoid arthritis. Which modifying antirheumatic drug (DMARD)?
one of the following features would not A) Sulfasalazine
provide support for this diagnosis? B) Naproxen
A) Morning stiffness lasting more than an C) Penicillamine
hour D) Sodium aurothiomalate (gold)
B) The presence of anti-double-stranded E) Azathioprine
DNA (anti-dsDNA) antibodies on blood
tests Question 12. All of the following except
C) Periarticular osteopenia and marginal one are documented side-effects of anti-
erosions on X-ray arthritic drugs. Which is the exception?
D) Symmetrical pattern of arthritis A) Renal impairment with ciclosporin
E) Duration of symptoms greater than 6 B) Pulmonary fibrosis with methotrexate
weeks C) Bone marrow suppression with
azathioprine
Question 8. All of the following except D) Keratitis with hydroxychloroquine
one are well-recognised extra-articular E) Gastritis with non-steroidal anti-
features of rheumatoid arthritis. Which is inflammatory drugs (NSAIDs)
the exception?
A) Episcleritis Question 13. All of the following except
B) Pyoderma gangrenosum one are characteristic features of
C) Pericarditis ankylosing spondylitis. Which is the
D) Alopecia exception?
E) Pulmonary nodules A) Raised erythrocyte sedimentation rate
and C-reactive protein
Question 9. A gentleman of 70 with B) Strongly positive rheumatoid factor
longstanding rheumatoid arthritis is found C) Association with HLA-B27
on a routine blood test to have a white cell histocompatibility antigen
count of < 1 × 109/l. He complains of D) Extraspinal features including anterior
abdominal pain, and on examination you uveitis and aortic incompetence
find generalised lymphadenopathy and E) 'Bamboo' spine appearance on X-ray
splenomegaly. Which of the following is
the most likely diagnosis? Question 14. Recognised patterns of
A) Malaria psoriatic arthritis include all of the
B) Acute lymphatic leukaemia (ALL) following except one. Which is the
C) Felty's syndrome exception?
D) Portal hypertension secondary to A) Asymmetric oligoarthritis
alcoholic liver disease B) Arthritis mutilans
E) Overwhelming septicaemia C) Symmetrical polyarthritis
D) Atlanto-axial instability
Question 10. All of the following clinical E) Distal interphalangeal joint arthritis
features affecting the hands except one are
characteristic of rheumatoid arthritis.
Which is the exception?
A) Bouchard's nodes
B) Swan neck deformity
C) Piano key deformity
D) Subluxed metacarpophalangeal joints
E) Rupture of the extensor tendons

99
Question 15. A 50-year-old man presents Question 19. Which one of the following
to his general practitioner with an acute statements regarding osteoporosis is true?
episode of gout. The GP takes a full A) The most common biochemical profile
history and decides that a likely risk factor is a normal serum calcium and phosphate
is one of his medications. Which one of with a raised alkaline phosphatase
the following drugs would be the most B) It is less likely to occur in women who
likely candidate? have an early menopause
A) Bendroflumethiazide C) It is highly unlikely in the absence of
B) Paracetamol symptoms
C) Perindopril D) Weight-bearing exercise is protective
D) Atenolol E) Bone mineral density Z-score of less
E) Clopidogrel than -2.5 is diagnostic

Question 16. All of the following Question 20. Which of the following
statements about gout are true except one. clinical features is least suggestive of
Which is the exception? systemic lupus erythematosus (SLE)?
A) A typical attack presents with rapid A) Raynaud's phenomenon
onset of severe pain in a single distal joint B) Scarring alopecia
B) There is a male preponderance of the C) A photosensitive, erythematous malar
disease rash
C) Allopurinol is often used in the D) Sclerodactyly
treatment of an acute attack E) Oral ulcers
D) High alcohol intake is a risk factor for
the condition in susceptible individuals Question 21. Which one of the following
E) The monosodium urate crystals statements is true with regard to the use of
deposited within affected joints are bisphosphonates in osteoporosis?
negatively birefringent when viewed under A) They inhibit bone resorption
polarised light B) They have been shown to reduce the
risk of fracture in patients with normal
Question 17. All of the following except bone mineral density
one are associated with pseudogout C) They reduce the risk of fracture but do
(calcium pyrophosphate dihydrate crystal not influence bone mineral density
deposition). Which is the exception? D) There is no effective oral preparation
A) Hyperparathyroidism E) They stimulate bone formation
B) Acromegaly
C) Wilson's disease Question 22. A 46-year-old builder
D) Haemochromatosis presents to his general practitioner with a
E) Advancing age 1-week history of back pain. Which one of
the following features in his presentation
Question 18. All of the following factors would provide the greatest cause for
except one would increase the likelihood concern?
of a patient developing osteoporosis in A) Sudden onset of the pain while lifting a
later life. Which is the exception? heavy load at work
A) Cigarette smoking B) A history of intermittent episodes of
B) Anorexia nervosa low back pain in the past
C) Hypothyroidism C) Pain varying with physical activity but
D) Family history of osteoporosis generally relieved by rest
E) Long-term therapy with oral D) Difficulty passing urine
corticosteroids E) Tenderness of the paraspinous muscles

100
Question 23. Which one of the following following statements is true regarding her
conditions is associated with a treatment?
seronegative arthritis? A) Calcium channel blockers such as
A) Systemic lupus erythematosus nifedipine may help symptoms of
B) Rheumatoid arthritis Raynaud's phenomenon
C) Systemic sclerosis B) Angiotensin-converting enzyme (ACE)
D) Psoriasis inhibitors are contraindicated due to the
E) Sjögren's syndrome risk of precipitating a hypertensive renal
crisis
Question 24. Which one of the following C) Systemic steroids are useful for treating
is not considered a risk factor for skin changes
development of gastrointestinal ulceration, D) H2-receptor antagonists are more
perforation and bleeding induced by non- effective than proton pump inhibitors for
steroidal anti-inflammatory drugs treatment of oesophageal reflux
(NSAIDs)? E) There are no effective oral treatments
A) Age > 60 years for pulmonary hypertension
B) Concomitant oral corticosteroid use
C) Concomitant respiratory disease Question 28. Which one of the following
D) Past history of peptic ulcer disease findings on clinical examination is
E) H. pylori colonisation of the stomach characteristic of dermatomyositis?
A) Weakness of the distal musculature
Question 25. Which one of the following B) Tender nodules on the finger pulps
is a characteristic feature of Reiter's C) A violaceous rash around the eyes
disease? D) Nail pitting
A) Conjunctivitis E) Peripheral oedema
B) Subcutaneous nodules
C) Strong female preponderance Question 29. As the admitting doctor on a
D) Precipitated by streptococcal throat general medical ward, you see a patient
infection who has been referred from primary care
E) Positive rheumatoid factor and is suspected of having
dermatomyositis. Which of the following
Question 26. Which one of the following investigations (all of which may support or
statements regarding dual energy X-ray refute the diagnosis) should you order
absorptiometry (DXA) bone scanning is first?
incorrect? A) Electromyography (EMG)
A) The Z-score compares the patient's B) Serum creatine kinase (CK)
bone density to that of an age-matched C) Muscle biopsy
control D) CT chest/abdomen/pelvis
B) A T-score of -2.5 is indicative of E) MRI
osteoporosis
C) DXA is indicated in patients with an Question 30. A 23-year-old lady who
early menopause who have not received complains of dry eyes and mouth has a
hormone replacement therapy positive Schirmer's test. Serological tests
D) The preferred measurement sites for are positive for antinuclear antibody
bone mineral density are the hip and the (ANA) and rheumatoid factor. Which of
lumbar spine the following is the most likely unifying
E) DXA bone scanning involves the diagnosis?
intravenous injection of radioisotope A) Systemic lupus erythematosus (SLE)
B) Sjögren's syndrome
Question 27. A 24-year-old woman with C) Conjunctivitis
sclerodactyly and symptoms suggestive of D) Rheumatoid arthritis
Raynaud's phenomenon is diagnosed with E) Dehydration
systemic sclerosis. Which one of the

101
Question 31. All of the following Question 35. With regard to the anatomy
statements about Sjögren's syndrome are of bone, which is the one incorrect
true except one. Which is the exception? statement below?
A) There is an association with HLA-B27 A) The normal skeleton consists of both
B) There is a female preponderance cortical and trabecular bone
C) Enlargement of the salivary glands is B) Cortical bone is formed from Haversian
frequently found systems
D) There is an association with other C) Osteoclasts are responsible for bone
autoimmune diseases such as primary formation
biliary cirrhosis and myasthenia gravis D) The most abundant protein in bone is
E) There is an increased risk of developing collagen
lymphoma E) Bone is mineralised with calcium and
phosphate
Question 32. Which one of the following
features is most typical of polymyalgia Question 36. With regard to the anatomy
rheumatica? of joints, which is the one incorrect
A) A normal erythrocyte sedimentation statement below?
rate (ESR) A) The costochondral joint is an example
B) Poor response to oral corticosteroid of a fibrocartilaginous joint
treatment B) Synovial joints always include two
C) Marked muscle wasting surfaces covered with articular cartilage
D) Muscle stiffness and tenderness C) Articular cartilage has a rich vascular
E) Mean age of onset of 30 years supply
D) Synovial fluid is an ultrafiltrate of
Question 33. A 75-year-old lady is plasma containing proteoglycans
admitted to the medical admissions unit in E) The joint capsule is in part formed from
the early hours of the morning with a ligament
temporal headache, visual disturbance and
pain on chewing. On examination, she has Question 37. Plain radiography of joints is
decreased visual acuity in her right eye often a helpful aid to diagnosis in bone
and tenderness over the right scalp. The and joint disease. Which is the one
erythrocyte sedimentation rate (ESR) is 95 incorrect association in the following list
mm/hr. Which is the most important action of radiographic findings?
to take? A) Bony osteophytes and subchondral
A) Order a CT of the brain sclerosis with osteoarthritis
B) Request an ophthalmology review in B) Diffuse narrowing of the joint space
the morning with rheumatoid arthritis
C) Request a temporal artery biopsy C) Soft tissue swelling with septic arthritis
D) Give 30 mg of dihydrocodeine for pain D) Looser's zones with osteomalacia
relief E) Osteopenia with Paget's disease of bone
E) Give 60 mg of prednisolone by mouth

Question 34. All of the following


statements except one are true of
polyarteritis nodosa. Which is the
exception?
A) Skin manifestations include palpable
purpura and livedo reticularis
B) There is an association with hepatitis B
infection
C) It is a large-vessel vasculitis
D) Peripheral nerve palsies may be
associated
E) Severe hypertension is a recognised
complication
102
Question 38. Which one of the following Question 40. A 12-year-old boy is brought
statements about blood tests often used in to clinic by his mother with a rash on his
rheumatology is incorrect? legs that has appeared over the last 3 days.
A) A high titre of rheumatoid factor at first He also complains of pain in his knees and
presentation of rheumatoid arthritis is a hands, and in the last 24 hours has
poor prognostic factor developed central abdominal pain. Prior to
B) Anti-double-stranded DNA (anti- this episode he was entirely well, apart
dsDNA) antibodies are highly specific for from what his mother describes as a 'cold'
systemic lupus erythematosus (SLE) 2 weeks ago. On examination, you find a
C) Antinuclear antibody (ANA) is a highly palpable purpuric rash over the buttocks
sensitive test for SLE and lower limbs. What is the likeliest
D) The serum creatine kinase is often diagnosis?
elevated in dermatomyositis A) Still's disease
E) Serum biochemistry is usually normal B) Henoch-Schönlein purpura
in osteomalacia C) Kawasaki disease
D) Rheumatic fever
Question 39. A number of weeks after E) Juvenile idiopathic arthritis
commencing a new tablet for her blood
pressure, a 49-year-old lady with Question 41. With reference to the patient
hypertension presents to her general in the previous question (Q40) who has
practitioner with a photosensitive rash suspected Henoch-Schönlein purpura,
affecting her face and hands and new- which of the following investigations is
onset arthralgia. The GP also finds least likely to contribute to the diagnosis?
microscopic haematuria on urinalysis and A) Dipstick and microscopy of urine for
suspects drug-induced lupus. Which of the red cells
following medications is most likely to B) Urea and electrolytes (U&E)
have caused this presentation? C) Full blood count (FBC)
A) Atenolol D) Chest X-ray
B) Lisinopril E) Skin biopsy
C) Minoxidil
D) Hydralazine Question 42. Which one of the following
E) Nifedipine conditions is least likely to occur as a
complication of Marfan's syndrome?
A) Lens dislocation
B) Spontaneous pneumothorax
C) Aortic dissection
D) Mitral valve prolapse
E) Pulmonary embolism

103
Chapter 25 NEUROLOGICAL DISEASE
Answers page 227

Question 1. The nerve root values Question 5. Which disorder is suggested


corresponding to the biceps tendon reflex by a 3 cycles/second 'spike and wave'
are: pattern on the electroencephalogram
A) C4/C5 (EEG)?
B) C5/C6 A) Juvenile myoclonic epilepsy
C) C6/C7 B) Childhood absence epilepsy
D) C7/C8 C) Secondary generalised epilepsy
E) C8/T1 D) Sporadic Creutzfeldt-Jakob disease
E) Motor neuron disease
Question 2. The myelin sheath
surrounding peripheral nerve axons is Question 6. Nerve conduction studies are
formed by: most likely to be helpful in which of the
A) Astrocytes following conditions?
B) Oligodendrocytes A) Motor neuron disease
C) Microglial cells B) Myasthenia gravis
D) Schwann cells C) Multiple sclerosis
E) Ependymal cells D) Carpal tunnel syndrome
E) Lambert-Eaton syndrome
Question 3. Which of the following
neurological deficits is most likely to Question 7. Which of the following
result from an isolated left temporal lobe imaging modalities is most likely to yield
lesion in a right-handed individual? diagnostic information in a patient with
A) Expressive dysphasia suspected multiple sclerosis?
B) Receptive dysphasia A) Plain skull X-ray
C) Right hemiparesis B) Carotid Doppler ultrasound (duplex)
D) Right hemisensory loss scan
E) Right-sided neglect C) CT head scan with contrast
D) CT head scan without contrast
Question 4. A 63-year-old man attends the E) MRI head scan
neurology outpatient clinic with his wife.
His wife reports that for the last few Question 8. A 48-year-old man presents to
months he has been behaving completely the hospital accident and emergency
out of character; for instance, he has been department with possible meningitis.
shouting and undressing in public. During Which one of the following would not be a
the consultation he makes inappropriate contraindication to immediate lumbar
sexual references. On examination he has puncture?
a positive grasp reflex. This presentation A) Thrombocytopenia
would fit best with a lesion in which of the B) A Glasgow coma score of 10/15
following sites? C) Reduced power in the left leg with an
A) Frontal lobe extensor plantar response
B) Dominant parietal lobe D) Current treatment with aspirin
C) Non-dominant parietal lobe E) Papilloedema
D) Dominant temporal lobe
E) Non-dominant temporal lobe

104
Question 9. Which of the following Question 13. Which one of the following
features is least consistent with the features is characteristic of cluster
headache of raised intracranial pressure? headache?
A) Associated with morning vomiting A) More frequent in females
B) Worsens as the day progresses B) Episodic attacks lasting 12-24 hours
C) Exacerbated by coughing C) Patients able to continue normal
D) Presence of papilloedema activities during pain
E) Relieved by simple analgesia D) Pain is predominantly occipital
E) Associated nasal congestion and
Question 10. A 33-year-old woman conjunctival injection
attends her general practitioner with a 4-
week history of persistent headache. She Question 14. In a patient presenting with
describes the sensation of a 'tight band' vertigo, which one of the following
around her head but denies any nausea or features most strongly suggests a central
visual disturbance. The headache is worse rather than peripheral (labyrinthine)
at the end of the day and responds poorly cause?
to paracetamol. She has managed to A) Episodes provoked by head movement
continue working as a shop assistant. What B) Persistent nystagmus
is the most likely diagnosis? C) Tinnitus and distorted hearing
A) Temporal arteritis D) Positive Hallpike manoeuvre
B) Migraine E) Vomiting
C) Tension headache
D) Viral meningitis Question 15. In a patient presenting with a
E) Intracranial space-occupying lesion blackout, which one of the following
features is most suggestive of a seizure?
Question 11. A 25-year-old woman A) No warning prior to blackout
attends hospital with a 12-hour history of B) Severely bitten tongue
intense, throbbing pain on the right side of C) Urinary incontinence
her head. She has vomited twice and finds D) Extreme pallor during the unconscious
it uncomfortable to look at bright light. period
She gives no history of any visual E) Rapid recovery following the collapse
disturbance or unusual sensations prior to
the onset of pain. She has experienced two Question 16. A 54-year-old man
similar but milder episodes over the last 3 experiences partial motor seizures
months. What is the most likely diagnosis? characterised by uncontrollable jerking of
A) Subarachnoid haemorrhage the right arm and leg. What is the most
B) Migraine likely anatomical focus of seizure activity?
C) Viral meningitis A) Right pre-central gyrus
D) Intracranial space-occupying lesion B) Right post-central gyrus
E) Cluster headache C) Left pre-central gyrus
D) Left post-central gyrus
Question 12. Which of the following E) Left temporal lobe
drugs is most useful in the treatment of an
acute severe migraine attack? Question 17. Which one of the following
A) Sumatriptan is not a cause of partial seizures?
B) Propranolol A) Astrocytoma
C) Pizotifen B) Cerebral abscess
D) Sodium valproate C) Cerebral infarction
E) Amitriptylin D) Alcohol withdrawal
E) Intracerebral haemorrhage

105
Question 18. A 49-year-old teacher is Question 23. Which one of the following
referred to the medical outpatient clinic involuntary movements is incorrectly
following a first seizure. He has no paired with its definition?
significant past medical history. What A) Chorea - jerky, small-amplitude,
advice would you offer him about driving purposeless movements
if he lived in the UK? B) Athetosis - slow writhing movements
A) He can continue driving provided that of the limbs
he is accompanied at all times C) Dystonia - involuntary adoption of an
B) He can resume driving once established abnormal posture
on anticonvulsant therapy D) Hemiballismus - brief, non-purposeful
C) He should refrain from driving until twitching of muscle groups
seizure-free for 3 months E) Tics - repetitive semi-purposeful
D) He should refrain from driving until movements
seizure-free for 1 year
E) He should not drive again Question 24. You are asked to examine a
59-year-old man admitted to the neurology
Question 19. Which of the following ward. Examination reveals loss of joint
drugs is first-line anticonvulsant therapy position and vibration sensation over the
for primary generalised tonic-clonic right side of the body, with loss of pain
epilepsy? and temperature sensation over the left
A) Phenytoin side of the body. There is weakness of the
B) Gabapentin right arm and leg associated with
C) Levetiracetam increased tone, brisk reflexes and an
D) Sodium valproate extensor plantar. There is no motor deficit
E) Topiramate on the left side of the body. Cranial
nerves, including facial sensation, are
Question 20. First-line drug treatment for intact. What is the most likely cause of
status epilepticus is: these findings?
A) IV phenytoin A) Left cerebral hemisphere lesion
B) IV propofol B) Right-sided cervical spinal cord lesion
C) IV diazepam C) Left-sided cervical spinal cord lesion
D) IV sodium valproate D) Transverse cervical spinal cord lesion
E) IV Phenobarbital E) Peripheral neuropathy

Question 21. Which one of the following Question 25. You are asked to review a
clinical signs is caused by lower motor patient with reduced consciousness. He is
neuron lesions? lying silently with his eyes closed. He
A) Hypertonia makes no response to any verbal prompts
B) Muscle fasciculations but when you apply supraorbital pressure
C) Brisk tendon reflexes he opens his eyes, makes moaning noises
D) Clonus and attempts to remove your hand. These
E) Extensor plantar responses findings are consistent on repeated testing.
Based on this data what is his Glasgow
Question 22. Which one of the following Coma Scale score?
conditions is most likely to be associated A) 6
with a resting tremor? B) 7
A) Cerebellar stroke C) 8
B) Hepatic encephalopathy D) 9
C) Essential tremor E) 10
D) Parkinson's disease
E) Thyrotoxicosis

106
Question 26. Which one of the following Question 30. A 74-year-old man
statements is true of dementia? complains of headache and double vision.
A) Prominent visual hallucinations suggest On examination the left eye is looking
Pick's disease downwards and abducted with a ptosis.
B) All patients with severe Alzheimer's The right eye appears normal. The left
disease should receive a trial of pupil is dilated and unresponsive but the
anticholinesterase drugs right pupil shows normal reactivity to
C) Disturbance of language function in the light. What is the most likely site of the
early stages is an indication for brain lesion?
biopsy A) Left oculomotor (III) nerve
D) Recurrent urinary tract infections are a B) Left trochlear (IV) nerve
frequent cause C) Left abducens (VI) nerve
E) Imaging of the brain (CT or MRI) D) Left levator palpebrae superioris
should be performed in most patients muscle
E) Left constrictor pupillae muscle
Question 27. Which one of the following
is not a feature of cerebellar disease? Question 31. A 68-year-old man with an
A) Ataxia apical bronchial carcinoma is found to
B) Dysarthria have a partial right ptosis associated with a
C) Hyperreflexia small right pupil and reduced sweating on
D) Nystagmus the right side of the face. What name is
E) Dysdiadochokinesis given to this constellation of features?
A) Cavernous sinus syndrome
Question 28. Which of the following B) Horner's syndrome
visual deficits is most likely to occur C) Lambert-Eaton syndrome
secondary to a right parietal stroke? D) Sjögren's syndrome
A) Right homonymous hemianopia E) Holmes-Adie syndrome
B) Left homonymous hemianopia
C) Right homonymous lower Question 32. Which one of the following
quadrantanopia statements regarding Bell's palsy is false?
D) Left homonymous lower A) Patients may experience hyperacusis
quadrantanopia B) Spontaneous recovery occurs in the
E) Left homonymous upper majority of patients
quadrantanopia C) Treatment with prednisolone may be of
benefit
Question 29. A 34-year-old woman D) The ability to wrinkle the forehead is
complains of double vision on looking to preserved
the right. Eye movement testing confirms E) Patients have difficulty closing the eye
diplopia on rightward gaze. Covering the on the affected side
right eye causes disappearance of the outer
image. What is the most likely cause of the Question 33. Which one of the following
diplopia? is an indication for an urgent CT head scan
A) Right oculomotor (III) nerve lesion in a patient presenting with a suspected
B) Left oculomotor (III) nerve lesion stroke?
C) Right trochlear (IV) nerve lesion A) Previous ischaemic stroke
D) Right abducens (VI) nerve lesion B) Presence of a carotid bruit
E) Left abducens (VI) nerve lesion C) Clinical features of a total anterior
circulation stroke
D) Treatment with aspirin
E) Treatment with warfarin

107
Question 34. Which of the following Question 37. A 43-year-old woman
presentations is least consistent with a attends the hospital accident and
diagnosis of transient ischaemic attack emergency department with a 1-hour
(TIA)? history of sudden-onset, severe occipital
A) Monocular blindness headache. On examination she is
B) Facial asymmetry and slurred speech photophobic with neck stiffness. What is
C) Expressive dysphasia the most appropriate first-line
D) Loss of consciousness investigation?
E) Left hemiparesis A) Skull X-ray
B) CT head scan
Question 35. A 73-year-old man presents C) CT cerebral angiography
with a right hemiparesis and expressive D) Lumbar puncture
dysphasia secondary to an infarct in the E) Carotid Doppler ultrasound (duplex)
territory of the left middle cerebral artery. scan
Blood pressure is 153/82 mmHg, serum
cholesterol is 4.4 mmol/l and the ECG Question 38. Which one of the following
shows sinus rhythm with no abnormalities. is not a risk factor for cerebral venous
Which of the following medications is thrombosis?
least likely to be of benefit in preventing a A) Oral contraceptive pill
further ischaemic stroke? B) Dehydration
A) Ramipril C) Facial erysipelas
B) Bendroflumethiazide D) Factor V Leiden deficiency
C) Aspirin E) Hypercholesterolaemia
D) Warfarin
E) Simvastatin Question 39. Which one of the following
statements regarding multiple sclerosis is
Question 36. Which of the following true?
ischaemic stroke patients who have A) The peak age of onset is between 50
undergone carotid duplex scanning would and 60 years
be most likely to benefit from left carotid B) It is more common in males
endarterectomy? C) It follows a relapsing and remitting
A) Right middle cerebral artery territory course in the majority of patients
infarct with good functional recovery, D) Impairment of memory is a common
75% left carotid artery stenosis presenting feature
B) Left middle cerebral artery territory E) The presence of oligoclonal bands of
infarct with good functional recovery, IgG in cerebrospinal fluid is diagnostic
60% left carotid artery stenosis
C) Left middle cerebral artery territory Question 40. Which one of the following
infarct with good functional recovery, treatments is first-line in treating acute
80% left carotid artery stenosis relapses of multiple sclerosis?
D) Left middle cerebral artery territory A) Interferon beta
infarct with persistent dense right B) Methylprednisolone
hemiparesis and profound dysphasia, 90% C) IV immunoglobulin
left carotid artery stenosis D) Plasmapheresis
E) Left cerebellar hemisphere infarct with E) Azathioprine
good functional recovery, 80% left carotid
artery stenosis

108
Question 41. Which one of the following Question 45. Which one of the following
statements about Alzheimer's disease is drugs is most likely to cause parkinsonism
true? as a side-effect?
A) 70% of individuals over the age of 80 A) Amitriptyline
years are affected B) Diazepam
B) Senile plaques and neurofibrillary C) Phenytoin
tangles are characteristic features on brain D) Fluoxetine
histology E) Haloperidol
C) Impairment of long-term memory is the
earliest feature in the majority of patients Question 46. Which one of the following
D) CT head findings are diagnostic for the statements regarding Huntington's disease
condition is false?
E) Progression can be halted by early A) It has autosomal recessive inheritance
treatment with anticholinesterases B) It is due to a trinucleotide repeat
C) Successive generations are often
Question 42. A 73-year-old man is affected at an earlier age
admitted to hospital with declining D) Cognitive impairment is a frequent
mobility. On examination there is a resting finding
tremor affecting the right hand. Muscle E) Genetic screening is available for
tone in the arms is increased. He has family members of affected individuals
difficulty when asked to fasten his buttons.
What is the most likely diagnosis? Question 47. A 68-year-old man is
A) Alzheimer's disease referred to the neurology outpatient clinic
B) Parkinson's disease with a 1-year history of progressive
C) Huntington's disease bilateral arm and leg weakness. On
D) Myotonic dystrophy examination there is global muscle
E) Motor neuron disease wasting with muscle fasciculations in the
upper arms and thighs. There is
Question 43. Which one of the following generalised muscle weakness, worse
is not a feature of the gait of Parkinson's distally and on the left. Reflexes are brisk
disease? throughout and plantars are extensor.
A) Reduced arm swing Sensory examination is normal and cranial
B) Wide-based gait nerves are intact. What is the most likely
C) Impaired balance on turning diagnosis?
D) Slow to start walking A) Myotonic dystrophy
E) Shortened stride length B) Motor neuron disease
C) Guillain-Barré syndrome
Question 44. Which one of the following D) Cerebrovascular disease
statements regarding the treatment of E) Myasthenia gravis
Parkinson's disease is true?
A) Levodopa can be used without a
peripheral dopa-decarboxylase inhibitor in
many patients
B) Young patients with Parkinson's
disease should be commenced on levodopa
as early as possible
C) Levodopa is more effective at relieving
tremor than bradykinesia or rigidity
D) Dyskinesia secondary to levodopa can
be improved by modifying the dosing
regimen
E) Anticholinergic drugs are
contraindicated

109
Question 48. A 34-year-old woman C) Cerebral abscess
attends the hospital accident and D) Creutzfeldt-Jakob disease (CJD)
emergency department with a 6-hour E) Herpes simplex encephalitis
history of headache, fever and neck
stiffness. On examination she has a Question 51. Which one of the following
temperature of 39.1°C, pulse of 108 bpm, statements regarding tetanus is true?
blood pressure of 110/58 mmHg and A) The responsible bacterium, Clostridium
oxygen saturation of 98% on air. Glasgow tetani, infects the spinal cord and brain
Coma Score is 15/15. A purpuric rash is B) Multiplication of the organism occurs
noted over her chest and arms. There is only in areas with good oxygenation
evidence of neck stiffness and C) The typical initial presentation is with
photophobia but no focal neurological headache, fever and drowsiness
deficit is identified. What would the most D) Antibiotics are of no value in treatment
appropriate initial management be? E) Tetanus antitoxin should be
A) Arrange a CT head scan administered in all suspected cases
B) Transfer the patient to the ward and
perform a lumbar puncture Question 52. Which one of the following
C) Administer IV benzylpenicillin statements regarding Creutzfeldt-Jakob
D) Administer IV acyclovir disease (CJD) is false?
E) Insert a central venous line A) The causative agent is a protein
B) Myoclonic jerks are a characteristic
Question 49. Which one of the following feature
statements regarding poliomyelitis is C) Cognitive function is usually preserved
false? D) Patients with variant CJD are typically
A) The condition is caused by a viral younger than those with sporadic CJD
infection E) No curative treatment is available
B) Upper motor neuron signs predominate
C) Physiotherapy is useful in maintaining Question 53. Which one of the following
mobility statements regarding intracranial
D) Invasive ventilation may be required neoplasms is true?
E) An effective oral vaccine is available A) The majority are primary neoplasms
B) Primary intracerebral tumours do not,
Question 50. A 73-year-old man is generally, metastasise outside the central
brought to the hospital accident and nervous system
emergency department after experiencing C) CT is superior to MRI for imaging
a seizure. His family state that he has been posterior fossa tumours
complaining of headaches and nausea for D) Dexamethasone aggravates oedema
3 months. His only past medical history is surrounding cerebral tumours
of type 2 diabetes mellitus. A contrast- E) Gliomas have a better prognosis than
enhanced CT head scan is shown below: meningiomas

Question 54. Which one of the following


statements regarding acoustic neuroma is
true?
A) Conductive deafness is a common
finding
B) There is an association with tuberous
sclerosis
C) Cerebellar signs may be present
D) CT is the investigation of choice
What is the most likely diagnosis? E) Radiotherapy is the treatment of choice
A) Subdural haematoma
B) Subarachnoid haemorrhage

110
Question 55. A 25-year-old woman is C) L5
referred to the neurology clinic with a 6- D) S1
month history of recurrent headaches. She E) S2
denies any other symptoms. Her only past
medical history is of acne, for which she Question 59. What is the most appropriate
takes oxytetracycline. On examination she first-line treatment for a patient presenting
has a body mass index of 34. Fundoscopy with severe lower back pain secondary to
reveals papilloedema but neurological lumbar disc herniation, associated with
examination is otherwise unremarkable. A nerve root compression?
CT head scan is normal. The cerebrospinal A) Bed rest
fluid (CSF) opening pressure at lumbar B) Analgesia and early mobilization
puncture is elevated, but CSF protein, C) Steroid injection
glucose and cell count are normal. What is D) Spinal radiotherapy
the most likely diagnosis? E) Surgical decompression
A) Normal pressure hydrocephalus
B) Cerebral astrocytoma Question 60. A 76-year-old man with
C) Idiopathic (benign) intracranial metastatic prostate cancer is admitted to
hypertension hospital with lower back pain and
D) Subarachnoid haemorrhage deteriorating mobility. He also describes a
E) Cerebellar abscess 2-day history of urinary incontinence.
Neurological examination of the lower
Question 56. Which of the following limbs reveals bilaterally increased tone
drugs is most likely to be effective in the with weakness, brisk reflexes and extensor
treatment of idiopathic (benign) plantars. No deficit is noted in the upper
intracranial hypertension? limbs or cranial nerves. There is reduced
A) Furosemide sensation below his mid-abdomen. What is
B) Bendroflumethiazide the most likely cause for the deterioration?
C) Spironolactone A) Cerebral metastases
D) Amiloride B) Cervical cord compression
E) Acetazolamide C) Thoracic cord compression
D) Cauda equina syndrome
Question 57. Which one of the following E) Peripheral neuropathy
clinical features is not characteristic of
normal pressure hydrocephalus? Question 61. Which investigation is most
A) Dementia likely to yield useful diagnostic
B) Urinary incontinence information in suspected cases of spinal
C) Headache cord compression?
D) Gait apraxia A) Plain X-ray of the spine
E) Higher incidence in old age B) Spinal CT scan
C) Spinal MRI scan
Question 58. A 45-year-old man attends D) Nerve conduction studies
his general practitioner after developing E) Isotope bone scan
sudden-onset lower back pain while lifting
a sofa. He also describes pain shooting Question 62. Which one of the following
down the back of his right leg. On is least likely to cause peripheral
examination the straight leg raise test is neuropathy?
positive on the right. Sensation is reduced A) Diabetes mellitus
over the sole and lateral border of the right B) Chronic alcohol excess
foot. Plantar flexion of the ankle is weak C) HIV infection
on the right, and the right ankle jerk is D) Multiple sclerosis
absent. Which nerve root is affected? E) Vitamin B12 deficiency
A) L3
B) L4

111
Question 63. Which one of the following Question 66. A 29-year-old woman is
drugs is most likely to cause peripheral referred to the medical outpatient clinic
neuropathy? with a 4-month history of intermittent
A) Amiodarone double vision. She also complains of
B) Furosemide difficulty combing her hair without taking
C) Haloperidol frequent rests. Examination of the upper
D) Amoxicillin limbs initially reveals normal power, but
E) Lansoprazole this declines with repeated testing.
Sensation is normal. What is the most
Question 64. A 54-year-old woman likely diagnosis?
attends her general practitioner with a 2- A) Duchenne muscular dystrophy
week history of tingling in her right hand. B) Oculopharyngeal muscular dystrophy
Her symptoms often wake her from sleep. C) Myasthenia gravis
On examination sensation is reduced over D) Chronic fatigue syndrome
the palmar aspect of the right thumb, index E) Temporal arteritis
and middle fingers and over the radial half
of the ring finger. There is weakness of Question 67. Which one of the following
thumb abduction. Which nerve is affected? statements about myasthenia gravis is
A) Radial nerve false?
B) Median nerve A) The condition is caused by
C) Ulnar nerve autoantibodies directed against
D) Axillary nerve acetylcholine receptors
E) Musculocutaneous nerve B) The condition may affect respiratory
muscles
Question 65. Which one of the following C) The Tensilon test can be helpful in
statements about Guillain-Barré syndrome making the diagnosis
is false? D) Thymectomy may improve prognosis
A) The condition is often preceded by a in patients without thymoma
diarrhoeal illness E) The presence of muscle paralysis with
B) There is a lower motor neuron pattern fasciculation, sweating and excessive
of muscle weakness salivation suggests a myasthenic crisis
C) Lumbar puncture demonstrates
increased cerebrospinal fluid protein with Question 68. Which one of the following
a normal white cell count features is characteristic of myotonic
D) Respiratory muscles are not affected dystrophy?
E) IV immunoglobulin reduces the A) Autosomal recessive inheritance
duration of illness B) Hirsutism
C) Delayed relaxation of skeletal muscle
D) Calf muscle hypertrophy
E) Autoantibodies directed against
presynaptic voltage-gated calcium
channels at the neuromuscular junction

112
Chapter 26 SKIN DISEASE
Answers page 238

Question 1. A 45-year-old gentleman Question 4. Which one of the following


presents with well-demarcated scaly systemic agents is not used in the
plaques on his scalp, elbows and knees. management of psoriasis?
On examination you also notice some nail A) Methotrexate
abnormalities: B) Psoralen
C) Ciclosporin
D) Acitretin
E) Gold

Question 5. Which one of the following


statements regarding urticaria is incorrect?
A) It may be caused by physical agents
such as cold and pressure
What is the diagnosis? B) Episodes often last longer than 24
A) Lichen planus hours
B) Drug eruption C) The pathophysiology involves
C) Chronic plaque psoriasis degranulation of mast cells
D) Atopic eczema D) Antihistamines may help the condition
E) Pityriasis rosea E) Appropriate investigations would
include total IgE and complement levels
Question 2. Which one of the following
statements regarding psoriasis is Question 6. A 15-year-old male presents
incorrect? with a rash consisting of small, scaly
A) It has a predilection for the extensor salmon-pink macules of raindrop
surface of the limbs appearance on his back. He gives a recent
B) It is the only skin disease to history of a sore throat. You suspect a
demonstrate the Köbner phenomenon diagnosis of guttate psoriasis. Which one
C) The Auspitz sign may be demonstrated of the following organisms is most likely
by lifting a psoriatic scale to be associated with this presentation?
D) Nail changes include pitting and A) Streptococcus pyogenes
subungual hyperkeratosis B) Trichophyton rubrum
E) Psoriatic pustules are sterile C) Proprionobacterium acnes
D) Borrelia burgdorferi
Question 3. Which of the following E) Sarcoptes scabiei
factors would be least likely to provoke a
worsening of psoriasis?
A) Beta-blockers
B) Lithium
C) Penicillin antibiotics
D) Stress
E) Antimalarial treatment

113
Question 7. A 28-year-old woman with Question 10. Which one of the following
type 1 diabetes mellitus presents to the statements about basal cell carcinoma is
clinic with these yellow waxy-looking true?
plaques on her anterior shin: A) Deep, penetrating lesions should be
treated with cryotherapy
B) Metastatic spread to regional lymph
nodes occurs early
C) Tumour recurrence following treatment
carries a poor prognosis
What is the diagnosis? D) Most lesions resolve spontaneously
A) Discoid eczema E) Local invasion is more common than
B) Granuloma annulare haematogenous spread
C) Psoriasis
D) Necrobiosis lipoidica Question 11. All of the following features
E) Erythema nodosum of a mole except one would give cause for
concern. Which is the exception?
Question 8. Which of the following A) Itch
statements regarding necrobiosis lipoidica B) Bleeding
is most accurate? C) Warty texture with 'stuck-on'
A) The condition only occurs in diabetics appearance
B) Severity of the lesion is closely D) Recent change in shape
correlated with glycaemic control E) Presence of irregular pigmentation
C) There is an infective component to the
pathogenesis Question 12. Which one of the following
D) Topical or intralesional steroids may be statements regarding classification of
useful malignant melanoma is incorrect?
E) The lesions very rarely ulcerate A) Lentigo maligna melanoma mainly
occurs on the exposed skin of the elderly
Question 9. A 68-year-old retired labourer B) Acral lentiginous melanoma occurs on
presents with the following lesion on the the palms and the soles
side of the nose: C) Superficial spreading melanoma has a
radial growth phase
D) Amelanotic melanomas are more
common than the pigmented variety
E) The excision margins of a nodular
melanoma are dependent on its Breslow
thickness
What is the diagnosis?
A) Squamous cell carcinoma Question 13. All of the following except
B) Nodular melanoma one are risk factors for development of
C) Cellulitis skin cancer. Which is the exception?
D) Actinic keratosis A) Dark skin and black hair
E) Basal cell carcinoma B) A strong history of sun exposure
C) A family history of melanoma
D) An outdoor occupation
E) The genetic defect, xeroderma
pigmentosum

114
Question 14. A 4-year-old girl is brought Question 18. A 19-year-old woman
to the clinic by her mother with a presents with this erythematous, painful,
distressing itchy rash on the flexor aspects nodular eruption on the anterior aspect of
of her elbows and knees and on her face. both shins:
She has had a similar rash since early
infancy. She takes inhalers for asthma but
has not been exposed to any other
medications. On examination, you find an
erythematous papular rash in the flexures,
which is obviously excoriated and has
prominent lichenification. Which one of
the following is the most likely diagnosis? Which of the following steps is least
A) Atopic eczema appropriate in her initial work-up and
B) Scabies management?
C) Contact dermatitis A) Chest X-ray
D) Psoriasis B) Full drug history
E) Lichen planus C) Antibiotics
D) Antistreptolysin O (ASO) titre
Question 15. Which one of the following E) Simple analgesia
would not feature in the management of a
child with atopic eczema? Question 19. Which one of the following
A) Greasy emollients statements concerning the definitions of
B) Topical steroids skin lesions is incorrect?
C) Patient education A) Vesicles are less than 5 mm in diameter
D) Dithranol B) Papules are solid elevations of skin less
E) Avoidance of known irritants than 5 mm in diameter
C) Macules are usually raised
Question 16. Involvement of the oral D) Telangiectasia are visibly dilated small
mucous membranes would be an cutaneous vessels
unexpected finding in which one of the E) Excoriation is the result of scratching
following disorders?
A) Systemic lupus erythematosus (SLE) Question 20. Which one of the following
B) Pemphigus skin conditions is not typically itchy?
C) Lichen planus A) Lichen planus
D) Bullous pemphigoid B) Atopic eczema
E) Stevens-Johnson syndrome C) Dermatitis herpetiformis
D) Bullous impetigo
Question 17. With regard to hair loss, E) Psoriasis
which one of the following statements is
incorrect? Question 21. Which one of the following
A) Discoid lupus erythematosus causes a is not associated with photosensitivity?
non-scarring alopecia A) Thiazide diuretics
B) Exclamation mark hairs are B) Tetracycline antibiotics
pathognomonic of alopecia areata C) Amiodarone
C) Vitiligo, autoimmune thyroid disease D) Hydroxychloroquine
and trisomy 21 are all associated with E) Atenolol
alopecia areata
D) Alopecia totalis describes complete
loss of scalp hair with preservation of
body hair
E) Androgenetic alopecia may occur in
both males and females

115
Question 22. All of the following D) Beau's lines Lichen planus
statements regarding staphylococcal E) Clubbing Atrial septal defec
scalded skin syndrome (SSSS) are correct
except one. Which is the exception? Question 24. Which one of the following
A) The causative organism is is not a cause of porphyria cutanea tarda?
Staphylococcus aureus A) Alcohol excess
B) Clinical features include fever, skin B) Haemochromatosis
tenderness and erythema C) Genetic defect
C) Patients are usually over the age of 18 D) Diabetes mellitus
years E) HIV
D) Blisters arise from a split in the skin
below the stratum corneum Question 25. The following are all
E) Intravenous antibiotics are an important recognised precipitants of erythema
part of initial management multiforme except one. Which is the
exception?
Question 23. Which one of the following A) Mycoplasma infection
nail changes is matched correctly with its B) Digoxin
associated condition? C) Herpes simplex infection
A) Koilonychia Folate deficiency D) Penicillin antibiotics
B) Onycholysis Psoriasis E) Underlying malignancy
C) Leuconychia Fibrosing alveolitis

116
PART 2
ANSWERS

117
Chapter 1 GOOD PRESCRIBING
Question 1. A) It causes adverse effects the absolute increase in harm in the
at doses close to the usual therapeutic treatment group = 4% - 2% = 2% = 0.02.
range The number needed to harm (i.e. the
Explanation: It is easier to think of a number of patients that need to be treated
'narrow' therapeutic index: that is, there is to produce one additional life-threatening
a narrow range between the dose haemorrhage) is the inverse of this: 1/0.02
necessary to have an effect and the dose = 50
necessary to cause toxicity.
Question 4. A) Taking this drug for 5
Question 2. B) On average, tow sever years will reduce your chance of a heart
exacerbations will be prevented for attack by 15%
every 50 patients treated for 1 year with Explanation: The only true statement is
the new inhaler. A. Relative risk (RR) for a given outcome
Explanation: The NNTB expresses the is calculated as: risk with treatment/risk
number of patients who must be treated without treatment. The principles
with a drug to obtain a single beneficial underlying the absolute and relative risk
outcome over a given time: in this case, to reductions and the number needed to treat
prevent a severe acute asthma are explained in the link below.
exacerbation during treatment for 1 year.
A is false because some patients may Question 5. E) The dose-response curve
experience benefits other than the for Potentol lies to the left of the curve
prevention of a severe exacerbation. At for Weakol
first glance it may appear that B and C are Explanation: The potency of a drug is
both true. Looking at B, it is correct to say merely a description of how much of it is
that the effect of a drug on the first 25 required to have a given effect. Thus, the
patients treated should, on average, be the more potent drug will have the same effect
same as the effect on the next 25, so if we at smaller doses, but the maximum effects
treat 50 patients we will prevent, on and adverse effects of the two drugs could
average, two severe exacerbations. be the same. A more potent drug gives a
However, C is one step too far; we cannot response at a lower dose; therefore the
infer from a trial lasting only 1 year that dose-response curve is shifted to the left.
the benefit will be identical in subsequent
years. In the words of the philosopher Question 6. C) Digoxin
Francis Bacon, 'The subtlety of nature is Explanation: Digoxin is extensively
many times greater than the subtlety of excreted via the kidneys and has a narrow
argument.' The only way to find out if C is therapeutic index, so severe toxicity can
true is to do another experiment, one that occur at usual doses in a patient with renal
lasts for 2 years. impairment. Paracetamol and simvastatin
are relatively safe in renal insufficiency.
Question 3. E) 50 Larger doses of furosemide are often
Explanation: The absolute risk of life- required to achieve an adequate diuresis in
threatening haemorrhage is 2% in the renal insufficiency, although clearly
placebo group (20/1000) and 4% in the furosemide and other diuretics should be
treatment group (40/1000). Thus, with avoided altogether if the cause of renal
respect to life-threatening haemorrhage, impairment is hypovolaemia.

118
Question 7. D) Allopurinol and Question 8. B) The route of
azathioprine administration of the drug
Explanation: Allopurinol inhibits Explanation: The essential elements of a
xanthine oxidase, preventing the drug prescription outside hospital can be
breakdown of a metabolite of azathioprine, found in the link below.
mercaptopurine. This leads to an effective
overdose of azathioprine, which can cause
bone marrow suppression, resulting in Question 9. A) Patient's date of birth
neutropenia and potentially catastrophic Explanation: All of the other components
immune compromise. The dose of are essential legal requirements for
azathioprine should be reduced by 75% if prescriptions for controlled drugs
allopurinol is co-prescribed. Paracetamol
500 mg plus dihydrocodeine 10 mg (co-
dydramol) is available in a combined Question 10. B) Amiodarone
formulation. Digoxin and warfarin are Explanation: A loading dose of
commonly prescribed together in atrial amiodarone achieves a more rapid effect.
fibrillation. Trimethoprim plus
sulfamethoxazole are used in combination
(as co-trimoxazole) in the treatment of Question 11. D) 0.5
Pneumocystis carinii (jirovecii) Explanation: Relative risk (RR) for a
pneumonia. Some calcium channel given outcome is calculated as: risk with
blockers increase ciclosporin blood treatment divided by risk without
concentrations, but there is no interaction treatment = 20/40 = 0.5.
of ciclosporin with ß- blockers

119
MOLECULAR

Chapter 2 MECHANISMS OF
DISEASE

Question 1. D) Neutrophils influence gene expression but rather bind


Explanation: The DNA in most human to G-protein-linked receptors on the cell
cells is packaged into 22 pairs of surface, and transmit their signal via
autosomal chromosomes, and one pair of second messenger systems.
sex chromosomes (XX in females and XY
in males), giving a total of 46. Exceptions Question 5. C) The cell membrane
are gametes (sperm and ova) which have remains intact
23 chromosomes, red blood cells which Explanation: Apoptosis refers to
are anucleate, and platelets which are programmed cell death in which cells
formed by cell fragmentation and contain undergo 'suicide'. The process is critical
no DNA. for cell turnover and tissue remodelling,
and occurs from embryogenesis through to
Question 2. B) Formation of messenger old age. It is mediated by the activation of
RNA from DNA by RNA polymerase specific genes. The cell membrane
Explanation: A gene is a region of DNA remains intact despite cell death; toxic
which encodes a specific protein. cellular enzymes are therefore not
Transcription refers to the creation of released, in contrast to necrosis, and there
RNA from cellular DNA; translation is the is no inflammatory response. The
synthesis of protein from RNA. apoptotic cell is removed by phagocytosis.
Cell death in any ischaemic process such
Question 3. C) Chloride transport as myocardial infarction involves necrosis
through channels on the cell membrane rather than apoptosis.
Explanation: Cystic fibrosis is an
autosomal recessive condition which Question 6.C) 50%
predominantly affects the lungs but also Explanation: Huntington's chorea is an
has consequences in the pancreas, gut and autosomal dominant disorder and so only
male reproductive system. Mutation of the one abnormal copy of the gene is required
CFTR gene results in defective chloride to cause the disease. Children with one
transport through channels in the cell affected parent therefore have a 50%
membrane. In the lungs, this leads to the chance of inheriting the abnormal gene
accumulation of thick respiratory from that parent. Non-identical (dizygotic)
secretions and superadded infection. twins are formed by the fusion of two
separate sperm with two separate ova;
Question 4. C) Noradrenaline genes are acquired independently from
(norepinephrine) both parents and the risk of disease is 50%
Explanation: Steroids (e.g. for each child. Identical (monozygotic)
glucocorticoids, mineralocorticoids, sex twins have the same genetic composition,
steroids), thyroid hormones, and vitamins and so if one twin has inherited the
A and D mediate their effect by binding to abnormal gene then so must the other.
intracellular receptors. The activated This highlights the importance of family
receptors act as transcription factors and counselling before genetic testing.
regulate gene expression. Catecholamines
(e.g. adrenaline/epinephrine and
noradrenaline/norepinephrine) do not

120
Question 7. D) Haemochromatosis - Question 9.E) Familial breast/ovarian
autosomal dominant cancer
Explanation: Haemochromatosis is a Explanation: Familial breast and ovarian
disorder characterised by iron overload cancer is a familial cancer syndrome with
and is inherited as an autosomal recessive autosomal dominant inheritance associated
trait. It is caused by mutation in the HFE with mutations in the BRCA1 or BRCA2
gene, and the carrier frequency of the genes.
abnormal gene in the general population is
10%. Clinical expression usually occurs Question 10.B) Congenital adrenal
earlier in males than in females, as hyperplasia
menstrual blood losses protect against iron Explanation: Congenital adrenal
overload. hyperplasia is an autosomal recessive
disorder due to a defect in one of the
Question 8. E) Trisomy 21 enzymes involved in steroid biosynthesis.
Explanation: Clinical features of Down's The most common abnormality is a
syndrome include learning difficulties, deficiency of 21-hydroxylase, resulting in
congenital heart disease, characteristic variable deficiencies of glucocorticoids
facies and a single palmar crease. and mineralocorticoids. Adult polycystic
kidney disease has autosomal dominant
inheritance, although the rarer infantile
polycystic kidney disease is autosomal
recessive.

121
Question 11. D) Secondary Question 13. C) Penetrance
amenorrhoea Explanation: Penetrance describes the
Explanation: Turner's syndrome (45,XO) proportion of individuals with an abnormal
is a chromosomal disorder that affects genotype who exhibit clinical disease.
only females. Affected individuals never Penetrance varies for different conditions,
menstruate (i.e. primary amenorrhoea) and and is often affected by the interaction of
are infertile. Secondary amenorrhoea, on genetic and environmental factors. Some
the other hand, refers to the cessation of conditions (e.g. Huntington's disease) have
periods in a female who has previously very high penetrance. Anticipation is the
menstruated. Other features of Turner's development of disease at a younger age
syndrome include widely spaced nipples, a and with greater severity in successive
wide carrying angle and the absence of generations; this phenomenon is often seen
secondary sex characteristics. in conditions caused by triplet repeats (e.g.
myotonic dystrophy). Genomic imprinting
Question 12. D) A son would be refers to the development of a different
unaffected and a daughter would have a phenotype depending on whether the
100% chance of being a carrier abnormal gene was inherited from the
Explanation: The abnormal gene in mother or father. Translocation is the
haemophilia A lies on the X chromosome, exchange of genetic material between two
and the condition is inherited as an X- different chromosomes; for instance, the
linked recessive trait. In this example, the t(9,22) translocation (the Philadelphia
prospective father has the abnormal gene chromosome) is strongly associated with
on his X chromosome, and the mother has chronic myeloid leukaemia. Lyonisation is
two normal X chromosomes. A son would the random inactivation of one of the two
inherit the Y chromosome (not the X chromosomes in the female fetus, and
abnormal X chromosome) from the father, explains why female carriers of X-linked
and so could not inherit the haemophilia recessive conditions (e.g. haemophilia A)
gene. A daughter would inevitably inherit can exhibit mild clinical disease if the
the abnormal X chromosome from the normal copy is inactivated in some cells.
father, and a normal X chromosome from
the mother. She would therefore be a Question 14. B) Von Hippel-Lindau
carrier for haemophilia A. Female carriers disease
often exhibit no clinical features, although Explanation: Von Hippel-Lindau disease
slightly reduced levels of factor VIII may is a familial cancer syndrome with
result in a mild bleeding disorder. autosomal dominant inheritance. It occurs
due to mutation in the VHL gene.

122
Chapter 3 IMMUNOLOGICAL
FACTORS IN DISEASE

Question 1. E) T lymphocytes Question 4. C) T-lymphocyte deficiency


Explanation: The immune system is Explanation: This is a typical
broadly divided into innate defences and presentation of T-lymphocyte deficiency,
adaptive responses. Innate defences are which can cause increased susceptibility to
non-specific, fast response systems that do viral,
not exhibit memory; the acquired (or fungal and mycobacterial infections. The
adaptive) immune response is targeted most common cause of T-lymphocyte
against the specific antigen, and changes deficiency worldwide is HIV infection.
in specificity and magnitude after initial
exposure (memory). T lymphocytes are an Question 5. A) Complement deficiency
essential component of the acquired Explanation: Formation of the membrane
immune response attack complex of complement is essential
in defence against Neisseria meningitides
Question 2. B) Neutrophils and other encapsulated bacteria.
Explanation: Neutrophils play an
important role in the initial stages of Question 6. D) Cryoglobulinaemia
infection, killing microorganisms directly Explanation: The combination of joint
and non-specifically amplifying the pain, microscopic haematuria and a
immune response. T and B cells form part purpuric rash is suggestive of small-vessel
of the adaptive immune response which is vasculitis. In patients with hepatitis C, this
more specific, but slower to respond. is most commonly caused by cryoglobulin
deposition. Autoimmune
Question 3 A) C-reactive protein (CRP) thrombocytopenia can cause a purpuric
Explanation: C-reactive protein (CRP) is rash but does not commonly cause
an acute phase protein which increases microscopic haematuria or joint pain.
within hours of infection and falls rapidly More on Cryoglobulinaemia
on resolution. Levels may increase up to
×1000 normal, making it a useful test for Question 7. D) Anticardiolipin
monitoring treatment response. antibodies
Complement levels may rise as part of an Explanation: Anticardiolipin antibodies
acute phase response, but do not exhibit are associated with recurrent fetal loss and
the same magnitude of response as CRP arterial and venous thrombosis. Patients
(×2-4-fold). In addition, complement may with primary or secondary
be consumed during infection, making it antiphospholipid syndrome may have a
an unreliable indicator of treatment positive lupus anticoagulant in addition to,
response. Immunoglobulin levels rise or instead of anticardiolipin antibodies, but
much more slowly in response to this cannot be assessed in patients on
infection (days to weeks), making these a warfarin.
less useful indicator.

123
Question 8. D) Antibody deficiency anti-blood group antibodies bind to blood
Explanation: Recurrent bacterial infection group determinants on the surface of red
of the upper and lower respiratory tract cells and cause cell killing.
with end organ damage is a common
presentation of patients with primary Question 13. A) CREST syndrome
antibody deficiency. Common organisms Explanation: If you have difficulty
include Haemophilus influenzae and remembering the various associations
Streptococcus pneumoniae. between autoantibodies and connective
tissue disorders you are certainly not
Question 9. B) Vaccination with live alone! However, specific autoantibody
attenuated vaccines profiles can provide useful diagnostic
Explanation: Live vaccines should always information and are favourites with
be avoided in immunocompromised postgraduate examiners. The CREST
patients (and patients in whom immune variant of scleroderma is associated with
deficiency is suspected) because of the anti-centromere antibodies (which can be
risk of iatrogenic infection. remembered because they both begin with
'c'). In contrast, the diffuse variant is
Question 10. A) Thymus commonly associated with anti-Scl70.
Explanation: DiGeorge syndrome is a
congenital disorder characterised by Question 14. E) Sjögren's syndrome
thymic aplasia. Normally, T lymphocytes Explanation: Anti-Ro/La antibodies are
mature in the thymus; in the absence of a associated with Sjögren's syndrome and
thymus, T cells fail to mature and patients systemic lupus erythematosus (SLE).
are susceptible to fungal, viral and Drug-induced SLE is associated with anti-
mycobacterial infections. Patients with histone antibodies.
DiGeorge syndrome also have failure of
parathyroid development, but this causes Question 15. B) Wegener's
abnormal calcium metabolism and does granulomatosis
not lead to immune deficiency. Explanation: Wegener's granulomatosis is
commonly associated with the presence of
Question 11. B) HLA-B27 a c-ANCA, and is often also associated
Explanation: Patients who carry HLA-B27 with antibodies to proteinase 3. Churg-
are 90 times more likely than other Strauss syndrome is associated with p-
individuals to develop ankylosing ANCA.
spondylitis.
Question 16. A) Extracellular
Question 12. C) ABO blood transfusion deposition of insoluble proteins
reaction Explanation: The amyloidoses are
Explanation: Type II reactions are characterised by the extracellular
characterised by antibodies to cell surface deposition of insoluble protein complexes.
antigens. In ABO transfusion reactions,

124
4
ENVIRONMENTAL AND
Chapter NUTRITIONAL FACTORS IN
DISEASE

Question 1. E) Incidence of the disease develop over a few days and include
Explanation: Incidence of a disease (often increased production of 2,3-DPG and
confused with prevalence) refers to the erythropoietin. Epo increases red cell
number of new cases occurring in the production while 2,3-DPG shifts the
population at risk during a defined period oxyhaemoglobin curve to the right,
of time. Prevalence is determined by both thereby decreasing the affinity of the
the incidence and the duration of the haemoglobin molecule for oxygen
disease. (therefore oxygen is 'off-loaded' to the
tissues more readily). Hyperventilation
Question 2. A) 5 occurs, rather than hypoventilation,
Explanation: The relative risk is the stimulated by hypoxic drive.
incidence in the exposed population
divided by the incidence in the unexposed Question 6.
population. In this case, that means 150 C) High-altitude pulmonary oedema
cases per 3000 people living in village X Explanation: This is a fairly classical
versus 30 cases per 3000 people living description of high-altitude pulmonary
remote from the nuclear power plant: oedema (HAPE). The most important
(150/3000)/(30/3000) = 150/30 = 5 treatments are immediate descent and
administration of oxygen.
Question 3. D) Ulcerative colitis
Explanation: Smoking tobacco Question 7. B) Visual loss
dramatically increases the chances of Explanation: Acute mountain sickness is
developing a large number of diseases, a common syndrome in travellers to
including A, B, C and E. However, it has altitudes of > 3000 m; it comprises
been noted in observational studies that headache, fatigue, anorexia, nausea and
smokers with ulcerative colitis appear to vomiting, difficulty sleeping and
have fewer flares in their disease. dizziness. Visual loss, as well as
behavioural change, confusion and loss of
Question 4. C) NRT plus behavioural consciousness, is more likely to be seen as
support part of the rare and life-threatening
Explanation: NRT plus behavioural syndrome, high-altitude cerebral oedema
support leads to rates of abstinence at 6 (HACE).
months of up to 12%. In the absence of
any intervention, the rate of success is Question 8. C) Metabolic alkalosis
only of the order of 2%. Explanation: Near-drowning is inevitably
characterised by hypoxaemia and a
Question 5. A) The oxyhaemoglobin metabolic acidosis. Acute lung injury
dissociation curve moves to the right results from inhalation of water, leading to
Explanation: The oxyhaemoglobin curve a ventilation-perfusion mismatch. This
describes the relationship between the tends to resolve over 48-72 hours unless
amount of O2 bound to haemoglobin and infection supervenes. Other complications
the partial pressure of oxygen. In response of near-drowning include haemoptysis,
to the reduction in partial pressure of hypotension, dehydration and cardiac
oxygen at high altitude and the resulting arrhythmias, in addition to hypothermia.
fall in arterial oxygen saturation, a number
of physiological adaptations occur. These

125
Question 9. D) Linoleic acid nutrition, hyperosmolar solutions (which
Explanation: NSP is, in its simplest are required in those patients with high
terms, the natural packaging of plant nutritional needs) may be administered
foods. By definition, dietary fibre cannot only into central veins, as they would
be digested by human enzymes. It cause thrombophlebitis if administered
therefore passes into the colon, improving peripherally. Although aspiration
transit and preventing constipation. pneumonia is a recognised complication of
Linoleic acid is a polyunsaturated fatty feeding by nasogastric tube, feeding via a
acid, i.e. a lipid rather than a carbohydrate. central line carries a higher overall risk of
infection.
Question 10. C) 9 kcal
Explanation: Fat is the most energy-dense Question 13. B) Combined oral
component of the human diet, providing 9 contraceptive pill
kcal/g. This compares with carbohydrate Explanation: Weight gain is a well-
and protein, both of which provide 5 recognised side-effect of oestrogen
kcal/g. Alcohol has a calorific value of 7 therapy. Metformin may lead to minor
kcal/g. weight reduction and is usually the most
appropriate first-line drug treatment for
Question 11. C) Some essential amino glycaemic control in obese patients with
acids cannot be obtained from type 2 diabetes. Sibutramine is one of the
vegetarian diets, necessitating the use of drug treatments available for obesity and
supplements reduces food intake through ß1-
Explanation: While the biological value adrenoceptor and 5-hydroxytryptamine
of protein of animal origin is generally receptor agonist activity in the central
higher than that of vegetable origin, a nervous system.
suitable combination of different vegetable
sources can provide the full range of Question `14. E) Obese people have a
essential amino acids with a sufficient higher basal metabolic rate than lean
protein nutritive value. This is known as people
the complementary value of proteins, and Explanation: Obesity is defined as a BMI
is the basis on which adequate protein may > 30 (weight in kilograms divided by
be obtained from a vegan or vegetarian height in metres squared). One of the few
diet without the use of supplements. 'benefits' of smoking is that it protects
against obesity, although of course it has
Question 12. E) Feeding via a central many other adverse effects. There are a
(subclavian or internal jugular) line number of potentially reversible causes of
carries a higher risk of sepsis than obesity, including hypothyroidism,
feeding via a percutaneous endoscopic Cushing's syndrome and insulinomas,
gastrostomy (PEG) tube which should be considered, particularly if
Explanation: Hospitalised patients are at the patient gives a history of having
high risk of nutritional deficiency; their recently begun to gain substantial weight
requirements are often increased by for the first time or at a faster rate than
illness, and yet their capacity to consume previously. Although approximately 60%
an adequate diet may be impaired. Many of variation in BMI is attributable to
therefore require some form of nutritional genetic factors, the genetic basis for
support. Whenever possible, the enteral obesity remains poorly understood. Single
route should be used as this preserves the mutations in a few genes (e.g. leptin) do
integrity of the mucosal barrier, reducing cause severe childhood obesity but in the
the risk of bacteraemia. 'Supplement' vast majority it is a polygenic disorder.
drinks should be used to support, not The basal metabolic rate of obese
replace, a normal diet. With parenteral individuals is actually higher than that of
lean subjects.

126
Question 15. C) Osteoporosis when it is deficient, defective collagen
Explanation: Obesity has adverse effects formation impairs healing of wounds,
on both mortality and morbidity. However, causes capillary haemorrhage and reduces
it protects against osteoporosis by platelet adhesiveness (normal platelets are
increasing both mechanical stress on bones rich in ascorbate). This manifests
and sex steroid synthesis in adipose tissue. clinically as perifollicular and petechial
haemorrhages, easy bruising, swollen
Question 16. A) Bariatric gastric gums which bleed easily, and occasionally
surgery gastrointestinal bleeding. Ascorbic acid is
Explanation: All of the interventions also a powerful reducing agent, which
above can be used in the appropriate converts the ferric form of iron found in
circumstances as part of the management the diet to the more readily absorbable
of obesity. Bariatric surgery is the most ferrous form, thereby increasing body iron
effective in the short and long (10-year) absorption.
term, although it also has significant risks
of complications. Question 20. C) Fluoride Discoloration
of teeth
Question 17. D) Vitamin C Explanation: There are about 16 essential
Explanation: Vitamin C is water-soluble. inorganic elements in the human diet. It is
This means that a large proportion of the chronic excessive intake of fluoride (e.g. >
vitamin C content of vegetables and fruit 10 ppm in the water supply) that causes
is lost when these foods are boiled in discoloration of teeth, via a process known
water. as fluorosis. Fluoride increases the
resistance of enamel to acid attack and
Question 18. B) Vitamin B2 protects against dental caries. A national
(riboflavin) Beri-beri policy of fluoridating the water in the
Explanation: A deficiency in vitamin B1 Republic of Ireland was introduced in the
(thiamin) causes beri-beri. This manifests 1950s, following which the country's
in three different forms: edentulous rate fell by more than 60%.
• infantile, occurring in the breastfeeding The addition of traces of fluoride (at 1
children of thiamin-deficient mothers ppm) to public water supplies is now
• dry or neurological beri-beri, widespread practice.
characterised by peripheral neuropathy
and wrist/foot drop Question 21. B) Increased skinfold
• wet beri-beri, characterised by thickness over the triceps
generalised oedema, due to congestive Explanation: In addition to body mass
cardiac failure. index, a useful measurement in the
Deficiency in vitamin B2, riboflavin, is assessment of nutritional status is skinfold
very rare because it is found in many food thickness over the triceps; this is decreased
types. When deficiency does occur, it in malnutrition due to loss of subcutaneous
mainly affects the tongue and lips, fat. This measurement can be subtracted
manifesting as glossitis, angular stomatitis from mid-arm circumference to estimate
and cheilosis. muscle mass. Many of the clinical features
of starvation represent physiological
Question 19. A) Vitamin C adaptations to undernutrition. For
Explanation: Elderly people who are example, it is disadvantageous to have
socially and geographically isolated may children born to an undernourished mother
not have ready access to fresh fruit and and into a famine environment, so
vegetables, the most abundant source of reproductive function is suppressed as part
vitamin C (ascorbic acid). This vitamin is of the physiological response to
important in the formation of collagen; undernutrition.

127
Chapter 5 PRINCIPLES OF
INFECTIOUS DISEASE

Question 1. C) Prions are highly given to children at the age of 2 months or


complex structures consisting of > 100 as soon as possible thereafter, as natural
different proteins immunity declines. MMR is given during
Explanation: Prions are the simplest of all the second year of life. Uptake of this
infectious agents, consisting of a single vaccine has declined in recent years
protein molecule which can catalyse a following reports of a link between receipt
change in a host protein. Viruses only of the vaccine and the development of
contain one type of genetic material, either autism. These claims are without
DNA or RNA, and require a host cell foundation, but have unfortunately shaken
(either prokaryotic or eukaryotic) in order public confidence in the vaccine.
to reproduce. Fungal cells have a defined
nucleus, in contrast to bacteria, whose Question 5. A) Yearly receipt of the
nuclear material is not organised into a polyvalent flu vaccine
specific organelle. Explanation: Asplenic patients have
heightened susceptibility to encapsulated
Question 2. D) Hepatitis A Needlestick organisms such as Streptococcus
injury pneumoniae, Neisseria meningitidis and
Explanation: In contrast to hepatitis B Haemophilus influenzae. While the flu
and C, which are transmitted by sexual vaccine is not contraindicated in asplenic
contact, blood transfusions and needlestick patients, it provides protection against a
injuries, hepatitis A is spread by the viral infection, and is, therefore, not
faecal-oral route. mandatory. All asplenic patients should
have prophylactic antibiotics prior to
Question 3. A) Pertussis dental procedures, as there is a risk of
Explanation: All of the above apart from disseminated infection. Daily penicillin
pertussis are live attenuated vaccines and prophylaxis is also a standard
should not cause infection in the recommendation. All asplenic patients
immunocompetent recipient. However, should be encouraged to carry a card or
they should be given with caution or wear a MedicAlert bracelet explaining
avoided in those who are their condition.
immunosuppressed or pregnant, or who
have an acute infection. Inactivated Question 6. E) Meningococcal
vaccines include pertussis, HiB meningitis
(Haemophilus influenzae type B), and Explanation: Certain diseases which are
pneumococcal. The 'awkward one' in this thought to have important implications for
classification system is poliomyelitis, for public health are classified as 'notifiable',
which both a live attenuated and an meaning that notification of cases to the
inactivated form exist. The Salk form is local public health authority is
inactive (remember the 'k' in Salk is also in compulsory. This is so that outbreaks of
killed) and the Sabin form is live. highly contagious conditions can be
recognised, and appropriate measures put
Question 4. D) MMR (measles, mumps, in place to control further spread of the
rubella) vaccine condition. In the UK both meningitis (of
Explanation: With the exception of the any cause) and meningococcal sepsis are
MMR vaccine, all of the above should be notifiable.

128
Question 7. C) Neisseria Question 10. D) Prevention of gas
gonorrhoeae Direct microscopy gangrene in a patient with a
Explanation: Strategies for identifying contaminated wound Flucloxacillin
micro-organisms can be broadly Explanation: The recommended
categorised into direct demonstration (e.g. prophylaxis for gas gangrene is
microscopy with or without appropriate benzylpenicillin 600 mg 6-hourly for 5
staining), molecular diagnostic methods days or metronidazole 500 mg 8-hourly
(e.g. PCR) and immunodiagnosis (e.g. for 5 days. Although chicken pox is
ELISA, Western blot, common in childhood and the vast
immunofluorescence). Strep. pyogenes is majority survive without sequelae, it is an
most commonly identified on culture, important cause of mortality in newborn
hepatitis B is identified initially by infants and in severely
immunodiagnosis, and if positive, viral immunocompromised patients. Both
load is subsequently determined by PCR. groups should therefore receive varicella
Gonococcus is most often identified on immune globulin if they have been
microscopic examination of biological exposed to the virus. The rationale behind
fluid such as urine or urethral discharge endocarditis prophylaxis prior to dental
(Gram-negative diplococci); sensitivities procedures is that transient bacteraemia,
may subsequently be determined on from organisms commensal to the mouth,
culture. Mycobacterium tuberculosis is often develops following invasive dental
also identified initially by microscopy procedures and may colonise damaged or
aided by the use of Ziehl-Neelsen staining. artificial heart valves, leading to
Candida albicans may be identified either endocarditis.
on direct microscopy or on culture of
biological fluids. Question 11. C) Most patients with
established allergy to benzylpenicillin
Question 8. C) Ciprofloxacin Inhibits are also allergic to cephalosporins
DNA gyrase, thereby blocking DNA Explanation: Approximately 10% of
replication penicillin-sensitive patients are allergic to
Explanation: Knowledge of how cephalosporins, although anaphylaxis to
antibiotics work is important for second- and third-generation
understanding their spectrum of activity cephalosporins is exceedingly rare, even in
and mechanisms of resistance. the presence of established penicillin
allergy. Beta-lactam antibiotics are
Question 9. A) Minimum inhibitory bactericidal, meaning that they kill
concentration (MIC) is the lowest bacteria. They act by inhibiting synthesis
concentration of an antibiotic required of the bacterial cell wall, which is essential
to inhibit 50% of the colonies of a for survival of bacteria in osmotically
particular organism challenging environments.
Explanation: The MIC is the lowest
concentration of an antibiotic required to Question 12. C) Clarithromycin
inhibit 90% of the colonies of a particular Explanation: Clarithromycin is a
organism. macrolide antibiotic useful in the treatment
of infections caused by Gram-positive,
rickettsial, chlamydial and Mycoplasma
infections. Had Legionella, another
'atypical' organism, been identified as the
cause (best diagnosed by polymerase chain
reaction detection of the Legionella
antigen in urine), then rifampicin would be
an appropriate antibiotic to add.

129
Question 13. A) IV ceftriaxone and Although newer quinolones such as
metronidazole moxifloxacin and levofloxacin have
Explanation: The history suggests the enhanced anti-Gram-positive activity,
possibility of an aspiration pneumonia, ciprofloxacin has excellent anti-Gram-
and empirical antibiotic therapy must negative activity with little or no anti-
therefore cover Gram-negative organisms Gram-positive activity. Side-effects of this
and anaerobes in addition to the usual drug are infrequent and generally mild. Its
causes of community-acquired pneumonia. oral bioavailability, as mentioned above, is
Anti-anaerobic activity will be provided excellent, but is impaired by co-
by metronidazole while ceftriaxone, a administration of food, antacids, ferrous
third-generation cephalosporin, has sulphates and multivitamins.
excellent Gram-negative activity as well as
providing Gram-positive cover. Given the Question 16. E) They have good oral
history of recent stroke and possible bioavailability
aspiration, the method of administration of Explanation: No useful oral absorption of
the antibiotic may be of importance, and glycopeptide antibiotics is achieved, and
the IV route may be the most appropriate. administration is always by the IV route
(except in the treatment of Clostridium
Question 14. D) Hepatotoxicity difficile diarrhoea). The anaphylactoid
Explanation: Aminoglycoside antibiotics reaction caused by rapid histamine release
are highly nephrotoxic, an effect from mast cells in response to a rapid
potentiated by concomitant administration infusion of vancomycin is known as 'red
of vancomycin, cisplatin, amphotericin B man' syndrome. Nephrotoxicity is another
or contrast media. The ototoxicity results potential side-effect, the risk of which is
from the enhanced cellular penetration greatly reduced by monitoring of antibiotic
demonstrated by aminoglycosides in the serum levels to ensure that toxic
hair cells in the cochlea. Hearing damage concentrations are not reached. No
and vertigo tend to be permanent. antibiotic in this class has any significant
Aminoglycosides impair neuromuscular activity against Gram-negative organisms
transmission and should not be given to but both vancomycin and teicoplanin are
patients with myasthenia gravis. Larger useful against MRSA.
doses given rapidly by the IV route may
result in a transient myasthenic syndrome Question 17. D)
in those with normal neuromuscular Ketoconazole Nephrotoxicity
function. Gastrointestinal upset is a Explanation: Ketoconazole is an oral
relatively common adverse effect but antifungal agent that can cause
hepatotoxicity does not occur. In order to hepatotoxicity. 'Grey baby syndrome'
avoid important complications, serum describes the dose-dependent cyanosis and
levels of the aminoglycosides must be circulatory collapse that occur in infants
checked on a regular basis, and the dosing due to inability to conjugate
regimen adjusted to ensure that toxic chloramphenicol and excrete it in urine.
levels are not reached. All tetracycline antibiotics bind to metallic
ions in bones and teeth, causing
Question 15. B) It has potent anti- discoloration, and therefore should be
Gram-negative properties, but little or avoided in children and in pregnancy.
no anti-Gram-positive activity Tetracyclines also photosensitise and
Explanation: Fluoroquinolones such as patients should be warned about this and
ciprofloxacin have excellent oral advised to use sun-screen. Erythromicin
bioavailability, a large volume of causes a cholestatic jaundice in some
distribution and good intracellular patients, which abates when the drug is
penetration (the drug concentrates in stopped.
phagocytes, facilitating its distribution).

130
Chapter 6 AGEING AND DISEASE
Question 1. D) Increased insulin Question 4. C) Stop amlodipine
secretion Explanation: Postural hypotension (drop
Explanation: Ageing is a normal in BP of > 20 mmHg systolic or > 10
physiological process and effects are seen mmHg diastolic on standing from supine)
in several organ systems. They do not is a cause of dizziness and falls, and is
usually interfere with organ function under often related to medication. This patient
normal conditions, but reserve capacity is has a documented postural BP drop, and a
reduced and so organ failure can be trial without amlodipine (a
precipitated by minor illness. Pancreatic _- dihydropyridine calcium channel blocker)
cell function declines with age, resulting in would be reasonable. Neither simvastatin
reduced insulin secretion. nor metformin causes postural
hypotension and there is no compelling
Question 2, D) Omeprazole reason to stop the drug. Fludrocortisone is
Explanation: Falls are an extremely a mineralocorticoid that causes salt and
common presentation in elderly patients water retention, which can be used to treat
and are often multifactorial. Sedative postural hypotension; however, it can
drugs (e.g. benzodiazepines, precipitate heart failure and should be used
antipsychotics, antidepressants), opiates with caution when withdrawal of
and diuretics are common culprits. hypotensive medication fails to resolve the
Polypharmacy in itself is also a risk factor. problem.
Proton pump inhibitors are generally well
tolerated and cause relatively few side- Question 5. D) Alzheimer's disease
effects. Explanation: Almost any acute illness can
present with confusion in the elderly;
Question 3. E) Bisphosphonate therapy examples include infections, vascular
Explanation: Elderly patients who present events, metabolic derangements and drugs
with recurrent falls are often frail with (particularly opioid analgesics and
multiple medical problems. The annual sedatives). Alzheimer's disease causes
risk of falling increases linearly with the chronic cognitive impairment rather than
number of risk factors, from 8% with no acute confusion. On the other hand, acute
risk factors to 78% with four or more. In cerebrovascular events may occur in
patients who experience recurrent falls, vascular dementia and can cause acute
osteoporosis prophylaxis with calcium and confusion.
vitamin D supplements, and treatment of
established osteoporosis with
bisphosphonates are important
considerations that may reduce the risk of
fracture.

131
Question 6. E) Laxatives may be helpful would therefore be the most useful initial
Explanation: Urinary incontinence affects test. Echocardiography may be helpful if
15% of women and 10% of men aged over there are clinical signs of valvular disease
65. It should never be considered to be due or an abnormal ECG. CT brain scans are
to ageing alone and requires investigation. useful if cerebrovascular disease or mass
Causes of incontinence include severe lesions are suspected, but these tend not to
constipation (treatable with laxatives) and present with episodic lightheadedness
hypercalcaemia. Catheterisation should alone.
not generally be considered as first-line
treatment but may be needed in refractory Question 8. C) Chronological age
cases or if there is local skin damage. Explanation: Decisions about the
appropriate level of intervention in elderly
Question 7. B) 24-hour ambulatory patients depend on individual
ECG circumstances and often require careful
Explanation: The self-limiting nature of thought. Multiple comorbidities, likely
this patient's symptoms, together with the tolerance of interventions, patient and
absence of a postural element or other family wishes, and advance directives
associated symptoms, suggests that ('living wills') should all be borne in mind.
transient arrhythmias may be the cause. Many elderly people remain fit and active
Ageing is associated with an increased into their nineties, whereas others become
incidence of sinoatrial disease. severely disabled by illness in their
Furthermore, this patient is on rate- seventies; 'biological' age is therefore
limiting treatment in the form of a _- generally more useful than 'chronological'
blocker. A 24-hour ambulatory ECG age in making decisions about treatment.

132
Chapter 7 CRITICAL CARE AND
EMERGENCY MEDICINE

Question 1. E) 250 ml of gelofusine over Question 4. B) Pulmonary embolism


15 minutes Explanation: Interpreting these data may
Explanation: The falling blood pressure, initially seem daunting. However, a logical
urine output and central venous pressure, approach to the causes of shock leads to
combined with thirst and tachycardia, are the correct answer.
highly suggestive of hypovolaemia. The There are a limited number of problems
diagnosis can be confirmed and partly that can cause shock. We can immediately
treated by giving a rapid fluid challenge rule out the 'distributive' causes (septic,
with gelofusine or saline. O-negative anaphylactic and neurogenic) because we
blood is precious and administration is know that cardiac output is low. This
associated with a higher complication rate, leaves hypovolaemic, cardiogenic and
so it should be conserved for acutely life- obstructive causes. Hypovolaemia is less
threatening emergencies. likely because the CVP is high. CVP
represents right atrial pressure, whereas
Question 2. C) Left atrial pressure PAWP represents left atrial pressure.
Explanation: As direct measurement of Cardiogenic shock tends to affect both
left atrial pressure is not practical, PAWP ventricles, so both CVP and PAWP would
is used as a surrogate. be elevated. The fact that PAWP is low
and CVP is high suggests some kind of
Question 3. B) Systemic arterial obstruction between the right and left
pressure ventricles. This pattern is typical of the
Explanation: Central venous pressure haemodynamic compromise caused by
(CVP) or right atrial pressure (RAP) is massive pulmonary embolus.
often measured in critically ill patients to If you found this question tricky, try
help guide intravascular fluid replacement. having a look at the link below. There is
Accurate interpretation of CVP values another question on shock later in the
requires a knowledge of the physiological chapter to test your understanding.
factors that influence it.
Central venous pressure is largely
determined by the combination of:
• intravascular volume
• venous tone
• pulmonary artery pressure (and right
ventricular function)
• intrathoracic pressure.
Systemic arterial pressure does not directly
affect right atrial pressure but may affect it
indirectly, through altering one of the
above parameters. For example, persistent
systemic hypertension may lead to left
heart failure, which in turn may lead to
increased pulmonary artery pressure,
ultimately resulting in a rise in right atrial
pressure.

133
Question 5. A) 1 ml of 1 in 1000 Question 7. D) Pulmonary artery wedge
adrenaline pressure (PAWP) < 15 mmHg
Explanation: Adrenaline (epinephrine) is Explanation: The four defining
one of the few drug doses that it is characteristics of ARDS are: hypoxaemia,
important to memorise, because there bilateral infiltrates on chest X-ray, PAWP
won't be much time to check when you < 15 mmHg (this rules out left ventricular
need to use it. Low volumes are used for failure as the cause of the pulmonary
intramuscular injections, and in true pathology) and decreased lung
anaphylaxis the dose is 1 mg. If it is given compliance.
intravenously, it is more likely to cause
arrhythmia, so the dose is titrated in 0.1 Question 8. A) Diamorphine overdose
mg boluses by an experienced doctor. Explanation: The others are all common
What mass of adrenaline is in 1 ml of each causes of type 1 respiratory failure in the
of the solutions listed? The system seems critically ill. Diamorphine overdose
unnecessarily complex but can be easily decreases respiratory drive, which
understood by memorising one key fact: decreases ventilation, which allows CO2 to
that all dose strengths refer to grams per accumulate.
millilitre.
Thus, 1 in 1000 is 1 gram in 1000 Question 9. B) Pulmonary oedema
millilitres = 1 gram/litre = 1 mg/ml. This secondary to left ventricular failure
is the concentrated solution for (LVF)
intramuscular injection in anaphylaxis. Explanation: CPAP is an effective
Likewise, 1 in 10 000 has 1 gram in 10 treatment to improve oxygenation in LVF.
000 millilitres = 0.1 grams/litre = 0.1 It does not increase ventilation (the
mg/ml. This is the concentration found in volume of air shifted in and out of the
minijet syringes on cardiac arrest trolleys. lungs) so it is not a good treatment for
In the context of a cardiac arrest, it is disorders characterised by hypoventilation,
given as a 10 ml bolus. In other situations, such as COPD, narcotic drug overdose and
it is normally titrated in 1 ml boluses. neuromuscular disorders. These patients
Finally, 1% means 1 gram in 100 may benefit from bi-level positive airway
millilitres = 10 grams/litre = 10 mg/ml. pressure, which does support ventilation.
Drug overdose patients are not normally
Question 6. treated in this way because they are at risk
B) Noradrenaline (norepinephrine) of aspiration pneumonitis, so the airway
Explanation: Noradrenaline must be protected with a cuffed
(norepinephrine) acts primarily in the endotracheal tube.
peripheral vasculature, causing intense CPAP is contraindicated in acute severe
vasoconstriction, with relatively little asthma as it worsens hyperexpansion, so
direct effect on the heart. Its major clinical decreasing ventilation, which may be fatal.
use is in hyperdynamic sepsis, when there
is low systemic vascular resistance and
blood pressure despite a high cardiac
output. In these circumstances
vasoconstriction is desirable to restore BP,
even at the expense of a small reduction in
cardiac output.
Dobutamine and dopexamine are
sometimes called 'ino-dilators', since they
increase cardiac contractility whilst
simultaneously causing systemic
vasodilatation. Sodium nitroprusside and
GTN are both nitric oxide donors that
cause profound systemic vasodilatation.

134
Question 10. C) Arterial bicarbonate that maintaining PaCO2 at the low end of
(HCO3-) must be lower in the second the normal range is the best management
ABG for control of intracranial pressure. This
Explanation: The first ABG shows causes mild cerebral vasoconstriction but
normal acid-base status and significant not enough to cause cerebral ischaemia.
hypoxia on air. The second ABG shows a Normal saline is used to prevent plasma
metabolic acidosis, and the patient is still sodium falling below 140 mmol/l.
very hypoxic despite receiving 40% O2.
The PaO2 should have increased but it has Question 14. D) Pneumonia
hardly changed at all, so pulmonary gas Explanation: Ranitidine reduces gastric
exchange has probably worsened in the acid secretion and is used to prevent stress
second ABG. ulceration. However, its use does increase
The metabolic acidosis is easy to diagnose the development of ventilator-associated
without being told the HCO3-. PaCO2 has pneumonia in patients on long-term
decreased but the patient is acidaemic. ventilation.
These H+ ions must have come from
somewhere but they have not come from Question 15. C) Central venous
CO2. Thus, there must be a metabolic catheterisation
acidosis. Explanation: This patient is almost
certainly going to require aggressive fluid
Question 11. D) Hyponatraemia resuscitation. Monitoring his central
Explanation: Hyponatraemia is venous pressure (CVP) will help to guide
particularly likely to cause coma when its the rate of intravascular fluid replacement
onset is acute. The other options do not and may help to reduce the risk of
directly obtund consciousness. precipitating pulmonary oedema.
There is no clear indication for ventilatory
Question 12. C) Disseminated support in this patient; he is sufficiently
intravascular coagulation (DIC) alert to communicate his symptoms, and
Explanation: Systemic sepsis may his SpO2 is 96% on air. Inotropes should
provoke DIC, a potentially life-threatening not be started at this point because
coagulopathy. This characteristically adequate volume resuscitation has not
consumes fibrinogen, platelets and clotting been achieved. There is no indication at
factors, and leaves the circulating blood present for haemofiltration. Diuretics are
lacking in adequate clotting factors to likely to be counterproductive since he
support normal clotting. This leads to already appears to be fluid-deplete.
abnormal bleeding and prolonged APTT
and PT. Question 16. B) Septic shock
Explanation: Hypovolaemia and
Question 13. D) Mild hyperventilation cardiogenic and obstructive shock are all
to reduce PaCO2 to 4-4.5 kPa associated with a low cardiac output.
Explanation: Intracranial pressure must Shock with a high cardiac output may be
be prevented from rising in head injury caused by sepsis, anaphylaxis, neurogenic
patients. Simple measures, such as a 30° shock and vasodilating drugs. There is no
head-up tilt, and tight control of fluid history of rash, bronchospasm,
balance to prevent dehydration or fluid angioedema or sudden circulatory
overload, are believed to help. PEEP collapse, so we can rule out anaphylaxis.
should be avoided, since it reduces venous Likewise, there is no history of spinal cord
return from the brain, potentially injury or administration of a large dose of
increasing intracranial pressure in head- a vasodilating drug.
injured patients. Current evidence suggests

135
Chapter 8 POISONING
Question 1. C) Organophosphate carbamazepine and dapsone poisoning.
insecticides Some drugs, particularly small molecules
Explanation: These are the classic acute and ions, are poorly adsorbed by activated
features ('toxidrome') of organophosphate charcoal.
poisoning. A full description of the
syndrome is found in the link below. Question 5. D) N-acetylcysteine
Cholinergic symptoms are a prominent Explanation: Intravenous infusion of N-
feature of organophosphate poisoning. A acetylcysteine is the standard treatment for
popular mnemonic for cholinergic paracetamol overdose in the UK.
symptoms is 'sludge': Methionine is a suitable oral alternative.
Salivation lacrimation
Urination diarrhoea Question 6. E) Communicate with the
gastrointestinal cramps semesis mother to be certain of the timing of the
overdose and to identify possible co-
Question 2. D) Ensure a patent airway ingestants
Explanation: The priority in a patient Explanation: Even in large doses in high-
with poisoning is the same as in any risk patients, paracetamol overdose is
acutely unwell patient: assessment of the often asymptomatic in the first 24 hours.
airway, breathing and circulation (ABC). The coma, hypotension and bradycardia
Assessing the airway is usually a simple cannot be explained by paracetamol alone
matter of asking the patient to speak, but and the most likely explanation is
requires a more detailed assessment in the concomitant overdose of another drug
unconscious patient. If there are any signs (probably opiates or benzodiazepines).
of compromise, a chin lift or jaw thrust Activated charcoal is not recommended
should be performed. All of the other for patients presenting more than 1 hour
measures are appropriate, but ultimately after paracetamol overdose. N-
futile if the airway is not patent. acetylcysteine can cause anaphylactoid
reactions; it is effective if given in the first
Question 3. E) Aspirin 10 hours after overdose, so it is safer to
Explanation: Aspirin is widely available wait for a paracetamol level before starting
and dangerous in overdose, so it is treatment. Vitamin K has no place in the
important to understand the key features of acute management of paracetamol
the management of salicylate poisoning. overdose. In fact, giving vitamin K would
Only a few drugs are 'able' to be removed be counterproductive because the
by haemodialysis and most of them can be prothrombin time is used to monitor
remembered easily: progress and potential need for liver
transplantation. Paracetamol levels less
Question 4. B) Ethanol than 4 hours after overdose are
Explanation: With the exception of uninterpretable.
ethanol, all of the drugs listed bind to
activated charcoal. It is usually only of
benefit if given within an hour of
ingestion, although repeated doses can be
used for theophylline, quinine,

136
Question 7. C) Ecstasy (MDMA) the cause of the acidosis, the lactate would
Explanation: The presentation includes be elevated. A raised anion gap metabolic
the classic features of ecstasy (MDMA) acidosis is a classic feature of advanced
abuse. GHB causes sedation and coma, salicylate poisoning, but methanol
and LSD is a hallucinogen with relatively poisoning is more likely in this case, as it
low intrinsic toxicity. often causes blindness and is a common
cause of poisoning in alcoholics. Methanol
Question 8. B) Naloxone poisoning is managed by administration of
Explanation: Although this presentation ethanol or 4-methylpyrazole (4MP), which
may be due to an overdose of opiates or competitively inhibits alcohol
benzodiazepines, the specific dehydrogenase and thus limits formation
benzodiazepine antagonist flumazenil is of toxic metabolites such as formate,
rarely used, as it can precipitate seizures which cause blindness.
and lethal cardiac arrhythmias. On the
other hand, naloxone, an opioid Question 11. E)Glucagon
antagonist, is relatively safe although, Explanation: N-acetylcysteine and
following administration, the patient may methionine both replenish hepatic
wake up very quickly and become agitated glutathione to protect the liver from the
or violent, especially if too much is given effects of paracetamol toxicity; calcium
too quickly. Aim to titrate to a Glasgow gluconate is used in overdose with calcium
coma score of 13-14. channel blockers; naloxone is effective in
opioid overdoses. IV glucagon and
Question 9. E) Digoxin adrenaline (epinephrine) are used if
Explanation: This is a classic description needed in overdoses with _-blockers
of digoxin toxicity, which due to
dependence on renal clearance and a Question 12. A) Iron
narrow therapeutic index, can occur Explanation: Desferrioxamine is a
without intentional overdose. The chelating agent used in severe iron
yellowing of vision is known as poisoning.
xanthopsia. Verapamil overdose can cause
vomiting and bradycardia but no visual Question 13. B) Diamorphine
disturbance. Quinine overdose causes Explanation: Opiate overdoses cause
visual disturbance, deafness, and nausea small 'pinpoint' pupils and respiratory
and vomiting. With large overdoses, depression. Overdose with tricyclic
cardiac effects can occur, including antidepressants, alcohol, cocaine,
impaired conduction and amphetamine or antihistamines can cause
tachyarrhythmias. It does not cause the large pupils. Benzodiazepines such as
classic 'reverse-tick' ECG changes seen temazepam cause respiratory depression,
with digoxin toxicity. Fluoxetine is but do not cause small pupils.
relatively safe in overdose; the major side-
effect of selective serotonin reuptake Question 14. E) Co-ingestion of ecstasy
inhibitors (SSRIs) is gastrointestinal upset. (MDMA)
Explanation: Malnourishment and
Question 10. D) Methanol poisoning anorexia deplete the liver of protective
Explanation: There is a raised anion gap glutathione. Alcohol, phenytoin and
metabolic acidosis. This indicates the several other drugs induce hepatic
presence of large quantities of an enzymes that increase the rate of
unmeasured anion, such as salicylate, conversion of paracetamol to highly toxic
ketone bodies, lactate or metabolites of metabolites
methanol. So, with the exception of
ethanol, all of the options listed could
cause this acidosis. DKA is unlikely with a
normal glucose, and if CO poisoning were

137
Question 15. D) Treat if the total 24- poison are ineffective and, indeed, are
hour dose was more than 150 mg/kg potentially harmful. Other investigations
Explanation: In a 'staggered overdose' may be necessary, but should not be
such as this, a paracetamol level is prioritised ahead of antivenin
uninterpretable and cannot reliably be used administration. For a full discussion on the
to guide treatment. Instead, the quantity of assessment and management of snake-bite
drug ingested per kg of body weight is victims, see the link below.
used as a guide. The patient does not
belong to a high-risk group so the dose to Question 18. D) Amitriptyline
determine the need for treatment will be Explanation: Overdose with tricyclic
150 mg/kg/24 hours. antidepressants such as amitriptyline,
alcohol, cocaine, amphetamines or
Question 16. C) Lithium antihistamines can cause large pupils.
Explanation: All of the other drugs listed Opiate and organophosphate overdoses
can cause severe bradycardia in overdose. cause small pupils.

Question 17. A) Inject 0.02 ml of saline- Question 19. A) Aspirin


diluted antivenin subcutaneously and Explanation: The management of
observe the injection site for 10 minutes overdose with aspirin, paracetamol or iron
Explanation: Anaphylactic reactions to is determined by the blood concentration,
antivenins are common and potentially the time since the overdose was taken and
lethal. Sadly, tourniquets and aspiration of the clinical features.

138
Chapter 9 MEDICAL PSYCHIATRY
Question 1. A) Mania or temporal lobe epilepsy. Pressure of
Explanation: Characteristic features of speech is characteristic of mania while
mania which may be apparent in the a flat affect is suggestive of depressive
mental state examination include disorder. Delusions are a strong pointer
restlessness, excess energy, elated mood, towards a psychotic disorder.
'pressure of speech' (rapid speech that is
difficult to interpret), 'flight of ideas' Question 5. C) It is rarely used due to
(flitting rapidly between loosely related the high incidence of serious side-effects
topics) and grandiose thoughts (e.g. Explanation: Despite its negative image
patients may believe they are particularly amongst the general public, ECT remains
gifted or have special powers). a highly useful treatment for severe
depressive disorder, particularly when
Question 2. C) Constipation complicated by psychosis or risk of
Explanation: Somatic symptoms may starvation. It is now remarkably safe, has
dominate the clinical picture in anxiety very few side-effects and is of proven
disorders. Diarrhoea is far more common efficacy in severe depressive disorder.
in anxiety than constipation, as evidenced
by the queue outside the toilets before Question 6. C) Electroconvulsive
final medical examinations! therapy
Explanation: Electroconvulsive therapy is
Question 3. E) Age under 45 reserved for the treatment of severe
Explanation: Although deliberate self- depressive disorder. Psychological
harm is more common in women than in treatments such as graded exposure,
men and in young adults than the elderly, cognitive behavioural therapy and
completed suicide is more common in men relaxation techniques have an important
and those over the age of 45. role in the management of anxiety
disorders. Beta-blockers help to alleviate
Question 4. A) Impaired cognition and the somatic symptoms of anxiety.
drowsiness Benzodiazepines are effective anxiolytics
Explanation: Impairment of cognition in the short term but readily lead to
and consciousness, particularly in a dependence and so are not a long-term
medical inpatient, would point to delirium treatment option. Where drug treatment is
as the cause of these hallucinations. required, an antidepressant with anxiolytic
Important causes to consider would properties such as citalopram is the
include infection, metabolic disturbance, treatment of choice.
hypoxia, alcohol withdrawal and drug
side-effects. The nature of the Question 7. C) Monoamine oxidase
hallucinations is also of diagnostic value inhibitors
as auditory hallucinations suggest a Explanation: These drugs are now rarely
functional psychosis such as prescribed in the UK but it is still
schizophrenia, while hallucinations in important to know of this potentially
other modalities (especially vision) dangerous interaction.
suggest an organic cause such as delirium

139
Question 8. E) Treatment should be Question 10. B) Poverty of speech
continued for at least 6 months to Explanation: First-rank symptoms are
reduce the high risk of relapse characteristic of schizophrenia and carry
Explanation: Cognitive behavioural high diagnostic value. Poverty of speech,
therapy and interpersonal therapy are as although not a first-rank symptom, is a
effective as antidepressants in the feature of chronic schizophrenia, along
treatment of mild to moderately severe with other 'negative symptoms' such as
depression, although the latter are blunted mood and apathy. More
preferred for severe depression. The information on the diagnosis of
various classes of antidepressant are schizophrenia can be found in the link
equally effective and hence choice is from the next question.
largely dictated by tolerability of side- Question 11. D) Ingestion of
effects and toxicity in overdose. SSRIs and amphetamines within the last 24 hours
newer antidepressants such as venlafaxine Explanation: Although the possibility of
cause less troublesome side-effects and are schizophrenia should still be considered,
less dangerous in overdose than tricyclics. recent drug misuse strongly suggests an
Although three-quarters of patients organic psychosis, which is one of the
respond to drug treatment, it commonly main differentials of schizophrenia.
takes up to 4 weeks for improvement to
occur and response within the first 2 Question 12. D) Driving under the
weeks is unusual. It is vital to explain this influence of alcohol
to patients on initiating treatment to Explanation: Alcohol consumption
maximise treatment concordance. associated with psychological, physical
and social problems constitutes harmful
Question 9. D) Diabetes mellitus use. Alcohol dependence is a more
Explanation: Lithium is the drug of restricted term, the criteria for which are
choice for the maintenance treatment of shown in the link below.
bipolar disorder. It has a narrow
therapeutic range, meaning there is only a Question 13. D) Wernicke's
small difference between the required encephalopathy
therapeutic dose and the dose at which Explanation: The combination of
toxicity develops. Features of lithium confusion, ataxia and nystagmus suggests
toxicity include tremor, nausea, vomiting Wernicke's encephalopathy. Other eye
and convulsions. Side-effects, as opposed signs, including ophthalmoplegia and
to toxic effects, occur even when levels ptosis, may also feature.
remain within the therapeutic range,
particularly with long-term treatment.
These include renal impairment,
hypothyroidism, nephrogenic diabetes
insipidus (not diabetes mellitus) and
weight gain. Lithium is highly teratogenic
and must never be prescribed in the first
term of pregnancy.

140
Question 14. C) High-dose intravenous Korsakoff's requires the immediate
thiamin (Pabrinex) administration of high-dose thiamin,
Explanation: Wernicke-Korsakoff usually in intravenous form (Pabrinex). IV
syndrome is a rare but important effect of dextrose may precipitate or exacerbate
chronic alcohol misuse caused by Wernicke's and must not be given before
deficiency of thiamin (vitamin B1). In adequate thiamin replacement in a patient
Wernicke's encephalopathy there is an in whom the diagnosis is suspected.
acute presentation with confusion, ataxia,
nystagmus and ophthalmoplegia. This Question 15. C) Self-induced vomiting
condition is reversible but without prompt or purgation
treatment it may progress to Korsakoff's Explanation: Recurrent bouts of binge
syndrome: severe and irreversible loss of eating followed by vomiting, purgation or
short-term memory with consequent dieting are a feature of bulimia nervosa
inability to make new memories. rather than anorexia nervosa.
Treatment of Wernicke's and prevention of

141
Chapter 10 ONCOLOGY
Question 1. D) Human papillomavirus Question 4. B) Alpha-fetoprotein (AFP)
and oesophageal cancer and bladder carcinoma
Explanation: Human papillomaviruses 16 Explanation: Tumour markers detected in
and 18 are associated with cervical intra- the blood can help in the diagnosis of
epithelial neoplasia (CIN) and subsequent certain types of cancer, but due to lack of
cervical carcinoma, not with oesophageal specificity are often more useful in
cancer. The detection of CIN by cervical monitoring response to treatment and
smear testing forms the basis of the UK identifying relapse. Alpha-fetoprotein
screening programme for cervical cancer. (AFP) may be elevated in hepatocellular
carcinoma and testicular cancers.
Question 2. A) Early menarche
Explanation: Risk factors for breast Question 5. B) Take blood cultures and
carcinoma include advancing age, early start broad-spectrum antibiotics
menarche, late menopause, post- immediately
menopausal obesity, personal or family Explanation: Neutropenic sepsis (fever of
history of breast malignancy, inheritance 38°C for over 1 hour in a patient with a
of BRCA 1 or 2 genes, some benign breast neutrophil count < 1.0 × 109/l) is most
conditions (e.g. lobular carcinoma in situ, often a complication of chemotherapy and
atypical hyperplasia), prolonged use of is a life-threatening emergency. Urgent
hormone replacement therapy, previous treatment with appropriate broad-spectrum
radiotherapy for Hodgkin lymphoma and IV antibiotics is essential and must not be
smoking. Early full-term pregnancy delayed pending results of cultures and X-
reduces the risk. Fibrocystic breast change rays. G-CSF can be used in selected cases
is a benign condition which results in to stimulate white blood cell production,
increased breast lumpiness and tenderness, but is not the initial treatment and should
but is not associated with breast cancer. In only be given on specialist advice. If the
MEN there is an increased risk of patient develops septic shock she may well
malignancy in endocrine glands; for require invasive monitoring, but this is
example, in MEN I, tumours affect the unnecessary in the current context.
pancreas, pituitary and parathyroids.

Question 3. B) Acanthosis nigricans


Explanation: Non-metastatic
manifestations of lung cancer include
finger clubbing, neurological problems
(e.g. Eaton-Lambert syndrome, cerebellar
signs) and excess hormone secretion
(ADH resulting in hyponatraemia, ACTH
leading to Cushing's syndrome, and
parathyroid hormone (PTH)-related
peptide causing hypercalcaemia).
Acanthosis nigricans, a velvety thickening
and pigmentation of the major flexures,
particularly the axillae, is associated with
tumours of the upper gastrointestinal tract.

Question 6. C) Spinal MRI scan

142
Explanation: The rapid onset of bilateral oedema of the upper body. Veins on the
upper motor neuron signs in a patient with chest wall become prominent as collateral
prostatic carcinoma is strongly suggestive channels open up. Treatment options for
of spinal cord compression secondary to SVCO include radiotherapy and stenting.
metastatic disease and is an oncological
emergency. Other important clues in the Question 9 B) Dexamethasone
history and examination would be Explanation: The CT scan shows multiple
sphincter disturbance or sensory loss. cerebral metastases associated with
Spinal MRI is the investigation of choice cerebral oedema; these are causing midline
to identify cord compression. CT is useful shift. The steroid dexamethasone reduces
to look at bony structures, but has poorer oedema and is therefore the initial
resolution than MRI for soft tissues such treatment of choice. Palliative
as the spinal cord and nerve roots. Nerve radiotherapy is often the next step, but
conduction studies and electromyography there is little evidence for this. Surgery in
are used in the assessment of peripheral the presence of multiple metastases and
neuropathies and myopathies. oedema would not be appropriate. There is
little evidence to support the use of
Question 7 A) Intravenous saline chemotherapy.
Explanation: Hypercalcaemia in Question 10 D) Peripheral
malignancy can occur due to metastatic neuropathy
bone disease or as a non-metastatic Explanation: Ionising radiation mediates
manifestation secondary to parathyroid its effect by causing damage to DNA.
hormone (PTH)-related peptide production Tumour cells and healthy cells are both
by tumour cells. The resulting osmotic affected, resulting in a range of adverse
diuresis often renders patients severely effects. Neuropathies are associated with
dehydrated. Treatment with large volumes various chemotherapeutic agents but not
of intravenous saline corrects the fluid typically with radiotherapy, although it
deficit and lowers serum calcium over a can cause a brachial plexopathy.
period of hours. Bisphosphonates (e.g.
pamidronate) lower serum calcium over a Question 11 E) It exerts its effect by
few days and can be given following acting as an oestrogen receptor
rehydration in severe cases. Forced antagonist
diuresis (using saline and furosemide) and Explanation: Tamoxifen is an oestrogen
calcitonin are occasionally used in very ill receptor antagonist and should be
patients. Calcium gluconate is used in the considered as adjuvant therapy in all
treatment of hypocalcaemia. women with breast cancer expressing
oestrogen receptors (ER); it has no role in
Question 8 D) Superior vena caval ER-negative breast cancer. Tamoxifen is a
obstruction hormonal treatment, not a
Explanation: This patient has features of chemotherapeutic agent, and is therefore
superior vena caval obstruction (SVCO), not associated with bone marrow
which is usually due to malignancy (e.g. suppression.
lung cancer, lymphoma). Obstruction of
the SVC leads to facial suffusion and

143
Chapter 11 PALLIATIVE CARE AND
PAIN MANAGEMENT

Question 1. D) Has a better response to Question 4. D) Respiratory depression is


non-steroidal anti-inflammatory drugs more likely to be a problem in acute
(NSAIDs) than tricyclic antidepressants dosing than with patients on regular
Explanation: Neuropathic pain is opioids
typically described as burning, stabbing or Explanation: Nausea and vomiting occur
pulsing. It often responds favourably to initially but usually settle after a few days.
tricyclic antidepressants and Dry mouth, rather than excessive
anticonvulsants, but suboptimally to salivation, is a common side-effect.
opioids and NSAIDs. Laxative agents are often prescribed
routinely in patients starting strong opioids
Question 2. B) Liver capsule to prevent problems with constipation.
pain Opioids Respiratory depression can be treated with
Explanation: Liver capsule pain is caused the opioid antagonist, naloxone. Opioids
by stretching of the capsule due to hepatic can cause dose-related sedation and
enlargement. It is usually associated with a drowsiness in patients of all ages, although
tender, palpably enlarged liver. It responds elderly patients tend to be more sensitive.
well to corticosteroids but is poorly
responsive to both NSAIDs and opioids.

Question 3. A) Replace codeine with a


strong opioid such as morphine
Explanation: The WHO analgesic ladder
dictates that analgesia should be
prescribed that is appropriate for the
severity of the pain, and increased until the
pain is controlled. For severe pain not
controlled by a combination of weak
opioid (codeine) and non-opioids
(paracetamol and NSAIDs), as in this case,
the appropriate step is to give the patient a
full trial of strong opioids in place of the
weak opioid. It is an important principle of
the ladder that patients should not move
'sideways', i.e. change from one drug to
another drug of similar potency. At each
step of the analgesic ladder adjuvant
agents (such as anticonvulsants for
neuropathic pain) should be considered.
However, the pain in this case is unlikely
to be neuropathic and a strong opioid is
definitely indicated.

144
Question 5. D) Change from CR • Increase the regular 24-hour dose by the
morphine to regular IR morphine at a sum of the breakthrough doses over the
lower equivalent dose and also prescribe previous 24 hours (unless there are
'as required' doses of IR morphine significant problems with side-effects).
Explanation: The patient developed • Only when the correct dose has been
opioid toxicity as a result of the steep established should the patient be converted
increase in her morphine dose. Whenever to a longer-acting preparation such as CR
the opioid requirements of a patient with oral morphine or a transdermal fentanyl
continuous pain are unclear (e.g. starting patch.
strong opioid for the first time, inadequate In the above case the patient may benefit
pain control or adjusting dose in light of from switching to an alternative strong
toxicity), the correct process is as follows: opioid once the correct dose has been
• Prescribe IR oral morphine 4-hourly. established. It would also be important to
• In addition, prescribe an 'as required' ensure adequate hydration and appropriate
dose of the same strength to be given use of adjuvant agents.
when the patient has pain that is not
controlled by the regular prescription Question 6 B) Metoclopramide
(breakthrough pain). Explanation: Metoclopramide is
• Review adequacy of pain control, contraindicated in the presence of colic
requirement for breakthrough analgesia because of its prokinetic effect.
and presence of side-effects daily.

145
Chapter 12 INFECTIOUS DISEASES
Question 1. C) Unpasteurised Question 5. D) Infants are protected for
milk Leptospirosis the first 6 months of life by passive
Explanation: Leptospirosis (Weil's immunity from maternal antibodies
disease) is caused by infection with the Explanation: The incubation period in
spirochaete, Leptospira interrogans, which measles is 14 days. Koplik's spots are
enters humans by direct contact, often pathognomonic of measles but appear on
through swimming in water contaminated the internal buccal mucosa not the retina.
with rat urine. Mosquitoes may transmit Bacterial pneumonia complicates the
dengue fever, filiariasis and, most disease in around 4% of cases and
importantly, malaria. Hepatitis B may be accounts for many measles-related deaths,
contracted from contaminated blood but antibiotics should not be given unless
products, unprotected sexual intercourse there is clinical evidence of superinfection
and the use of intravenous drugs. (see evidence-based medicine box in link).
Brucellosis and salmonellosis may be Vaccination after recovery is pointless, as
contracted through drinking unpasteurised survival of infection confers life-long
milk. immunity.

Question 2. C) Giardiasis Question 6. A) Rubella Myocarditis


Explanation: The flagellate protozoon, Explanation: Rubella infection in early
Giardia lamblia, colonises the small pregnancy may lead to severe congenital
bowel, where it may cause diarrhoea, malformation but rarely causes
abdominal discomfort and malabsorption. complications in childhood, with most
cases being subclinical. Infection with
Question 3. B) Cutaneous larva migrans human erythrovirus 19 (previously known
Explanation: The history and description as parvovirus B19) transiently depresses
of the rash are classical for cutaneous larva bone marrow function. This is usually
migrans (CLM), which is caused by the mild but, in individuals with reduced red
dog hookworm, Ancylostoma caninum. cell lifespan (e.g. spherocytosis), it may
The foot is the most common site, but lead to life-threatening anaemia.
lesions of CLM may also be seen on the
breasts, buttocks and elbows. Treatment Question 7. C) Varicella zoster virus
options include albendazole and Explanation: The clinical description and
ivermectin. rash are typical of shingles, caused by
herpes zoster. The virus remains dormant
Question 4. D) Tuberculosis in the dorsal root ganglion of sensory
Explanation: Eosinophils play an nerves and can be reactivated at any time.
important role in combatting helminths,
phagocytosing immune complexes and
modulating type 1 hypersensitivity
reactions. Eosinophilia is therefore most
commonly seen with parasitic infections
and allergy. Tuberculosis causes a
monocytosis.

146
Question 8. E) It is caused by a DNA the principal vector, and the incubation
poxvirus period is 2-7 days, with the rash appearing
Explanation: Smallpox has been between days 3 and 5 of the symptomatic
eradicated world-wide since 1980, when infection. A more serious form of the
WHO coordinated a successful infection, dengue haemorrhagic fever, can
vaccination campaign. Since eradication, also occur, particularly in children in
the vaccine is no longer routinely used but South-east Asia.
is still available. The disease has a 30%
mortality rate. In contrast to chickenpox, Question 12. E) Norovirus
the rash is worst on the face and Explanation: Norovirus (Norwalk agent)
extremities. Smallpox has recently is the cause of many of the outbreaks of
returned to media prominence due to 'winter vomiting disease' that close wards
concern over its potential as a bioterrorist in hospitals each winter in the UK. After a
weapon. 48-hour incubation period, the virus
causes a vomiting illness that lasts 2-3
Question 9. A) Infectious mononucleosis days.
Explanation: The symptoms, clinical
findings, lack of travel history and age Question 13. C) Hepatitis
group all point towards a diagnosis of Explanation: Staph. aureus is a normal
infectious mononucleosis, caused by the commensal of human skin, but can cause
Epstein-Barr virus. serious infection when it breaches this
barrier. This is particularly so if foreign
Question 10. D) The peak incidence of material is present: for example, in
acquisition is in the sixth decade prosthetic joints and cannulae.
Explanation: The first peak in virus
acquisition is in early childhood, when it is Question 14. B) Staphylococcal toxic
passed readily in communal environments shock syndrome
such as nurseries and school. The second Explanation: Staphylococcal toxic shock
peak occurs in teenage years. In most syndrome is a life-threatening condition
cases primary infection is asymptomatic, that presents with high fever, systemic
but carriers often continue to excrete virus upset, a generalised erythematous
for months or years. Serious consequences blanching rash and haemodynamic
of the infection usually occur in the compromise, before progressing rapidly to
immunosuppressed, in whom it may cause multi-organ failure. It is caused by
CMV retinitis, and in pregnant women, infection with certain strains of
when there is a 40% risk of spread to the Staphylococcus aureus that produce a
fetus, 10% of whom will have serious toxin known as toxic shock syndrome
neurological sequelae. toxin 1 (TSST1). A major aetiological
factor is the use of vaginal tampons,
Question 11. D) Dengue particularly when they are not removed in
Explanation: Despite the apparent a timely fashion, as Staph. aureus can
compliance with chemoprophylaxis, it grow in and around the tampon with
would be vital to consider the diagnosis of subsequent liberation of TSST1. In the
malaria in this and any other patient case above, the diagnosis of
recently returned from an endemic area meningococcal sepsis would also have to
who has a fever. Such patients should have be considered.
urgent thick and thin blood films sent to
look for the Plasmodium falciparum
parasite. Nevertheless, by far the more
likely diagnosis in this scenario would be
dengue fever, considering the recent travel
history, mosquito bites and the symptoms
described. The mosquito, Aedes aegypti, is

147
Question 15. E) Haemolytic uraemic Question 18. A) 14-day course of
syndrome doxycycline
Explanation: Scarlet fever most often Explanation: In the absence of
results from uncomplicated streptococcal contraindications, a 14-day course of
pharyngitis or tonsillitis. Various amoxicillin or a tetracycline antibiotic
streptococcal species, including the such as doxycycline is indicated.
viridans group, may be responsible for Disseminated Lyme disease may require
endocarditis. In rheumatic fever, more prolonged therapy, with up to 30
antibodies generated in response to days of treatment. Where there is
infection with certain strains of group A neurological involvement
streptococci cross-react with cardiac (neuroborreliosis), parenteral therapy,
myosin and sacrolemmal membrane usually with a broad-spectrum
proteins, leading to inflammation in the cephalosporin, is required for 3-4 weeks.
endocardium, myocardium and
pericardium. An immune-mediated Question 19. D) Weil's disease
reaction following streptococcal infection (leptospirosis
may also lead to glomerulonephritis but Explanation: This dramatic presentation
haemolytic uraemic syndrome is a with fever, haemorrhages, jaundice,
consequence of infection with certain purpura, renal impairment and conjuctival
strains of E. coli, most notably E. coli hyperaemia is a classical of Weil's disease,
O157. a spirochaetal infection caused by
leptospires. These highly motile organisms
Question 16. A) Pyoderma are shed in the urine of animal hosts,
gangrenosum especially rats, and infection may occur
Explanation: Brucellae are intracellular during prolonged immersion in
organisms that can survive within the body contaminated water. The disease is
for long periods of time, causing non- therefore more common is freshwater
specific symptoms and signs in various sports enthusiasts, such as this patient.
organ systems. The acute phase of the Meningococcal sepsis must be considered
illness is marked by high swinging in any patient presenting with fever and
temperature, rigors and lethargy. Physical purpura, but this man's age and the
findings that are non-specific include presence of jaundice and hyperaemic
lymphadenopathy and splenomegaly. conjuctivae make it less likely. His age
Brucellosis may cause erythema nodosum also reduces the likelihood of infectious
but is not associated with pyoderma mononucleosis, which does not usually
gangrenosum. present in such dramatic fashion. Hepatitis
A may present with fever, jaundice and
Question 17. E) Erythema chronicum raised transaminases, but would not
migrans account for some of the other features.
Explanation: Erythema chronicum Listeriosis tends to affect
migrans is the rash associated with Lyme immunocompromised patients and
disease, caused by infection with pregnant women.
spirochaetes of the genus Borrelia. The
main vector is the tick Ixodes, whose bites Question 20. C) Faecal-oral
transmit the disease to humans. The early Explanation: Typhoid and paratyphoid
disease is localised, manifesting as a skin fevers (the enteric fevers) are transmitted
reaction around the site of the tick bite. by the faecal-oral route. It is important not
The infection may then disseminate to to confuse them with typhus, a rickettsial
produce widespread systemic effects such infection transmitted by ticks.
as arthralgia, meningitis, cranial nerve
palsies and peripheral neuropathy. Late
advanced disease produces arthritis,
encephalopathy and acrodermatitis
chronica atrophicans.

148
Question 21. B) Refrigeration of food Question 24. A) Cholera
products encourages the growth of heat- Explanation: The bacterium, Vibrio
stable enterotoxins cholerae, causes acute watery diarrhoea.
Explanation: Correct storage of food and The stool is typically 'rice water' in nature,
good food-handling practices limit the consisting of clear fluid with flecks of
incidence of staphylococcal food mucus. This contrasts with the other
poisoning. As Staph. aureus is a organisms listed, which may directly
commensal of human skin (and invade the mucosa of the small bowel, or
particularly nasal passages), poor hand produce cytotoxins that damage and
hygiene among food servers and handlers ulcerate the mucosa with inflammation,
is a frequent cause of cases. Nausea and producing a bloody diarrhoea.
profuse vomiting are the predominant
symptoms and are due to production of an Question 25. C) Oral rehydration
enterotoxin by the organism rather than solution (ORS) contains carbohydrate,
active infection. This explains the rapid sodium, potassium and chloride
onset of symptoms, which first arise 1-6 Explanation: Replacement of fluid losses
hours after food ingestion. The enterotoxin is by far the most important aspect of the
is also responsible for the typical rise in management of acute gastroenteritis. The
neutrophil count. average adult with 48 hours of moderate
diarrhoea will be 1-2 litres depleted.
Question 22. E) Entero-invasive E. coli Adequate rehydration requires
(EIEC) causes symptoms through replacement of this deficit and on-going
production of an enterotoxin diarrhoeal losses in addition to normal
Explanation: EIEC causes symptoms by daily fluid requirement. While intravenous
direct invasion of the colonic mucosa and replacement may be required for those
presents in a very similar fashion to who cannot keep up with losses by oral
Shigella dysentery. Approximately 10-15 intake alone (e.g. in persistent vomiting),
percnt; of patients with the O157 serotype the vast majority of cases can be managed
of EHEC will develop haemolytic uraemic with appropriate oral rehydration. ORS
syndrome, characterised by acute renal contains both electrolytes and a source of
failure, and intravascular haemolysis. It carbohydrate to allow absorption to take
occurs with greatest frequency in the very place. Prolonged bouts of diarrhoea will
young and the very old. Management may result in a metabolic acidosis, due to the
include dialysis and/or transfusion. E. coli loss of bicarbonate-rich secretions from
is also the most common cause of the gastrointestinal tract distal to the
travellers' diarrhoea. stomach. Infants with gastroenteritis
should definitely continue to breastfeed, as
Question 23. B) Metronidazole milk is sterile and nutritious, and contains
Explanation: A 10-day course of oral the correct balance of electrolytes.
metronidazole is first-line therapy for
established Cl. difficile infection; oral
vancomycin is used if there is no response.
Almost all antibiotics predispose to Cl.
difficile infection, particularly broad-
spectrum penicillins (e.g. co-amoxiclav),
second/third generation cephalosporins
(e.g. cefuroxime) and clindamycin.

149
Question 26. B) Diphtheria More than 12 million patients have now
Explanation: Although diphtheria was been successfully treated with MDT.
eradicated from the developed world in the
mid-20th century, it remains an important Question 30. D) Bartonella henselae
source of illness in the developing world, Explanation: B. henselae causes cat
and recent years have seen the occurrence scratch disease, a common benign
of outbreaks in South-east Asia and parts condition that arises in children and young
of the former USSR. The 'bull neck' and adults following a scratch from a cat. A
'wash-leather' grey-green membrane vesicle or papule develops at the site of the
covering the tonsils are characteristic of injury, and subsequently the patient
the acute presentation. Treatment is develops lymphadenopathy which may
threefold, including administration of last for up to 4 months after the skin lesion
diphtheria antitoxin, antibiotics (penicillin has resolved. Bartonella isolates are
or erythromycin) and isolation. A sensitive to ß-lactam antibiotics,
streptococcal sore throat will rifampicin, erythromycin and
characteristically look red and raw; oral tetracyclines. Organisms of the Bartonella
candidiasis is whitish in appearance, and group may also cause a life-threatening
has a predilection for the tongue rather endocarditis, particularly in the homeless,
than the tonsils; mumps usually affects a which requires intravenous antibiotics and
younger age group and may cause a valve replacement.
swollen neck, but not the other findings;
tonsillar cancer is exceedingly rare and Question 31. B) Falciparum malaria
does not really fit with this acute Explanation: A history of intermittent
presentation. fevers following return from travel in an
endemic area should always arouse
Question 27. D) Leprosy suspicion of malaria, the most dangerous
Explanation: A history of skin lesions form of which is falciparum malaria.
and thickened peripheral nerves should Jaundice is common due to a combination
lead to strong suspicion of leprosy. About of haemolysis and hepatic dysfunction,
70% of the world's leprosy patients live in anaemia develops rapidly and tender
India, so this gentleman's ethnic origin is hepatosplenomegaly is typical. Infectious
also a clue. mononucleosis could potentially present
with similar features but the lack of sore
Question 28. C) Microscopy of split skin throat, lymphadenopathy or rash and the
smears recent travel history favour malaria. With
Explanation: Although the diagnosis of a corresponding history of unprotected
leprosy is largely clinical, based on the sexual intercourse, the timing would be
presence of cardinal features, the finding right for HIV seroconversion illness (2-6
of acid-fast bacilli in split skin is weeks after initial infection), which may
confirmatory and also useful for present non-specifically with fever and
monitoring response to treatment. M. malaise but would not produce anaemia
leprae still cannot be grown in vitro and and jaundice. The other two options are
neither serology nor PCR testing for highly unlikely.
leprosy DNA is sensitive enough for
diagnostic purposes. Question 32. C) Nephrotic syndrome
Explanation: Infection with Plasmodium
Question 29. D) Rifampicin malariae but not Plasmodium falciparum
Explanation: Rifampicin is a potent may cause glomerulonephritis and
bactericidal for Mycobacterium leprae but nephrotic syndrome in children. Renal
should always be given in combination complications of falciparum malaria
with other antileprotics. Multidrug include acute renal failure secondary to
treatment (MDT) is crucial to treatment acute tubular necrosis, occasionally with
success because of the emergence of
resistant strains of the bacillus.

150
Question 33. A) Thin blood film sudden death.
Explanation: A thin blood film allows
identification of the species of parasite as Question 38. B) Tsetse fly
well as estimation of parasitic load. Full Explanation: The bite of the tsetse fly is
blood and coagulation screen will painful and often causes local
commonly reveal abnormalities but are not inflammation.
diagnostic. Culture and serological tests
play no role in the diagnosis. Question 39. E) The most common
presenting features of acquired
Question 34. B) Falciparum malaria is toxoplasmosis are fever, photophobia
still largely sensitive to quinine and a non-specific rash
Explanation: P. falciparum is now Explanation: The most common clinical
resistant to chloroquine almost world- presentation of toxoplasmosis is with
wide, but not to quinine which is the drug painless enlargement of lymph nodes,
of choice. Mefloquine, not quinine, may particularly cervical nodes. Photophobia
occasionally cause severe neuropsychiatric and a rash do not normally feature. The
side-effects. Mefloquine or doxycycline risks of transmission to the fetus are
chemoprophylaxis should be started 1 greatest in the third trimester; however, the
week before travel to a malarious area and overall incidence and severity of
continued for 4 weeks after. Fansidar has congenital disease are greatest in the first
resulted in deaths from both trimester.
agranulocytosis and Stevens-Johnson
syndrome and should not be used for Question 40. C) Clinically, the disease
chemoprophylaxis. manifests as diarrhoea alternating with
constipation, with the presence of blood
Question 35. D) Blood tests typically and mucus
reveal thrombocytosis and neutrophilia Explanation: Amoebiasis is a form of
Explanation: Characteristically, dysentery, which is spread to humans by
pancytopenia is found on full blood count, ingestion of cysts of E. histolytica in water
rather than thrombocytosis and or uncooked food. Vegetative trophozoite
neutrophilia. forms emerge from cysts in the large
bowel and invade the mucous membrane.
Question 36. B) Cutaneous The incubation period ranges from 2
leishmaniasis weeks to many years. Diagnosis is made
Explanation: The history of the lesion, by direct microscopy of stools, looking for
recent travel to an endemic area and motile trophozoites. Intestinal amoebiasis
biopsy findings of parasites in the dermis responds rapidly to oral metronidazole.
all make cutaneous leishmaniasis (CL) by
far the most likely diagnosis. The Question 41. E) Neutrophilia
incubation period is typically 2-3 months. Explanation: Strongyloidiasis is a disease
Although all of the above are protozoal caused by the nematode Strongyloides
infections, only CL gives a cutaneous stercoralis, which is common in the Far
ulcer as described. East. It is characterised by a marked
eosinophilia. Allergic phenomena, such as
Question 37. B) Trypanosoma cruzi urticaria, wheeze and arthralgia, are often
Explanation: American trypanosomiasis seen in conjunction with the
(Chagas disease) commonly occurs in gastrointestinal symptoms. Ivermectin is
South and Central America, and is the treatment of choice.
transmitted to humans by contact with the
faeces of the reduviid bug. Between 10%
and 30% of cases with chronic infection
develop a cardiomyopathy characterised
by cardiac dilatation, varying degrees of
atrioventricular block, arrhythmias and

151
Question 42. B) Filiariasis every year. Eye disease is heralded by
Explanation: Filarial worms are tissue- itching, lacrimation and conjunctival
dwelling nematodes; toxins released by the injection. 'Snowflake' deposits are
adult worm after dissemination in the classically seen at the edge of the cornea.
human blood stream cause dilatation of the The treatment of choice is ivermectin,
lymphatic vessels, leading to lymphatic which is also in widespread use as a
dysfunction and the chronic limb prophylactic agent.
manifestation of elephantiasis. In the acute
phase, the presentation is as described Question 44. D) Urticaria
above, with fever, erythema and Explanation: Urticaria is a manifestation
tenderness along the course of the of acute schistosomal infection.
inflamed lymphatic vessels and Schistosoma haematobium
epididymo-orchitis. The treatment of characteristically affects the urinary tract,
choice is diethylcarbamazine (DEC). causing the chronic complications of
recurrent urinary tract infections, bladder
Question 43. A) Vision or ureteric stone formation, and
Explanation: Onchocerciasis, otherwise hydronephrosis. The eggs of S. mansoni
known as river blindness, is the result of and S. japonicum are deposited in the
infection by the filarial Onchocerca liver, where they may lead to portal
volvulus, transmitted by flies of the genus hypertension and consequent development
Simulium. It is endemic in sub-Saharan of splenomegaly, progressive ascites and
Africa, where it leaves almost half a oesophageal varices.
million people with visual impairment

152
Chapter 13 HIV INFECTION AND
THE AIDS

Question 1. A) Single-stranded RNA Question 4. B) CD4 count falls, viral


virus load rises
Explanation: HIV is a single-stranded Explanation: Primary infection with HIV
RNA virus. It consists of a protein core, is symptomatic in 70-80% of cases,
surrounded by a lipid membrane which is presenting with symptoms suggestive of a
lined by a matrix protein studded with flu-like illness. This coincides with a surge
glycoprotein. The virus requires a host cell in the plasma HIV-RNA levels to > 1
(a CD4-positive cell) in order to replicate, million copies/ml, and a fall in the CD4
and to integrate its own genetic material count to 300-400 cells/mm3.
into that of the host.
Question 5. D) Oropharyngeal
Question 2. C) Homosexual candidiasis
transmission Explanation: The acquired
Explanation: The relative importance of immunodeficiency syndrome (AIDS) is
the various modes of HIV transmission defined by the development of specified
shows marked geographical variation. In opportunistic infections, tumours etc. (see
northern Europe, where blood products are link below) in a patient with HIV
routinely and scrupulously screened for infection. These tend to become manifest
HIV and where highly active antiretroviral with varying degrees of
therapy (HAART) and needle exchange immunosuppression, so that there is a
programmes are more readily available, correlation between the CD4 count and the
the main mode of transmission is from specific diseases likely to occur.
men who have sex with men (MSM). In Oesophageal but not oropharyngeal
Africa, by contrast, the most frequent candidiasis is considered an AIDS-
modes of transmission are heterosexual defining illness.
and vertical. China and Eastern Europe
have a much greater proportion of cases in Question 6. A) Patients typically present
intravenous drug-users. with profuse, bloody diarrhea
Explanation: One of the cardinal features
Question 3. E) Receipt of infected blood of Cryptosporidium infection is that the
products diarrhoea is watery and cholera-like, as
Explanation: Receipt of infected blood or opposed to bloody. When prolonged, it is
blood products carries a 90% risk of an AIDS-defining illness that occurs when
transmission to the recipient. The other the CD4 count falls below 200 cells/mm3.
transmission risks are: As the cysts are chlorine-resistant, all
• Vertical transmission 15-40% HIV-infected patients are advised to boil
• Intravenous drug users 0.5-1% their drinking water.
• Genital mucous membrane contact 0.2-
0.5%
• Non-genital mucous membrane contact <
0.1%

153
Question 7. E) Toxoplasma Amphotericin given intravenously is the
Explanation: Toxoplasma is a pathogen treatment of choice. Once infection has
of the nervous system, causing space- been successfully treated, fluconazole can
occupying cysts within the brain that lead be given as prophylaxis until immunity
to focal neurological signs. The typical has recovered.
radiological appearance is of multiple
ring-enhancing lesions on contrast CT or Question 10. D) Cytomegalovirus
MRI of brain. Toxoplasma causes (CMV) retinitis
subclinical infection in the Explanation: The symptoms given in the
immunocompetent individual; it occurs in history, along with the time-scale of the
HIV-positive patients when the CD4 count visual loss and the physical findings, are
falls below 100 cells/mm3. All of the other all suggestive of CMV retinitis. This is an
pathogens listed can cause chronic important disease; although the incidence
diarrhoea in the immunosuppressed. has plummeted since the introduction of
highly active antiretroviral therapy
Question 8. B) A cough productive of (HAART), the infection is sight-
green sputum threatening, particularly if the macula is
Explanation: Pneumocystis carinii (now affected. It tends to occur when the CD4
renamed jirovecii) pneumonia is an AIDS- count falls below 50 cells/mm3. Treatment
defining illness that affects the respiratory involves the initiation of antivirals for
tract, causing a dry cough, pyrexia, and CMV (e.g. ganciclovir, cidofovir) and
breathlessness disproportionate to clinical HAART, if not already instituted. Visual
and radiological signs. Exercise-related loss may result if treatment is not started
desaturation is a common feature. promptly.
Diagnosis is based on cytology of induced
sputum samples or bronchoalveolar Question 11. E) Kaposi's sarcoma
lavage, as the organism cannot be Explanation: Options A-C form part of
cultured. First-line treatment is high-dose the differential diagnosis of HIV-related
co-trimoxazole and steroids. Pneumocystis skin disease but do not fit well with this
infection occurs when the CD4 count falls presentation. Although HIV-associated
below 200 cells/mm3. Many patients who lymphoma may cause lymphadenopathy
are HIV-positive with CD4 counts at or and hepatosplenomegaly, it would not
below this level will be on prophylactic account for the cutaneous appearances
co-trimoxazole or an alternative. described and tends to occur with lower
CD4 counts (median CD4 count at
Question 9. C) Deafness and blindness diagnosis is 50 cells/mm3). Kaposi's
are the most common complications sarcoma, an AIDS-defining disease, is a
Explanation: Cryptococcus is identified tumour that affects cutaneous, oral and
on microscopy of CSF that has been visceral sites. It may follow an indolent or
stained with India ink. It is the most a fulminant course, with visceral
common cause of meningitis in late-stage involvement often following cutaneous
HIV infection, and tends to occur only disease. The pattern of disease mirrors the
once the CD4 count has fallen below 100 CD4 count, with accelerated disease more
cells/mm3. The typical signs of meningism common in those with low CD4 counts.
are often absent, and the presenting
features may be non-specific, with
headache, fever, vomiting and mild
confusion. Deafness and blindness may
result from prolonged elevation of
intracranial pressure. Cryptococcus
neoformans is a budding encapsulated
yeast, and therefore penicillin-based
antibiotic regimens are ineffective.

154
Question 12. A) Cranial nerve palsies Question 14. E) Low incidence of drug
Explanation: The neurological side- rashes
effects of NRTIs tend to affect peripheral Explanation: Rash/drug eruption is a
rather than central nerves. All drugs in this major class-specific side-effect of
class act by incorporating themselves into NNRTIs, up to and including
the host DNA, inhibiting further manifestations as severe as Stevens-
lengthening of the complementary strand Johnson syndrome. The action of this class
to the viral RNA template. NRTIs form of drug is to inhibit the action of reverse
one of the cornerstones of highly active transcriptase by binding near to the active
antiretroviral therapy (HAART, most enzyme site. NNRTIs have a half-life of >
usually two NRTIs and a protease 24 hours, allowing for once-daily dosing.
inhibitor). Below is a list of side-effects Their incorporation into a HAART regime
paired with the drug with which they are involves combination with two NRTIs.
most strongly associated: The other major disadvantage of NNRTIs
• Pancreatitis ddI is the potential for development of cross-
• Anaemia Zidovudine (ZDV) resistance to all drugs in the class, through
• Peripheral neuropathy d4T, ddI, ddC a single mutational change. Other side-
• Lipoatrophy d4T, ZDV effects include dizziness, vivid dreams,
insomnia, poor concentration and
Question 13. D) They improve the lipid depression.
profile of most patients by lowering
total cholesterol and low-density Question 15. B)
lipoprotein (LDL), and by raising high- Toxoplasmosis Erythromycin
density lipoprotein (HDL) Explanation: Appropriate prophylaxis for
Explanation: Protease inhibitors are toxoplasmosis would be co-trimoxazole.
associated with a worsening of lipid The decision to institute prophylaxis is
profile, causing a rise in total cholesterol generally dependent on CD4 levels, in that
and LDL, and a fall in HDL in 30-50% of prophylaxis will be started when the CD4
patients. This effect, combined with the count falls to a level at which the patient is
insulin resistance also caused by PIs, leads deemed to have much greater
to an overall increased risk of ischaemic susceptibility to that particular infection.
heart disease in these patients. The other For example, for Pneumocystis carinii
major side-effect of PI therapy is fat pneumonia, the indication to commence
redistribution, characterised by peripheral co-trimoxazole is when the CD4 count
fat wasting (cheeks, temples, limbs and falls below 200 cells/mm3
buttocks) and central adiposity.

155
Chapter 14 SEXUALLY TRANSMITTED
INFECTIONS

Question 1. D) Trichomonas vaginalis Question 4. B) HPV 16 and 18


Explanation: There are approximately Explanation: HPV 16 and 18 have been
340 million curable STIs reported on a linked to neoplasia of the genital tract.
yearly basis to the World Health HPV 6 and 11 are the relatively benign
Organization, of which 170 million cases strains that cause anogenital warts.
represent Trichomonas vaginalis.
Chlamydia is responsible for 92 million Question 5. E) Chlamydial infection
cases, gonorrhoea for 62 million cases, Explanation: The most commonly
and syphilis for 12 million cases. In the diagnosed curable STI in the UK is
UK, the most common treatable STIs are chlamydia, with 100 000 new cases
chlamydia and gonorrhoea. It is important detected each year. Chlamydia is far more
to remember that some STIs, especially frequently diagnosed than the next most
chlamydial infection, may remain silent common curable STI, gonorrhoea, with 22
for many years and therefore go 000 cases reported each year. Chlamydia
undiagnosed. is usually asymptomatic, but may cause
symptoms of pelvic inflammatory disease
Question 2. D) Gonorrhoea such as those described above. In either
Explanation: The presence of Gram- scenario, it may eventually cause damage
negative diplococci on a smear of urethral to the Fallopian tubes, and subsequently
exudate makes gonococcal urethritis the infertility or ectopic pregnancy. Other rare
most likely diagnosis; culture may complications in either sex include
subsequently confirm the organism. Note conjunctivitis and Reiter's syndrome.
that if a patient has been sexually active
with two new partners in the past month, Question 6. A) Trichomoniasis
then the likelihood of an STI is increased. Explanation: Motile flagellate protozoa
seen on a wet mount of vaginal material
Question 3. D) Swab of the contents of a are diagnostic of trichomoniasis. The
vesicle organism involved is Trichomonas
Explanation: The most likely diagnosis in vaginalis, and the treatment of choice is
the above case is genital herpes, which is metronidazole.
diagnosed by sending swabs of vesicular
fluid or ulcers for polymerase chain
reaction examination for detection of
herpes simplex virus (HSV) DNA.
Although HSV-2 is classically associated
with anogenital herpes, HSV-1 is found in
about 50% of cases in the UK, as well as
being the most common strain found in
orolabial herpes.

156
Question 7. E) Syphilitic chancre chances of ever encountering any of these
Explanation: A chancre is an indurated signs in clinical practice are infinitesimally
ulcer, which is the primary lesion of small but some of them do still appear in
syphilis; it occurs in the first days of postgraduate examinations! Webbed neck
symptomatic infection. It begins as a dull is, of course, associated with Turner's
red macule, which becomes papular and syndrome.
eventually ulcerates. Draining lymph
nodes may also become enlarged but are Question 9. B) Vaginal candidiasis
painless. The chancre is usually located on Explanation: The clinical picture best fits
the penis in men or on the labia in women. candidiasis. Options A, C and E would be
Chancres may also occur on the vaginal unlikely in a woman of this age who is not
wall and on the cervix. In 10% of cases, sexually active, and the history would not
extra-genital sites such as fingers, lips, fit with any of these. Although TSS could
tongue and nipple may be involved. occur in this patient, it would present in an
Secondary syphilis occurs 6-8 weeks after entirely different way. A further clue to
the chancre, when the treponemes have the diagnosis lies in the fact that the
disseminated to produce multi-system patient is on long-term antibiotic therapy
disease. for acne; this would, of course, predispose
her to candidal infection, as the antibiotics
Question 8. D) Webbed neck disrupt the natural flora of the female
Explanation: Congenital syphilis is genital tract, allowing overgrowth of
associated with a number of stigmata, pathogens such as Candida albicans.
many of which are eponymous. The

157
Chapter 15 CLINICAL BIOCHEMISTRY
AND METABOLISM

Question 1. B) 15 litres Question 4. A) Sodium


Explanation: In a typical adult male, total Explanation: Since the great majority of
body water is around 40 litres and the body's sodium content is located in the
constitutes around 60% of overall body ECF, total body sodium is a principal
weight. The majority of this, around 25 determinant of ECF volume. Importantly,
litres, is contained within cells (the when sodium balance is disturbed, any
intracellular fluid compartment). The bulk tendency for plasma sodium concentration
of the remaining 15 litres (the extracellular to change is usually corrected by the
fluid compartment) is found within the osmotic mechanisms controlling water
interstitial fluid between cells and only a balance. Consequently, disorders in
small proportion of this (3 litres) sodium balance present chiefly as altered
comprises plasma. ECF volume (hyper- or hypovolaemia)
rather than altered sodium concentration.
Question 2. C) Potassium
Explanation: The dominant cation in the Question 5. B) 65%
intracellular fluid (ICF) compartment is Explanation: Around 65% of the filtered
potassium while the dominant cation in the sodium load is reabsorbed in the proximal
extracellular fluid (ECF) compartment is tubule, a further 25% in the thick
sodium. The major force maintaining the ascending limb of the loop of Henle and
difference in cation concentration between some 6% in the early distal tubule. Sodium
the ICF and ECF is the activity of the handling in the late distal tubule and
sodium-potassium pump, integral to all collecting duct is under the influence of
cell membranes. aldosterone, which can increase
reabsorption over the range of 2-3% of the
Question 3. B) High potassium filtered sodium load. A further small
Explanation: Haemolysis during or after component (< 1%) occurs in the medullary
collection of a blood sample causes collecting duct; this component is
contamination of the plasma compartment inhibited by atrial natriuretic peptide. A
by intracellular components. As potassium knowledge of the processes mediating
is the dominant cation within the sodium reabsorption and the factors which
intracellular fluid compartment, this leads regulate it is essential for understanding
to a spuriously high plasma potassium disturbances of extracellular fluid volume
concentration. In clinical practice this is and their management.
one of the most common explanations for
the finding of a high plasma potassium Question 6. C) Bradycardia
concentration and it is usually prudent to Explanation: Tachycardia, not
repeat the test, particularly if the result is bradycardia, is a clinical feature of
unexpected. True hyperkalaemia is, hypovolaemia.
however, a life-threatening emergency and
treatment should not be delayed pending
the result of a repeat sample if significant
ECG changes are present.

158
Question 7. D) Increased release of Question 10. B) Hyponatraemia
atrial natriuretic peptide Explanation: Treatment with either
Explanation: Atrial natriuretic peptide thiazide or loop diuretics may produce a
(ANP) is one of several neurohumoral wide range of metabolic abnormalities,
mediators that act to inhibit sodium though these are frequently subclinical.
reabsorption, contributing to natriuresis Both may lead to hyponatraemia through
(increased urinary sodium excretion) excess sodium loss with relative retention
during periods of sodium and volume of water, especially in high vasopressin
excess. Release of ANP is reduced in (ADH) states such as congestive heart
response to loss of sodium and water. failure. Note that the effect of loop and
Activation of the renin-angiotensin- thiazide diuretics on calcium excretion
aldosterone system is the most important differs, with the former increasing urinary
neurohumoral response to salt and water calcium excretion and the latter reducing
depletion. it. Thus a thiazide diuretic may lead to
mild hypercalcaemia, not hypocalcaemia.
Question 8. E) Addison's disease
Explanation: Addison's disease is a cause Question 11. E) Diuretic therapy
of sodium and water depletion and may Explanation: Hyponatraemia (Na < 135
present with features of profound mmol/l) is a common electrolyte
hypovolaemia. Note that in normal health abnormality but it is easily misunderstood.
a number of physiological mechanisms This is because disturbances in sodium
serve to match urinary sodium excretion balance are manifest principally as
closely to sodium intake, such that excess changes in ECF volume status (hypo- or
sodium intake does not lead to hypervolaemia) rather than changes in
hypervolaemia. Various disease states plasma sodium concentration (the plasma
may, however, impair these mechanisms sodium concentration is often normal).
or render them counterproductive, Abnormalities of plasma sodium
triggering a sequence of events that concentration are, instead, usually due to
ultimately results in sodium and water disturbances of body water content. Thus
excess (see link below). An awareness of in all cases of hyponatraemia there is
these events is important both for retention of water relative to sodium and
understanding the pathophysiology of the most useful guide to the underlying
hypervolaemia in various clinical cause is assessment of the ECF volume
conditions and for providing a rational status (which in turn reflects total body
basis for treatment. sodium). In hypovolaemic hyponatraemia
Question 9. C) Loop of Henle (e.g. diuretic therapy, diarrhoea and
Explanation: All diuretics act by vomiting) there is a sodium deficit with a
inhibiting sodium reabsorption at some relatively smaller water deficit; in
point in the nephron. Furosemide belongs euvolaemic hyponatraemia (e.g. SIADH,
to the potent class of 'loop diuretics' which primary polydipsia) there is water
inhibit sodium reabsorption in the thick retention alone with no change in total
ascending limb of the loop of Henle by body sodium; and in hypervolaemic
blocking the apical Na,K,2Cl transporter. hyponatraemia (e.g. congestive cardiac
The link below is very useful for failure, nephrotic syndrome) there is
understanding the mechanisms by which sodium retention with relatively greater
different classes of diuretic achieve both water retention. Hyperlipidaemia is an
their therapeutic effects and their side- artefactual cause of apparent
effects. hyponatraemia.

159
Question 12. D) Syndrome of peaking of the T wave and flattening or
inappropriate ADH secretion (SIADH) loss of the P wave. A more serious change
Explanation: The pointers towards a is broadening of the QRS complex that
diagnosis of SIADH in this case are: often heralds an impending serious
• no features on examination of hypo- or arrhythmia. Flattened T waves, ST
hypervolaemia (i.e. euvolaemic depression and U waves are all changes
hyponatraemia) associated with hypokalaemia.
• urinary osmolality that is inappropriately
high for the plasma osmolality. (In the Question 16. C) Conn's syndrome
face of a plasma osmolality of 250 Explanation: The corticosteroid hormone,
mmol/kg, the urine should be maximally aldosterone, is the most important factor in
dilute ( 50 mmol/kg) and should certainly adjusting potassium secretion to match
not have an osmolality greater than potassium load. It acts on the nephron to
plasma.) stimulate both potassium and hydrogen ion
Diabetes insipidus is a cause of secretion and increase sodium
hypernatraemia rather than hyponatraemia reabsorption, effectively conserving
due to inadequate production of ADH. sodium at the expense of potassium and
hydrogen. The altered potassium excretion
Question 13. D) Fluid restriction of 1- usually acts to maintain tight control of
1.5 litres per day plasma potassium concentration within the
Explanation: Patients with SIADH have narrow normal range. In Conn's syndrome
normal total body sodium but an excess of there is excess production of aldosterone
water (euvolaemic hyponatraemia). The by a unilateral adrenocortical adenoma,
most appropriate treatment is therefore not resulting in primary hyperaldosteronism.
to increase sodium intake but to restrict This leads to hypertension through excess
water intake. Demeclocycline may be a sodium and water retention, usually
useful second-line treatment if the rise in accompanied by hypokalaemia through
plasma Na with fluid restriction is excessive potassium secretion, and
inadequate. An important general point metabolic alkalosis. All of the other
regarding the management of options would be more likely to result in
hyponatraemia is that the rate of correction hyperkalaemia.
of the plasma Na should be determined by
the rate of onset of the hyponatraemia. An Question 17. C) Ramipril
over-rapid correction in chronic Explanation: ACE inhibitors such as
hyponatraemia may lead to large water ramipril inhibit the conversion of
shifts out of neurons and consequent angiotensin I to angiotensin II. One of the
irreversible brain damage (central pontine effects of angiotensin II is to stimulate
myelinolysis). aldosterone production in the zona
glomerulosa of the adrenal cortex. The
Question 14. B) Carbamazepine major function of aldosterone is to
Explanation: SIADH has many promote reabsorption of sodium in the late
underlying causes including tumours, distal tubule and collecting ducts of the
intracranial pathology, pulmonary kidney at the expense of potassium and
disorders and a variety of drugs. hydrogen. Thus a reduction in aldosterone
Carbamazepine is a well-recognised and activity will lead to decreased potassium
common cause of drug-induced SIADH. secretion and decreased sodium
reabsorption, explaining why ACE
Question 15. D) Peaked T wave inhibitors (and aldosterone antagonists
Explanation: Hyperkalaemia is a such as spironolactone) may lead to
dangerous electrolyte abnormality hyperkalaemia and reduced body sodium.
requiring prompt attention as it may lead
to ventricular arrhythmias and asystolic
cardiac arrest. Early ECG changes include

160
Question 18. E) Calcium gluconate Question 20. C) Metabolic alkalosis
Explanation: All of the above treatments Explanation: More on Patterns of acid-
have a role in the management of base disturbance
hyperkalaemia but calcium gluconate is
used to stabilise myocardial conductive Question 21. A) Gastric outlet
tissue membranes in severe hyperkalaemia obstruction
(usually only in the presence of significant Explanation: The patient has a
ECG changes) and not to lower plasma hypochloraemic, hypokalaemic metabolic
potassium concentration. Inhaled _2 alkalosis with paradoxically acidic urine.
agonists (e.g. salbutamol) and intravenous This fairly classical pattern of biochemical
glucose and insulin (e.g. 50 ml of 50% disturbance is seen with persistent severe
dextrose with 5 units Actrapid) act by vomiting, as might occur with gastric
shifting potassium into cells, whereas outlet obstruction. The factors that
dialysis and calcium resonium remove contribute are explained in detail in the
potassium from the body. All medical link below. Essentially, large quantities of
students and junior doctors should be sodium, chloride, potassium and hydrogen
familiar with the management of ions, as well as water, are lost in the
hyperkalaemia, as severe hyperkalaemia is vomit. The overriding response of the
a life-threatening but reversible medical kidney is salt and water conservation,
emergency. principally through increased aldosterone
release. This maintains the alkalosis and
Question 19. E) Renal tubular acidosis hypokalaemia, as sodium is conserved at
Explanation: The anion gap is the the expense of hydrogen and potassium
difference between the main measured ions. Thus the most effective method of
cations (Na and K) and anions (Cl and correcting the alkalosis is intravenous
HCO3) in plasma. It is composed mainly sodium chloride (with potassium
of phosphate, sulphate and negative supplementation) to interrupt the volume-
charges on plasma proteins. The normal conserving mechanisms and allow the
value of the anion gap is around 15 kidney to excrete excess alkali in the
mmol/l. urine.
Renal tubular acidosis and diarrhoea are
the two most common causes of a normal Question 22. E) High-density
anion gap acidosis. Sepsis may lead to lipoprotein
tissue hypoxia and a consequent lactic Explanation: Reverse cholesterol
acidosis; salicylate poisoning typically transport refers to the transfer of
produces an initial respiratory alkalosis cholesterol from cholesterol-laden cells to
followed by a raised anion gap metabolic the liver (for excretion) and cholesterol-
acidosis. Both diabetic ketoacidosis and requiring tissues. This protects peripheral
starvation ketosis result in accumulation of tissues (including blood vessel walls) from
acidic ketone bodies. excessive cholesterol accumulation. High-
density lipoprotein (HDL) is primarily
responsible for reverse cholesterol
transport and low levels of HDL
cholesterol are associated with increased
risk of atherosclerosis.

161
Question 23. C) Eruptive xanthomas Studies have demonstrated significant
Explanation: Xanthelasma and corneal reduction in stroke as well as coronary
arcus are fairly non-specific signs of heart disease. The lipid response includes
hyperlipidaemia seen in a variety of lipid a mild increase in HDL cholesterol, whilst
disturbances and the elderly. On the other the decrease in triglyceride is usually less
hand, xanthomas of the Achilles or than that in LDL.
extensor digitorum tendons are highly
suggestive of familial Question 25. B) Bullae on sun-exposed
hypercholesterolaemia (moderate to severe areas of skin
hypercholesterolaemia resulting from an Explanation: The porphyrias are
autosomal dominantly inherited mutation disorders of the haem biosynthetic
of the LDL receptor gene). Although a pathway that can be broadly divided into
correlation exists between two categories according to pattern of
hypertriglyceridaemia and risk of coronary presentation:
heart disease, hypercholesterolaemia is a • Non-acute or cutaneous porphyrias (e.g.
far more important risk factor and more porphyria cutanea tarda) are characterised
closely associated with the development of by photosensitive skin manifestations that
cardiovascular disease. Eruptive occur due to excess production and
xanthomas, as well as lipaemia retinalis accumulation of porphyrins in the skin.
and hepatosplenomegaly, may be found in • Acute or neurovisceral porphyrias (e.g.
severe hypertriglyceridaemia. acute intermittent porphyria) present with
acute episodes of abdominal pain and
Question 24. C) They inhibit features of autonomic dysfunction (e.g.
endogenous cholesterol synthesis hypertension, tachycardia, constipation),
Explanation: The primary action of often in conjunction with neuropsychiatric
statins is to inhibit endogenous synthesis manifestations and/or acute neuropathy
of cholesterol in the liver. This leads to up- (usually motor). There are no skin
regulation of the LDL receptor and hence manifestations in acute intermittent
increased clearance of LDL cholesterol porphyria.
from the circulation. The incidence of
rhabdomyolysis is very low, less than 1%.

162
Chapter 16 KIDNEY AND URINARY
TRACT DISEASE

Question 1. E) Renal artery stenosis independent of production or GFR.


Explanation: A bruit over one of the renal
arteries may raise suspicion of renal artery Question 5. A) Negative exponential
stenosis, though it is neither a sensitive nor Explanation: The importance of
a specific sign. Other situations in which understanding this relationship is clear
the diagnosis may be suspected include from the graph in the link below. A large
significant deterioration of renal function fall in GFR may occur before serum
after starting an angiotensin-converting creatinine is elevated outside the normal
enzyme (ACE) inhibitor or the finding of range.
significant renal impairment in a patient
with widespread vascular disease Question 6. B) Glucose
elsewhere. Explanation: Glucose is freely filtered at
the glomerulus and reabsorbed actively in
Question 2. C) Diabetic nephropathy the proximal tubules. Glycosuria results
Explanation: Diabetic nephropathy is not when the capacity of proximal
a cause of enlarged kidneys but is one of reabsorption is saturated, because of either
the most common causes of chronic renal high serum glucose or low reabsorption
failure, often with small shrunken kidneys. capacity (low renal threshold). Large
proteins such as albumin and fibrinogen,
Question 3. D) 500 ml as well as cells, are not normally filtered.
Explanation: The minimum urine volume
required to remove the daily solute load in Question 7. E) Post-streptococcal
maximally concentrated urine is 300-500 glomerulonephritis
ml. Higher volumes are clearly preferable Explanation: Red cell casts are
in health. characteristic of glomerular bleeding (the
casts are formed in the renal tubules) and a
Question 4. C) Malnutrition common finding in cases of post-
Explanation: Urea is excreted by the streptococcal glomerulonephritis. The
kidneys, so levels rise following a other options listed cause bleeding from
reduction in GFR. However, serum urea lower down in the urinary tract and do not
concentration is an imperfect guide to lead to the formation of casts.
renal function since it is also influenced by
several other parameters. As urea is a by-
product of protein catabolism, its
production and serum levels are increased
by high-protein diets or catabolic states.
Upper gastrointestinal haemorrhage is
effectively a large protein meal while
malnutrition is a cause of low protein
intake. Urea is also reabsorbed, to a
variable extent, in the renal tubules;
dehydration increases the proportion of
filtered urea that is reabsorbed and
therefore causes a rise in blood levels

163
Question 8. B) Reduced muscle bulk from being a contraindication, acute renal
Explanation: Creatinine is freely filtered failure is one of the most common
in glomeruli and not reabsorbed in tubules, indications for renal biopsy. Haematuria
so that serum creatinine is an index of may also be an indication, especially if
glomerular filtration rate (GFR). Under associated with proteinuria or renal
normal circumstances, creatinine is impairment. Antiplatelet drugs (aspirin
released from muscle at a fairly constant etc.) and anticoagulants, but not ACE
rate, so the greater a patient's muscle bulk, inhibitors, should be omitted for elective
the more creatinine he or she will produce. biopsies; indeed, uncontrolled
Changes in serum creatinine levels are hypertension is a contraindication to
therefore a better guide to renal function biopsy.
than a single one-off value. It is important
to note that patients with significant renal Question 12. A) Escherichia coli
impairment may well have a serum Explanation: E. coli is by far the most
creatinine within the normal range, common cause of community-acquired
particularly if frail or malnourished. UTI. The other options listed occur with
Because urea is a by-product of hepatic greater frequency in the hospital setting,
protein metabolism, liver failure, protein although E. coli remains the most
catabolism and dietary protein intake have prevalent.
a much bigger effect on urea concentration
than creatinine. A significant upper Question 13. E) Rhabdomyolysis
gastrointestinal bleed is equivalent to a Explanation: All of the options listed
large protein meal and will likewise lead could conceivably account for
to increased urea production. deteriorating renal function accompanied
by dipstick-positive haematuria, but the
Question 9. A) Detection of urothelial absence of red cells on microscopy is
malignancy characteristic of myoglobinuria occurring
Explanation: Renal ultrasound has as a consequence of rapid muscle
superseded IVU for many purposes and is breakdown. This patient is at risk of
usually the first-line investigation for renal rhabdomyolysis from having lain on the
imaging. However, IVU provides floor for 24 hours.
excellent definition of the collecting
system and ureters and remains superior to Question 14. C) Cystoscopy and renal
ultrasound for detecting stones and ultrasound
urothelial tumours. Explanation: The priority in persistent
haematuria is to exclude an anatomical
Question 10. D) Dimercaptosuccinic bleeding lesion. This is particularly
acid (DMSA) isotope renogram important in older patients who are at
Explanation: DMSA scans allow greater risk of malignancy. IVU may be
quantification of each kidney's relative considered if cystoscopy and ultrasound
contribution to renal function and can fail to reveal a cause, as it provides
detect the presence of scarring. In general, detailed imaging of the ureters. It would
the other investigations listed yield little be unusual for prostate cancer or urinary
functional information regarding the tuberculosis to present with isolated
kidneys. macroscopic haematuria as the only
symptom. Similarly, the absence of
Question 11. B) Kidneys < 60% normal additional symptoms, lack of proteinuria
size and presence of normal renal function do
Explanation: Small shrunken kidneys are not provide evidence of inflammatory
difficult to biopsy and usually indicate renal disease or an indication for renal
irreversible chronic renal disease. biopsy.
Consequently, complications are more
frequent and results rarely helpful. Far

164
Question 15. E) Microalbuminuria also causes an osmotic diuresis). Cranial
Explanation: Microalbuminuria is usually diabetes insipidus is often idiopathic but
the earliest sign of renal dysfunction in may occur secondary to head injury or an
diabetes mellitus and is associated with an intracranial tumour.
increased risk of cardiovascular events in
people without diabetes. It is not detected Question 19. A) Urge incontinence
on standard urine dipstick tests and must Explanation: Anticholinergics reduce
therefore be specifically screened for in detrusor muscle tone and may give
diabetic patients using ultra-sensitive symptomatic relief to patients with urge
sticks and/or through formal measurement incontinence. Stress incontinence is a
of the urinary albumin:creatinine ratio. common problem, particularly in
multiparous women, but anticholinergics
Question 16. E) Focal segmental are not usually effective. Treatment is with
glomerulosclerosis physiotherapy to strengthen pelvic floor
Explanation: This is a very high level of musculature and, in severe cases, surgery.
proteinuria. More than 3 g/24 hrs is in the
'nephrotic' range and is invariably caused Question 20. E) Atenolol
by glomerular disease. Focal segmental Explanation: Beta-blockers are a
glomerulosclerosis is a relatively common common cause of erectile dysfunction.
cause of nephrotic syndrome; the other Sildenafil (Viagra) is a phosphodiesterase
disorders listed affect the urinary tract 5 inhibitor used as a treatment for erectile
distal to the glomerulus. dysfunction. Fluoxetine is a selective
serotonin reuptake inhibitor in common
Question 17. E) Hypercholesterolaemia use as an antidepressant. Interestingly, in a
Explanation: Nephrotic syndrome randomised, double-blinded trial, valsartan
describes the consequences of heavy appeared to boost sexual performance in
urinary protein loss. The most middle-aged Italian males.
characteristic manifestation is generalised
oedema from a combination of decreased Question 21. D) Bilateral small kidneys
serum oncotic pressure on renal ultrasound
(hypoalbuminaemia) and avid renal Explanation: The other options given
sodium retention (secondary here (haematuria, proteinuria, oliguria,
hyperaldosteronism due to decreased dehydration and hyperkalaemia) may all
intravascular volume). Other well- be present in either acute or chronic renal
recognised features, which are explained failure. In some cases it may be difficult to
in the link below, include marked determine the rate of onset of the renal
hypercholesterolaemia, hypercoagulability problem, but radiological evidence of
and increased susceptibility to infection. shrunken kidneys usually indicates chronic
disease.
Question 18. D) Hypocalcaemia
Explanation: Causes of polyuria are: Question 22. B) Ibuprofen
• excessive fluid consumption (polydipsia) Explanation: Non-steroidal anti-
• osmotic diuresis (e.g. in diabetes mellitus inflammatory drugs such as ibuprofen
due to hyperglycaemia) have been shown to increase the risk of
• reduced secretion of antidiuretic contrast nephropathy. Intravenous fluid
hormone (cranial diabetes insipidus) and N-acetylcysteine are thought to have a
• resistance of the renal tubules to the protective role.
actions of antidiuretic hormone
(nephrogenic diabetes insipidus).
Nephrogenic diabetes insipidus has a
number of underlying causes including
interstitial renal disease, lithium therapy,
hypokalaemia and hypercalcaemia (which

165
Question 23. D) Amoxicillin Question 26. C) Bilateral nephrostomies
Explanation: Furosemide is a loop Explanation: Nephrostomies are
diuretic and bendroflumethiazide radiologically inserted drains that allow
(formerly known as bendrofluazide) is a urine to drain directly from the renal pelvis
thiazide diuretic. They are particularly and thereby decompress obstructed urinary
potent in combination and will impair systems. Most patients with post-renal
renal sodium and water reabsorption, acute renal failure will recover with relief
exacerbating hypovolaemia and of the obstruction without the need for
hypoperfusion. Lisinopril is an renal support and, from the information
angiotensin-converting enzyme (ACE) given, there is no compelling indication
inhibitor, which will prevent the for urgent renal replacement therapy here.
compensatory constriction of efferent Fluid restriction is not indicated and,
renal arterioles. Diclofenac, a non- indeed, fluid supplementation may be
steroidal anti-inflammatory drug, will required once the obstruction is relieved as
prevent the compensatory dilatation of patients often experience a 'post-
renal resistance arteries. Both of these obstructive diuresis'. Retroperitoneal
drugs worsen renal failure in patients with fibrosis causes ureteric obstruction, so
critically impaired renal perfusion. catheterisation would be unhelpful and
potentially dangerous as the bladder is
Question 24. A) Promotes water likely to be empty.
reabsorption from urine
Explanation: The medullary Question 27. C) Reduced phosphate
concentration gradient is a key mechanism Explanation: Hyperphosphataemia is
of water reabsorption, enabling the typical in chronic renal failure due to a
production of highly concentrated urine. It combination of reduced phosphate
is often lost in acute tubular necrosis excretion and secondary
(ATN) and this can lead to a 'diuretic hyperparathyroidism. The elevated PTH
phase' in which an inappropriately large occurs as an 'appropriate' physiological
volume of urine is produced during response to hypocalcaemia (hence
recovery from ATN. secondary hyperparathyroidism), which in
turn is due to deficiency of active (1V-
Question 25. E) Haemodialysis hydroxylated) vitamin D.
Explanation: Severe hyperkalaemia may Declining renal function is associated with
lead to life-threatening arrhythmias, and metabolic acidosis that should be treated
resistant hyperkalaemia in the context of with sodium bicarbonate.
acute renal failure is one of the strongest Hypercholesterolaemia is very common in
indications for dialysis. Calcium gluconate patients with chronic renal failure
may also be appropriate treatment here but (particularly with significant proteinuria)
is not used to lower serum potassium; and may contribute to the greatly
rather it acts to stabilise the myocardium increased risk of cardiovascular events in
and should be given if the hyperkalaemia these patients.
is accompanied by ECG changes. Sodium
bicarbonate would be of little benefit as
the patient is not acidotic, and the other
conservative measures listed are unlikely
to achieve a sufficient reduction in serum
potassium.

166
Question 28. E) Complications of Platelet function is actually impaired in
erythropoietin include hypertension and chronic renal failure ('uraemic platelets')
hypercoagulability and this is accompanied by an increased
Explanation: In patients with significant bleeding tendency.
anaemia, plasma erythropoietin (EPO)
should be elevated; a level within the Question 31. A) 1L-hydroxylase
normal range is therefore inappropriately Explanation: One of the functions of the
low and indicates inadequate EPO kidney is conversion of the inactive form
production. The target haemoglobin level of vitamin D (25-hydroxycholecalciferol)
is around 100-120 g/dl; significantly to its active metabolite (1,25
higher targets may be associated with dihydroxycholecalciferol) by the enzyme
increased cardiovascular mortality. Iron 1V-hydroxylase. Diminished activity of
deficiency is a common contributing factor this enzyme leads to deficiency of active
to anaemia in chronic renal failure and a vitamin D, resulting in hypocalcaemia and
cause of suboptimal response to EPO; secondary hyperparathyroidism (see link
transferrin saturation should therefore be below). Hyperphosphataemia then occurs
maintained above 20%. Human from a combination of the secondary
recombinant EPO is currently only hyperparathyroidism and reduced renal
available in intravenous and subcutaneous phosphate excretion. Accumulation of
preparations. Hypertension and aluminium often occurs in dialysis patients
hypercoagulability are both well- and may be associated with a wide range
recognised side-effects of EPO. of pathology.

Question 29. B) Angiotensin-converting Question 32. D) Anaemia


enzyme (ACE) inhibitors Explanation: Dialysis essentially replaces
Explanation: Aggressive control of blood the functions of urine - excretion of solutes
pressure is very important in patients with and excess fluid, control of acid-base and
chronic renal disease, both to slow the removal of toxins - but it does not replace
deterioration in renal function and to the endocrine functions of the kidney, such
reduce the risk of cardiovascular as production of erythropoietin and
complications such as myocardial metabolism of vitamin D.
infarction and stroke. All of the above
drugs may be used for this purpose; Question 33. D) Symptomatic fluid shift
however, ACE inhibitors are anti- is less common with PD than HD
proteinuric and have been shown to have Explanation: Peritoneal dialysis is less
'renoprotective effects' independent of efficient than haemodialysis and must be
blood pressure lowering. performed more often. However, fluid
shifts are less rapid, so peritoneal dialysis
Question 30. D) Enhanced platelet is less likely to cause symptoms associated
function with hypotension such as lightheadedness
Explanation: Cardiovascular disease is by and cramps. Abdominal scarring from
far the leading cause of death in patients previous surgery may make peritoneal
with chronic renal failure. Hypertension dialysis impossible, but will not affect
occurs in approximately 80% of patients haemodialysis.
and is a major risk factor for myocardial
infarction, stroke and heart failure.
Hypercholesterolaemia is very common
and may also contribute to accelerated
atherosclerosis. Anaemia is an
independent predictor of left ventricular
hypertrophy. Patients with chronic renal
failure often have a heavily calcified
arterial tree and this may be so severe as to
cause limb ischaemia.

167
Question 34. B) Active anti-glomerular The link below provides a simple and
basement membrane (GBM) disease memorable summary of the spectrum of
Explanation: Active anti-GBM disease is clinical presentations of
considered an absolute contraindication to glomerulonephritis.
renal transplant because of the likelihood
of recurrence in the transplanted kidney. Question 37. C) Goodpasture's disease
Transplantation is routinely offered to (anti-glomerular basement membrane
patients in their fifties in most centres. (anti-GBM) disease)
Previous skin cancer is not directly Explanation: As the name suggests,
relevant if the lesion has been successfully rapidly progressive glomerulonephritis
removed. However, 50% of Caucasian (RPGN) may lead to a rapid deterioration
renal transplant patients develop skin in renal function over days to weeks. It is
malignancy within 15 years of usually associated with the histological
transplantation, so patient 'D' must be appearance described as focal segmental
advised to present early with any necrotising glomerulonephritis or
suspicious lesions. Some patients with
chronic renal failure and diabetes mellitus Question 38. A) Minimal change
may be suitable for simultaneous pancreas nephropathy
and kidney transplantation. Explanation: Minimal change disease
remits, in most cases, with corticosteroid
Question 35. D) Renal artery stenosis therapy. In contrast, the other conditions
Explanation: The response of the kidney listed characteristically exhibit a poor
to a reduction in renal perfusion pressure response to steroids.
(such as occurs with flow-limiting stenosis
in a renal artery) includes activation of the Question 39. E) Sensorineural deafness
renin-angiotensin-aldosterone system Explanation: Alport's syndrome is an X-
through increased renin production. The linked recessive disorder that may present
resulting rise in angiotensin II with new renal failure in adulthood. It is
concentration mediates constriction of the associated with sensorineural deafness and
efferent arteriole and thereby maintains ocular abnormalities.
glomerular perfusion pressure distal to the
stenosis. ACE inhibitors abolish this Question 40. E) Renal cell carcinoma
compensatory mechanism and may Explanation: APKD is not associated
precipitate a marked reduction in with an increased risk of renal cancer. The
glomerular filtration rate; close monitoring liver may be massively enlarged but liver
of renal function is therefore essential on failure is unusual. Subarachnoid
starting any patient on an ACE inhibitor or haemorrhage occurs in 10% of patients.
angiotensin receptor blocker.
Question 41. B) Alpha-adrenoceptor
Question 36. B) Membranous blockers
nephropathy Explanation: Alpha-adrenoceptor
Explanation: Membranous nephropathy is antagonists (e.g. tamsulosin) are used to
the most common cause of nephrotic improve urinary flow in BPH. All of the
syndrome in adults. In contrast, anti-GBM other options are precipitants of acute
disease, post-streptococcal urinary retention.
glomerulonephritis and small-vessel
vasculitis tend to cause a 'nephritic' pattern
of clinical findings. Acute tubular necrosis
causes renal interstitial dysfunction and
usually presents with acute renal failure.

168
Question 42. A) Lytic bony metastases Question 43. B) Adrenaline
Explanation: Bone is by far the most (epinephrine)
common site of metastatic deposits in Explanation: Renal adenocarcinomas
prostate cancer but these characteristically may secrete a wide variety of peptides that
produce osteosclerotic rather than can produce a range of systemic effects.
osteolytic lesions on plain X-ray. Bony Adrenaline (epinephrine) is not usually
involvement may also be indicated by an secreted by renal adenocarcinoma, but
isolated rise in alkaline phosphatase and may be secreted by a
'hotspots' on radioisotope bone scans. In phaeochromocytoma; these two tumours
practice, high levels of PSA (> 100 ng/ml) may coexist in the von Hippel-Lindau
are highly suggestive of distant bony syndrome.
metastases. In prostate cancer the gland is
usually enlarged and feels hard and Question 44. D) Penicillamine
irregular, often with obliteration of the Explanation: All of the agents listed
median sulcus. However, it is worth noting above are potentially nephrotoxic through
that 10-15% of tumours are not palpable a variety of mechanisms (see link below)
on rectal examination. but penicillamine is a rare cause of
membranous nephropathy.

169
Chapter 17 CARDIOVASCULAR
DISEASE
Question 1. D) Large systolic waves are effects on the sinoatrial and
associated with tricuspid regurgitation atrioventricular nodes but do not influence
Explanation: The internal jugular vein force of contraction. Parasympathetic
drains directly into the right atrium and the effects are mediated by acetylcholine and
height of the pulse reflects right atrial are therefore also termed 'cholinergic'.
pressure. It should be viewed with the
patient at 45 degrees. Abdominal Question 5. B) Evaluation of unstable
compression causes the JVP to rise angina
(hepato-jugular reflux). This may aid Explanation: Stress tests are
identification of the pulsation and help contraindicated in the presence of unstable
distinguish it from an arterial pulsation. As angina, decompensated heart failure and
a waves are caused by atrial systole, they severe hypertension.
are absent in atrial fibrillation.
Question 6. C) Exertional breathlessness
Question 2. D) Right coronary artery and throat tightness
Explanation: The right coronary artery Explanation: Myocardial ischaemia can
supplies the AV node in 90% of sometimes present with breathlessness
individuals, and supplies the sinoatrial rather than chest pain. A sensation of
node in 60%. It also supplies the inferior throat tightness often accompanies this.
aspect of the heart. Inferior myocardial Epigastric discomfort after eating could
infarction is therefore often associated represent inferior wall ischaemia but is
with transient sinus bradycardia and AV more likely to be dyspeptic. Wheeze is a
block. feature of bronchospasm and not
myocardial ischaemia. Accompanying
Question 3. A) Left anterior descending paraesthesia would suggest
artery hyperventilation syndrome. Exertional
Explanation: Although the exact coronary palpitation would suggest either an
anatomy varies greatly between exercise-related arrhythmia or possibly
individuals, the left anterior descending anaemia.
artery normally gives branches to the
anterior part of the septum and the anterior Question 7. B) Hypercholesterolaemia
wall and apex of the left ventricle. Explanation: The abnormality shown in
the picture is xanthelasma, which, along
Question 4. D) Cholinergic nerve fibres with tendon xanthomas and corneal arcus,
act on the sinoatrial node to slow heart is a clinical manifestation of
rate hypercholesterolaemia. Eruptive
Explanation: Sympathetic stimulation xanthomas would be more suggestive of
increases heart rate (positively hypertriglyceridaemia.
chronotropic) and force of contraction
(positively inotropic) via _1-adrenoceptors.
Beta2-receptors are found predominantly
in the vasculature where they mediate
vasodilatation. Parasympathetic fibres run
in the vagus nerve and act to slow heart
rate (negatively chronotropic) through

170
Question 8. B) Exercise ECG Question 11. C) Fine bi-basal
Explanation: The exercise tolerance test crepitations
(stress ECG) is highly useful in patients Explanation: A raised JVP and pitting
with symptoms suggestive of angina. It oedema may accompany the marked salt
may help to confirm or refute the and water retention in left heart failure but
diagnosis as well as identify patients with are also characteristic findings in right
high-risk features. Such patients may heart failure. Isolated right heart failure
require further evaluation with coronary does not lead to pulmonary venous
angiography. congestion. Hepatomegaly and ascites are
features of severe right heart failure.
Question 9. C) Spironolactone
Explanation: The goal of secondary Question 12. C) Displaced apex beat
prevention is to reduce the risk of future Explanation: A displaced apex beat
adverse events such as myocardial suggests left ventricular enlargement. In
infarction. This entails measures to correct acute de novo heart failure there is no time
modifiable risk factors such as smoking for left ventricular dilatation and hence the
cessation, weight loss, optimal blood finding of a displaced apex beat implies
pressure control and lipid-lowering chronic, underlying cardiac pathology.
therapy, as well as antiplatelet therapy. The other features listed are all simply
Although both an ACE inhibitor and a manifestations of acute heart failure.
statin are clearly indicated in this patient to
correct hypertension and Question 13. B) Amlodipine
hypercholesterolaemia respectively, recent Explanation: Peripheral oedema is a
trial data suggest that all patients with common side-effect of dihydropyridine
vascular disease benefit from angiotensin- calcium channel blockers, such as
converting enzyme (ACE) inhibition and amlodipine and nifedipine, as they lead to
that all patients with coronary heart increased capillary permeability.
disease should receive statin therapy
irrespective of serum cholesterol. Question 14. E) Heavy proteinuria
Spironolactone is an aldosterone Explanation: A raised jugular venous
antagonist beneficial in the treatment of pressure and hepatomegaly are both signs
severe chronic heart failure and also as a of chronic heart failure. Severe chronic
diuretic, particularly in cirrhotic patients heart failure is a catabolic state and
with ascites. commonly causes weight loss and muscle
wasting (cardiac cachexia).
Question 10. E) Recurrent pulmonary Hyponatraemia may be caused by diuretic
emboli therapy but is also a feature of severe heart
Explanation: Repeated pulmonary failure.
thromboembolic events may lead to Heavy proteinuria of greater than 3.5
increased pulmonary vascular resistance, g/day suggests nephrotic syndrome.
pulmonary hypertension and ultimately
right heart failure. The other disorders
present more commonly with left-sided
heart failure. Note, however, that
chronically elevated left atrial pressure
will also lead to pulmonary hypertension,
so any cause of left heart failure may
eventually result in concurrent right
ventricular failure.

171
Question 15. B) Ramipril Question 18. D) Loop diuretics
Explanation: The mineralocorticoid Explanation: Loop diuretics are first-line
hormone aldosterone acts to conserve salt treatment to relieve symptoms in patients
and water at the expense of potassium (and with chronic heart failure but have not
hydrogen ions) in the distal convoluted been shown to improve outcomes. It
tubule and collecting ducts of the kidney. would be ethically difficult to conduct a
The principle stimulus to aldosterone heart failure trial in which diuretics were
production is angiotensin II. Thus agents withheld. The other agents listed have all
that inhibit the renin-angiotensin- been shown to reduce cardiovascular
aldosterone system (e.g. ACE inhibitors, mortality and morbidity in patients with
angiotensin receptor blockers) or the chronic heart failure in large-scale
actions of aldosterone (e.g. randomised controlled trials.
spironolactone) may lead to The treatment of chronic heart failure is a
hyponatraemia and hyperkalaemia. topic of major importance and a
Loop diuretics such as furosemide comprehensive guide, including the
typically cause hypokalaemia. evidence base for treatments, is provided
in the link below.
Question 16. C) Beta-blockers
Explanation: Although a useful treatment Question 19. B) Transient ischaemic
in patients with chronic stable heart attack
failure, _-blockers must be avoided in Explanation: Transient loss of
patients with acute or decompensated heart consciousness is due to temporary global
failure due to their negatively inotropic cerebral dysfunction from either cerebral
effects. The other treatments listed are all hypoperfusion (syncope) or generalised
commonly used to good effect in the electrical discharge (seizure). Transient
management of acute pulmonary oedema. ischaemic attacks, by definition, produce
focal cerebral dysfunction and the term
Question 17. C) Dry cough should not be applied to patients
Explanation: As well as converting presenting with blackouts. Basilar
angiotensin I to angiotensin II, the ischaemia can cause loss of consciousness
angiotensin-converting enzyme (ACE) is but is very uncommon. Cardiac syncope
also responsible for breakdown of occurs with arrhythmia or structural heart
bradykinin, a vasodilating peptide. disease. The former is usually due to
Increased bradykinin levels may be profound bradycardia (e.g. Stokes-Adams
responsible for the dry cough associated attack) or ventricular tachyarrhythmias.
with ACE inhibitors. As shown in the link The latter commonly occur with exertion
below, ARBs do not interfere with (e.g. severe aortic stenosis), when
bradykinin metabolism and are much less profound hypotension results from a
likely to cause cough. Unfortunately they combination of reduced cardiac output and
share all of the more serious adverse fall in peripheral vascular resistance. In
effects of ACE inhibitors such as renal carotid sinus syndrome, hypersensitivity of
dysfunction and electrolyte disturbance. the carotid baroreceptors causes
inappropriate vasodilatation, provoking
recurrent episodes of blackout or altered
consciousness.

172
Question 20. E) Rapid recovery (< 1 Question 24. E) Atrial septal defect
min) following the collapse Explanation: In atrial septal defect there
Explanation: Events immediately before, is a left to right shunt because left atrial
during and after a blackout may provide pressure exceeds right atrial pressure. The
useful clues to the cause. Cardiac syncope resulting increase in right ventricular
often occurs without any warning (e.g. stroke volume prolongs right ventricular
Stokes-Adams attack) or may be preceded ejection time and hence pulmonary valve
by chest pain, palpitation or closure - accounting for the wide splitting.
lightheadedness (e.g. tachyarrhythmia). A The split between the sounds does not vary
history of aura preceding the blackout with respiration because the septal defect
suggests seizure but is frequently lacking, equalises pressures between the left and
perhaps due to post-ictal retrograde right side of the heart throughout the
amnesia. For this reason a lack of respiratory cycle. The increased right
premonitory symptoms is a fairly poor ventricular stroke volume may also
discriminator between the two. Urinary produce a systolic flow murmur over the
incontinence and brief twitching activity pulmonary valve.
are often taken as evidence of seizure but
may actually occur with any cause of Question 25. D) Atrial fibrillation
blackout. Cyanosis during the unconscious Explanation: A fourth heart sound is due
period suggests seizure, while extreme to atrial contraction against a stiff left
'death-like' pallor is characteristic of ventricle and is therefore not present in
cardiac syncope. A rapid recovery with atrial fibrillation.
full reorientation within 1 minute is very
suggestive of cardiac syncope and Question 26. C) Mitral regurgitation
virtually excludes seizure, which is Explanation: In contrast to a fourth heart
characterised by prolonged confusion, sound, which is always pathological, a
drowsiness and disorientation. third heart sound may be a normal
physiological finding (e.g. in young adults
Question 21. E) Defibrillation or pregnancy). The combination of a third
Explanation: The rhythm strip shows heart sound and soft first heart sound may
ventricular fibrillation characterised by occur in mitral regurgitation or heart
completely chaotic electrical activity. The failure.
only effective treatment is defibrillation. If
delivered immediately this will restore Question 27. D) Aortic regurgitation
cardiac output in more than 80% of Explanation: The murmurs of VSD,
patients, but the chances of successful tricuspid regurgitation and pulmonary
outcome fall by around 10% with each stenosis all occur in systole. Mitral
minute's delay. stenosis typically produces a mid-diastolic
murmur. The soft early diastolic murmur
Question 22. B) Atropine of aortic regurgitation is best heard with
Explanation: In a cardiac arrest scenario, the patient sitting forward in held
atropine should be administered as a single expiration and is often described as 'the
dose of 3 mg when the rhythm is either absence of silence' following the second
asystole or pulseless electrical activity heart sound. The link below provides a
with an electrical rate of less than 60 beats comprehensive guide to the evaluation of
per minute. heart murmurs.

Question 23. B) Mitral stenosis


Explanation: The classical low-pitched
rumbling diastolic murmur of mitral
stenosis is often very difficult to hear and
a loud first heart sound may be the most
prominent auscultatory finding.

173
Question 28. B) Raised intracranial On the other hand, the dihydropyridine
pressure CCBs (e.g. amlodipine, nifedipine) act on
Explanation: Raised intracranial pressure the peripheral vasculature to produce
is associated with hypertension and vasodilatation. These agents are used
bradycardia. mainly as antihypertensives and may
actually cause reflex tachycardia.
Question 29. C) Atrial flutter Diltiazem exerts both rate-limiting and
Explanation: Vagal manoeuvres, such as vasodilating properties and may be used
carotid sinus massage or Valsalva for rate control in AF.
manoeuvre, slow conduction through the
AV node and may terminate AV nodal re- Question 33. C) Atrioventricular (AV)
entry tachycardia or reveal flutter waves nodal re-entrant tachycardia
(as in this case) in atrial flutter. Explanation: Adenosine transiently
blocks conduction through the AV node
Question 30. A) Bisoprolol and may therefore terminate AV nodal re-
Explanation: Beta-blockers are the drug entrant tachycardia. It may be of
of choice to prevent occasional paroxysms diagnostic value in atrial flutter as the
of AF and are particularly useful when slowing of conduction often reveals flutter
there is associated hypertension (as in this waves but does not cardiovert this rhythm.
case), ischaemic heart disease or cardiac It can also be of use in distinguishing
failure. Verapamil and digoxin are both between ventricular tachycardia and a
used as rate-limiting agents in persistent supraventricular tachycardia with aberrant
AF but are not effective for preventing conduction. Vagal manoeuvres such as
paroxysms. ACE inhibitors are effective carotid sinus massage also slow or
antihypertensives but have no anti- transiently block AV conduction and may
arrhythmic properties. Amiodarone is be tried before adenosine.
associated with troublesome side-effects
that restrict its use to those in whom other Question 34. C) Wolff-Parkinson-White
measures have failed. syndrome
Explanation: The resting ECG shows a
Question 31. D) Subtherapeutic INR short PR interval and slurred upstroke of
Explanation: Therapeutic anticoagulation the QRS complex (known as a delta
for at least 4 weeks before and 4 weeks wave), suggesting an accessory pathway
after elective DC cardioversion is essential between the atria and ventricles. The
to minimise the risk of cerebral embolism. presence of an accessory pathway in
All of the other factors listed would reduce association with symptoms is known as
the chance of a successful long-term Wolff-Parkinson-White syndrome.
outcome from cardioversion but are not, in
themselves, contraindications. In patients Question 35. D) Catheter ablation of
with relapse following previous DC accessory pathway
cardioversion, anti-arrhythmic therapy Explanation: Percutaneous catheter
may reduce the risk of further recurrence ablation of the accessory pathway is the
and help maintain sinus rhythm. treatment of choice in patients with
symptomatic Wolff-Parkinson-White
Question 32. E) Amlodipine (WPW) syndrome. It is important to note
Explanation: Digoxin and _-blockers are that both digoxin and verapamil should be
both suitable options for rate control in avoided in WPW as they shorten the
atrial fibrillation. The other three options refractory period of the accessory
listed are calcium channel blockers pathway. Amiodarone prolongs the
(CCBs). The non-dihydropyridine CCBs refractory period and can be used as
(e.g. verapamil) act predominantly by prophylactic therapy but is associated with
slowing conduction at the AV node and troublesome long-term side-effects and is
exert powerful rate-limiting effects. less effective than catheter ablation.

174
Question 36. C) Presence of capture and Question 40. C) Clarithromycin
fusion beats on ECG Explanation: Macrolide antibiotics are a
Explanation: A history of ischaemic heart well-known cause of QT interval
disease (particularly previous myocardial prolongation, should be avoided in
infarction) and a QRS duration of > 140 patients with long QT syndrome, and
ms both favour the diagnosis of VT. should not be used with other drugs known
However, capture and fusion beats (see to cause QT prolongation, such as
link below) are pathognomonic of VT and amiodarone and sotalol.
therefore the best answer. Other features
that favour VT are extreme left axis Question 41. A) Magnesium
deviation (not right axis), lack of response Explanation: IV magnesium should be
to carotid sinus massage/IV adenosine and given in all cases. Class III anti-
evidence of atrioventricular dissociation. arrhythmics, such as amiodarone and
Note: VT is by far the most common cause sotalol, prolong the QT interval and may
of a broad complex tachycardia and if exacerbate the situation.
there is any doubt, manage as VT.
Question 42. D) Third-degree
Question 37. B) IV amiodarone (complete) heart block
Explanation: In any patient with VT the Explanation: Third-degree AV block
priority is to determine if there is a cardiac (complete heart block) results from
output. Pulseless VT is managed as a complete failure of conduction at the AV
cardiac arrest in the same way as node. Atrial impulses occur as usual,
ventricular fibrillation. In patients with a giving rise to normal P waves. The AV
cardiac output the next step is to establish block results in a bradycardic ventricular
if there is haemodynamic compromise escape rhythm, usually associated with
(e.g. systolic BP < 90; features of shock), wide QRS complexes due to the abnormal
in which case the appropriate treatment is activation of the ventricles. In some cases,
urgent DC cardioversion (under general the escape rhythm can arise from the AV
anaesthetic unless unconscious). node itself (nodal escape) and a narrow-
Patients, such as the one above, who complex bradycardia develops. There is no
tolerate the arrhythmia well may be treated relationship between atrial and ventricular
in the first instance with IV amiodarone activity on the ECG (AV dissociation).
or, less commonly, IV lidocaine. They
must be monitored closely for signs of Question 43. D) Dual-chamber
deterioration with full resuscitation pacemaker
facilities nearby. Explanation: This patient has a double
indication for pacemaker. Patients with
Question 38. C) Torsades de pointes symptomatic bradyarrhythmias associated
Explanation: Torsades de pointes literally with atrioventricular block should receive
means 'twisting points' and the ECG above permanent pacemakers. In addition,
shows the characteristic pattern of rapid patients with complete heart block may
irregular complexes that seem to oscillate derive prognostic benefit from pacemaker
around a fixed central axis. implantation, even in the absence of
symptoms.
Question 39. D) Prolonged QT interval
Explanation: Torsades de pointes is a
complication of prolonged QT interval.

175
Question 44. A) Nephrotoxicity The test result in scenario C should be
Explanation: Digoxin has a narrow classed as 'inconclusive'; coronary
therapeutic range and is predominantly angiography may provide useful
renally excreted. Consequently renal diagnostic information but other forms of
impairment may lead to digoxin toxicity stress test could also be considered (see
and any of features B-E, but link).
nephrotoxicity does not result from
digoxin toxicity. Question 48. A) Clopidogrel
Explanation: Clopidogrel is an
Question 45. C) Lipid-rich core antiplatelet drug used in patients intolerant
Explanation: Complications of of aspirin. It reduces the risk of adverse
atherosclerotic plaque rupture, notably events such as myocardial infarction in
myocardial infarction, stroke and acute patients with stable angina but does not
limb ischaemia, are the most common relieve symptoms. The combination of
cause of premature death in the developed aspirin and clopidogrel is also used in the
world. Understanding the mechanisms that management of acute coronary syndromes
lead to plaque instability and developing and after percutaneous coronary
novel therapeutic strategies to reduce the intervention. A comprehensive discussion
risk of rupture are therefore areas of major of anti-anginal drugs can be found in the
importance. link below.

Question 46. B) Raised high-density Question 49. B) Persistent ST segment


lipoprotein (HDL) cholesterol elevation on ECG
Explanation: Elevated HDL cholesterol is Explanation: Although unstable angina
protective against atherosclerosis. Raised may be associated with transient ST
low-density lipoprotein (LDL) cholesterol elevation, the presence of persistent ST
and, to a lesser extent, triglycerides elevation in the context of ischaemic-
increase the risk of atherosclerosis. sounding chest pain suggests an acute ST
elevation myocardial infarction. Other
Question 47. E) Chest pain and 1-2 mm causes of ST elevation include pericarditis
downward-sloping ST depression in and a left ventricular aneurysm (usually a
inferior leads during stage 1 of the long-term complication of a large anterior
Bruce protocol myocardial infarction).
Explanation: The most convincing ECG
evidence of myocardial ischaemia is the Question 50. C) Elevated serum
demonstration of reversible ST segment cholesterol
depression or elevation that corresponds Explanation: Elevated serum cholesterol
with the timing of the patient's symptoms. is a risk factor for the development of
Horizontal and downward-sloping ST coronary artery disease and should be
depression usually indicate myocardial addressed as part of a secondary
ischaemia, while upward-sloping ST prevention strategy, but it is not a marker
depression is less specific. Of the above for adverse outcomes in patients with
scenarios, D and E would both constitute a acute coronary syndromes.
positive test, but in E there is ischaemia at
low workload - a high-risk finding and
therefore an indication for coronary
angiography. In scenario D, ischaemia
occurs only at high workload and effort
tolerance is good, both of which are
reassuring features. A rise in pulse and BP
are normal haemodynamic responses to
exercise, whereas a fall in BP during a
stress test is a high-risk finding.

176
Question 51. D) Aspirin, clopidogrel and Primary percutaneous coronary
low molecular weight heparin intervention (PCI) is an alternative
Explanation: The link below includes the reperfusion therapy to thrombolysis that is
evidence base for this combined treatment. resource-intensive but produces better
Additional treatment with a glycoprotein outcomes. It is used in patients in whom
IIb/IIIa inhibitor (e.g. abciximab, thrombolysis is contraindicated. In some
tirofiban) may further improve outcomes countries, it is supplanting thrombolysis as
in refractory cases, particularly in the the first-line reperfusion treatment for ST
context of percutaneous coronary elevation MI. The term 'rescue PCI' refers
intervention. to the use of PCI in patients for whom
thrombolysis has failed to achieve
Question 52. E) Coronary angiogram reperfusion.
Explanation: This patient with unstable
angina has failed to settle with medical Question 55. B) IV atropine
therapy and has several high-risk features Explanation: The admission ECG shows
including recurrent pain at rest, ECG acute inferolateral myocardial infarction
changes and an elevated troponin. He (MI). Inferior MI is often complicated by
should therefore undergo coronary sinus bradycardia and transient
angiography with a view to atrioventricular (AV) block, due to
revascularisation through either ischaemia of the sinus node and AV node
percutaneous coronary intervention or, if respectively. The correct initial treatment
appropriate, bypass surgery. for symptomatic sinus bradycardia is
atropine 0.6 mg IV repeated, if necessary,
Question 53. B) Acute anterior to a maximum of 3 mg. AV block may
myocardial infarction also respond to atropine or resolve with
Explanation: The ECG shown was reperfusion but, if symptomatic, may
actually taken from a 48-year-old man require temporary pacemaker insertion. A
who had developed severe chest pain 6 permanent pacemaker is very rarely
hours earlier. It shows acute full-thickness needed as the block almost always
anterior myocardial infarction. He has ST resolves within 7-10 days.
elevation in leads I, aVL and V2-V6, and
has already developed Q waves in leads Question 56. E) Isosorbide mononitrate
V3-V5. Explanation: In addition to lifestyle
modifications, and in particular smoking
Question 54. D) IV metoprolol cessation, the first four agents listed form
Explanation: This patient has the core of secondary prevention following
uncontrolled hypertension which is a acute MI. Aspirin reduces the risk of
relative contraindication to thrombolysis. further infarction and other vascular events
If thrombolysis is the reperfusion strategy by 25% and _-blockers reduce long-term
of choice for this patient, then urgent mortality by 25%. The benefits of ACE
control of blood pressure is required inhibitors and statins following MI extend
before initiating therapy. This could be to patients with preserved left ventricular
achieved with either sublingual nifedipine function and serum cholesterol levels
or IV metoprolol (oral ACE inhibitor within the normal range. Isosorbide
would take too long to have an effect). In mononitrate offers no prognostic benefit
patients without contraindications (e.g. but might improve symptoms in patients
heart failure or bradycardia) intravenous _- who continue to experience exertional
blockers relieve pain, reduce arrhythmias angina post-infarction.
and improve short-term mortality, making
them the best option here. Large-scale
trials in acute MI have shown no survival
benefit from use of magnesium or calcium
channel blockers.

177
Question 57. B) Night pain Question 61. E) Addison's disease
Explanation: Chronic lower limb Explanation: The vast majority (95%) of
ischaemia manifests as two distinct patients with hypertension have no
clinical entities: intermittent claudication specific underlying pathology but in a
(IC) and critical limb ischaemia (CLI). small proportion of patients the elevated
The latter is by far the more dangerous blood pressure may be due to endocrine
condition; it is almost always due to severe disease, renal disease, drugs or coarctation
multi-level disease and carries a high risk of the aorta. Addison's disease (primary
of limb loss and mortality within a matter adrenal insufficiency) frequently presents
of months. Night pain is one of the with hypotension.
cardinal features of CLI.
Diminished pedal pulses are a common Question 62. D) Thiazide diuretic
finding in both IC and CLI and the Explanation: Thiazide diuretics (e.g.
presence of a bruit provides no bendrofluazide) or calcium channel
information on disease severity. ABPI, on blockers are recommended first-line
the other hand, is a simple but useful guide therapy for elderly patients with
to severity. A ratio of 0.8 is typical of IC; hypertension.
it is usually < 0.5 in CLI.
Question 63. C) Calcium channel
Question 58. C) Previously undiagnosed antagonist
atrial fibrillation Explanation: See Guide to management
Explanation: An obvious embolic source of hypertension
such as atrial fibrillation would increase
the probability of embolism, particularly if Question 64. C) Acute rheumatic fever
it were a new finding, as the patient would Explanation: This teenage patient has
almost certainly not be anticoagulated. All evidence of recent streptococcal infection
of the other features listed would favour (ASOT > 200 U/ml), pericarditis and
thrombotic occlusion of a previously mitral regurgitation, making rheumatic
diseased artery. fever the most likely diagnosis.
Question 65. E) Flitting polyarthritis
Question 59. D) New early diastolic Explanation: The diagnosis of acute
murmur rheumatic fever is based on the revised
Explanation: Type B dissections may Jones criteria, and requires evidence of
involve the descending aorta and aortic recent streptococcal infection as well as
arch but, by definition, spare the ascending two or more major manifestations, or one
aorta. A new early diastolic murmur major and two or more minor
suggests aortic regurgitation and hence manifestations. This patient already has
involvement of the aortic root. sufficient evidence of streptococcal
infection in the form of elevated
Question 60. A) Hyperlipidaemia antistreptolysin O titres so a positive throat
Explanation: The pathogenesis of swab is incorrect. He currently fulfils one
essential hypertension is poorly of the major criteria ('carditis') and cardiac
understood but all of the factors listed dilatation would simply provide further
other than hyperlipidaemia (which is a risk evidence of this criterion. Fever and a
factor for atherosclerosis) are contributing prolonged P-R interval are both minor
environmental factors. criteria so neither one would be sufficient
for diagnosis. Flitting polyarthritis is
another of the major criteria (seen in 75%
of patients) and would therefore clinch the
diagnosis. The revised Jones criteria are
included in the link below.

178
Question66. D) Mitral stenosis Question 70. B) Third heart sound (S3)
Explanation: The signs listed are typical Explanation: Atrial fibrillation,
of mitral stenosis, as is the history of pulmonary congestion and pulmonary
slowly progressive symptoms. Atrial hypertension (loud P2 and right ventricular
fibrillation is an extremely common heave) commonly complicate both mitral
complication owing to left atrial stenosis and regurgitation.
enlargement. Aortic regurgitation typically A third heart sound is caused by rapid
causes an early diastolic murmur and large filling of the left ventricle during diastole;
volume pulse. therefore, its presence effectively excludes
significant mitral stenosis but is
Question 67. E) Loud pulmonary characteristic of mitral regurgitation.
component of second heart sound (P2)
Explanation: A loud S1, opening snap Question 71. E) Slow-rising carotid
and tapping apex beat are all classical pulse
signs of mitral stenosis. The presence of a Explanation: A slow-rising pulse is a
pansystolic murmur radiating to the axilla characteristic finding in aortic stenosis,
would suggest concomitant mitral often accompanied by a narrow pulse
regurgitation. Clinical findings suggestive pressure, heaving but non-displaced apex
of pulmonary hypertension include a loud beat and a soft A2.
P2 and parasternal heave (right ventricular In general, left-sided heart murmurs are
hypertrophy), signs of tricuspid louder in expiration; right-sided heart
regurgitation (secondary to right murmurs are louder in inspiration.
ventricular dilatation) such as systolic
waves in the jugular venous pulse and, Question 72. A) Syncopal episodes
ultimately, signs of right ventricular Explanation: Reduced valve area and a
failure such as pitting oedema and ascites. high pressure gradient across the valve are
both indicators of severity in aortic
Question 68. B) Concomitant mitral stenosis, but the presence of symptoms is
regurgitation by far the most important indication for
Explanation: Mitral balloon valvuloplasty valve replacement.
is the treatment of choice for patients who
fulfil the necessary criteria. The presence Question 73. B) Acute aortic dissection
of mitral regurgitation is a Explanation: Bounding pulses, a wide
contraindication. pulse pressure and displaced apex beat are
characteristic of severe chronic aortic
Question 69. E) Congenitally bicuspid regurgitation due to left ventricular
mitral valve dilatation and increased stroke volume
Explanation: This one is a bit of a trick consequent upon chronic volume overload
question, as the normal mitral valve is of the left ventricle. Syphilitic aortitis,
bicuspid. A congenitally bicuspid aortic rheumatic heart disease, Marfan's
valve (usually tricuspid) may predispose to syndrome and ankylosing spondylitis are
either aortic stenosis or regurgitation. all well-recognised causes. However, in
acute aortic regurgitation (e.g. secondary
to involvement of aortic root in acute
aortic dissection) there is no time for
compensatory hypertrophy and dilatation
of the ventricle. The apex beat is therefore
not displaced and the clinical picture tends
to be dominated by features of heart
failure and hypotension.

179
Question 74. A) Viridans streptococci Question 77. C) Patent ductus
Explanation: Streptococci of the viridans arteriosus
group are the most common cause of Explanation: More on Patent ductus
native valve endocarditis, accounting for arteriosus
30-40% of cases. They tend to cause
subacute endocarditis with a presentation Question 78. A) Ventricular septal
like the one described above. In contrast, defect
Staph. aureus endocarditis usually Explanation: This patient has developed
presents as a severe acute febrile illness. It Eisenmenger's syndrome and therefore the
is a common organism in IV drug original underlying abnormality must have
misusers, often affecting right-sided involved a left-right shunt, e.g. ventricular
valves. Coagulase-negative staphylococci, septal defect or patent ductus arteriosus. In
such as Staph. epidermidis, are frequent patent foramen ovale there is a connection
culprits in prosthetic valve endocarditis. between the left and right side of the heart
Coxiella burnetti is the causative agent in but no significant shunt. The other options,
Q fever, a cause of culture-negative in isolation, are not associated with
endocarditis. shunting of blood and are not causes of
cyanotic heart disease.
Question 75. D) Benzyl penicillin and
gentamicin Question 79. D) Patent ductus
Explanation: In this patient with subacute arteriosus
endocarditis, streptococci are the most Explanation: More on Tetralogy of Fallot
likely causative agents and, therefore,
combined treatment with benzyl penicillin Question 80. A) Tuberculosis
and gentamicin is the best choice of Explanation: The patient has symptoms
empirical therapy. The management of and signs of severe right heart failure with
infective endocarditis, including rationale no obvious features of left heart failure, a
for choice of antibiotic, is described in the normal-sized heart and heavy calcification
link below. of the pericardium. The diagnosis is
constrictive pericarditis. In a patient from
Question 76. D) Vegetations on West Africa the most likely underlying
echocardiography diagnosis would be tuberculosis. This is a
Explanation: See Modified Duke criteria common presenting feature of AIDS in
West Africa and he should be offered HIV
testing. Haemochromatosis and
trypanosomiasis may both cause a
cardiomyopathy.

180
Chapter 18 RESPIRATORY
DISEASE
Question 1. C) PaCO2 of 5.8 kPa Question 4. D) Smoking cessation
Explanation: The other features all Explanation: Smoking cessation is one of
suggest a severe asthma attack but only the very few interventions shown to
answer C is a life-threatening feature. prolong life in COPD. Nebulised and
Patients with acute severe asthma should inhaled ß2-agonists provide symptomatic
have a depressed PaCO2 on account of the relief but do not influence prognosis. Oral
increased alveolar ventilation. Even a steroids are useful in the treatment of acute
normal PaCO2 is a sign that the patient is exacerbations but have no role in
struggling to overcome the obstruction to maintenance treatment; inhaled steroids
airflow and, perhaps, beginning to tire may reduce the number of acute
from the effort of breathing. In this case exacerbations and hospital admissions in
the PaCO2 is at the high end of the normal patients with severe disease and frequent
range and, in addition to other appropriate exacerbations. Tiotropium is a once-daily
measures, the intensive treatment unit inhaled anticholinergic treatment which
should be alerted. improves both symptoms and effort
tolerance but, like ß2-agonists, has not
Question 2. B) Chronic obstructive been shown to reduce mortality.
pulmonary disease (COPD)
Explanation: This lifelong smoker has an Question 5. D) Transfer to a critical
obstructive ventilatory defect that is not care environment and consider
reversible by inhaled bronchodilators. Her endotracheal intubation by an
lungs are hyperexpanded and there is no experienced anaesthetist
clinical evidence of pulmonary oedema or Explanation: The likely diagnosis is acute
fibrosis. epiglottitis. The acute onset makes non-
infectious causes of stridor, such as
Question 3. B) Compensated respiratory tumour compression of the trachea, less
acidosis likely, so a CT would be of limited use. In
Explanation: The results show a epiglottitis, the problem is the potential for
respiratory acidosis (high PaCO2) that is sudden airway occlusion. Nebulised
completely offset by a metabolic alkalosis bronchodilators will offer no benefit,
(high HCO3-), so that the overall pH of the while arterial blood gas analysis is
blood is normal. By far the most likely unlikely to provide much useful
explanation for this is chronic type II information and may exacerbate the
respiratory failure with metabolic situation by distressing the patient.
compensation. The only theoretical Examination of the throat is extremely
alternatives are complete respiratory dangerous and should be avoided. The
compensation for metabolic alkalosis or patient must be managed with intravenous
concomitant but unrelated respiratory antibiotics in facilities appropriate for
acidosis and metabolic alkalosis. In endotracheal intubation if necessary.
practice, the former does not occur and the
latter is extremely unlikely.

181
Question 6. C) Stony dull percussion Question 9. A) Psychogenic
note at left base hyperventilation
Explanation: The patient has a moderate Explanation: Psychogenic
to large left-sided pleural effusion which hyperventilation is, to a large extent, a
will manifest clinically as reduced diagnosis of exclusion, and alternative
expansion of the left hemithorax possibilities such as pneumothorax and
accompanied by stony dullness to particularly pulmonary embolism should
percussion, reduced or absent breath always be considered. However, there are
sounds, and reduced or absent vocal several pointers towards the diagnosis
resonance over the left base and mid-zone. here, most notably the inability to take a
Large effusions can cause tracheal and deep breath, history of recurrent episodes,
mediastinal shift although, from the chest absence of chest signs and reassuring
X-ray, there does not appear to be any oxygen saturations. The hyperventilation
tracheal deviation in this case. causes a respiratory alkalosis that may
lead to paraesthesiae in the hands and feet
Question 7. D) Pleural aspiration and carpopedal spasm - both useful
Explanation: The patient has a unilateral distinguishing features.
pleural effusion. The most helpful
investigation in establishing the cause will Question 10. B) Rusty, golden yellow-
be evaluation of the pleural fluid obtained coloured sputum
by simple aspiration. The appearance of Explanation: Pneumocystis carinii
the fluid should be noted (e.g. blood- pneumonia (PCP) (caused by
stained, purulent) and it should then be Pneumocystis jirovecii) typically presents
sent for biochemical, microbiological and with a dry, non-productive cough; rusty-
cytological analysis. brown spit is a classical finding in
pneumococcal pneumonia. PCP is a
Question 8. B) Parapneumonic effusion common and life-threatening infection in
Explanation: The pleural effusion is an AIDS sufferers. At presentation, dyspnoea,
exudate. We know this because the pleural fever and a dry cough are usual. The
fluid protein is more than half the plasma dyspnoea is out of proportion to the chest
protein content and also because the X-ray changes; indeed, many patients have
pleural fluid LDH is > 60% of the plasma a normal chest X-ray and most have no
LDH (see Light's criteria in the link focal abnormalities on chest auscultation.
below). Nephrotic syndrome and cardiac Induced sputum is the diagnostic test of
failure cause transudative effusions. Acute choice for PCP. Clinical findings
pancreatitis is highly unlikely in the suggestive of HIV infection, such as
absence of either abdominal pain or a oropharyngeal candidiasis or oral hairy
raised serum amylase. Thoracic duct leukoplakia, would also raise suspicion.
occlusion causing chylothorax is very rare
and the pleural aspirate would have a
milky rather than serous appearance. The
diagnosis of parapneumonic effusion is
further supported by the patient's
confusion, the raised white cell count and
the low pH of the pleural fluid.
Establishing the cause of a pleural effusion
is a commonly encountered problem in
clinical practice and more information can
be found in the link below.

182
Question 11. E) Intravenous Question 14. D) Viral pneumonia
aminophylline Explanation: Viral pneumonia is very
Explanation: This patient has several unlikely to have lasted several months and
features to suggest this is an acute severe would be associated with chest X-ray
asthma exacerbation (respiratory rate > changes. The other options in this question
25/min, heart rate > 115, PEF < 50% are reasonable differential diagnoses for a
predicted, inability to speak in sentences) patient with a persistent cough, although
and she should be managed accordingly. bronchogenic carcinoma must also be
Options A-D are all key steps but considered, particularly in smokers. Cough
intravenous aminophylline should only be may be the dominant symptom in asthma
used in patients who fail to respond to with little or no breathlessness or wheeze.
these and other measures (e.g. intravenous
magnesium) and, even then, usually under Question 15. B) Aggressive fluid
specialist supervision. More information resuscitation is necessary immediately
on assessment of asthma severity can be Explanation: Community-acquired
found in Q1 and further details of the pneumonia is by far the likeliest diagnosis
management of asthma exacerbations in here; non-specific elevation of D-dimer
the link below. occurs in a number of conditions other
than pulmonary embolism, including
Question 12. C) Refer to the intensive pneumonia. She has severe sepsis with
treatment unit for consideration of evidence of haemodynamic compromise
mechanical ventilation and requires prompt, aggressive fluid
Explanation: The patient has deteriorated resuscitation, ideally with monitoring of
significantly despite maximal medical both urine output and central venous
therapy and this is now a life-threatening pressure; she may require inotropic
asthma attack (PEF < 33% predicted, high support and invasive ventilation if her
alveolar/arterial oxygen gradient, rising condition does not improve. Current
PaCO2, quiet breath sounds, exhaustion). British Thoracic Society guidelines
She is likely to require intubation and specify four markers of severity in
mechanical ventilation imminently and community-acquired pneumonia, known
should be referred to ITU without delay. as the CURB score:
Note that, although non-invasive • confusion
ventilation may be useful in exacerbations • urea > 7 mmol/l
of COPD, it has no role in the • respiratory rate > 30/min
management of acute severe asthma. • low BP (systolic < 90 mmHg or diastolic
Likewise, in acute asthma, the presence of < 60 mmHg).
a high or rising PaCO2 must not be taken This patient has all four markers and is
as an indication to reduce oxygen undoubtedly in the severe category. She
concentration but rather as a warning of requires immediate treatment with
deterioration and possible need for intravenous third-generation cephalosporin
intubation and mechanical ventilation. and macrolide; it is inappropriate to delay
pending the results of cultures. Her
Question 13. D) Anticholinergic management should be supervised in a
Explanation: Ipratropium is an high-dependency or intensive care unit.
anticholinergic bronchodilator. Tiotropium Management on a general medical or
is a newer, once-daily preparation. general respiratory ward is inappropriate
Nebulised ipratropium forms part of the for someone this ill.
treatment of acute severe asthma but its
main use is in the management of COPD.

183
Question 16. B) Pulmonary embolism Question 19. C) Fibrotic lung disease
Explanation: The sudden onset of Explanation: These are all classical
symptoms, pleuritic chest pain and lack of findings in pulmonary fibrosis. Cystic
change in sputum favour the diagnosis of fibrosis presents at an earlier age;
pulmonary embolism over infective however, severe bilateral bronchiectasis
exacerbation of COPD. She also has a risk might explain the findings, although the
factor for thromboembolism in the form of crackles would tend to be coarse.
oestrogen-containing hormone
replacement therapy. One would expect Question 20. A) Larynx and central
chest X-ray changes in acute left airways
ventricular failure or pneumonia; Explanation: A useful fact to help
psychogenic hyperventilation should not understand examination findings.
be considered as a diagnosis in the
presence of hypoxia. Question 21. D) Three lobes on the
right, two on the left
Question 17. A) Computed tomography Explanation: This crucial fact is essential
pulmonary angiogram (CTPA) for correct interpretation of all lung
Explanation: In a patient with pre- imaging.
existing lung disease such as COPD, in Question 22. C) Sarcoidosis
whom a pulmonary embolism (PE) is Explanation: The skin lesion described is
suspected, CTPA is undoubtedly the characteristic of erythema nodosum. The
investigation of choice. A V/Q scan may combination of erythema nodosum,
be hard to interpret, as existing lung arthralgia, constitutional symptoms and
disease may cause perfusion defects and isolated BHL on chest X-ray is highly
the majority of such cases are reported as suggestive of sarcoidosis.
'indeterminate'. Echocardiography may
help in the urgent diagnosis of massive PE Question 23. D) Reduced mid-
in patients with circulatory collapse by expiratory flow indicates collapse of
identifying features of acute right heart small intrathoracic airways
strain, but is less likely to be helpful in this Explanation: The loop shows the classical
situation. The value of D-dimer is the high pattern of intrathoracic small airways
negative predictive value conferred by a obstruction, as seen in chronic obstructive
low result. It is, however, non-specific and pulmonary disease (COPD). Large airway
a positive D-dimer, no matter how high, obstruction limits peak inspiratory and/or
would not confirm the diagnosis. HRCT is peak expiratory airflow (depending on the
mainly used in the assessment and site of obstruction), producing a plateau on
diagnosis of diffuse interstitial lung the loop. Vital capacity is frequently
disease and bronchiectasis. reduced in obstructive ventilatory defects,
but the proportional reduction in forced
Question 18. B) Percussion and expiratory volume in 1 second (FEV1) is
auscultation of the lung fields greater, giving a reduced FEV1/vital
Explanation: The most likely diagnosis is capacity (VC) ratio. The loop shows that
tension pneumothorax, which can and peak expiratory flow is reduced. At a
must be diagnosed on clinical given lung volume, the limitation of flow
examination. Reduced breath sounds and is much greater during expiration than
hyper-resonant percussion note on the inspiration, indicating dynamic collapse of
affected side confirm the diagnosis, and the small intrathoracic airways.
immediate needle thoracocentesis should Question 24. E) Functional residual
be performed. capacity
Explanation: A full explanation of all of
the above terms and uses of respiratory
function tests in clinical practice can be
found in the link below.

184
Question 25. A) Chest X-ray Type II respiratory failure is hypoxia
Explanation: Pneumothorax should be (PaO2 < 8 kPa) in the presence of
excluded in any apparent acute severe hypercapnia (PaCO2 > 6.6 kPa) and is due
asthma exacerbation, especially when to inadequate ventilation (not 'blowing off'
there are supportive clinical findings. sufficient CO2). It is important to bear in
mind that any cause of type I respiratory
Question 26. A) Long-term oxygen failure may progress to type II if
therapy exhaustion supervenes.
Explanation: Long-term domiciliary
oxygen therapy has been shown to Question 32. C) A trial of itraconazole
improve survival in selected patients with should be considered
COPD. Explanation: The most likely diagnosis is
Question 27. D) Calcification on chest allergic bronchopulmonary aspergillosis
X-ray (ABPA), caused by a hypersensitivity
Explanation: Calcification may be found reaction to Aspergillus fumigatus.
in malignant tumours, particularly on CT, Common manifestations include fever,
but more often it indicates a granuloma, breathlessness, cough productive of
perhaps from old tuberculosis. bronchial casts and worsening of asthmatic
symptoms, which may be accompanied by
Question 28. C) Protuberant mandible radiographic evidence of transient diffuse
Explanation: A recessed mandible is pulmonary infiltrates and lobar or
associated with airway obstruction during segmental pulmonary collapse. Other
sleep. characteristic features include an elevated
peripheral blood eosinophilia (> 0.5 ×
Question 29. B) Arterial blood gas 109/l), positive skin test to an extract of A.
(ABG) analysis fumigatus, serum precipitating antibodies
Explanation: The most likely cause of to A. fumigatus and an elevated total
this patient's deterioration is type II serum IgE. Fungal hyphae of A. fumigatus
respiratory failure. An ABG is essential to may be seen on microscopic examination
confirm this, assessing the severity and of sputum (which will be rich in
guiding further management. eosinophils). The optimum maintenance
therapy for ABPA is uncertain. Regular
Question 30. E) Mechanical ventilatory therapy with low-dose oral corticosteroid
support therapy (prednisolone 7.5-10 mg daily) is
Explanation: This patient is in extremis prescribed to suppress the
and at high risk of imminent cardiac arrest immunopathological responses and
because of hypoxia. Her normal prevent progression to tissue damage.
bicarbonate shows that this is an acute not Therapy with itraconazole may be
a chronic deterioration, therefore probably beneficial in some patients. As this patient
reversible. Her oxygen must not be has never smoked and lobar collapses have
reduced, but unfortunately increasing it is occurred in two different sites, lung cancer
unlikely to be enough to save her, is unlikely.
particularly as this may further decrease
her respiratory drive. The best solution is Question 33. B) Non-steroidal anti-
mechanical ventilatory support, which can inflammatory drugs
be delivered by non-invasive bi-level Explanation: Non-steroidal anti-
ventilation or by intubation in an ITU. inflammatory drugs and ß-blockers are the
drugs most frequently implicated in
Question 31. E) Severe kyphoscoliosis exacerbations of asthma. Beta-agonists
Explanation: Type I respiratory failure and leukotriene receptor antagonists are
refers to hypoxia (PaO2 < 8 kPa) with a used in the management of asthma.
normal or low PaCO2 (i.e. failure of
oxygenation) and is primarily caused by
ventilation-perfusion mismatch.

185
Question 34. B) A 20% improvement in Interestingly, heterozygotes (who have
forced expiratory volume in 1 second one normal and one abnormal CFTR gene)
(FEV1) following administration of a appear to have some protection from fluid
bronchodilator loss in gastroenteritis such as cholera.
Explanation: All of these may be present Question 38. B) Amiodarone
in an asthmatic patient, but only B Explanation: See list of Drug-induced
clinches the diagnosis according to current respiratory diseases
criteria. Asthma is a very common disease,
so a knowledge of the diagnosis is Question 39. B) Tuberculosis
essential. Explanation: Caseating granulomas are a
classic finding in tuberculosis, which is
Question 35. B) Addition of long-acting often discovered after a 'cryptic'
ß2-adrenoceptor agonist presentation such as this, particularly in
Explanation: It is always important in this the elderly.
situation to check inhaler technique,
adherence to treatment and avoidance of Question 40. C) Right upper lobe
aggravating factors first before increasing collapse
drug therapy. Long-acting inhaled ß2- Explanation: This chest X-ray has the
agonists (e.g. salmeterol) are first choice classic appearance of right upper lobe
of add-on therapy for patients whose collapse. The link below shows the chest
symptoms are inadequately controlled on X-ray features associated with collapse of
regular inhaled corticosteroid and an as- the other lobes.
required short-acting ß2-agonist. Asthma
treatment is guided by a stepwise Question 41. B) Small-cell bronchogenic
treatment model, based on good clinical carcinoma
evidence and detailed in the link below. Explanation: Lobar collapse in a smoker
is commonly due to lung cancer. Of the
Question 36. C) 76-year-old ex-smoker various histological subtypes of bronchial
with bilateral pitting oedema to knees carcinoma, small-cell carcinoma most
and maximum PaO2 of 7.7 kPa on two commonly causes hyponatraemia through
ABGs 3 weeks apart ectopic inappropriate secretion of
Explanation: Criteria for LTOT are based antidiuretic hormone (ADH).
on two sets of ABG measurements taken
at least 3 weeks apart in clinically stable Question 42. C) Squamous carcinoma
patients on optimal medical therapy. To Explanation: Squamous cell cancers may
qualify, patients must have stopped cause hypercalcaemia by secreting ectopic
smoking and demonstrate either: parathyroid hormone (PTH). Small-cell
• PaO2 < 7.3 kPa (and FEV1 < 1.5 litres) carcinomas commonly secrete antidiuretic
• PaO2 7.3-8 kPa plus pulmonary hormone (ADH) and adrenocorticotrophic
hypertension, peripheral oedema or hormone (ACTH).
nocturnal hypoxaemia.
Only the patient in C meets these Question 43. A) Continued cigarette
requirements. LTOT is one of the few smoking
treatments that favourably alters prognosis Explanation: Cigarette smoking is a
in COPD. More information can be found contraindication to domiciliary oxygen
in the link below. therapy but this is on the grounds of safety
(oxygen is a fire hazard). It is not a
Question 37. A) Autosomal recessive contraindication to surgical resection of
Explanation: The gene for CFTR (cystic bronchial carcinoma.
fibrosis transmembrane conductance
regulator) is on the long arm of
chromosome 7. A defect in the genes
encoding CFTR from both parents is
required for clinical disease.

186
Question 44. E) Asthma disease is often progressive. The patient
Explanation: Asthmatic patients often should, therefore, be removed from the
have relatively normal lung function at offending environment as soon as
rest so asthma is not usually associated possible. Enlargement of the hilar glands
with chronic hypoxaemia and the majority with an 'egg-shell' pattern of calcification
of patients have normal pulmonary artery is said to be a characteristic chest X-ray
pressures. In contrast, the other conditions appearance but is uncommon and non-
listed cause progressive pulmonary specific. The appearances described are
hypertension, which is associated with not those of miliary tuberculosis; however,
increased complications and mortality. it should be remembered that patients with
silicosis may be at increased risk of
Question 45. D) Rheumatic fever tuberculosis - so-called 'silicotuberculosis'.
Explanation: Connective tissue disorders The recognition of occupational lung
may lead to pulmonary fibrosis and a disease should prompt screening of the
variety of other respiratory complications. workplace to identify other affected
individuals.
Rheumatic fever may cause pleurisy or a
pleural effusion, but not fibrosis. Question48. B) The clinical scenario
could be explained by bleomycin
Question 46. E) Intravenous endothelin
toxicity
Explanation: Endothelin is a potent
Explanation: It is correct to consider a
pulmonary vasoconstrictor. The endothelin
diagnosis of PCP; however, induced
receptor antagonist bosentan has shown
sputum is generally the investigation of
some promising results in pulmonary
choice. Bronchoalveolar lavage may be
hypertension.
considered as a second-line investigation.
In this case induced sputum samples for
Question 47. B) The most likely
PCP and cytomegalovirus were negative
diagnosis is silicosis and the most likely diagnosis was
Explanation: This history and the chest
bleomycin-induced pneumonitis. Lung
X-ray appearances are highly suggestive
function tests classically show a restrictive
of silicosis. Silicosis results from the
defect and patients typically desaturate on
inhalation of crystalline or free silica,
exertion. HRCT will confirm the presence
usually in the form of quartz. Occupations
of interstitial inflammation but is not
at risk include stonemasons, sandblasters
usually diagnostic.
and miners. Silica is highly fibrogenic and
even when exposure to dust ceases, the

187
Chapter 19 ENDOCRINE DISEASE
Question 1. B) Thyroid-stimulating Question 3. E) Toxic thyroid adenoma
hormone (TSH) Explanation: In toxic adenoma there is
Explanation: The thyroid is the only increased isotope uptake by the adenoma.
tissue which makes significant However, excessive thyroxine production
physiological use of iodine. The functional by the adenoma inhibits pituitary thyroid-
unit of the thyroid gland is the follicle, stimulating hormone (TSH) secretion so
consisting of central colloid surrounded by that the remainder of the thyroid gland is
follicular epithelial cells. Inorganic iodide suppressed, resulting in negligible isotope
is trapped and transported into these cells uptake. In toxic multinodular goitre there
and organified by the enzyme thyroid is patchy uptake by the hyperfunctioning
peroxidase; thereafter the iodine is bound nodules. Thyroiditis is associated with low
to thyroglobulin protein. This iodinated isotope uptake by the thyroid as the
molecule is cleaved to release the thyroid follicular cells are not functioning
hormones thyroxine (T4) and normally during a temporary inflammation
triiodothyronine (T3). T4 can be converted of the gland. (The excess thyroid hormone
to either T3 or rT3 in peripheral tissues. A is released from stores within the follicle,
dietary iodide intake greater than 100 µg rather than synthesised de novo.) Thyroid
per day is necessary for thyroid hormone malignancies are not usually associated
synthesis. Thyroid-stimulating hormone with derangements of thyroid function and
(TSH, thyrotrophin) is a glycoprotein are visible as 'cold' (low-uptake) areas on a
released from the anterior pituitary which scintigraphy scan. Medullary carcinoma
stimulates the whole process, and is arises from the parafollicular C cells
subject to negative feedback from T4 and which produce calcitonin. A very useful
T3. TSH does not contain iodine. The anti- guide to establishing the diagnosis in
arrhythmic drug amiodarone contains thyrotoxicosis is provided in the link
large amounts of iodine. below.

Question 2. C) Graves' thyrotoxicosis Question 4. B) Lid lag


Explanation: The raised T4 with Explanation: Lid retraction (visible sclera
undetectable TSH levels indicates primary above the iris) and lid lag (delay in
thyrotoxicosis. The most common cause of downward movement of the upper eyelid
thyrotoxicosis is the autoimmune during downgaze) can occur with all
condition, Graves' disease. TSH receptor causes of thyrotoxicosis. They result from
antibodies are very rare in patients without the effect of increased sympathetic activity
autoimmune thyroid disease and their on the levator palpebrae superioris
presence therefore confirms the diagnosis. muscles. The other listed ocular features
Note, however, that 5-20% of patients are only seen in Graves' disease, although
with Graves' disease are negative for these their presence is unrelated to the severity
antibodies; had the result been negative of the thyrotoxicosis. Ophthalmoplegia
rather than positive, this would not have and diplopia result from swelling and
excluded the diagnosis of Graves' disease eventual fibrosis of the extraocular
and further tests (thyroid scintigraphy) muscles. Other features specific to Graves'
might have been required. are pretibial myxoedema and thyroid
acropachy (finger clubbing).

188
Question 5. D) Thyroid-stimulating Hyperprolactinaemia does not cause
hormone (TSH) receptor antibodies hypothyroidism but may occur as a result
(TRAb) of primary hypothyroidism due to
Explanation: Stimulating TRAb increased hypothalamic thyrotrophin-
antibodies are responsible for stimulating releasing hormone (TRH) secretion (TRH
the TSH receptors and causing stimulates prolactin release).
thyrotoxicosis. Occasionally, blocking
TRAb antibodies cause episodes of Question 8. C) Start treatment with
hypothyroidism in patients with thyroxine
autoimmune thyroid disease. Routine Explanation: This patient has primary
assays cannot distinguish stimulating from hypothyroidism, as indicated by the low T4
blocking antibodies but TRAbs are present level with raised TSH. She has clinical
in up to 95% of patients with Graves' symptoms of hypothyroidism and
disease and are rare in patients with other treatment with thyroxine is required. The
forms of thyroid disease. positive anti-thyroid peroxidase antibody
result suggests an autoimmune cause such
Question 6. B) Propranolol as Hashimoto's disease. Imaging of the
Explanation: Thyrotoxicosis is an thyroid gland is unlikely to provide any
important cause of atrial fibrillation (AF). additional information, and such cases can
Of the agents listed above, the non- generally be managed in the primary care
selective _-blocker propranolol is best for setting. Carbimazole is used in the
AF associated with hyperthyroidism and is treatment of hyperthyroidism.
also useful for many other symptoms of
hyperthyroidism (particularly tremor) Question 9. B) Normal ageing
which are mediated by increased Explanation: As phenytoin and
sympathetic activity. Ventricular rate in rifampicin are liver enzyme inducers, they
thyrotoxic patients with AF increase metabolism of thyroxine, often
characteristically responds poorly to necessitating an increase in the thyroxine
digoxin. Prolonged use of amiodarone can dose. In coeliac disease there is
lead to hyper- or hypothyroidism. Acute malabsorption, so a higher dose of oral
DC cardioversion is reserved for those thyroxine may be required. Pregnancy is
patients who are haemodynamically associated with increased levels of
compromised by the rapid ventricular rate. thyroid-binding globulin; a greater
Two other key elements of this patient's proportion of thyroid hormone is therefore
management will be definitive treatment bound to protein, and an increased
of her thyrotoxicosis and anticoagulation thyroxine dose may be needed to maintain
(thromboembolic complications are the same levels of free active thyroid
particularly common in thyrotoxic AF). hormone. Ageing leads to a reduction in
thyroxine clearance and smaller doses of
Question 7. D) Hyperprolactinaemia thyroxine are often required.
Explanation: The most common causes of
hypothyroidism in the UK are autoimmune
disease (including Hashimoto's thyroiditis)
and thyroid failure following
thyroidectomy or radio-iodine therapy.
Transient thyroiditis (e.g. de Quervain's
thyroiditis, post-partum thyroiditis) is
often associated with an initial thyrotoxic
phase, followed by transient
hypothyroidism. Although Graves' disease
usually causes hyperthyroidism, the
thyroid-stimulating hormone (TSH)
receptor antibodies can sometimes have a
blocking effect rather than a stimulatory
one, resulting in hypothyroidism.
189
Question 10. D) Give IV Since episodic relapse and remission is the
triiodothyronine and IM hydrocortisone most common pattern in the natural
Explanation: Myxoedema coma is a rare history of Graves' thyrotoxicosis,
presentation of hypothyroidism. It has a carbimazole is usually given for 12-18
50% mortality rate and needs urgent months in the anticipation that the disease
treatment even before biochemical will be in remission when the drug is
confirmation of the diagnosis. IV discontinued. However, thyrotoxicosis
triiodothyronine replacement is required. recurs in at least 50% of patients within 2
Unless it is clear that the patient has years of stopping treatment. Subtotal
primary hypothyroidism (e.g. thyroidectomy or radioactive iodine
thyroidectomy scar), hydrocortisone therapy may then be preferred.
should also be administered to cover the Agranulocytosis is a rare side-effect of
possibility of coexisting corticosteroid carbimazole, occurring in 0.2% of
deficiency due to hypothalamic or patients; patients are therefore advised to
pituitary disease. In hypopituitarism, undergo urgent testing of their full blood
administration of thyroid hormone in count in the event of a sore throat or fever.
advance of hydrocortisone replacement Although carbimazole and
can be dangerous, as the increase in propylthiouracil are only excreted in small
metabolic rate may precipitate an adrenal amounts in breast milk, propylthiouracil is
crisis. Supportive measures (e.g. oxygen, thought to be safer.
IV fluids and rewarming) are also
important, but the bradycardia is Question 13. A) Medullary carcinoma
appropriate to her slow metabolic rate and Explanation: Medullary carcinoma arises
is likely to respond to hormone from the parafollicular C cells of the
replacement. thyroid. Tumour cells secrete calcitonin,
and serum levels can be used to monitor
Question 11. A) Facial nerve injury response to treatment. The condition
Explanation: Complications of subtotal presents in middle age with a firm thyroid
thyroidectomy are rare. A hoarse voice or mass, and treatment is by total
change in voice quality can result from thyroidectomy. Thyroid function tests are
damage to the recurrent laryngeal branch normal and there is no role for radio-
of the vagus (X) nerve, but there is no risk iodine therapy.
of injury to the facial (VII) nerve. Around
25% of patients become hypothyroid Question 14. B) Carbimazole
following surgery and require temporary Explanation: 'Differentiated' (papillary
treatment with thyroxine, but permanent and follicular) thyroid carcinomas usually
hypothyroidism is rare. Transient (10%) or present as palpable nodules for which the
permanent (rare) hypocalcaemia can occur initial investigation of choice is fine
due to damage to the parathyroid glands or needle aspiration cytology. They are
their blood supply, and careful monitoring usually treated by total thyroidectomy
of serum calcium is important during the followed by a large dose of 131iodine to
post-operative period. Exacerbation of ablate any residual thyroid tissue.
Graves' ophthalmopathy is thought to be a Subsequent long-term treatment with
complication of radio-iodine treatment thyroxine at a dose sufficient to suppress
(not surgery), and prophylactic therapy thyroid-stimulating hormone (TSH)
with corticosteroids reduces this risk. secretion is required, and recurrence can
be detected by secretion of thyroglobulin
Question 12. C) The risk of relapse is into the blood. Papillary and follicular
greater than 30% within 2 years of carcinoma have 20-year survival rates of
stopping the drug 95% and 60% respectively.
Explanation: The antithyroid drugs
carbimazole and propylthiouracil act by
inhibiting the iodination of tyrosine and
thereby reducing thyroid hormone
synthesis.
190
Question 15. D) Autoimmune gonadal Bromocriptine, on the other hand, is a
failure dopamine agonist used in the treatment of
Explanation: In hypogonadotrophic prolactinoma. Primary hypothyroidism
hypogonadism there is deficiency of leads to increased hypothalamic
follicle-stimulating hormone (FSH) and thyrotrophin-releasing hormone (TRH)
luteinising hormone (LH), the secretion by negative feedback, which in
gonadotrophins. This may be due to turn stimulates prolactin release from the
structural disease of the pituitary or pituitary.
hypothalamus (e.g. pituitary adenoma), or
to 'functional' gonadotrophin deficiency Question 18. C) Gliclazide
secondary to other influences (e.g. chronic Explanation: PCOS is associated with
illness, stress, malnutrition, excessive insulin resistance and obesity. Weight loss
exercise). Primary gonadal failure, on the and treatment with insulin-sensitising
other hand, is associated with elevated drugs such as metformin are beneficial;
gonadotrophin levels due to a loss of gliclazide is a sulphonylurea which
negative feedback (hypergonadotrophic enhances insulin secretion but not insulin
hypogonadism). sensitivity, and has no role in PCOS. The
combined oral contraceptive pill enables
Question 16. B) Women with an intact cyclical delivery of oestrogen and
uterus should receive both oestrogen progesterone, which regulates menstrual
and progesterone cycles and improves hirsutism by
Explanation: Oestrogen replacement in suppressing androgen production from the
post-menopausal women reduces ovary. Cyproterone acetate is an androgen
vasomotor symptoms (e.g. hot flushes, receptor antagonist (which also has
sweats) and prevents osteoporotic progesterone agonist effects) and is useful
fractures. There is, however, an increased in combating hirsutism in PCOS.
risk of breast and endometrial cancer,
thromboembolic disease, ischaemic heart Question 19. E) Low follicle-stimulating
disease and stroke. For these reasons, HRT hormone (FSH) and luteinising
should not be continued long-term, and hormone (LH) levels
many authorities recommend that Explanation: Turner's syndrome affects 1
treatment is only continued beyond the age in 2500 females and is usually associated
of 50 if there are unacceptable symptoms with the 45XO karyotype. Ovarian
of oestrogen deficiency. In women with an development during gestation is abnormal.
intact uterus, unopposed oestrogen therapy The ovaries are therefore unable to
is more likely to cause endometrial produce adequate oestrogen, and the loss
malignancy; combined cyclical treatment of negative feedback leads to elevated
with progesterone is therefore levels of FSH and LH. The short stature
recommended to induce regular responds in part to growth hormone
withdrawal bleeds. therapy.

Question 17. A) Bromocriptine


Explanation: High serum prolactin levels
may be physiological (e.g. stress,
pregnancy) or pathological. Prolactin
secretion from the anterior pituitary is
normally inhibited by dopamine from the
hypothalamus. Pituitary tumours can cause
hyperprolactinaemia by directly secreting
prolactin (prolactinoma) or by
compressing the infundibular stalk and
thereby interfering with the inhibitory
effect of dopamine. Anti-dopaminergic
drugs (e.g. antiemetics and antipsychotics)
can also cause high prolactin levels.
191
Question 20. C) There is an increased Question 23. B) Secondary
incidence of gynaecomastia hyperparathyroidism
Explanation: Klinefelter's syndrome Explanation: Primary
affects 1 in 1000 males and is usually hyperparathyroidism is usually due to a
associated with the 47XXY karyotype. parathyroid adenoma, but can be due to
There is dysgenesis and subsequent parathyroid hyperplasia or, in rare cases,
fibrosis of the seminiferous tubules, and parathyroid carcinoma. Elevated
affected individuals usually have small, parathyroid hormone (PTH) levels result
firm testes. Tall stature (due to androgen in hypercalcaemia. Secondary
deficiency and failure to close the hyperparathyroidism occurs as a response
epiphyseal growth plates) is common, and to hypocalcaemia (e.g. in chronic renal
growth hormone treatment is not required. failure), and calcium levels are therefore
Impaired testicular function results in normal or low. Prolonged secondary
androgen deficiency, which in turn leads hyperparathyroidism can ultimately lead to
to elevated follicle-stimulating hormone autonomous PTH hypersecretion with
(FSH) and luteinising hormone (LH) consequent hypercalcaemia (tertiary
levels due to loss of negative feedback hyperparathyroidism). Malignancy can
(hypergonadotrophic hypogonadism). The lead to elevated serum calcium due to
lack of androgens leads to gynaecomastia, bony metastases, paraneoplastic
failure of development of secondary sex phenomena (mediated by PTH-related
characteristics and infertility. peptide, e.g. in squamous cell bronchial
carcinoma) or myeloma.
Question 21. D) Alpha-fetoprotein
(AFP) Question 24. C) Hyperparathyroidism,
Explanation: Teratomas often secrete V- medullary thyroid carcinoma,
fetoprotein (AFP), and both teratomas and phaeochromocytoma
seminomas secrete _-human chorionic Explanation: MEN syndromes are rare
gonadotrophin (_-HCG); serum levels of autosomal dominant disorders
these tumour markers are potentially characterised by tumours or hyperplasia in
useful in diagnosis and prognosis. multiple glands. Differentiated thyroid
cancers (e.g. follicular carcinoma) are not
Question 22. B) It inhibits 1L- a feature. Option A describes MEN type 1,
hydroxylation of vitamin D in the and option E describes von Hippel-Lindau
kidney syndrome.
Explanation: PTH is a polypeptide
hormone secreted by the parathyroid Question 25. C) Low calcium, high
glands in response to low serum calcium phosphate, low parathyroid hormone
levels. Its actions lead to an increase in Explanation: This patient has
serum calcium. This is mediated by hypoparathyroidism resulting from
increased calcium resorption from bone damage to the parathyroid glands during
and increased renal absorption of calcium. her thyroidectomy. Since the net effect of
Vitamin D derived from sunlight and from parathyroid hormone (PTH) is to increase
the diet initially undergoes 25- serum calcium and decrease phosphate,
hydroxylation in the liver to 25-hydroxy- this results in hypocalcaemia and
vitamin D, which then undergoes 1V- hyperphosphataemia. Carpopedal spasm is
hydroxylation in the kidney to form active a manifestation of tetany and is
1,25-dihydroxy-vitamin D. PTH stimulates characteristic of hypocalcaemia. In carpal
the latter step. Activated vitamin D spasm, the metacarpophalangeal (MCP)
increases gut absorption of calcium, joints are flexed and the interphalangeal
thereby further increasing calcium levels. (IP) joints extended. Other features of low
calcium include tingling in the hands and
feet and around the mouth, stridor and
seizures. Chvostek's and Trousseau's signs
may be positive.

192
Question 26. A) Osteophyte formation The overnight dexamethasone suppression
Explanation: Hyperparathyroid bone test involves administration of 1 mg of the
disease is now rare thanks to earlier use of synthetic glucocorticoid dexamethasone at
surgical parathyroidectomy. Osteitis bedtime with measurement of plasma
fibrosa cystica results from bone cortisol as soon as practicable after waking
resorption by osteoclasts with subsequent the next morning. In healthy individuals,
fibrous tissue deposition within the endogenous adrenocorticotrophic hormone
lacunae. Bone mineral density is reduced (ACTH) secretion is inhibited and plasma
and subperiosteal erosions and terminal cortisol is therefore suppressed to < 60
resorption can be seen in the phalanges. nmol/l. Failure to suppress endogenous
Calcium pyrophosphate crystal deposition cortisol secretion is consistent with
within the articular cartilage (especially Cushing's syndrome. Alternative screening
the knees) leads to chondrocalcinosis and tests include measurement of cortisol in a
pseudogout. Osteophytes are a 24-hour urine sample or of
characteristic radiological finding in cortisol:creatinine ratio in an overnight
osteoarthritis. urine sample.

Question 27. B) Cushing's syndrome Question 30. B) Low blood pressure,


Explanation: Cushing's syndrome is low sodium, high potassium
caused by excessive glucocorticoid levels, Explanation: In Addison's disease there is
which may be exogenous or endogenous primary adrenocortical insufficiency.
in origin. Addison's disease results from Mineralocorticoid (aldosterone) deficiency
primary adrenocortical insufficiency. In leads to hypotension, hyponatraemia and
Conn's syndrome there is excess hyperkalaemia. Glucocorticoid (cortisol)
mineralocorticoid secretion by an adrenal deficiency causes hypotension and
adenoma. Phaeochromocytoma arises hyponatraemia. Patients can present with
from a tumour of catecholamine- chronic features (e.g. fatigue, weight loss,
producing enterochromaffin cells, most anorexia) or with an acute adrenal crisis
commonly from the adrenal medulla. associated with circulatory collapse. The
latter is a medical emergency requiring
Question 28. D) Chronic glucocorticoid urgent treatment with IV fluids and
therapy hydrocortisone.
Explanation: Prolonged administration of
glucocorticoids in supra-physiological Question 31. E) Spironolactone
quantities (e.g. in patients with asthma or Explanation: In Conn's syndrome there is
inflammatory bowel disease) is the most an aldosterone-secreting adrenal adenoma.
common cause of Cushing's syndrome. In Aldosterone promotes sodium and water
patients who have not received retention and increases renal potassium
glucocorticoid therapy, pituitary adenoma excretion. High mineralocorticoid levels
is the most common cause (Cushing's therefore result in hypertension and
disease). hypokalaemia. The diuretic spironolactone
is a mineralocorticoid receptor antagonist
Question 29. C) Overnight and is therefore first-line treatment.
dexamethasone suppression test Surgical resection of the diseased adrenal
Explanation: The first step is to establish gland is usually recommended.
whether or not the patient has Cushing's
syndrome; further investigations are then
required to determine the cause. Random
plasma cortisol levels are unhelpful in
view of the diurnal variation in cortisol
secretion.

193
Question 32. B) Phenoxybenzamine the portal circulation are metabolised by
Explanation: The clinical features and the liver, carcinoid syndrome usually
investigation findings are consistent with occurs only when such tumours have
phaeochromocytoma. Hypertension results spread to the liver or outside the GIT.
from excessive secretion of
catecholamines (adrenaline/epinephrine Question 35. C) Cranial diabetes
and noradrenaline/norepinephrine) from insipidus
the tumour. Initial antihypertensive Explanation: Diabetes insipidus is
treatment is with the V-blocker, characterised by the production of large
phenoxybenzamine. If the V-blockade volumes of dilute urine. It is secondary to
induces a marked reflex tachycardia or if deficient production/release of ADH by
there is persistent hypertension, then a _- the hypothalamus/posterior pituitary
blocker can be added. A _-blocker must (cranial diabetes insipidus) or to reduced
not be given before an V-blocker, as responsiveness of the renal tubules to
blockade of _2-receptors (which have a ADH (nephrogenic diabetes insipidus). In
vasodilatory action when activated) can healthy individuals, water deprivation
result in unopposed V-mediated leads to increased ADH secretion. This
vasoconstriction and precipitate a causes increased water retention by the
hypertensive crisis. kidneys with the resultant formation of
concentrated urine (> 600 mOsm/kg) and
Question 33. D) Administration of maintenance of normal plasma osmolality
exogenous insulin (280-290 mOsm/kg). In diabetes insipidus,
Explanation: Endogenous insulin is co- the kidneys are unable to retain water
secreted with C-peptide (both are formed despite water deprivation. Plasma
from the cleavage of pro-insulin from the therefore becomes concentrated (> 300
pancreas). High insulin levels associated mOsm/kg) and the urine is not maximally
with low C-peptide levels during concentrated (i.e. it is < 600 mOsm/kg). In
hypoglycaemia therefore indicate an cranial diabetes insipidus (ADH
exogenous origin of the insulin. The deficiency) the urine does concentrate
family history of diabetes in this patient after administration of the ADH analogue
suggests that she may have access to a DDAVP, but in nephrogenic diabetes
relative's insulin at home. In insulinoma insipidus (ADH resistance) there is a
and sulphonylurea ingestion, the minimal response to DDAVP.
hyperinsulinaemia is endogenous and C-
peptide levels are therefore also elevated. Question 36. A) Oral GTT.
Hypoglycaemia in alcohol excess and Explanation: This patient has
hypopituitarism is not due to increased acromegaly. It is caused by excessive
insulin secretion; insulin and C-peptide growth hormone (GH) secretion by a
levels are therefore low. pituitary adenoma. The diagnosis is
confirmed by measuring plasma GH levels
Question 34. D) 5-hydroxytryptamine during an oral glucose tolerance test; in
(5-HT) normal subjects plasma GH suppresses to
Explanation: Carcinoid syndrome is < 2 mU/l, but fails to do so in patients with
caused by ectopic production of the acromegaly. It is important to evaluate the
vasoactive hormone 5-HT by a carcinoid other pituitary hormones (including
tumour. Clinical manifestations include thyroid function tests) and to image the
flushing, wheezing and diarrhoea, as well pituitary (MRI scan); however, as in most
as cardiac valvular abnormalities. The circumstances in endocrinology, scanning
most common site of carcinoid tumours is is usually performed after the biochemical
the small bowel, but for carcinoid diagnosis has been established. Insulin
syndrome to occur, the vasoactive tolerance tests may be used to diagnose
hormones must gain access to the systemic GH deficiency (not GH excess).
circulation. Since hormones secreted into

194
Chapter 20 DIABETES MELLITUS
Question 1. D) Decreased glycogenolysis Addison's disease and thyroid disease.
Explanation: Insulin is an anabolic
hormone, and its secretion increases in Question 4. C) Ketoacidosis is a
response to a rise in blood glucose. Its common complication
actions are to reduce blood glucose levels Explanation: Diabetic ketoacidosis
and promote energy storage (therefore develops in type 1 diabetes because of
only D is true), inhibit ketogenesis, and severe insulin deficiency, but is rarely a
increase fat and protein synthesis. feature of type 2 diabetes, as sufficient
insulin secretion to suppress lipolysis is
Question 2. E) Addison's disease usually retained. Type 2 diabetes has a
Explanation: Growth hormone, much higher prevalence and is a major risk
catecholamines, corticosteroids and factor for cardiovascular disease. Unlike
glucagon are all counter-regulatory type 1 diabetes, it is often asymptomatic
hormones, as they respond to the stress of and detected incidentally. One of the
hypoglycaemia and oppose the actions of characteristics of type 2 diabetes is insulin
insulin. They may all therefore cause resistance. The combination of type 2
diabetes when present in excess, e.g. in diabetes, hypertension, hyperlipidaemia
acromegaly, phaeochromocytoma or and central obesity constitutes the
Cushing's syndrome. Addison's disease metabolic syndrome, also known as the
involves immune destruction of the insulin resistance syndrome, which
adrenals, resulting in reduced strongly predisposes to ischaemic heart
glucocorticoid production, and so blood disease.
glucose levels tend to be low. In
haemochromatosis, pancreatic iron Question 5. D) Explain to the patient
overload can impair insulin secretion, that she has diabetes mellitus
leading to diabetes. Explanation: The World Health
Organization (WHO) criteria for
Question 3. E) Concordance rates diagnosing diabetes mellitus are shown in
between monozygotic twins are greater the link below. C-peptide is the connecting
than 90% peptide which links the A and B chains of
Explanation: Type 1 diabetes is mediated insulin, and is secreted from the islet ß-
by autoimmune destruction of the insulin- cells with insulin in equimolar
producing ß-cells in the pancreas. concentrations. It can be measured in the
Treatment with insulin is therefore assessment of hypoglycaemia to establish
essential to prevent uncontrolled whether excess insulin originates from an
hyperglycaemia and ketoacidosis. The exogenous or endogenous source and,
condition is thought to be triggered by occasionally, to determine endogenous
environmental factors (e.g. viral infection) insulin secretory reserve in a patient with
in genetically susceptible individuals. diabetes; however, it is not used to make
However, concordance between the diagnosis of diabetes.
monozygotic twins is only 35% in type 1
diabetes, compared with rates approaching
100% in type 2 diabetes. Type 1 diabetes
is associated with other autoimmune
conditions, including pernicious anaemia,

195
Question 6. C) A low renal threshold for Although serum potassium levels are often
glucose in an otherwise healthy initially high, patients are usually depleted
individual in total body potassium, and intravenous
Explanation: The most common cause of infusion of potassium should be given
glycosuria is a low renal threshold for once potassium falls below 5.0 mmol/l.
glucose, which results in glucose 'spilling Sodium bicarbonate is rarely needed, may
over' into the urine. This is most often be dangerous and should be administered
found in pregnancy and in young people. only under expert supervision. Central
In some healthy individuals there is a rapid venous access may be required to assist
rise in blood glucose after meals, which with fluid management, but is not the most
can exceed the renal threshold and lead to urgent step. Antibiotic therapy would be
transient glycosuria. Pyelonephritis can required if there was evidence of overt
lead to protein, blood, nitrites and infection as the precipitating cause of the
leucocytes in urine, but not typically DKA.
glucose.
Question 9. B) Subcutaneous heparin
Question 7. D) Measuring glycated should be administered within the first
haemoglobin (HbA1C) 24 hours
Explanation: Glycated haemoglobin Explanation: Hyperosmolar non-ketotic
(HbA1C) is formed by the non-enzymatic hyperglycaemia (HONK) principally
attachment of glucose to haemoglobin. affects patients with type 2 diabetes. It is
Measurement of HbA1C provides an characterised by severe hyperglycaemia
estimate of glycaemic control over the and marked dehydration, but patients are
lifespan of red blood cells. (Red cells have not acidotic because ketone production is
a lifespan of 120 days, but the turnover of not increased. Smaller doses of insulin are
the pool in the circulation means that required than for treatment of diabetic
HbA1C reflects blood glucose over the ketoacidosis. There is a high risk of
previous 8 weeks.) The target level is thromboembolic complications in HONK
usually 7% or less, as this reduces the risk due to the severe dehydration, and low
of developing microvascular molecular weight heparin should therefore
complications. Pre-prandial blood glucose be given.
measurements are useful to patients at
home for monitoring glycaemic control Question 10. B) Kussmaul breathing
and adjusting insulin doses. Assessment of Explanation: Sweating and tachycardia
urine glucose is of limited value since it are stimulated by sympathetic activation in
varies according to the renal threshold. hypoglycaemia. Confusion and altered
consciousness result from
Question 8. A) Administration of neuroglycopenia. Kussmaul breathing (air
intravenous insulin and intravenous hunger) is a feature of diabetic
saline ketoacidosis as a response to the metabolic
Explanation: In diabetic ketoacidosis acidosis.
(DKA), severe insulin deficiency leads to
hyperglycaemia and the generation of
ketone bodies. The hyperglycaemia results
in glycosuria and an osmotic diuresis,
through which patients can become
severely dehydrated. The initial priorities
in managing DKA are therefore
administration of intravenous saline and
insulin.

196
Question 11. D) Oral carbohydrate Question 13. C) Insulin alone
Explanation: Although the accuracy of Explanation: The development of
blood glucose strip testing kits is variable, diabetes during pregnancy is associated
this patient has clinical features and a with increased maternal and fetal
glucose reading consistent with morbidity and mortality. Hyperglycaemia
hypoglycaemia, so prompt treatment is can result in teratogenic effects and
required. If a patient is conscious and able macrosomia. Strict blood glucose control
to swallow, then oral carbohydrate is the with insulin is therefore important. Oral
treatment of choice. If not, then IV anti-diabetic agents may be harmful to the
dextrose or IM glucagon should be fetus and should not be given during
administered. Laboratory samples should pregnancy.
be obtained (for glucose, insulin, C-
peptide, toxicology drug screen) before Question 14. D) Start an intravenous
treatment in patients with unexplained infusion of dextrose and insulin in place
suspected 'spontaneous' hypoglycaemia, as of the patient's usual therapy
this avoids potentially complex attempts to Explanation: Adequate control of
reproduce the hypoglycaemia later. hyperglycaemia is important in the peri-
However, hypoglycaemia in diabetic operative period. For type 2 diabetic
patients who are treated with insulin or patients undergoing minor operations, it is
with a sulphonylurea is probably related to reasonable to place them first on the
their treatment. operating list and to omit their morning
oral anti-diabetic agent or insulin until
Question 12. E) Stop metformin and after the operation. However, for type 2
commence intravenous insulin diabetic patients undergoing major
Explanation: Studies suggest that strict surgery, and for all type 1 diabetic
glycaemic control in the peri-infarct period patients, blood glucose should be
improves long-term prognosis after an MI. controlled by means of an intravenous
This is usually achieved with insulin, often insulin infusion.
in the form of an intravenous infusion
(sliding scale). The study most often cited Question 15. B) Metformin
is the DIGAMI study (diabetes mellitus, Explanation: Sulphonylureas (e.g.
insulin glucose infusion in acute gliclazide) act by stimulating endogenous
myocardial infarction), which showed a insulin secretion. A major side-effect of
reduction in mortality of 25% in patients this class of drug is weight gain, and so
treated with an insulin infusion, compared they would not generally be considered
with standard treatment over a follow-up first-line treatment in obese patients.
period of 3.5 years (though this has not Metformin (a biguanide) works by
been confirmed in subsequent studies). increasing insulin sensitivity and does not
Oral anti-diabetic agents are usually cause weight gain, so is a better choice in
discontinued until the recovery period this patient. Thiazolidinediones (e.g.
after MI. Metformin in particular should rosiglitazone, pioglitazone) also act by
be withheld because of the small risk of improving insulin sensitivity, but are not
lactic acidosis in the event of impaired usually prescribed as first-line treatment.
systemic perfusion. Insulin treatment is sometimes required in
type 2 diabetes, but tends to be initiated
when glycaemic control with oral agents is
suboptimal.

197
Question 16. C) They are particularly Question 19. C) Flame haemorrhages
useful in diabetic patients with left Explanation: Flame haemorrhages are a
ventricular dysfunction feature of hypertensive retinopathy but are
Explanation: The thiazolidinedione class not usually a feature of diabetic
of drugs, also known as glitazones or retinopathy. Hard exudates (due to leakage
TZDs, includes rosiglitazone and from abnormal capillaries), blot and dot
pioglitazone. They act by binding to a haemorrhages, and cotton wool spots
nuclear receptor called peroxisome (caused by areas of retinal ischaemia) are
proliferator-activated receptor-W (PPAR-W), found in both diabetic and hypertensive
particularly in fat cells. This results in retinopathy. Microaneuryms and venous
altered expression of genes involved in beading occur in diabetic retinopathy.
metabolism and thereby enhances insulin
sensitivity. Insulin secretion itself is not Question 20. D) Venous beading with
affected. They are useful in obese patients cotton wool spots
with type 2 diabetes, in whom insulin Explanation: Microaneurysms, dot
resistance is often severe. Although they haemorrhages and hard exudates are
do cause weight gain, this is usually in features of background diabetic
subcutaneous rather than intra-abdominal retinopathy. The finding of hard exudates
adipose tissue and is therefore thought to near the macula indicates diabetic
be metabolically 'benign'. They have few maculopathy, which is sight-threatening.
side-effects, but can cause salt and water Venous beading, cotton wool spots and
retention, and are therefore contraindicated intra-retinal microvascular abnormalities
in heart failure. (IRMAs) occur in pre-proliferative
disease. New vessel formation
Question 17. D) (neovascularisation) is the key feature of
Glipizide N hypoglycaemia proliferative diabetic retinopathy. Rupture
Explanation: Sulphonylureas (e.g. of these vessels can result in pre-retinal
gliclazide, glipizide) stimulate insulin haemorrhage. Papilloedema is a feature of
secretion and hence cause weight gain and grade IV hypertensive retinopathy but may
can induce hypoglycaemia. Metformin also be seen in other conditions, including
increases insulin sensitivity and does not raised intracranial pressure. The
cause hypoglycaemia, but frequently classification of retinopathy is detailed in
causes nausea and diarrhoea. It is also the link below.
contraindicated in patients with renal
impairment, liver dysfunction, Question 21. C) Regular follow-up in
symptomatic heart failure and sepsis the diabetic clinic but no referral to
because of the risk of lactic acidosis. ophthalmology
Explanation: Background retinopathy
Question 18. A) Ischaemic heart disease requires regular fundoscopy to identify
Explanation: Macrovascular disease is progression but is not in itself an
the major cause of mortality in diabetes, indication for ophthalmology referral.
particularly myocardial infarction and Patients with pre-proliferative and
stroke. Macrovascular disease also causes proliferative retinopathy should be
significant morbidity in the form of referred urgently to ophthalmology for
coronary heart disease, cerebrovascular further assessment and consideration of
disease and peripheral vascular disease. retinal laser photocoagulation. Progression
Microvascular complications include of retinopathy is related to poor glycaemic
diabetic retinopathy, nephropathy and control and elevated blood pressure, and
neuropathy. Their incidence is related to so anti-diabetic agents and
poor glycaemic control and causes serious antihypertensives should not be stopped.
morbidity but is not the major cause of
death.

198
Question 22. C) Ramipril stocking' distribution of sensory loss
Explanation: Diabetic nephropathy is a distally. Motor involvement can cause
major cause of end-stage renal failure. Its progressive proximal muscle weakness,
development is related to poor glycaemic commonly accompanied by severe pain.
control. The presence of microalbuminuria Motor and sensory function of individual
heralds the development of overt cranial and peripheral nerves can also be
nephropathy. The risk of progression can affected (mononeuropathy and
be reduced by good glycaemic control and mononeuritis multiplex). Peripheral
aggressive treatment of elevated blood sensory neuropathy can ultimately result in
pressure. Angiotensin-converting enzyme grossly deforming neuroarthropathy of the
(ACE) inhibitors provide greater benefit feet (Charcot joint) due to loss of pain
from equal blood pressure lowering than sensation. Although neuropathy can give
other antihypertensive agents. They reduce rise to foot ulcers, these are typically
intra-glomerular pressure and urinary painless (unlike ischaemic ulcers).
protein excretion and are therefore the
preferred agents in patients with Question 24. D) Gastroparesis
microalbuminuria. Explanation: Autonomic neuropathy can
affect sympathetic or parasympathetic
Question 23. E) It is typically associated nerves. Gastroparesis in patients being
with painful foot ulcers treated with insulin can predispose to poor
Explanation: Like all diabetic glycaemic control and hypoglycaemia,
microvascular complications, the because of unpredictable matching
development of neuropathy can be between carbohydrate absorption and
prevented or delayed by good glycaemic plasma insulin concentrations. Fast-acting
control. Sensory, motor and autonomic insulin analogues should be avoided.
nerves can all be affected, but patients are Agents that promote gastric emptying,
often asymptomatic, particularly in the such as metoclopramide, may be helpful.
early stages. Sensory neuropathy
classically gives rises to a 'glove and

199
Chapter 21 ALIMENTARY TRACT AND
PANCREATIC DISEASE

Question 1. E) Rounded mass Question 7. B) Helicobacter pylori


Explanation: The other findings are all infection
characteristic of an enlarged spleen. Explanation: Urea breath testing is, in
most cases, the best test for detecting H.
Question 2. A) Trypsin pylori infection as it is accurate, simple
Explanation: Amylase is involved in and non-invasive. Bacterial overgrowth
carbohydrate digestion. Lipase cleaves and lactose intolerance may be detected by
long chain triglycerides, yielding fatty the glucose hydrogen breath test and
acids and monoglycerides. lactose hydrogen breath tests respectively.
Cholecystokinin is a hormone with Fat malabsorption can be diagnosed by
important effects on gut motility. Pepsin is either 3-day faecal fat assessment or the
14
active in the stomach, not small bowel, C-triolein breath test. Tests for coeliac
where it plays a modest role in protein disease include coeliac antibodies and
digestion. small bowel (usually duodenal) biopsy.

Question 3. C) Glycogen Question 8. E) Achalasia


Explanation: More on Carbohydrate Explanation: Upper gastrointestinal
digestion and absorption endoscopy is, in general, the investigation
Question 4. D) Increased gastric acid of choice in dysphagia to exclude
secretion structural causes and allow biopsy with or
Explanation: In addition to the other without dilatation of any suspicious
factors listed above, CCK suppresses strictures. However, oesophageal
gastric acid secretion. manometry is useful in achalasia where
the characteristic pattern of high-pressure,
Question 5. D) Oesophageal carcinoma non-relaxing lower oesophageal sphincter
Explanation: The barium swallow shows with poorly contractile body of
a long irregular stricture, typical of an oesophagus confirms the diagnosis.
oesophageal carcinoma. None of the other
options would produce this appearance. Question 9. E) Associated difficulty in
swallowing
Question 6. A) Faecal elastase Explanation: Dyspepsia is extremely
Explanation: Elastase is a proteolytic prevalent and usually does not require
enzyme produced by the pancreas. Levels investigation, especially in younger
are reduced in pancreatic exocrine failure, patients. In general, upper gastrointestinal
though it has low sensitivity for detecting endoscopy should be reserved for patients
mild disease.Abdominal CT is useful for over 55 years old, younger patients
assessing pancreatic size and structure, but unresponsive to empirical treatment or
not function. Three-day faecal fat is a test those with 'alarm' features, such as
for fat malabsorption. Fasting blood dysphagia, which may point to serious
glucose is a test of pancreatic endocrine underlying pathology.
function. The lactose hydrogen breath test
is used in the detection of lactose
intolerance.

200
Question 10. C) Atenolol Orlistat is a lipase inhibitor that reduces
Explanation: Many antibiotics, analgesics absorption of dietary fat and is used in the
and cytotoxins may lead to vomiting. Side- management of obesity. Side-effects
effects of atenolol include tiredness, include liquid or oily stools and faecal
impotence and nightmares, but not urgency rather than constipation.
vomiting.
Question 16. E) Psoriasis
Question 11. B) Mallory-Weiss tear as Explanation: See list of Causes of oral
the cause of bleeding ulceration
Explanation: Mortality risk in patients
presenting with acute upper Question17. C) Weight loss
gastrointestinal bleed is defined by the Explanation: Weight gain may precipitate
'Rockall risk score' which takes into or aggravate symptoms by increasing
account age, features of shock, intra-abdominal pressure, whereas weight
comorbidity, diagnosis and endoscopic loss may improve symptoms.
findings. Mallory-Weiss tear is the
diagnosis associated with the most Question18. D) Patients with the
favourable outcome. condition should undergo regular
endoscopic surveillance with biopsies
Question12. E) Anal fissure Explanation: Barrett's oesophagus refers
Explanation: Anal fissure does not cause to metaplastic rather than neoplastic
severe acute lower gastrointestinal change. Although a major risk factor for
bleeding but rather small amounts of fresh oesophageal adenocarcinoma, the vast
rectal bleeding associated with pain during majority of patients do not go on to
defaecation. develop this condition. Neither potent acid
suppression nor antireflux surgery has
Question 13. C) Blood and mucus in been shown to induce regression or even
stool stop progression of Barrett's oesophagus.
Explanation: A colonic aetiology is Regular endoscopic surveillance is
suggested by cramping lower abdominal recommended.
pain, blood and mucus in stool, and high-
frequency, low-volume stool, often with Question 19. D) Recurrence of
tenesmus. symptoms following cessation of
medical treatment is common
Question 14. D) Peripheral Explanation: The patient is young, is
neuropathy Vitamin C otherwise well, and has a typical history of
Explanation: Vitamin C deficiency GORD and no worrying features. He can
produces bleeding gums, poor wound therefore be treated empirically without
healing and perifollicular or petechial need for further investigation. A history of
bruising (scurvy). Peripheral neuropathy weight gain prior to onset of symptoms is
may be due to vitamin B12 deficiency. common in GORD and would lend
support to the diagnosis. PPIs are superior
Question 15. E) Verapamil to H2-receptor antagonists in both
Explanation: Constipation is a side-effect relieving symptoms and healing
of various calcium channel blockers but oesophagitis (see the evidence-based
particularly verapamil. Diarrhoea is a medicine box in the link below).
common side-effect of macrolide Recurrence of symptoms after stopping
antibiotics, especially erythromycin. acid-suppressing therapy is common and
Likewise, metformin is far more likely to some patients require long-term low-dose
cause diarrhoea than constipation. Side- maintenance therapy. Antireflux surgery is
effects of digoxin include nausea, highly effective in alleviating symptoms.
vomiting, arrhythmia and disturbed vision,
but not constipation.

201
Question 20. B) Achalasia The pain of chronic pancreatitis and
Explanation: The barium swallow shows mesenteric ischaemia tends to be
the characteristic appearance of a dilated precipitated rather than relieved by eating,
barium-filled oesophagus with fluid level as does the pain of biliary colic,
and distal tapering and a closed lower particularly with fatty foods. It would be
oesophageal sphincter. Clues in the history unusual for Crohn's disease to produce
include the long duration of symptoms, the pain well localised to the epigastrium.
easing of swallowing difficulties by
drinking liquids, and the intermittent Question 23. D) The patient should be
episodes of severe chest pain which are given specific advice on dietary
due to oesophageal spasm. Diffuse restriction
oesophageal spasm produces transient Explanation: Most peptic ulcers are now
rather than slowly progressive dysphagia. cured by H. pylori eradication and acid-
Myasthenia gravis, as with other suppressing therapy with the result that
neuromuscular causes of dysphagia, tends elective surgery for peptic ulcer disease
to be worse for liquids rather than solids. has become a rare event. No special
In systemic sclerosis there is usually a dietary advice is required, although
history of severe heartburn. An endoscopy patients may themselves wish to avoid
may still be necessary to exclude certain foods they feel aggravate
oesophageal carcinoma. symptoms. If failure to eradicate H. pylori
infection occurs with standard triple
Question 21. D) The presence of therapy, the patient should first be asked
hoarseness suggests mediastinal about compliance with treatment and then
invasion offered second-line therapy.
Explanation: The overall 5-year survival
rate is very poor at 6-9% and even after Question 24. C) It exclusively colonises
'potentially curative' surgery remains only gastric-type epithelium
30%. Endoscopic ultrasound is the most Explanation: H. pylori is a Gram-
sensitive modality for staging, as CT tends negative spiral-shaped rod. The enzyme
to understage tumours. Dysphagia is 'urease' produces ammonia from urea,
classically persistent and progressive and thereby raising the surrounding pH and
is initially worse for solids. Pain is often making the environment less hostile for
absent due to destruction of mucosal the bacterium. It is true that H. pylori only
innervation by the tumour. Endoscopically colonises gastric-type epithelium and is,
directed tumour ablation using laser therefore, only found in the duodenum in
therapy or stent insertion is the major association with patches of gastric
method of improving swallowing. metaplasia. CagA is one of several well-
Mediastinal invasion with involvement of recognised virulence factors (see Fig.
the recurrent laryngeal nerve may cause 22.32 in the link below). Colonisation is
hoarseness. associated with depletion of somatostatin
(from D cells) but stimulates release of
Question 22. A) Chronic peptic ulcer gastrin (from G cells).
Explanation: Of the possibilities above,
chronic peptic ulcer fits best with the
pattern of symptoms. The three
characteristics of ulcer pain are the
recurrent, episodic nature, a relationship to
food and localisation to the epigastrium.
Occasional vomiting occurs in about 40%.
It is worth noting, though, that the history
is actually a poor predictor of the presence
of an ulcer.

202
Question 25. C) Around 40% of patients Question 28. B) Zollinger-Ellison
with duodenal ulcer are infected with H. syndrome
pylori Explanation: A reduction in the acidity of
Explanation: H. pylori infection may be the stomach (hypochlorhydria or
responsible for 60-70% of cases of gastric achlorhydria) appears to increase gastric
carcinoma. Around 90% of duodenal ulcer cancer risk, perhaps by permitting
patients and 70% of gastric ulcer patients colonisation of the stomach with nitrite-
are infected. The development of a low- reducing bacteria which form carcinogenic
grade lymphoma known as a 'MALToma' N-nitroso-compounds from nitrates.
is closely associated with H. pylori Autoimmune chronic gastritis and H.
infection, and superficial MALTomas may pylori infection may both lead to
be cured by H. pylori eradication. Despite achlorhydria, unlike Zollinger-Ellison
all of the above, only a minority of H. syndrome, which markedly increases
pylori-infected patients develop clinical gastric acidity. (Although H. pylori
disease. infection normally results in normal or
increased acid secretion, a proportion of
Question 26. A) Weight gain infected individuals become
Explanation: Weight loss rather than hypochlorhydric or achlorhydric.)
weight gain is common. The usual cause is
reduced dietary intake due to early satiety Question 29. C) Conn's syndrome
from a small gastric remnant. Both Explanation: See list of Disease
diarrhoea and anaemia may be associations of coeliac disease
multifactorial. Rapid gastric emptying of
food of high osmotic potential may Question 30. E) Mesenteric ischaemia
produce large fluid shifts into the small Explanation: The most common cause of
intestine, provoking autonomic symptoms. subtotal villous atrophy in the UK is
This is known as 'dumping syndrome'. coeliac disease but there are a number of
other important causes.
Question 27. C) Zollinger-Ellison
syndrome Question 31. E) There is an increased
Explanation: In Zollinger-Ellison risk of T-cell lymphoma
syndrome there is severe peptic ulceration Explanation: Patients are intolerant of
from gastric acid hypersecretion due to a wheat, rye, barley and, to a lesser extent,
gastrin-producing non-ß-cell islet tumour oats, but not rice, maize or potatoes. Peak
of the pancreas ('gastrinoma'). Ulcers are onset is in infancy and early childhood; in
often multiple and refractory to standard adults the highest incidence occurs in the
therapy, and may occur in unusual places 40-59 age group. Around 50% of patients
such as oesophagus or duodenum. It are asymptomatic and many remain
comprises part of the MEN type 1 undiagnosed. Metabolic bone disease is
syndrome. Inactivation of pancreatic less common in patients who stick strictly
enzymes and bile salts by increased acidity to a gluten-free diet. There is an increased
in the small intestine can result in risk of T-cell lymphoma as well as small
diarrhoea (seen in more than one-third of bowel carcinoma and squamous carcinoma
patients) and steatorrhoea. High-dose of the oesophagus, although the absolute
proton pump inhibitor therapy heals risk of developing these malignancies is
ulcers, alleviates diarrhoea and has largely very low.
removed the need for total gastrectomy.

203
Question 32. E) Serum amylase Question 36. D) Recurrent
Explanation: The above account is hypoglycaemia
classical of acute pancreatitis. Pain Explanation: Hyperglycaemia, due to the
radiates to the back in 65% of cases; development of diabetes mellitus, rather
guarding and rebound tenderness are than hypoglycaemia is a common feature
typically absent in the early stages as the of chronic pancreatitis.
inflammation is retroperitoneal. A
markedly elevated serum amylase often Question 37. D) Carcinoma of pancreas
confirms the diagnosis, although in Explanation: This is a fairly typical
difficult cases, ultrasound or CT can help presentation of pancreatic carcinoma
by demonstrating evidence of pancreatic (apart from the vanishingly rare sign of
swelling. Abdominal and erect chest X- palpable gallbladder). The diagnosis will
rays may exclude other conditions such as be confirmed with abdominal ultrasound
intestinal obstruction or perforated viscus or CT. According to 'Courvoisier's law', a
and an ECG should be performed to palpable gallbladder in a jaundiced patient
exclude atypical presentation of is unlikely to be due to gallstones, and is
myocardial infarction. C-reactive protein usually the result of distal biliary
is not helpful diagnostically but is a useful obstruction by a carcinoma of the head of
indicator of severity and prognosis. pancreas.

Question 33. C) Hypocalcaemia Question 38. D) The liver is a common


Explanation: Hypercalcaemia not site for metastatic deposits
hypocalcaemia is a rare cause of acute Explanation: Pancreatic carcinoma has a
pancreatitis. By far the two most common dismal prognosis; only 15% of tumours
aetiologies are alcohol and gallstones. are amenable to curative resection and,
even in patients who undergo complete
Question 34. D) Amylase 1000 resection, 5-year survival is around 20%.
U/l (normal range < 100 U/l) Around two-thirds of tumours arise from
Explanation: Although an elevated serum the head of the pancreas; these frequently
amylase is useful in diagnosing acute involve the common bile duct, resulting in
pancreatitis, it has no prognostic value. obstructive jaundice. Coeliac plexus
The other parameters listed are included in neurolysis may be useful in relieving pain,
the Glasgow criteria of adverse prognostic which often proves extremely difficult to
factors. control with other measures. Ampullary
carcinomas behave less aggressively than
Question 35. B) Enteral feeding should pancreatic carcinomas and have a more
be started at an early stage in patients favourable outlook.
with severe pancreatitis
Explanation: Hypocalcaemia need only
be corrected if tetany occurs.
Thromboprophylaxis is advised. The
greatest benefit from emergency ERCP
occurs in patients who have ascending
cholangitis (see EBM Box 22.58 in the
link below). Opiate analgesia is required in
nearly all cases. Nutritional support,
preferably by the enteral route, forms an
important part of the management of acute
pancreatitis.

204
Question39. E) Inflammation limited to Question 43. B) The patient should be
the mucosa treated with intravenous corticosteroids
Explanation: There are several important Explanation: Intravenous corticosteroids
pathological differences between are given as a constant infusion in the
ulcerative colitis (UC) and Crohn's disease treatment of severe ulcerative colitis.
(CD), including but not confined to the
following: Question 44. C) The patient should be
• UC only involves the colon, while referred for urgent colectomy
Crohn's disease can involve any part of the Explanation: The X-ray shows a grossly
gastrointestinal tract from mouth to anus. dilated colon due to severe ulcerative
• In UC inflammation is confluent and colitis. Toxic megacolon occurs most
more severe distally (thus it invariably commonly during the first attack of colitis
involves the rectum but may spread and is associated with a high risk of
proximally to involve the remainder of the perforation. It is, therefore, an indication
colon). In CD changes are patchy with for colectomy.
areas of inflammation interrupted by
islands of normal mucosa. (This may Question 45. E) Diarrhoea in ulcerative
produce the characteristic 'skip lesion'.) colitis may be due to enteroenteric
• In UC the inflammatory process is fistulae
superficial, confined to the mucosa and Explanation: Fistulous connections
sparing deeper layers of the bowel wall, between loops of affected bowel
while in CD inflammation is deep and (enteroenteric), or between bowel and
seen through all layers of the bowel wall. bladder (enterovesical) or vagina
(Deep ulcers may penetrate through the (enterovaginal) are specific complications
bowel wall to initiate fistulae.) of Crohn's disease and do not occur in
ulcerative colitis. Patients with extensive
Question 40. B) Cigarette smoking active colitis of more than 8 years'
Explanation: Ulcerative colitis is more duration are at increased risk of colon
common in non-smokers and, in fact, cancer and should, therefore, have regular
cessation of smoking can provoke a colonoscopic surveillance.
relapse of symptoms. Emotional stress,
intercurrent illness, non-steroidal anti- Question 46. B) Primary sclerosing
inflammatory drugs (NSAIDs) and cholangitis
antibiotics are all well-recognised factors Explanation: Primary sclerosing
which may precipitate flares. cholangitis and cholangiocarcinoma are
both more common in ulcerative colitis
Question 41. D) Serum albumin but appear to be unrelated to intestinal
Explanation: Serum albumin is a marker disease activity. The other complications
of disease severity in ulcerative colitis but listed tend to be more of a problem during
does not contribute to the diagnosis. It is active bowel disease.
suppressed in any condition producing an
acute phase response.

Question 42. B) Stool volume 200 g/24


hrs
Explanation: Stool volume > 400 g/24 hrs
suggests severe disease. These parameters
are utilised in clinical practice.

205
Question 47. C) Corticosteroids are Question 50. B) Acute small bowel
preferred to methotrexate for ischaemia
maintaining remission Explanation: The rapidity of onset,
Explanation: Chronic corticosteroid history of vascular disease and atrial
treatment is avoided due to the high fibrillation (not on warfarin), paucity of
incidence of complications with long-term abdominal findings despite severe
use and poor efficacy in preventing symptoms and metabolic acidosis all point
relapse. All of the other statements are true to acute small bowel ischaemia. Given her
and are useful learning points. history of atrial fibrillation, a likely
Azathioprine and methotrexate are both aetiology would be a thromboembolism
used in maintenance of remission. from the heart.
Infliximab is highly effective in inducing
remission in active Crohn's but relapse Question 51. E) Stool analysis for
commonly occurs around 12 weeks after Clostridium difficile toxin
treatment. It should therefore be given in Explanation: Cl. difficile is the most
combination with either azathioprine or likely cause of this patient's diarrhoea. It is
methotrexate. Surgery is not curative in a very common cause of diarrhoea in
Crohn's disease and repeated surgery may hospital inpatients, particularly those > 65
lead to the development of short bowel years with comorbid disease who have had
syndrome. Cigarette smoking is more recent broad-spectrum antibiotic therapy.
important than any drug treatment in Legionella pneumonia can be associated
maintaining remission. with gastrointestinal upset but the timing
certainly favours Cl. difficile infection in
Question 48. C) Symptoms disturbing this case.
sleep
Explanation: Nocturnal symptoms are Question 52. D) Polyps with a villous
uncommon in IBS and pain that wakens a architecture carry a higher risk of
patient from sleep is often a useful pointer malignant change than those with
to organic disease. Other features that tubular architecture
would raise doubt as to the diagnosis Explanation: Nearly all forms of
include rectal bleeding and weight loss. colorectal carcinoma develop from
adenomatous polyps, usually over a period
Question 49. D) Reassurance alone may of 5-10 years, although certain features
lead to resolution of symptoms such as increased size and villous
Explanation: Anxiety over the possibility architecture increase the risk for malignant
of serious organic disease, particularly change. Polyps are usually asymptomatic
cancer, is often a major contributing factor and are often found incidentally during
to symptoms in IBS and many patients colonoscopy; when discovered, they
respond to simple explanation and should be removed whenever possible to
reassurance. Such patients have a far better reduce the risk of developing colorectal
long-term outlook than those who fail to cancer. Peutz-Jeghers syndrome is
respond. characterised by the presence of non-
neoplastic hamartomatous polyps in the
colon and small bowel (although it is
associated with increased risk of certain
malignancies, e.g. breast, ovary, colon).

206
Question 53. C) High dietary calcium urgency by colonoscopy. Colonoscopy is
Explanation: Dietary calcium decreases preferable to barium enema in most
the risk of developing colorectal cancer as instances as it has better sensitivity and
it binds and precipitates faecal bile acids. specificity and allows biopsy of lesions
and polypectomy. If the diagnosis is
Question 54.D) Colonoscopy confirmed then a CT scan may be useful
Explanation: The combination of occult for detecting hepatic metastases. CEA is
gastrointestinal bleeding, weight loss and a neither sensitive nor specific for
palpable right iliac fossa mass in a patient diagnosing colorectal cancer, but in those
of this age group should lead to strong patients with a raised CEA who go on to
suspicion of colonic (caecal) carcinoma have resection of the tumour, it may help
and should be investigated as a matter of in follow-up to detect early recurrences

207
Chapter 22 LIVER AND BILIARY TRACT
DISEASE

Question 1. B) Production of vitamin K in the previous question, a predominant


Explanation: Vitamins are, by definition, rise in ALT suggests a hepatocellular
not synthesised in the human body so an cause. Spider naevi are a pointer towards
adequate dietary intake and chronic liver disease.
gastrointestinal absorption of vitamin K is
essential. Vitamin K is a fat-soluble Question 6. D) Factor IX
vitamin and absorption is impaired in Explanation: The vitamin K-dependent
obstructive jaundice, resulting in a clotting factors are II, VII, IX and X. No-
prolonged prothrombin time which is one has ever devised a decent mnemonic
corrected by vitamin K administration. for this fact, so you just have to remember
it. Here they are again: 2, 7, 9 and 10.
Question 2. E) It is the primary site of (1972)
lipolysis in the liver
Explanation: Below is a quick reminder Question 7. E) Prothrombin time (PT)
of the structure of the hepatic acinus. Explanation: Albumin, fibrinogen and
most coagulation factors are synthesised in
Question 3. C) Phenytoin the liver so all of the above tests are, to
Explanation: Phenytoin is a microsomal some extent, dependent on hepatic
enzyme-inducing drug; all of this type of synthetic function, with the exception of
drug may cause an isolated increase in the urea which is a by-product of protein
microsomal enzyme GGT. This is a metabolism. The prothrombin time
reflection of hepatic enzyme induction and changes most quickly in response to an
does not necessitate any change to therapy. acute liver insult, because of the short
A list of enzyme-inducing drugs can be half-life of the clotting factors on which it
found in the link below. It is important to depends. Albumin has a much longer half-
learn them as they have interactions with life and hypoalbuminaemia is a feature of
other drugs, such as reducing the chronic liver disease.
effectiveness of the oral contraceptive pill
and increasing the toxicity of paracetamol Question 8. A) Chronic obstructive
in overdose. pulmonary disease (COPD) with forced
expiratory volume in 1 second
Question 4. A) Viral hepatitis (FEV1)/forced vital capacity (FVC) of
Explanation: This is the classic picture of 37%
hepatocellular jaundice. Explanation: The conditions required for
a safe liver biopsy are included in the link
Question 5. E) Raised reticulocyte count below. There is a risk of causing
Explanation: Part of the bone marrow pneumothorax if the pleura is punctured
response to excessive red cell destruction and, in a patient with severe COPD, this is
is premature release of reticulocytes, not only more likely but also potentially
resulting in a reticulocytosis. Nucleated catastrophic.
red cell precursors may also appear in the
blood. Bilirubinuria and pale stools both
suggest cholestatic jaundice. As discussed

208
Question 9. D) Abdominal ultrasound Question 12. E) Ascitic fluid white cell
Explanation: The LFTs show a count
cholestatic picture. The most useful initial Explanation: This is a classic presentation
investigation is abdominal ultrasound, of spontaneous bacterial peritonitis, a
particularly to detect biliary obstruction. common and life-threatening complication
Should more detailed imaging of the of cirrhosis.
biliary tree prove necessary, then either
ERCP or magnetic resonance Question 13. B) Spironolactone
cholangiopancreatography (MRCP) may Explanation: Aldosterone excess, caused
be helpful. MRCP (followed if necessary by impaired hepatic steroid metabolism, is
by therapeutic ERCP) is the diagnostic test a key factor in the pathogenesis of ascites
of choice, as it carries a lower risk of in chronic liver disease (explained in the
complications than ERCP. A very useful link below). Spironolactone is an
algorithm for the investigation of patients aldosterone antagonist, which makes it the
with jaundice is provided in the link best choice to prevent sodium retention in
below. cirrhotic patients. Furosemide (a loop
diuretic) is sometimes added to
Question 10. B) Primary biliary spironolactone to further promote sodium
cirrhosis (PBC) Antimitochondrial excretion in the urine.
antibody (AMA) Bendroflumethiazide (a thiazide diuretic)
Explanation: This is difficult to is not often used to treat ascites because
remember, but genuinely useful because better drugs are available. Vasopressin is a
AMA has high sensitivity for PBC, being potent vasoconstrictor and powerfully
present in over 95% of cases. promotes water retention. It is used in the
management of hepatorenal syndrome.
Question 11. B) Cholestatic jaundice Digoxin is not useful in the management
Explanation: Cholestatic jaundice is the of ascites.
only option here that explains both yellow
sclerae and dark urine. Conjugated Question 14. A) Prothrombin time (PT)
bilirubin from the liver enters the blood > 100 secs
because of a failure of bile flow. As Explanation: PT is a better predictor of
conjugated bilirubin (unlike unconjugated poor outcome than liver function tests, and
bilirubin) is water-soluble, it is filtered by is used in standard criteria for determining
the kidneys and appears in the urine. More the need for liver transplant. The following
information on cholestatic jaundice, two prognostic factors (see the link below)
including causes and clinical features, is have been shown to confer a > 90%
provided in the link below. Carotenaemia, mortality in acute liver failure secondary
caused by eating too much carotene, is to paracetamol overdose:
almost always benign and causes • H+ > 50 nmol/l (pH < 7.3)
yellowing of the skin but not the sclera. • Creatinine > 300 µmol/l with PT > 100
Gilbert's syndrome describes a congenital secs and grade 3 or 4 encephalopathy.
impairment of glucuronyl transferase, that
leads to reduced hepatic uptake of Question 15. C) May be balloted
bilirubin from the blood. This therefore Explanation: Distinguishing a grossly
causes unconjugated hyperbilirubinaemia, enlarged kidney (for example, in
so the sclerae are yellow but the urine is polycystic kidney disease) from
not dark. In contrast, rhabdomyolysis splenomegaly can be tricky. The correct
causes large amounts of myoglobin to technique for palpation of the liver and
enter the urine, which can make the urine spleen is shown in the link below.
appear black but does not cause jaundice.
Dehydration results in highly concentrated
urine but does not cause jaundice.

209
Question 20. E) It may cause cirrhosis
Question 16. C) Pruritus is a more Explanation: Hepatitis B is commonly
common presenting feature than transmitted through blood or sexual
jaundice contact. The most common route of
Explanation: Primary biliary cirrhosis is transmission world-wide is vertical during
more common in women by a factor of 9. childbirth; this route of infection carries a
Pruritus is the most common presenting much higher risk of chronic infection (see
complaint and may precede jaundice by the link below), which may ultimately lead
years. Positive antinuclear antibodies are to cirrhosis and hepatocellular carcinoma.
found in only 15% of cases, whereas A vaccine is given to all health-care staff
antimitochondrial antibodies are of in the UK.
considerable diagnostic value, present in
over 95% of patients. Immunosuppressive Question 21. E) Anti-HBc IgG and
drugs are not effective, and treatment HBsAb
mainly palliates symptoms until transplant Explanation: This is difficult but
becomes necessary. Although 30% of important to understand. Both the natural
patients will exhibit disease recurrence in history of infection and the serological
a transplanted liver, this takes years to tests are explained in the link below. A
develop and is not a contraindication to few things are particularly important to
transplant. commit to memory: a vaccinated patient
will only have antibody to the surface
Question 17. D) Terlipressin protein (content of vaccine) but a patient
Explanation: Terlipressin lowers the who is immune (previously exposed and
portal pressure and is the drug of choice in cleared) will have antibodies to the whole
this scenario. The somatostatin analogue, virus, i.e. core and surface antibodies.
octreotide, is also useful. Noradrenaline Note that HbsAg is present in chronic
(norepinephrine) and dopamine infection but, likewise, appears in acute
predominantly cause peripheral infection; acute infection can be
vasoconstriction. However, in a patient differentiated from chronic infection by
dying from haemorrhage, fluid the presence of anti-HBc IgM.
replacement, rather than vasoconstriction,
is the best method to correct hypotension. Question 22. C) It effectively rules out
Activated protein C is contraindicated in hepatitis B as the cause of his illness
haemorrhage. Explanation: Anti-HBs is found in the
blood of patients who have been
Question 18. D) Hepatitis D successfully immunised against hepatitis
Explanation: Hepatitis D is an incomplete B. As a nurse, this patient will have been
virus and cannot cause infection without immunised before taking up employment.
concurrent infection with hepatitis B. If you're still having trouble with
interpretation of these serological tests,
Question 19. B) It may be spread by the have another look at the link below
faecal-oral route
Explanation: Hepatitis A is not usually Question 23. E) Treatment with
spread through blood or conventional interferon is ineffective in chronic
sexual contact, and, unlike hepatitis B and infection
C, does not induce a chronic carrier state. Explanation: Interferon treatment is
A vaccine is available, and immune serum effective in some patients with hepatitis B.
globulin may be useful in the management
of outbreaks and immunocompromised or
pregnant patients. Anti-HAV antibodies of
IgM type may be useful diagnostically.

210
Question 24. E) Acute infection is Question 28. E) Hereditary
usually asymptomatic haemochromatosis
Explanation: Hepatitis C is only rarely Explanation: The combination of joint
symptomatic in the acute phase, and is pain (caused by chondrocalcinosis),
often only discovered once cirrhosis has diabetes mellitus ('bronzed diabetes),
developed. About 80% of infected patients hepatomegaly and hyperpigmentation is
develop chronic infection, which usually characteristic of hereditary
progresses ultimately to cirrhosis. haemochromatosis, one of the most
Treatment with pegylated interferon and common inherited diseases in northern
ribavirin can eradicate the virus Europeans. About 90% of patients are
completely in some patients. Although male, perhaps due to the protective effect
reinfection almost inevitably occurs after of iron loss with menstruation.
transplant, this treatment may give
otherwise terminally ill patients many Question29. B) Weekly venesection of
years of a high quality of life, and should 500 ml blood
not be withheld for this reason. Explanation: The complications of
haemochromatosis arise because of
Question 25. B) There is a genetic massive iron overload. The aim of
predisposition treatment is to remove this excess iron
Explanation: Twin studies have revealed safely, and this is done by weekly
a genetic contribution to ALD, although venesection until iron levels return to
there is at least one obvious environmental normal.
factor involved. After a presenting
complication of cirrhosis, most patients Question30. C) Wilson's disease
who become abstinent will be alive after 5 Explanation: Recurrent acute hepatitis
years. with parkinsonian features and
choreoathetoid movements suggest
Question 26. C) Non-alcoholic fatty liver Wilson's disease. The Kayser-Fleischer
disease rings (shown here) around the cornea are
Explanation: Obesity is by far the most pathognomonic but easily missed. Of the
common cause of non-alcoholic fatty liver other options, hereditary
disease, which also shows an association haemochromatosis usually presents later in
with type 2 diabetes mellitus and the life with advanced liver disease. The
metabolic syndrome. It is the most likely others may cause acute liver failure but do
cause in this patient. The introduction of not cause Kayser-Fleischer rings or
tests to detect and exclude infected blood extrapyramidal manifestations. Budd-
units has massively reduced the risk of Chiari syndrome is hepatic venous
transmission of hepatitis B and other thrombosis, Amanita phalloides is a
viruses through blood transfusion. The profoundly hepatotoxic mushroom, and
chances of hepatitis B transmission from a ecstasy (MDMA) is a drug of abuse that
unit of blood in the UK in 2002 were may cause liver failure.
around 0.24 per 1 000 000 units.
Question31. A) Penicillamine
Question 27. A) Co-amoxiclav Explanation: Penicillamine is a copper
Explanation: Co-amoxiclav (amoxicillin chelator and is the treatment of choice in
plus clavulanic acid) may cause a Wilson's disease. It removes copper in the
cholestatic hepatitis up to 42 days after a urine and confers an excellent prognosis,
course of treatment has finished. provided it is commenced before the
Paracetamol in normal doses can cause development of irreversible liver and
liver necrosis, but this only occurs in high- cerebral damage.
risk patients, such as those with pre-
existing alcoholic liver disease.

211
Question32. E) Chronic viral hepatitis Question36. B) Abdominal ultrasound
Explanation: All of these conditions scan
predispose to cirrhosis and hepatocellular Explanation: Ultrasound is quick and
carcinoma. Hepatitis B- and C-related inexpensive and does not expose the
cirrhosis is particularly carcinogenic. In patient to radiation. Stones may be
chronic hepatitis B infection, but not visualised within the gallbladder and bile
chronic hepatitis C infection, duct, and dilatation of the bile duct may be
hepatocellular carcinoma can also occur in seen, indicating distal obstruction of bile
the absence of cirrhosis. flow.
Question37. B) Correction of any
Question33. C) Acute cholecystitis coagulopathy with fresh-frozen plasma
Explanation: This is a classic presentation Explanation: The liver is a highly
of acute cholecystitis. Biliary colic causes vascular organ and life-threatening
pain but not inflammation, and so the haemorrhage may occur following injury
pyrexia and leucocytosis make this to it. Most patients presenting for TIPSS
diagnosis less likely. Acute pancreatitis is have cirrhosis, and impairment of hepatic
highly unlikely, given the normal amylase. synthetic function leading to coagulopathy
Mirizzi's syndrome (a stricture in the is common. This is effectively corrected
common hepatic duct caused by gallstones by replacing clotting factors by
impacted in the cystic duct) and administering fresh-frozen plasma.
choledocholithiasis (stones in the bile
duct) would be expected to cause Question38. C) Cholestasis and hepatitis
abnormal LFTs in a patient with disease are both common presentations
severe enough to present with systemic Explanation: Alcoholic liver disease is a
symptoms. common cause of substantial suffering, so
a good understanding of it is important. It
Question34. C) Cholecystokinin may be detected incidentally in an
Explanation: Cholecystokinin secretion is asymptomatic individual or may present
increased after a meal. It promotes satiety, with features of cholestasis, hepatitis or
increases pancreatic enzyme secretion, advanced cirrhosis. Alcohol causes several
stimulates gallbladder contraction and different pathological lesions including
relaxes the sphincter of Oddi. macrovesicular steatosis. Fatty liver
secondary to alcohol carries a good
Question35. A) Recurrent epigastric prognosis and may be reversible with as
pain, radiating to the back, lasting 2 little as 3 months of abstinence. Cirrhosis
hours then subsiding spontaneously is found in only 10% of alcoholics at post-
Explanation: 'Biliary colic' usually lasts mortem. The total amount of alcohol
for around 2 hours and occurs ingested rather than the concentration
intermittently. It is most commonly felt in determines the risk of liver damage.
the epigastrium (70%) or right upper
quadrant (20%), and often radiates to the Question39. E) Chronic hepatitis C
interscapular region or the tip of the right infection
scapula. Of the other options, D is
characteristic of dyspepsia and E is Question 40. D) Hypoglycaemia
suggestive of an acute myocardial Explanation: Hypoglycaemia does not
infarction. B and C are relatively non- precipitate hepatic encephalopathy but is
specific presentations but serious an important differential diagnosis in a
diagnoses such as dissecting thoracic patient with liver disease presenting with
aneurysm and acute pancreatitis must be confusion or drowsiness.
considered.

212
Chapter 23 BLOOD DISORDERS
Question1. A) They are derived from hypertension secondary to cirrhosis of the
megakaryocytes liver is a common cause of splenomegaly
Explanation: Red blood cell precursors in the UK. Multiple myeloma, a malignant
are called erythroblasts or normoblasts. proliferation of plasma cells associated
These nucleated cells divide and acquire with excess production of a monoclonal
haemoglobin. The nucleus is then extruded immunoglobulin (paraprotein), may
from the cell. The first non-nucleated red produce a diverse array of clinical
cell is the reticulocyte. Increased red cell manifestations but does not cause
production is therefore associated with splenomegaly.
increased numbers of circulating
reticulocytes. Megakaryocytes give rise to Question7. B) Coagulation by the
platelets. extrinsic pathway is initiated by the
interaction of factor VII with tissue
Question2. D) Pneumococcal infection factor
Explanation: Eosinophils have Explanation: Clotting factors are
phagocytic potential and are involved in synthesised in the liver. The coagulation
combating parasitic infection. They are process can be activated by one of two
also implicated in allergic reactions (e.g. pathways (extrinsic and intrinsic). The
atopic asthma, hay fever, eczema). extrinsic pathway is thought to be the
Bacterial infection typically causes a major physiological mechanism in vivo,
neutrophilia. and is initiated by the interaction of factor
VII with tissue factor. The final step in the
Question3. C) Carbimazole cascade is the conversion of fibrinogen to
Explanation: See list of Drugs causing fibrin. Antithrombin, protein C and protein
neutropenia S are natural inhibitors of the clotting
system. Antithrombin has inhibitory
Question4. C) Thalassaemia activity against thrombin and factor Xa.
Explanation: Iron deficiency, The binding of heparin to antithrombin
thalassaemia and sideroblastic anaemia all accelerates this inhibitory activity.
cause a microcytosis (reduced red cell size
or mean cell volume, MCV). Vitamin B12 Question8. B) Factor V
or folate deficiency, chronic liver disease, Explanation: Clotting factors are
alcohol excess and hypothyroidism cause a synthesised by the liver. Factors II, VII, IX
macrocytosis (i.e. increased MCV). and X are produced as inactive proteins.
They are activated by a carboxylase
Question5. B) Cirrhosis enzyme which requires vitamin K as a co-
Explanation: Target cells have a central factor. In this process, vitamin K is
area of haemoglobinisation and are seen inactivated to an epoxide; it must therefore
typically in liver disease, thalassaemia and be reactivated by a reductase enzyme.
post-splenectomy. Warfarin inhibits the latter reaction,
reducing the availability of active vitamin
Question6. B) Multiple myeloma K and thereby exerting an anticoagulant
Explanation: Chronic myeloid leukaemia, effect.
myelofibrosis and malaria may all produce
massive splenomegaly. Portal

213
Question9. C) It provides an accurate Question12. C) Doppler ultrasound
assessment of platelet function scan of the leg
Explanation: The prothrombin time (PT) Explanation: A patient's clinical risk of
assesses the extrinsic pathway of the having a DVT can be established using the
coagulation cascade and is prolonged by Wells scoring system. Those with a high
deficiencies of factors II, V, VII and X. or medium risk should undergo definitive
Warfarin leads to prolongation of the PT. investigation by Doppler ultrasound or
The international normalised ratio (INR) is venography. For patients with a low risk,
used to monitor warfarin treatment; it is the plasma D-dimer should be measured; a
the ratio of the patient's PT to a normal normal level excludes a DVT, but a raised
control PT corrected to the international level should prompt ultrasound or
thromboplastin. The activated partial venography. A positive D-dimer is not
thromboplastin time (APTT) assesses the diagnostic of a DVT, but a negative D-
intrinsic pathway and is prolonged in dimer in patients at low risk of a DVT
patients with haemophilia A. essentially excludes the diagnosis.
Thrombocytopenia and abnormal platelet Depending on the clinical context, a
function lengthen the bleeding time but do thrombophilia screen may be indicated
not affect the PT or APTT. after the diagnosis of venous thrombosis
has been confirmed.
Question10. B) Haemarthrosis
Explanation: Muscle and joint bleeds Question13. E) ABO incompatibility
suggest a coagulation defect. Purpura, reactions are mediated by complement
prolonged bleeding from superficial cuts, Explanation: The ABO system comprises
epistaxis, gastrointestinal haemorrhage four different blood groups: O, A, B and
and menorrhagia suggest AB. Individuals have antibodies directed
thrombocytopenia, abnormal platelet against the A or B antigens that are not
function or von Willebrand's disease. expressed on their own red cells. Group O
blood can therefore be given to all
Question11. D) Desmopressin patients, as the infused red cells do not
aggravates the tendency to bleeding express any AB antigens ('universal
Explanation: Haemophilia A is an X- donor'), while patients with group AB
linked recessive condition characterised by blood can safely receive blood of all four
factor VIII deficiency. There is a spectrum groups, as they do not have antibodies to
of severity. Large joint haemarthroses and either A or B antigens ('universal
muscle haematomas are characteristic. recipient'). The transfusion of ABO-
Recurrent bleeds into joints can lead to incompatible blood is the main cause of
secondary osteoarthritis. Treatment is fatal acute transfusion reactions, and the
usually with factor VIII concentrate. risk is greatest if group A red cells are
Desmopressin (vasopressin, antidiuretic infused into a group O recipient. The
hormone recipient's antibodies bind to antigens on
) can also be used to treat mild bleeds or the transfused cells, activating
cover minor surgery in patients with mild complement. The resulting lysis of red
haemophilia. blood cells may lead to disseminated
intravascular coagulation and renal failure.

214
Question14. C) Transfusion of RhD- Reduced gastric acid secretion following
negative blood to an RhD-positive long-term proton pump inhibitor therapy
woman may result in production of can therefore lead to iron malabsorption.
anti-RhD antibodies Aspirin causes iron deficiency by
Explanation: IgG antibodies to RhD- increasing gastrointestinal blood loss, not
positive red cells are produced if such cells by interfering with its absorption.
enter the circulation of an RhD-negative
individual (e.g. via blood transfusion or Question18. C) Upper gastrointestinal
fetomaternal haemorrhage). If an RhD- endoscopy
negative woman is so sensitised and Explanation: Iron deficiency causes a
subsequently becomes pregnant with a microcytic, hypochromic anaemia. The
RhD-positive fetus, then the anti-RhD most common cause in men and post-
antibodies can cross the placenta and menopausal women is gastrointestinal
cause severe anaemia and neurological blood loss. This may result from upper or
damage (haemolytic disease of the lower gastrointestinal malignancy, peptic
newborn). Anti-RhD immunoglobulin ulcer disease, inflammatory bowel disease,
(anti-D) can be given to RhD-negative diverticular disease, colonic polyps and
pregnant women after potentially angiodysplasia. Aspirin and non-steroidal
sensitising events to prevent the anti-inflammatory drugs (NSAIDs) can
development of endogenous Rhesus exacerbate any blood loss. The
antibodies. gastrointestinal tract must therefore be
investigated by endoscopy (upper
Question15. D) Give paracetamol and gastrointestinal endoscopy and
continue transfusion at a slower rate colonoscopy) or barium studies.
Explanation: Febrile non-haemolytic
transfusion reactions are not uncommon. If Question19. C ) Cold agglutinin disease
temperature rises by less than 1.5°C and Explanation: Iron deficiency occurs when
the patient is otherwise well, then it is iron losses or physiological requirements
reasonable to administer paracetamol and exceed absorption. (see link below). Cold
continue the transfusion at a slower rate. agglutinin disease is a cause of
Allergic reactions (urticaria, intravascular haemolysis resulting from
bronchospasm, angio-oedema) should be the development of IgM antibodies to red
treated with IV chlorphenamine and cells. It can occur transiently in association
hydrocortisone, nebulised salbutamol and with Mycoplasma pneumoniae infection or
IM adrenaline (epinephrine) according to infectious mononucleosis, or may follow a
their severity. The transfusion should be chronic course in patients with underlying
stopped, and any unused blood should be lymphoma.
returned to the blood bank together with a
repeat sample for a 'group and screen' test. Question 20. D) Reduced ferritin
Explanation: All of the above typically
Question16. E) Creutzfeldt-Jakob occur in iron deficiency but plasma ferritin
disease (CJD) is the best single confirmatory test on
Explanation: Blood products are account of its high specificity (i.e. a low
routinely screened for HIV, human T ferritin is strongly suggestive of iron
lymphotrophic virus (HTLV), hepatitis B deficiency). On the other hand, a normal
and C, and syphilis. ferritin level does not exclude iron
deficiency as levels may be elevated in
Question17. A) Omeprazole systemic inflammation (ferritin is an acute
Explanation: Iron is absorbed in the phase protein) or liver disease, despite
upper small intestine. Hydrochloric acid absent bone marrow stores.
secreted by the stomach helps to maintain
iron in the soluble reduced ferrous (Fe2+)
state, as opposed to the oxidised ferric
(Fe3+) state; the ferrous form is more easily
absorbed.
215
Question 21. D) It is often due to dietary Question 24. D) Hereditary
deficiency spherocytosis
Explanation: Vitamin B12 and folate Explanation: The normocytic anaemia,
deficiency lead to impaired DNA reticulocytosis, elevated bilirubin and
synthesis. This results in a megaloblastic elevated LDH suggest a haemolytic
anaemia associated with a raised mean cell process, consistent with either
volume (MCV). The blood film shows autoimmune haemolytic anaemia or
oval macrocytes and hypersegmented hereditary spherocytosis. The presence of
neutrophils. In severe cases there may be reticulocytes, which are large, accounts for
peripheral blood cytopenias. Vitamin B12 the MCV at the upper end of the normal
is absorbed in the terminal ileum. range. One would expect the direct
Deficiency may therefore arise from Coombs test to be positive in autoimmune
Crohn's disease or surgery affecting this haemolytic anaemia, making hereditary
part of the bowel. B12 deficiency is very spherocytosis the more likely diagnosis in
rarely dietary in origin, only affecting this case. The anaemia of chronic disease
strict vegans. may cause a normocytic anaemia but
would not account for the features
Question 22. B) Nystagmus suggesting haemolysis. Coeliac disease
Explanation: Vitamin B12 deficiency is may lead to anaemia through iron and/or
associated with neurological disease in up folate malabsorption but both are normal
to 40% of patients. Peripheral nerve in this case. The normal B12 level excludes
involvement gives rise to a 'glove and pernicious anaemia.
stocking' peripheral neuropathy. Spinal
cord damage results in subacute combined Question 25. B) Haemosiderinuria
degeneration of the cord; posterior column Explanation: Haemosiderinuria occurs
involvement causes loss of vibration and when all other mechanisms for 'mopping
joint position sensation (resulting in up' free intravascular haemoglobin have
ataxia), and corticospinal tract damage been exhausted and is always indicative of
leads to upper motor neuron signs. In rare intravascular haemolysis. Earlier features
cases, chronic B12 deficiency can lead to include a fall in haptoglobin levels and
dementia. formation of methaemalbumin (detected
by the Schumm's test). Reticulocytosis and
Question 23. C) Total body stores are elevated LDH may result from any cause
small of haemolysis. Splenomegaly may occur
Explanation: Foods rich in folate include in extravascular haemolysis where
leafy vegetables, fruits and animal protein. physiological destruction occurs in the
Pregnancy is a common cause of folate fixed reticulo-endothelial cells in the liver
deficiency, especially in twin pregnancies or spleen. Positive direct Coombs test
and hyperemesis gravidarum. Pernicious indicates immune-mediated destruction of
anaemia is an autoimmune condition in red cells.
which there is antibody-mediated
destruction of gastric parietal cells. This
leads to reduced secretion of intrinsic
factor and impaired absorption of vitamin
B12; folate is unaffected. The Schilling test
is helpful in distinguishing B12 deficiency
secondary to pernicious anaemia from that
resulting from malabsorption. It is,
however, rarely performed in practice.
Serum folate is very sensitive to dietary
intake, and red cell folate provides a more
accurate indicator of folate stores.

216
lead to a vaso-occlusive crisis, where
Question 26. C) It may lead to the plugging of small blood vessels produces
development of cholesterol gallstones acute severe bone pain (e.g. dactylitis,
Explanation: In most cases of hereditary avascular necrosis of the hip, pain in the
spherocytosis there is a compensated limbs or vertebrae). Recurrent sickling in
chronic haemolytic state. However, the the spleen can cause splenic infarction and
condition can be complicated by ultimately hyposplenism. Bone marrow
haemolytic, aplastic or megaloblastic infarction can result in fat emboli to the
crises. The latter results from folate lungs and cause pulmonary infarction
deficiency, which in turn arises from (sickle chest syndrome).
increased red cell turnover. This can be
precipitated by pregnancy, and patients Question 30. C) Beta-thalassaemia
with hereditary spherocytosis should minor
receive folic acid supplements. The Explanation: Normal adult haemoglobin
hyperbilirubinaemia resulting from (HbA) consists of two alpha and two beta
haemolysis gives rise to pigment (not globin chains (V2_2). The thalassaemias are
cholesterol) gallstones. The spleen is the a group of inherited disorders in which
main site of haemolysis, and splenectomy there is impairment of globin chain
may therefore improve the anaemia. production. In alpha-thalassaemia the
alpha genes are deleted; in beta-
Question 27. C) Glucose-6-phosphate thalassaemia (seen most commonly in the
dehydrogenase Mediterranean area) beta chain production
Explanation: Glucose-6-phosphate is defective. In beta-thalassaemia major
dehydrogenase is essential for the (seen in homozygotes), HbA production is
intracellular production of NADPH which absent or grossly reduced; individuals
protects the red cell against oxidative develop a profound microcytic
stress. Deficiency can result in haemolysis. hypochromic anaemia, and electrophoresis
reveals increased levels of HbF (V2W2).
Question 28. D) Sickle-cell anaemia is Beta-thalassaemia minor (seen in
usually accompanied by a heterozygotes) results in a microcytic
reticulocytosis blood picture with no clinical
Explanation: Sickle-cell disease is caused manifestations; there is a raised fraction of
by an amino acid substitution in the beta HbA2 (V2a2).
globin polypeptide chain. The condition is
inherited as an autosomal recessive trait. Question 31. C) Acute myeloid
Homozygotes only produce abnormal beta leukaemia (AML)
chains which form abnormal HbS (sickle- Explanation: This patient has a high
cell anaemia), while heterozygotes make a white cell count associated with anaemia
mixture of normal HbA and abnormal HbS and thrombocytopenia. The hypercellular
(sickle-cell trait). Individuals with sickle- bone marrow indicates increased white
cell trait are usually asymptomatic and cell proliferation with 'spill over' into the
may be relatively resistant to falciparum peripheral blood. The excess of immature
malaria. Patients with sickle-cell disease blast cells of the myeloid lineage suggests
usually have a compensated anaemia a diagnosis of AML. Although white cells
associated with a reticulocytosis. The high are massively increased in number, they
reticulocyte count represents the body's are immature and functionally useless.
attempt to maintain an adequate They take up increasing amounts of
haemoglobin concentration in the face of marrow space at the expense of the normal
red cell sickling. Even with optimal haematopoietic stem cells.
medical care, life expectancy is greatly
reduced. Question 32. D) Chronic myeloid
leukaemia (CML)
Question 29. Peripheral neuropathy Explanation: This patient has a marked
Explanation: Hypoxia, dehydration and leucocytosis originating from the
infection precipitate sickling. This can granulocyte series. The picture is
217
dominated by mature neutrophils. This Question 35. D) Bone marrow
pattern of granulocyte proliferation with aspiration reveals a hypocellular
fairly normal cell maturation is consistent marrow
with chronic myeloid leukaemia. In some Explanation: Myelodysplastic syndrome
cases, CML may transform into an acute (MDS) comprises a group of
leukaemia (blast crisis phase) haematopoietic stem cell disorders
characterised by proliferation of immature characterised by peripheral blood
blast cells. Blast crisis is relatively cytopenias and abnormal-looking
refractory to treatment and is the major (dysplastic) blood cells. The marrow is
cause of death in patients with CML. hypercellular with dysplastic changes. It
predominantly affects elderly patients,
Question 33. E) Chronic myeloid presenting with symptoms of anaemia,
leukaemia (CML) recurrent infections or bleeding. The
Explanation: Around 95% of patients condition is incurable in most patients, and
with CML possess the Philadelphia supportive care with transfusion and
chromosome. This is a chromosomal antibiotics is the mainstay of treatment.
abnormality characterised by reciprocal Given time, all forms of MDS will
translocation of genetic material between ultimately progress to AML, although the
chromosomes 9 and 22. The fragment time course to progression is highly
from chromosome 9 carries the abl variable.
oncogene and joins the breakpoint cluster
region (BCR) on chromosome 22. The Question 36. B) Reed-Sternberg cell
resultant BCR ABL chimeric gene codes Explanation: The characteristic
for a tyrosine kinase that stimulates cell histological abnormality in Hodgkin
proliferation and plays a causative role in lymphoma is the presence of Reed-
the disease. The development of imatinib, Sternberg cells - large, malignant
a specific inhibitor of BCR ABL tyrosine lymphoid cells of B-cell origin. Ringed
kinase activity, has revolutionised the sideroblasts are erythroid precursors in
management of CML. which iron has accumulated in the
mitochondria, seen in sideroblastic
Question 34. C) Hyposplenism is a anaemia. Auer rods are present in acute
frequent finding myeloid leukaemia. Burr cells are
Explanation: Chronic lymphocytic irregularly shaped red blood cells typical
leukaemia (CLL) has an insidious onset, of uraemia, and Howell-Jolly bodies are
and the diagnosis is made incidentally nuclear remnants commonly seen post-
following a routine full blood count in splenectomy; both are detected on the
70% of patients. The condition most peripheral blood film.
commonly arises in the elderly, and is
characterised by a monoclonal
proliferation of B lymphocytes. Clinical
features include symptomatic anaemia,
infection, lymphadenopathy, night sweats
and weight loss. Splenomegaly and warm
autoimmune haemolytic anaemia may also
be present. Treatment is generally only
required for patients with systemic
symptoms, bone marrow failure,
progressive lymphadenopathy or
splenomegaly. The oral chemotherapeutic
agent chlorambucil is usually first-line
therapy.

218
Question 37. B) Multiple myeloma Question 39. E) Aspirin is
Explanation: Multiple myeloma is a contraindicated
malignant proliferation of plasma cells Explanation: Polycythaemia rubra vera
associated with excess production of a (PRV) is a myeloproliferative disorder
monoclonal immunoglobulin characterised by an increased red cell mass
(paraprotein). The plasma cells release and elevated haemoglobin concentration.
cytokines which stimulate osteoclasts and Neutrophil and platelet counts are often
result in bone resorption, bone pain, also raised. Patients may present with
pathological fractures and hypercalcaemia. symptoms of hyperviscosity (e.g.
Bone marrow involvement can lead to a headache, dizziness, loss of concentration,
normocytic anaemia. Paraprotein lethargy, pruritus). Splenomegaly is
deposition in the kidneys, hypercalcaemia common. The hyperviscosity predisposes
and dehydration can all contribute to cause to arterial thrombosis; aspirin reduces this
renal failure. The erythrocyte risk. Venesection reduces red cell mass
sedimentation rate is often elevated. The and relieves hyperviscosity symptoms.
paraprotein (most often of the IgG
subtype) can be measured in plasma and Question 40. B) Thrombotic
detected by protein electrophoresis. thrombocytopenic purpura
Explanation: Haemolytic uraemic
Question 38. D) Bence Jones protein in syndrome (HUS) and thrombotic
urine and lytic lesions on plain X-ray thrombocytopenic purpura (TTP) represent
Explanation: This is a difficult question two conditions on a continuum
but contains a number of useful learning characterised by microangiopathic
points. The diagnosis of multiple myeloma haemolytic anaemia, the formation of
requires two of the following criteria: platelet thrombi in the microcirculation
• increased malignant plasma cells in bone and resultant thrombocytopenia. Other
marrow features include renal failure and
• serum and/or urinary paraprotein fluctuating neurological signs due to
skeletal lesions (characteristically lytic microvascular thrombosis.
lesions).
• Thus, although options A and B would Question 41. D) Non-Hodgkin
both be highly suggestive of myeloma, lymphoma
only option D satisfies the criteria. Bence Explanation: The combination of
Jones protein is an immunoglobulin light lymphadenopathy and systemic upset in an
chain produced by plasma cells that elderly patient raises the possibility of
appears in urine. Hypercalcaemia is a non-Hodgkin lymphoma (NHL).
common feature in myeloma, resulting Hodgkin's disease usually affects young
from lytic bony lesions. The ESR may be adults, although there is a second peak at
elevated as a result of increased plasma age 50-70. CML is usually associated with
viscosity from the paraproteinaemia, a marked leucocytosis and does not
although it is worth noting that only 5% of typically cause generalised
patients with an ESR persistently > 100 lymphadenopathy.
mm/hr actually have myeloma. Lytic bony
lesions, the skeletal hallmark of myeloma,
are best detected by skeletal survey and
not a radioisotope bone scan; in fact, in the
absence of fractures, both plasma ALP and
radioisotope bone scan are normal in
myeloma.

219
Question 42. C) Fibrinogen levels are Explanation: Unfractionated IV heparin
elevated exerts its anticoagulant affect by
Explanation: Disseminated intravascular enhancing the effect of antithrombin,
coagulation (DIC) is triggered by which itself inhibits the procoagulant
conditions causing exposure of tissue activity of factors IIa, VIIa, IXa, Xa and
factor with consequent activation of the XIa. The level of anticoagulation is
coagulation system via the extrinsic monitored using the APTT. Low
pathway. Common causes of DIC include molecular weight heparins are
Gram-negative septicaemia and administered subcutaneously on a once- or
antepartum haemorrhage. Intravascular twice-daily basis. They augment
coagulation results in thrombosis with antithrombin activity against factor Xa and
consumption of platelet and coagulation do not affect the APTT. The half-life of IV
factors. The resultant thrombocytopenia, heparin is around 1 hour, and if a patient
prolonged prothrombin and activated bleeds it is usually sufficient simply to
partial thromboplastin time lead to a discontinue the heparin; in severe cases,
haemorrhagic state. Fibrinogen levels are heparin can be neutralised with protamine.
low, and levels of D-dimer (a fibrin It takes warfarin takes several days to
degradation product indicating fibrin exert its therapeutic effect, and treatment
turnover and breakdown) are raised. with heparin is required during this
window. Heparin-induced
Question 43. C) The anticoagulant effect thrombocytopenia (HIT) is a recognised
of low molecular weight heparin can be complication, requiring immediate
assessed using the activated partial discontinuation of the drug.
thromboplastin time (APTT)

220
Chapter 24 MUSCULOSKELETAL
DISORDERS

Question 1. D) Paget's disease Question 4. C) Exacerbations of


Explanation: Paget's disease is a common symptoms are often associated with
condition characterised by increased and stressful life events
disorganised bone remodelling. The Explanation: The principal features of
biochemical picture is that of a normal fibromyalgia are multiple sites of regional
calcium, phosphate and PTH, in pain, particularly over the neck and back,
conjunction with a raised alkaline and fatigability. In general there is a
phosphatase. Osteoporosis (in the absence discrepancy between the level of disability
of a fracture) does not cause any reported by the patient and the physical
biochemical abnormality. The fact that the findings found on examination. Joint
lady has been complaining of difficulty in disease, neurological deficit, muscle
hearing may be significant, as deafness is wasting and a raised ESR are not features
a clinical feature of Paget's disease. of fibromyalgia and any of these should
prompt a search for an alternative
Question 2. C) Septic arthritis explanation. Treatment is symptomatic
Explanation: Although all of the above and patient education is important, as is
may cause an acute monoarthritis, septic the provision of a graded exercise
arthritis is the most likely cause, given the programme.
progressive pain, overlying erythema,
pyrexia, recent episode of cellulitis Question 5. C) Prolonged morning
(representing a possible portal of entry for stiffness
infection) and presence of risk factors Explanation: Morning stiffness in
(diabetes mellitus, pre-existing joint osteoarthritis tends to be brief, lasting only
disease, age). With crystal synovitis (urate, a few minutes. Thereafter, pain is related
pyrophosphate) pain is at its maximum to movement and weight-bearing, and
within the first day; progressive pain and relieved by rest. This pattern contrasts
overlying erythema should always suggest markedly with inflammatory arthritides
sepsis. such as rheumatoid arthritis, in which
morning stiffness is typically prolonged
Question 3. D) Joint aspiration (lasting longer than an hour) and
Explanation: Although a full blood count, symptoms are exacerbated by inactivity.
CRP test and X-ray of the joint might
provide useful information, only an Question 6. D) Methotrexate
aspirate of fluid from the knee sent for Explanation: Osteoarthritis is extremely
microscopy, culture and sensitivity can common and knowledge of the various
definitively confirm the diagnosis. Blood aspects of its management is therefore
cultures also identify the organism in essential. Methotrexate is a disease-
around 50% of cases. modifying drug used in the treatment of
the inflammatory arthritides and has no
role in the management of osteoarthritis.

221
Question 7. B) The presence of anti- Question 11. B) Naproxen
double-stranded DNA (anti-dsDNA) Explanation: DMARDs have the
antibodies on blood tests potential to reduce target tissue damage in
Explanation: Anti-dsDNA antibodies are rheumatoid arthritis and may therefore
highly specific for systemic lupus favourably alter the rate of disease
erythematosus and would therefore make progression. Naproxen is a non-steroidal
the diagnosis of rheumatoid arthritis anti-inflammatory drug (NSAID) and, as
unlikely. The presence of rheumatoid such, provides symptomatic relief only.
factor, on the other hand, would be
consistent with the diagnosis and, in Question 12. D) Keratitis with
common with the other options listed, is hydroxychloroquine
included in the American Rheumatism Explanation: All of the above
Association diagnostic criteria for associations are true with the exception of
rheumatoid arthritis. Because of its low D. At very high doses hydroxychloroquine
specificity, however, a high titre of (though more commonly chloroquine)
rheumatoid factor at presentation has more may cause retinitis (classically a 'bull's
prognostic value (it suggests a worse eye' maculopathy) and not keratitis
prognosis) than diagnostic value. (inflammation of the cornea).

Question 8. D) Alopecia Question 13. B) Strongly positive


Explanation: All of the above are rheumatoid factor
recognised features of rheumatoid arthritis, Explanation: As one of the seronegative
with the exception of alopecia. Alopecia is spondarthritides (see Box 25.51 in the link
more commonly associated with systemic below), ankylosing spondylitis has no
lupus erythematosus. association with seropositivity for
rheumatoid factor. Rheumatoid factor is
Question 9. C) Felty's syndrome negative or positive only in low titre.
Explanation: Although an enlarged
spleen may potentially be found in any of Question 14. D) Atlanto-axial instability
the above conditions, Felty's syndrome Explanation: Psoriatic arthritis may
refers to the association of splenomegaly present with a variety of different patterns
and neutropenia with rheumatoid arthritis of joint disease. Atlanto-axial subluxation,
and is, therefore, the most likely diagnosis however, is a potentially serious
here. It has a peak incidence in the 50-70- manifestation of rheumatoid arthritis and
year-old age group, typically occurring in occasionally other conditions (e.g. classic
patients with longstanding disease and a ankylosing spondylitis, Down's
positive rheumatoid factor. In addition to syndrome).
splenomegaly, clinical features include
recurrent infections, weight loss, leg ulcers Question 15. A) Bendroflumethiazide
and lymphadenopathy. Explanation: Thiazide diuretics such as
bendroflumethiazide reduce urinary
Question 10. A) Bouchard's nodes excretion of uric acid and may therefore
Explanation: Bouchard's nodes are lead to hyperuricaemia and eventual urate
posterolateral hard swellings of the crystal formation. Other drugs that cause
proximal interphalangeal joints, hyperuricaemia include aspirin,
characteristically seen in nodal generalised ciclosporin, pyrazinamide and alcohol.
osteoarthritis along with Heberden's nodes
(similar swellings arising from the distal
interphalangeal joints).

222
Question 16. C) Allopurinol is often Question 20. D) Sclerodactyly
used in the treatment of an acute attack Explanation: All the skin and mucous
Explanation: Allopurinol is not used to membrane conditions listed are associated
treat acute gout. Management of acute with SLE except for sclerodactyly, which
gout includes appropriate analgesia (often is a feature of systemic sclerosis.
a non-steroidal anti-inflammatory drug),
oral colchicine, joint aspiration and intra- Question 21. A) They inhibit bone
articular injection with a long-acting resorption
steroid. The most common side-effects of Explanation: Bisphosphonates act by
colchicine are nausea and diarrhoea. adsorbing to bone surfaces and becoming
Definitive diagnosis of gout requires incorporated into the bone matrix. When
demonstration of negatively birefringent they are then ingested by osteoclasts in the
needle-shaped crystals in a joint aspirate process of bone resorption, large amounts
when viewed under polarised light. of the drug are released within the cell,
bringing about osteoclast death and a
Question 17. B) Acromegaly reduction in bone resorption. They have no
Explanation: Sporadic, familial and direct stimulatory effect on bone
metabolic-associated forms of pseudogout formation. The increased bone mineral
exist. The common feature is deposition of density is thought to be due to decreased
calcium pyrophosphate crystals in the joint removal of minerals from existing bone.
(most commonly the knee, followed in
frequency by the wrist, shoulder, ankle Question 22. D) Difficulty passing urine
and elbow), leading to chondrocalcinosis. Explanation: The most important lesson
Acromegaly is associated with a secondary from this question is that there are so-
osteoarthritis, but not with pseudogout. called 'red flag signs' (see the link below)
which must not be ignored, as they may be
Question 18. C) Hypothyroidism indicative of serious underlying pathology.
Explanation: Hyperthyroidism, not All of the other symptoms described are
hypothyroidism, is associated with consistent with simple mechanical low
osteoporosis. back pain, which is likely to improve with
time and simple analgesia.
Question 19. D) Weight-bearing exercise
is protective Question 23. D) Psoriasis
Explanation: Protective factors include Explanation: The term 'seropositive'
weight-bearing exercise, high dietary refers to arthritis that associates with an
calcium and late menopause. The increased prevalence of rheumatoid factor.
biochemical profile in osteoporosis should Psoriasis is one of a family of conditions -
be entirely normal, and any deviation from the seronegative spondarthritides - that
this, other than in the context of a fracture manifest arthritis with no such increased
(which may cause a rise in alkaline association with rheumatoid factor.
phosphatase), should arouse suspicion of Rheumatoid factor has low specificity for
an alternative diagnosis. The diagnosis is rheumatoid arthritis and occurs with
based on the T-score, which measures the increased prevalence in a wide range of
number of standard deviations difference conditions.
between the bone mineral density of the
patient and the mean of a normal healthy
population with peak bone mass. The Z-
score is the number of standard deviations
from the mean for an age-matched
population, which is a less precise
predictor of risk of fracture than the T-
score.

223
Question 24. C) Concomitant Both T- and Z-scores refer to numbers of
respiratory disease standard deviations from the mean.
Explanation: Comorbidity does not Osteoporosis is defined as a T-score of < -
increase the incidence of NSAID- 2.5. The preferred sites for measurement
associated gastrointestinal ulceration, but are the lumbar spine and the hip. DXA
if a person develops a complication scans should not be confused with isotope
(bleeding, perforation, obstruction) bone scans, which involve the intravenous
comorbidity, especially cardiovascular injection of radioisotope.
disease and diuretic therapy, increases the
associated morbidity and mortality. Co- Question 27. A) Calcium channel
prescription of a proton pump inhibitor blockers such as nifedipine may help
such as omeprazole can reduce the symptoms of Raynaud's phenomenon
incidence of NSAID-associated peptic Explanation: The dihydropyridine
ulceration, and is particularly important to calcium channel blockers help prevent
consider in those with any of the risk vasospasm and are a useful treatment for
factors listed above. A past history of Raynaud's phenomenon. An ACE inhibitor
peptic ulcer bleeding or perforation is is an important prophylactic agent in
usually taken as an absolute reducing the risk of development of a
contraindication to oral NSAIDs. hypertensive renal crisis, particularly in
patients who have diffuse cutaneous
Question 25. A) Conjunctivitis disease, who are most at risk of this
Explanation: Reiter's disease is the most complication. Other useful treatments
common cause of inflammatory arthritis in include proton pump inhibitors for
men aged 16-35. It consists of the classic oesophagitis (which is often severe) and
triad of arthritis, urethritis and the oral endothelin antagonist bosentan for
conjunctivitis; additional extra-articular pulmonary hypertension. On the other
features include keratoderma hand, no agent is known to improve the
blenorrhagica, nail dystrophy, buccal cutaneous manifestations of the disease.
erosions and circinate balanitis.
Precipitants for Reiter's include bacterial Question 28. C) A violaceous rash
infections such as Salmonella or around the eyes
Campylobacter, and also sexually Explanation: Recognised cutaneous
transmitted infections such as Chlamydia, features of dermatomyositis include a
but not streptococcal throat infection. violaceous rash around the eyes, Gottron's
Nodules are a feature of rheumatoid papules over the proximal interphalangeal
arthritis. and distal interphalangeal joints,
periorbital oedema and abnormal nailfold
Question 26. E) DXA bone scanning capillary loops. Weakness of proximal
involves the intravenous injection of musculature is a cardinal feature.
radioisotope
Explanation: DXA bone scanning Question 29. A) There is an association
measures the extent to which the passage with HLA-B27
of X-ray beams through bone is Explanation: HLA-B27 is strongly
attenuated. Since this is determined by the associated with ankylosing spondylitis.
degree of mineralisation, the relative Sjögren's is associated with HLA-B8 and
density of the bone can be calculated. Two HLA-DR3.
scores are calculated; the T-score
compares the bone mineral density (BMD)
of the subject to that of a healthy young
person, and the Z-score to that of an age-
matched control.

224
Question 30. B) Sjögren's syndrome Question 34. C) It is a large-vessel
Explanation: The case described fits best vasculitis
with a diagnosis of Sjögren's syndrome. Explanation: Polyarteritis nodosa (PAN)
Although the serology would be is a rare vasculitis of small and medium-
compatible with both SLE and rheumatoid sized (not large conduit) vessels, which
arthritis, the description of predominately affects men, is ANCA-
keratoconjunctiva sicca (dry eyes) and negative and may have a number of
xerostomia (dry mouth) and the manifestations, including arteritis of the
combination of both positive rheumatoid vasa nervorum, leading to peripheral
factor and ANA are typical of Sjögren's. neuropathy, abdominal pain, fever and
severe hypertension secondary to renal
Question 31. A) There is an association infarctions. Hepatitis B is a risk factor for
with HLA-B27 development of PAN, and incidence of
Explanation: HLA-B27 is strongly PAN is higher in places such as Alaska
associated with ankylosing spondylitis. where the infection is endemic.
Sjögren's is associated with HLA-B8 and
HLA-DR3. Question 35. C) Osteoclasts are
responsible for bone formation
Question 32. D) Muscle stiffness and Explanation: All of the above are true
tenderness with the exception of option C; osteoclasts
Explanation: Polymyalgia rheumatica is a are responsible for bone resorption (C for
disease of older people; patients are almost chewing away at the bone), osteoblasts are
always over 55 years, but there is a mean responsible for bone formation (B for
age of onset of 70. The ESR is invariably building up bone).
elevated. There is a rapid and dramatic
response to systemic corticosteroids, with Question 36. C) Articular cartilage has
most patients experiencing relief of a rich vascular supply
symptoms within 72 hours of first dose. Explanation: The major learning point
The most common complaint is of muscle here is that articular cartilage is avascular.
stiffness and there is often marked muscle
tenderness but no muscle weakness. Question 37. E) Osteopenia with Paget's
Muscle weakness and wasting suggest disease of bone
primary muscle disease such as Explanation: X-rays in Paget's disease
polymyositis. commonly show a mixed picture of
osteosclerosis and focal osteolysis, but not
Question 33. E) Give 60 mg of osteopenia. All of the other radiological
prednisolone by mouth features described are true.
Explanation: The history and exam
findings detailed above are absolutely
classical for temporal or giant cell arteritis.
This is a large-vessel vasculitis affecting,
predominantly, the branches of the
temporal and ophthalmic arteries. It is
sight-threatening, and the correct treatment
once the diagnosis is suspected is
immediate administration of high-dose
oral corticosteroid. Such steroid treatment
will not alter any histological
abnormalities obtained on temporal artery
biopsy if this procedure is undertaken
within 1-2 weeks.

225
Question 38. E) Serum biochemistry is Question 40. B) Henoch-Schönlein
usually normal in osteomalacia purpura
Explanation: Osteomalacia is commonly Explanation: The combination of
associated with low serum calcium and arthritis, abdominal pain and purpura in a
phosphate, accompanied by elevated patient of this age suggests a diagnosis of
alkaline phosphatase. In contrast, Henoch-Schönlein purpura. The condition
osteoporosis is associated with normal often follows an upper respiratory tract
serum biochemistry. Options B and C infection.
highlight the difference in the sensitivity
and specificity of tests. ANA has a high Question 41. D) Chest X-ray
sensitivity for lupus (almost 100%), so if a Explanation: A chest X-ray is the least
person tests negative for ANA this likely to aid diagnosis. A skin biopsy,
virtually excludes the diagnosis. (High although invasive, would confirm the
sensitivity is a desirable quality in a good diagnosis by demonstrating on
screening test.) Anti-dsDNA antibodies, immunofluorescence IgA deposition in the
however, provide a highly specific test for vessel walls. The serum IgA levels are
lupus, meaning that if a person tests also often raised. Henoch-Schönlein
positive for anti-dsDNA antibodies, then purpura may cause an acute nephritis
the diagnosis is almost certainly lupus. leading to haematuria and/or renal failure,
Unfortunately, anti-dsDNA has low thus U&E and urinalysis are important. An
sensitivity for lupus. FBC should be checked in all cases of
purpura to exclude the possibility of
Question 39. D) Hydralazine thrombocytopenia (low platelet count).
Explanation: Hydralazine, along with
procainaminde, isoniazid, minocycline and Question 42. E) Pulmonary embolism
chlorpromazine, can cause a drug-induced Explanation: Cardiovascular
lupus syndrome that commonly presents as complications are the leading cause of
skin rash and arthralgia. Renal death in patients with Marfan's,
involvement is less common in drug- particularly aortic dissection which
induced lupus than in primary lupus, but accounts for 27-48% of deaths. There is no
can occur. increased risk of pulmonary embolism.

226
Chapter 25 NEUROLOGICAL DISEASE
Question 1. B) C5/C6 Question 5. B) Childhood absence
Explanation: The biceps and supinator epilepsy
jerks both correspond to the C5/C6 nerve Explanation: Absence epilepsy (formerly
roots. known as petit mal) is a form of
generalised epilepsy encountered in
Question 2. D) Schwann cells childhood and is characteristically
Explanation: The myelin sheath associated with a 3 cycles/second 'spike
surrounding axons is essential for rapid and wave' pattern on EEG. Individuals
conduction of the action potential. Myelin develop altered awareness and often have
is formed by Schwann cells in the a staring appearance lasting for only a few
peripheral nervous system, and by seconds, followed by rapid recovery.
oligodendrocytes in the central nervous
system (CNS). Astrocytes form the Question 6. D) Carpal tunnel syndrome
structural framework for neurons within Explanation: The principal use of nerve
the CNS. Microglia are specialised conduction studies is to identify damage to
macrophages. Astrocytes, peripheral nerves, determine whether such
oligodendrocytes and microglial cells are damage is focal (e.g. carpal tunnel
collectively known as glial cells. syndrome) or diffuse (e.g. polyneuropathy
Ependymal cells line the cerebral such as Guillain-Barré syndrome), and
ventricles. establish if it is predominantly axonal or
demyelinating in nature.
Question 3. B) Receptive dysphasia Electromyography (rather than nerve
Explanation: The language centres are conduction) is abnormal in motor neuron
located in the dominant hemisphere. The disease and myasthenia gravis. Multiple
left hemisphere is dominant in almost all sclerosis is a disease of the central rather
right-handed individuals and 50% of left- than the peripheral nervous system.
handed individuals. Lesions to Broca's
area within the frontal lobe cause Question 7. E) MRI head scan
expressive dysphasia; damage to Explanation: Although there is no
Wernicke's area in the temporal lobe specific test for multiple sclerosis, the
results in expressive dysphasia. demonstration of patchy demyelination of
white matter within the central nervous
Question 4. A) Frontal lobe system may assist in the diagnosis. MRI is
Explanation: The frontal lobe is far more sensitive than CT in detecting
responsible for personality, emotional soft tissue abnormalities and is therefore
control and social behaviour, and lesions the investigation of choice.
here commonly present with disinhibited
or antisocial behaviour. Associated
physical phenomena may include 'frontal
release signs', i.e. primitive reflexes such
as grasp reflex, rooting reflex and
palmomental response.

227
Question 8. D) Current treatment with Question 11. B) Migraine
aspirin Explanation: The history of severe,
Explanation: Lumbar puncture is throbbing, episodic, unilateral headache
contraindicated in the presence of raised associated with nausea, vomiting and
intracranial pressure, as the procedure can photophobia in a patient of this age is
result in the downward shift of intracranial highly suggestive of migraine. 'Classical'
contents into the spinal canal (coning). migraine consists of a triad of paroxysmal
Lumbar puncture should therefore not be headache, nausea and vomiting, and an
performed if there is papilloedema, a 'aura' of focal neurological events.
reduced level of consciousness or focal However, many patients experience
neurological signs that may suggest an paroxysmal headache (with or without
underlying space-occupying lesion. It is vomiting) without aura; this is termed
also contraindicated in the presence of a 'common' migraine. When present, the
bleeding tendency (e.g. thrombocytopenia, aura is most often visual, consisting of
disseminated intravascular coagulation, shimmering zigzag lines ('fortification
treatment with warfarin). Aspirin spectra').
treatment is not a contraindication.
Question 12. A) Sumatriptan
Question 9. B) Worsens as the day Explanation: Avoidance of obvious
progresses precipitants may prevent migraine attacks.
Explanation: Headache due to raised Acute migraine may respond to simple
intracranial pressure tends to be maximal analgesics (e.g. paracetamol) and anti-
on waking in the morning, and improves emetics. Severe attacks can be treated with
or resolves through the day as the upright one of the 'triptans' (e.g. sumatriptan);
posture reduces intracranial pressure. It these are 5-hydroxytryptamine agonists
often responds well to simple analgesia. that cause vasoconstriction of the
extracranial arteries, which are thought to
Question 10. C) Tension headache be dilated in migraine. The other agents
Explanation: Tension headache is listed can all be used to prevent recurrent
extremely common and often follows a migraines but are not useful during an
chronic course. The pain is usually acute attack.
generalised and becomes more noticeable
as the day progresses. Although it Question 13. E) Associated nasal
responds poorly to analgesia, individuals congestion and conjunctival injection
are typically able to continue with their Explanation: Typical cluster headache
daily activities. comprises episodic severe, unilateral,
periorbital pain. It is associated with
unilateral lacrimation, nasal congestion
and conjunctival injection. Episodes
characteristically last 30-90 minutes and
can occur daily for a few weeks, followed
by respite for several months before
recurrence of another 'cluster'. The
condition is five times more common in
men than women.

228
Question 14. B) Persistent nystagmus Question 16. C) Left pre-central gyrus
Explanation: Disorders affecting either Explanation: Partial (focal) seizures
the vestibular nucleus in the brain stem or result from paroxysmal discharge of
its central connections are distinguished neurons in a focal area of the cerebral
from disorders of the labyrinth by the cortex. The clinical manifestations depend
persistence of both vertigo and nystagmus. on the area of cortex involved.
Positionally induced central vertigo Involvement of the pre-central gyrus
persists for as long as the head position is (frontal lobe, motor cortex) causes partial
maintained, as does the accompanying motor seizures affecting the contralateral
nystagmus. In contrast, peripheral side of the body, while post-central gyrus
positional vertigo (and any associated (parietal lobe, sensory cortex) activity
nystagmus) fatigues quite quickly if the leads to partial sensory seizures. Occipital
inducing position is maintained. This foci result in partial visual seizures, and
phenomenon of fatigability is seen with temporal lobe foci can cause rhythmic
the Hallpike maneouvre (see the link smacking of the lips or alterations of
below). This test is positive in benign perception (e.g. déjà vu). Partial seizures
paroxysmal positional vertigo - a are classified as simple partial if
condition, more common in the elderly, consciousness remains intact, and complex
which causes brief episodes of vertigo partial if consciousness is altered. Any
with certain head movements. Severe partial seizure may subsequently spread to
vertigo lasting a few days associated with the rest of the brain, resulting in a tonic-
vomiting and ataxia is characteristic of clonic phase (secondary generalisation).
labyrinthitis. Ménière's disease, another
cause of paroxysmal labyrinthine vertigo, Question 17. D) Alcohol withdrawal
usually presents initially with tinnitus and Explanation: Alcohol withdrawal causes
distorted hearing. generalised seizures. The other listed
conditions cause focal cerebral pathology
Question 15. B) Severely bitten tongue and can therefore result in partial seizures;
Explanation: Determining the cause of a these may become secondarily
collapse can be difficult and obtaining a generalised. Individuals with partial
witness account is crucial. Details of the seizures should undergo brain imaging to
events immediately before (prodrome) and identify focal structural defects.
after (recovery) provide useful clues.
Vasovagal syncope is typically preceded Question 18. D) He should refrain from
by lightheadedness, sensations of heat, driving until seizure-free for 1 year
tinnitus and dimming of vision. A history Explanation: Patients should be advised
of aura preceding the blackout suggests to inform the Driver and Vehicle
seizure but is often lacking (perhaps due to Licensing Authority (DVLA) of their
post-ictal retrograde amnesia). Cardiac seizure and to refrain from driving until
syncope (e.g. Stokes-Adams attack) they are fit-free for 1 year.
frequently occurs without any warning.
With regard to the event itself, urinary Question 19. D) Sodium valproate
incontinence and brief twitching of limbs Explanation: Anticonvulsant treatment
are non-specific features that may occur should generally be considered after two
during any syncopal attack. On the other seizures have occurred. The majority of
hand, cyanosis and tongue biting are more patients can be controlled with
specific to seizure and the finding of a monotherapy. Sodium valproate is first-
severely bitten tongue after loss of line treatment for primary generalised
consciousness is pathognomonic of a epilepsy; carbamazepine is the usual drug
generalised seizure. Extreme 'death-like' of choice for partial seizures.
pallor is characteristic of cardiac syncope.
A prolonged recovery phase characterised
by confusion, headache, drowsiness or
amnesia strongly suggests seizure.

229
Question 20. C) IV diazepam Fibres conveying joint position and
Explanation: Status epilepticus describes vibration sensation enter the spinal cord at
prolonged seizure activity or recurrent the posterior horn and pass without
seizures without full recovery of synapsing into the ipsilateral dorsal
consciousness between attacks. It usually columns, from where they ascend to the
refers to generalised tonic-clonic seizures brain stem. They synapse with second-
and is a medical emergency. order neurons in the gracile and cuneate
nuclei, which then cross the midline and
Question 21. B) Muscle fasciculations ascend to the thalamus. Pain and
Explanation: Muscle wasting and temperature sensory fibres, on the other
fasciculations are manifestations of lower hand, enter the spinal cord and synapse
motor neuron lesions (anterior horn cell, with second-order neurons. These cross
motor root, nerve plexus or peripheral the midline at the level of entry and then
nerve). All of the other options listed are ascend in the contralateral spinothalamic
signs of upper motor neuron pathology tract to the thalamus. A unilateral cord
(motor cortex or corticospinal tracts). lesion will therefore cause ipsilateral loss
of joint position and vibration sensation,
Question 22. D) Parkinson's disease with contralateral loss of pain and
Explanation: Parkinson's disease is temperature below the level of the lesion.
characteristically associated with a 'pill- Ipsilateral upper motor neuron signs
rolling' tremor which is most pronounced develop due to involvement of the
at rest. The tremor of thyrotoxicosis is best ipsilateral corticospinal tracts.
detected by asking the patient to hold the
arms outstretched (i.e. sustained posture). Question 25. D) 9
Cerebellar lesions cause an intention Explanation: •Eye-opening To pain = 2
tremor which worsens as the hand •Motor response Localises = 5
approaches a fixed target (identified using •Verbal response Incomprehensible
the 'finger-nose' test). A flapping tremor sounds = 2
(asterixis) is observed with metabolic The Glasgow Coma Scale provides a tool
derangements such as hepatic for objectively assessing a patient's level
encephalopathy and hypercapnia; holding of arousal and response to stimuli. It is
the arms outstretched with hyperextended particularly useful for monitoring trends in
wrists leads to intermittent flapping of the the level of consciousness.
hands.
Question 26. E) Imaging of the brain
Question 23. D) Hemiballismus - brief, (CT or MRI) should be performed in
non-purposeful twitching of muscle most patients
groups Explanation: Anticholinesterases (e.g.
Explanation: Hemiballismus refers to donepezil) may produce modest
dramatic, flinging movements of the improvements in cognitive function in
limbs. These are usually unilateral, and selected patients with mild or moderate
result from vascular lesions of the Alzheimer's disease. Prominent visual
subthalamic structures. The definition hallucinations would be more suggestive
given in option D is that of myoclonus. of Lewy body dementia than Pick's
disease. Infections (e.g. urinary tract
Question 24. B) Right-sided cervical infection, pneumonia) can cause acute
spinal cord lesion confusion (delirium) and can lead to
Explanation: This patient has Brown- decompensation in patients with dementia,
Séquard syndrome, caused by a unilateral but are not in themselves a cause of
spinal cord lesion. dementia. Brain imaging should be
performed to exclude potentially treatable
pathology (e.g. cerebral tumour, chronic
subdural haematoma), but brain biopsy is
very seldom required.

230
Question 27. C) Hyperreflexia A Painful third nerve palsy is classical of a
Explanation: Features of cerebellar posterior communicating artery aneurysm.
pathology include ataxia, incoordination of
limb movements (e.g. past pointing), Question 31. B) Horner's syndrome
intention tremor, dysdiadochokinesis Explanation: Sympathetic nerve fibres
(inability to perform alternating originating in the hypothalamus pass down
movements), dysarthria and nystagmus. the brain stem and cervical cord to emerge
at T1, then return back up to the eye in
Question 28. D) Left homonymous association with the internal carotid artery
lower quadrantanopia and supply the dilator pupillae muscle to
Explanation: The anatomy of the visual stimulate pupil dilatation. Lesions of the
pathways is shown in the link below. sympathetic pathway (e.g. due to locally
Lesions anterior to the optic chiasm cause invasive apical lung tumour) therefore
visual field defects restricted to one eye, result in a unilateral small pupil associated
while lesions posterior to the chiasm cause with a partial ptosis and reduced sweating
homonymous field defects affecting the on that side (Horner's syndrome).
contralateral side of the visual field in both
eyes. From the lateral geniculate body in Question 32. D) The ability to wrinkle
the thalamus, lower fibres running in the the forehead is preserved
temporal lobe represent the upper part of Explanation: The term 'Bell's palsy' refers
the field, while upper fibres in the parietal to an isolated idiopathic facial nerve (VII)
lobe represent the lower part of the field. palsy. There is lower motor neuron
A right parietal lesion will therefore cause weakness on the affected side, with a
a left homonymous lower quadrantanopia. facial droop and an inability to wrinkle the
forehead. Other features include difficulty
Question 29. D) Right abducens (VI) closing the affected eye, reduced tear and
nerve lesion saliva secretion, and reduced taste
Explanation: Horizontal diplopia on sensation. Hyperacusis may occur if the
rightward gaze implies a problem with nerve to stapedius (a branch of the facial
right eye abduction (right VI nerve nerve) is involved. Preservation of the
stimulates lateral rectus) or left eye ability to wrinkle the forehead on the
adduction (left III nerve stimulates medial affected side indicates an upper motor
rectus). A useful tip to remember is that neuron pattern of facial weakness.
the outer image always arises from the
paretic eye; the lesion here must therefore Question 33. E) Treatment with
be within the right VI (abducens) nerve or warfarin
its nucleus. Explanation: Patients who are
Question 30. A) Left oculomotor (III) anticoagulated with warfarin or who have
nerve a non-iatrogenic coagulopathy require
Explanation: The trochlear (IV) nerve urgent imaging of the brain to rule out the
supplies the superior oblique muscle, possibility of an intracerebral
while the abducens (VI) nerve supplies haemorrhage.
lateral rectus. All other extraocular
muscles are innervated by the oculomotor
(III) nerve. This nerve also supplies
levator palpebrae superioris (which keeps
the upper eyelid open) and the constrictor
pupillae muscle (which mediates papillary
constriction in response to light). Third
nerve palsies therefore lead to ptosis with
a dilated unreactive pupil, and the eye
rests in a 'down and out' position due to
unopposed activity of the lateral rectus and
superior oblique muscles.

231
Question 34. D) Loss of consciousness The evidence to support surgery in
Explanation: The definition of a TIA is symptomatic patients with moderate (50-
the rapid onset of a focal neurological 69%) stenosis and asymptomatic patients
deficit, of presumed vascular origin, that with severe stenosis is less conclusive, as
resolves within 24 hours. It also includes these patients have a smaller benefit/risk
transient monocular blindness due to ratio than patients with severe
vascular occlusion in the retina (amaurosis symptomatic stenosis. Patients with
fugax). Basilar ischaemia can cause loss of stenosis of < 50% do not benefit from
consciousness but is very uncommon and, carotid endarterectomy, irrespective of
as a general rule, the diagnosis of TIA symptoms. Finally, a patient with severe
should not be made in patients who residual disability would gain little benefit
present with episodes of syncope, from preventing a further stroke within the
dizziness or confusion as these do not same territory and may have a greater risk
reflect focal cerebral dysfunction. of surgical complications.

Question 35. D) Warfarin Question 37. B) CT head scan


Explanation: In addition to lifestyle Explanation: The clinical signs and
modifications, antiplatelet, lipid-lowering symptoms in this patient are suggestive of
and antihypertensive therapy forms the a subarachnoid haemorrhage. An
cornerstone of secondary prevention for emergency head CT scan is essential.
most patients with an ischaemic stroke. About 15% of patients with a
Recent large-scale randomised trials have subarachnoid haemorrhage will have a
demonstrated the benefit of statins and normal CT scan; in these cases, a lumbar
antihypertensives in these patients, even puncture should be performed 12 hours
with blood pressure and cholesterol levels following the onset of headache to look for
within the 'normal' range. There is no net xanthochromia (breakdown products of
benefit in prescribing anticoagulants after red blood cells).
ischaemic stroke to patients in sinus
rhythm. The evidence base for stroke Question 38. E) Hypercholesterolaemia
secondary prevention is included in the Explanation: Cerebral venous thrombosis
link below. is uncommon. Hypercholesterolaemia and
hypertension are risk factors for arterial
Question 36. C) Left middle cerebral thromboembolic disease, but not for
artery territory infarct with good venous occlusion.
functional recovery, 80% left carotid
artery stenosis
Explanation: Carotid endarterectomy
reduces the risk of stroke in patients with
severe stenosis of the internal carotid
artery but carries a significant risk of
perioperative mortality and stroke.
Decisions on whether to operate must,
therefore, be based on a careful
benefit/risk analysis. The absolute
reduction in risk of future stroke is greatest
for symptomatic patients with 70-99%
stenosis and, in general, outweighs the risk
of surgical complications. Importantly,
symptomatic patients are defined as
patients with a TIA or non-disabling
stroke in the territory of the carotid artery
on the same side as the stenosis in the
preceding 6 months.

232
Question 39. C) It follows a relapsing Question 41. B) Senile plaques and
and remitting course in the majority of neurofibrillary tangles are
patients characteristic features on brain
Explanation: Multiple sclerosis (MS) is histology
characterised by inflammation and patchy Explanation: Alzheimer's disease is the
demyelination within the central nervous most common cause of dementia, affecting
system (CNS). The condition is more 5% of the population over 65 years of age
common in females and peak age of onset and 30% over 80. Histology from the brain
is between 30 and 40 years. MS can typically reveals senile plaques
present with diverse neurological features (containing amyloid) and neurofibrillary
according to the areas of the CNS tangles. Although both short- and long-
involved. Optic neuritis (optic nerve), term memory are affected, defects in the
sensory symptoms and leg weakness former are generally more obvious and
(spinal cord), and ataxia (cerebellar fibres) occur earlier. CT head scan typically
occur frequently but intellectual function shows cerebral atrophy, but there are no
is usually preserved until the later stages specific appearances that are
of the disease. Most patients experience a pathognomonic for Alzheimer's disease.
relapsing-remitting course, while 20% Treatment with anticholinesterase drugs
have primary progressive MS. Although may lead to modest improvements in
there is no single diagnostic investigation, cognitive function in selected patients but
plaques of CNS demyelination can be seen they do not halt progression of the disease.
on MRI. The presence of oligoclonal
bands of IgG in the cerebrospinal fluid is Question 42. B) Parkinson's disease
consistent with (but not specific for) a Explanation: Parkinson's disease is an
diagnosis of MS. idiopathic condition characterised by
depletion of dopaminergic neurons in the
Question 40. B) Methylprednisolone substantia nigra. It is classically composed
Explanation: Pulsed high-dose steroids of the triad of tremor, rigidity and
(methylprednisolone) reduce the duration bradykinesia. A fine resting tremor is
of relapses by means of their anti- usually first seen in the fingers and thumb,
inflammatory effect. Interferon beta is an and is characteristically 'pill-rolling' in
immune modulator which reduces the nature. Rigidity (increased muscle tone)
number of relapses by 30%. Azathioprine causes stiffness and a flexed posture.
also has some effect in reducing relapses Coexistent tremor causes cogwheel
and improving long-term outcome. rigidity. Bradykinesia (slowness of
Plasmapheresis and IV immunoglobulin movement) is manifest by slowness of gait
are occasionally used in severe disease. and difficulty with tasks such as fastening
buttons and writing.

Question 43. B) Wide-based gait


Explanation: An unsteady wide-based
gait (ataxia) is a feature of cerebellar
disease. It is also seen in conditions with
impaired joint position sensation.

233
Question 44. D) Dyskinesia secondary to Question 47. B) Motor neuron disease
levodopa can be improved by modifying Explanation: Motor neuron disease is a
the dosing regimen progressive disorder caused by
Explanation: The rationale for giving degeneration of motor neurons in the
levodopa (a dopamine precursor) in spinal cord and cranial nerve nuclei, and
Parkinson's disease is to boost dopamine of pyramidal neurons in the motor cortex.
production in the remaining dopaminergic Patients present with muscle weakness,
neurons of the substantia nigra. However, and neurological examination reveals a
if it is given as monotherapy, more than combination of lower motor neuron (e.g.
90% is decarboxylated peripherally, wasting, fasciculations) and upper motor
preventing it from reaching the brain and neuron (e.g. increased tone, brisk reflexes,
leading to a high incidence of side-effects. extensor plantars) signs. Sensation is
This is overcome by coadministering a normal. Involvement of the lower cranial
peripheral dopa-decarboxylase inhibitor nerves may cause bulbar palsy with
(carbidopa or benserazide). As these do dysarthria and dysphagia.
not cross the blood-brain barrier they
prevent conversion to dopamine in the Question 48. C) Administer IV
periphery but allow it to occur in the brain. benzylpenicillin
Involuntary movements (dyskinesia) are Explanation: The presence of headache,
related to long-term use of levodopa fever and neck stiffness in association with
therapy. Initiation of levodopa should a purpuric rash is strongly suggestive of
therefore be delayed until there is meningococcal meningitis. This is a
significant disability, especially in younger medical emergency requiring resuscitation
patients. Dyskinesia can be improved by and immediate treatment with empirical
modification of the drug dose and IV antibiotics. Although a lumbar
frequency of administration. Levodopa puncture is required, this must not delay
may help tremor in some patients but is antimicrobial therapy. A CT head scan
generally more effective at relieving should be performed before lumbar
bradykinesia and rigidity. Anticholinergic puncture if there are clinical signs of
drugs may be useful in treating tremor and possible raised intracranial pressure (e.g.
rigidity. drowsiness, focal neurological deficit,
seizures).
Question 45. E) Haloperidol
Explanation: Neuroleptic agents (e.g.
haloperidol, chlorpromazine) and certain Question 49. B) Upper motor neuron
anti-emetics (e.g. metoclopramide) have signs predominate
anti-dopaminergic actions and can Explanation: Poliomyelitis is caused by
therefore cause parkinsonism an enterovirus. Involvement of anterior
horn cells results in lower motor neuron
Question 46. A) It has autosomal signs (flaccid weakness, fasciculations,
recessive inheritance areflexia). Treatment consists of bed rest
Explanation: Huntington's disease is an and physiotherapy. Respiratory muscle
autosomal dominant disorder characterised involvement may necessitate invasive
by involuntary choreiform movements. ventilation via a tracheostomy. Childhood
Symptoms usually develop in middle age, immunisation with an oral vaccine has
by which time individuals have often had dramatically reduced the incidence of
children. Genetic screening is available for polio in many countries.
asymptomatic family members after
appropriate counselling. Involuntary
movements are accompanied by
psychiatric symptoms which eventually
progress to frank dementia.

234
Question 50. C) Cerebral abscess However, metastases to the brain can
Explanation: The CT scan shows a single occur from extracranial primary tumours
low-density area with ring enhancement in (e.g. lung, breast, gastrointestinal tract).
the right temporal lobe, consistent with a MRI offers better resolution than CT, and
cerebral abscess. There is associated is particularly useful for imaging
cerebral oedema and midline shift to the structures in the posterior fossa (brain
left. The condition typically presents over stem and cerebellum). The steroid
days or weeks with features of raised dexamethasone reduces oedema
intracranial pressure, seizures and focal surrounding cerebral tumours, and can
neurological signs. The main differential often lead to an improvement in conscious
diagnosis is a cerebral tumour. level. Meningiomas are benign tumours
arising from the meninges, whereas
Question 51. E) Tetanus antitoxin gliomas (e.g. astrocytoma) arise from the
should be administered in all suspected brain substance; the former are therefore
cases easier to resect completely and carry a
Explanation: The bacterium Clostridium better prognosis.
tetani is found in soil and usually enters
the body through contaminated wounds. It Question 54. C) Cerebellar signs may be
multiplies only in anaerobic conditions, present
e.g. necrotic tissue. The organism itself Explanation: Acoustic neuroma is a
remains localised but produces an benign tumour arising from the Schwann
exotoxin that affects anterior horn cells. cells of the VIII (vestibulocochlear)
The most important early symptom is cranial nerve. The condition may arise
spasm of the masseter muscles, which sporadically or in association with
causes difficulty in opening the mouth and neurofibromatosis type 2. Sensorineural
chewing (trismus or lockjaw). The rigidity deafness is extremely common. The
spreads to the rest of the body and causes tumour is located in the cerebellopontine
violent convulsions. Treatment comprises angle; distortion of the cerebellum can
IV human tetanus antitoxin to neutralise therefore lead to ataxia and cerebellar
absorbed toxin, and wound debridement signs in the limbs. As the lesion is situated
and benzylpenicillin to prevent further in the posterior fossa, MRI is the
toxin production (toxoid). investigation of choice. Treatment is by
surgical resection.
Question 52. C) Cognitive function is
usually preserved Question 55. C) Idiopathic (benign)
Explanation: Creutzfeldt-Jakob disease intracranial hypertension
(CJD) is a spongiform encephalopathy Explanation: Idiopathic (benign)
mediated by a prion protein. Spongiform intracranial hypertension is characterised
change in the brain is associated with by raised intracranial pressure in the
neuronal loss and gliosis in the grey absence of a space-occupying lesion or
matter. Clinical features include ventricular dilatation. The condition tends
myoclonic jerks and a rapidly progressive to affect young obese women. Other risk
dementia. Repetitive slow wave factors include drugs (e.g. tetracyclines,
complexes on EEG are characteristic, and vitamin A), Addison's disease and
life expectancy is reduced to 4-6 months. withdrawal of steroid therapy. Patients
complain of headache, and papilloedema
Question 53. B) Primary intracerebral is usually the only clinical finding. CT of
tumours do not, generally, metastasise the head is normal and lumbar puncture
outside the central nervous system demonstrates elevated CSF pressure with
Explanation: Primary intracerebral normal constituents.
tumours do not metastasise outside the
central nervous system, even if malignant.

235
Question 56. E) Acetazolamide Cervical cord compression would produce
Explanation: Treatment of idiopathic motor and sensory signs in all four limbs
intracranial hypertension includes weight (see the link below). The spinal cord ends
loss and withdrawal of precipitating at the T12/L1 spinal level; below this
factors. Other measures to lower hangs a collection of nerve roots called the
intracranial pressure include the carbonic cauda equina. Lesions compressing the
anhydrase inhibitor, acetazolamide, and cauda equina therefore only cause lower
repeated therapeutic lumbar puncture. motor neuron signs in the lower limbs.

Question 57. C) Headache Question 61. C) Spinal MRI scan


Explanation: Normal pressure Explanation: Spinal MRI is the
hydrocephalus is characterised by the triad investigation of choice for identifying
of gait apraxia, dementia and urinary compression of the spinal cord or nerve
incontinence. Headache is not a feature. roots. Urgent investigation is required to
The condition predominantly affects older enable prompt treatment. Soft tissue detail
patients. is excellent, and it is often possible to
determine the reason for the cord
Question 58. D) S1 compression (e.g. spinal metastases). This
Explanation: Sudden onset of lower back helps guide further therapy in terms of
pain while straining, accompanied by surgery or radiotherapy.
'sciatica' and a positive straight leg raise
test, suggests lumbar disc herniation with Question 62. D) Multiple sclerosis
resultant nerve root compression. The Explanation: Peripheral neuropathies can
neurological findings described affect a single nerve (mononeuropathy) or
correspond to pressure on the S1 root. several discrete nerves (mononeuritis
multiplex), or be generalised
Question 59. B) Analgesia and early (polyneuropathy). There are multiple
mobilization causes, the most common of which are
Explanation: The vast majority of diabetes mellitus, alcohol excess, Guillain-
patients with sciatica recover with a Barré syndrome, hereditary neuropathies,
combination of analgesia and early vitamin deficiencies and drug toxicity.
mobilisation. Bed rest retards recovery.
Local anaesthetic or steroid injections are Question 63. A) Amiodarone
useful if there is associated ligamentous Explanation: See list of Drugs causing
injury or joint dysfunction. Surgery is peripheral neuropathy
usually considered only if there is no
response to conservative treatment or if Question 64. B) Median nerve
there is progressive neurological disability. Explanation: This patient has carpal
Radiotherapy is useful only in patients tunnel syndrome, a mononeuropathy
with spinal cord or nerve root compression affecting the median nerve. Symptoms are
secondary to malignancy. often self-limiting and a wrist splint may
provide some relief. More severe or
Question 60. C) Thoracic cord persisting symptoms may require surgical
compression decompression at the wrist.
Explanation: This patient has signs of
spinal cord compression, which is likely to
be secondary to bony metastases. The
presence of bilateral upper motor neuron
signs in the legs (spastic paraparesis), with
preserved upper limb function and a
sensory level on the abdomen, suggests
involvement of the thoracic spinal cord.

236
Question 65. D) Respiratory muscles are synaptic membrane of the neuromuscular
not affected junction. IV injection of the short-acting
Explanation: Guillain-Barré syndrome is anticholinesterase edrophonium (the
an acute inflammatory polyneuropathy Tensilon test) increases the available
caused by cell-mediated demyelination of levels of acetylcholine and produces a
peripheral and cranial nerves. It is often transient improvement in muscle power.
preceded by a respiratory infection or Longer-acting anticholinesterases (e.g.
diarrhoeal illness (usually pyridostigmine) are used in the treatment
Campylobacter). Patients develop rapidly of myasthenia. Thymectomy in patients
ascending muscle weakness, starting in the with antibody-positive disease is
lower limbs and spreading up to the upper associated with a better prognosis,
limbs. Involvement of the respiratory irrespective of whether or not a thymoma
muscles can lead to respiratory failure and is present. Sudden, severe weakness in
necessitate ventilatory support. The myasthenia may be due to exacerbation of
majority of patients make a complete the disease (myasthenic crisis) or
recovery over 3-6 months. Early treatment excessive treatment with
with IV immunoglobulin or plasma anticholinesterase drugs (cholinergic
exchange shortens the duration of crisis). Differentiating between the two is
ventilation and improves prognosis. vital and the presence of fasciculations,
sweating and excessive salivation suggests
Question 66. C) Myasthenia gravis a cholinergic crisis.
Explanation: The fatigable muscle
weakness is typical of myasthenia gravis. Question 68. C) Delayed relaxation of
Symptoms worsen with exercise and as the skeletal muscle
day progresses. Features include Explanation: Myotonic dystrophy is an
intermittent ptosis, diplopia (due to autosomal dominant condition caused by
extraocular muscle involvement), and an expanded triplet repeat on chromosome
weakness of chewing, swallowing, 19. The characteristic finding is myotonia,
speaking and limb movement. There is no an abnormality of muscle relaxation. This
sensory deficit. The condition is more can be elicited by asking the patient to
common in women and usually presents make a fist and then rapidly straighten the
between the ages of 15 and 50 years. fingers; the latter can only be performed
slowly in patients with myotonic
Question 67. E) The presence of muscle dystrophy. Other features include cardiac
paralysis with fasciculation, sweating conduction abnormalities, lens opacities,
and excessive salivation suggests a frontal balding and cognitive impairment.
myasthenic crisis Autoantibodies to presynaptic calcium
Explanation: Myasthenia gravis is caused channels are a feature of Lambert-Eaton
by autoantibodies directed against myasthenic syndrome.
acetylcholine receptors in the post-

237
Chapter 26 SKIN DISEASE
Question 1. C) Chronic plaque psoriasis Question 4. E) Gold
Explanation: The picture shows nail Explanation: All of the above except gold
pitting and onycholysis (separation of the are used to good effect in the management
nail from the nail bed), both of which are of refractory psoriasis; gold is a disease-
classical of psoriasis. Nail changes are modifying drug used in rheumatoid
common in psoriasis and also include arthritis. Psoralens are natural
subungual hyperkeratosis. Neither the photosensitisers used in combination with
description nor the clinical photograph long-wavelength ultraviolet A (PUVA)
would be consistent with any of the other therapy.
differentials listed.
Question 5. B) Episodes often last longer
Question 2. B) It is the only skin disease than 24 hours
to demonstrate the Köbner Explanation: Urticaria refers to an area of
phenomenon Explanation: The Köbner focal dermal oedema secondary to an
phenomenon describes the tendency of a increase in capillary permeability. It is
lesion to arise in areas of previous trauma, more commonly known by patients as
such as skin biopsy sites and old surgical 'hives', and demonstrates a characteristic
scars. Although psoriasis does exhibit the weal and flare reaction. Episodes, by
Köbner phenomenon, so do a number of definition, last less than 24 hours. There
other conditions, including lichen planus are numerous possible precipitants, all of
and vitiligo. Psoriasis classically has a which trigger mast cell degranulation,
predilection for the extensor surfaces of leading to release of histamine and a range
the limbs. The Auspitz sign is of other inflammatory mediators.
pathognomonic of psoriasis, and describes Correspondingly, antihistamines improve
the minute capillary bleeding that occurs symptoms but do not abolish them
on lifting a psoriatic scale. Nail changes completely.
are common and also include onycholysis.
The pustules which may feature are sterile. Question 6. A) Streptococcus pyogenes
Explanation: Guttate psoriasis often
Question 3. C) Penicillin antibiotics occurs in children and adolescents after a
Explanation: All of the above have been streptococcal sore throat. The fungus T.
associated with flares in psoriasis, except rubrum is a common cause of ringworm of
for penicillin antibiotics. Although not the trunk. P. acnes is a bacterium which
mentioned above, alcohol intake is colonises the pilosebaceous ducts in acne
described by many patients as an and produces pro-inflammatory factors. B.
exacerbant of psoriasis. burgdorferi is the causative organism in
Lyme disease, one of the features of which
is erythema chronicum migrans, an
annular skin lesion which spreads, clearing
centrally. S. scabiei causes scabies, an
intensely itchy dermatosis characterised by
'burrows' in the web-spaces of the hands.

238
Question 7. D) Necrobiosis lipoidica Question 11. C) Warty texture with
Explanation: The clinical description and 'stuck-on' appearance
the photograph correlate best with the Explanation: In assessing any mole, the
diagnosis of necrobiosis. Skin conditions principal concern is to exclude malignant
which occur with greater frequency in melanoma. A naevus with a warty, 'stuck-
diabetics than in the normal population on' appearance is much less worrisome
also include acanthosis nigricans and than one which demonstrates any of the
granuloma annulare. other features listed. A warty, 'stuck-on'
appearance is more suggestive of a
Question 8. D) Topical or intralesional seborrhoeic wart, which is a benign
steroids may be useful epidermal tumour. A useful mnemonic to
Explanation: Necrobiosis lipoidica occurs help remember worrying features in a
with greater frequency but not exclusively mole is 'ABCDE' this is explained in the
in patients with diabetes. There is no link below.
correlation between degree of glycaemic
control and severity of the skin disease, Question 12. D) Amelanotic melanomas
and although superinfection may occur, no are more common than the pigmented
infective organism is implicated in the variety
pathogenesis. The lesions readily ulcerate Explanation: Melanoma is classified into
and subsequently tend to heal slowly. four basic subtypes: superficial spreading,
lentigo maligna, nodular and acral
Question 9. E) Basal cell carcinoma lentiginous. The characteristics of each
Explanation: The appearance of this should be known. Amelanotic melanomas
lesion is classical for a basal cell (those without pigment) are very rare;
carcinoma (BCC) or 'rodent ulcer'. These careful examination with a dermatoscope
are slow-growing tumours of the basal cell will usually reveal a few flecks of colour.
layer of the epithelium that occur most
commonly on the face of elderly Question 13. A) Dark skin and black
individuals. Their clinical appearance is hair
characterised by a pearly, telangiectatic Explanation: The phenotype of red hair
edge, with a central ulcer. and pale skin is associated with an
increased risk of skin cancer, due to
Question 10. E) Local invasion is more exposure to ultraviolet radiation in the
common than haematogenous spread absence of adequate melanin
Explanation: Basal cell carcinomas pigmentation. The corollary of this is that
(BCCs) are slow-growing tumours which those with dark skin have a relatively
very rarely metastasise. They are usually reduced risk of developing skin cancer. All
removed by surgical excision. Cryotherapy of the other factors mentioned are
has significant comorbidity when used for associated with an increased risk of skin
anything other than superficial lesions; cancer.
radiotherapy can be used for larger lesions,
but surgery tends to produce the best
cosmetic result. Mohs' surgery is a special
kind of 'tissue-conserving' surgery, in
which excised tissue margins are
examined histologically then extended
successively by millimeters until they are
clear. Although time-consuming, this
method ensures that the bare minimum of
tissue is taken.

239
Question 14. A) Atopic eczema A more extensive form of hair loss, called
Explanation: This history is pretty alopecia totalis, describes loss of all scalp
classical for atopic eczema. The child has hair with preservation of body hair.
an itchy rash in the skin creases, which Alopecia universalis describes loss of all
began within the first 2 years of life, and scalp and body hair. Androgenetic
there is also a history of asthma, one of the alopecia may occur in both males and
atopic spectrum of disorders. (These females.
comprise hay fever, asthma and a tendency
towards allergy: for example, to animal Question 18. C) Antibiotics
dander.) Lichenification describes the skin Explanation: The lesions are those of
changes that occur with prolonged periods erythema nodosum, a panniculitis
of scratching and rubbing, resulting in (inflammation in the deep dermis and
increased skin markings. It is a feature of subcutaneous fat). The nodules are
many skin diseases. characteristically painful, and although
there are many infective causes such as
Question 15. D) Dithranol tuberculosis and streptococcal infections
Explanation: Dithranol is an anthralin (hence a chest X-ray and ASO titre would
used in the topical treatment of psoriasis. be appropriate investigations), antibiotics
All of the other agents are important do not form part of the management of this
elements of the management of childhood condition. There a number of drug causes,
atopic eczema. Topical steroids should be and it is particularly important to ask
used at an appropriate strength for the site females about oral contraceptive use.
to which they are applied; for instance, as Systemic diseases such as sarcoidosis are
a general rule, nothing stronger than 1% also associated.
hydrocortisone should be applied to the
face, where the skin is thinner than the rest Question 19. C) Macules are usually
of the body. It may necessary to use more raised
potent steroids elsewhere. Explanation: A macule is a small, flat
area of altered colour or texture.
Question 16. D) Bullous pemphigoid
Explanation: The paucity of oral mucous Question 20. D) Bullous impetigo
membrane involvement in bullous Explanation: All of the above rashes are
pemphigoid is one of the key characteristically itchy with the exception
distinguishing features between this of bullous impetigo, which is a
condition and pemphigus. Pemphigus and staphylococcal infection of skin, common
Stevens-Johnson syndrome may feature in young children and the
bullous lesions of the mucous membranes. immunocompromised. It occurs when the
SLE may feature oral ulceration. infecting staphylococci produce
Examination of the mouth of a patient with exfoliatoxin A or B, which causes
lichen planus may reveal Wickham's striae blistering lesions to arise.
(a fine white lacy network overlying the
buccal mucosa). Question 21. E) Atenolol
Explanation: All the drugs listed above
Question 17. A) Discoid lupus may cause photosensitivity, with the
erythematosus causes a non-scarring exception of atenolol. None of the drugs in
alopecia the ß-blocker class are associated with
Explanation: Discoid lupus causes a photosensitivity.
scarring alopecia. Alopecia areata
describes a non-scarring patchy hair loss Question 22. C) Patients are usually
which is associated with other over the age of 18 years
autoimmune conditions, including vitiligo Explanation: SSSS is a serious exfoliative
and thyroid disease. skin condition that predominantly affects
children, particularly neonates.

240
Question 23. B) Onycholysis Psoriasis Question 24. D) Diabetes mellitus
Explanation: Nail changes are common in Explanation: Diabetes is associated with
psoriasis and include pitting, onycholysis several skin conditions but not porphyria
(separation of the nail plate from the nail cutanea tarda.
bed) and subungual hyperkeratosis.
Koilonychia (spoon-shaped nails) is a sign
of iron deficiency, while leuconychia
(whitening of the nails) is a rare
manifestation of hypoalbuminaemia. Question 25. B) Digoxin
Fibrosing alveolitis may cause clubbing, Explanation: Erythema multiforme (EM)
as may cyanotic congenital heart disease. is characterised by targetoid lesions that
Beau's lines, transverse grooves in the have a 'bull's-eye' appearance. Blisters
nails, are a non-specific manifestation of may feature; severe bullous EM is called
any acute illness. Stevens-Johnson syndrome.

241

You might also like